You are on page 1of 159

lE.

Irodov
,Basic laws
ofelectromagnetism

> l(
1

"
Basic laws of electromagnetism
r If. E. HPOJJ,OB
T

OCHOBHhIE 3AKOHbI
3JlEKTPOMAl'HETH3:\IA I.E.IrodoY
1f3~aTeJlhCTBO «BhlCillaH illKOJIa)
MOCHBa
Basic laws
ofelectromagnetism
Translated from Russian
by
Natasha Deineko and Ram Wadhwa

,
I
! f
I

Mir Publishers IOSCOI'

~ ,
~
CBS
CBS PUBLISHERS & DISTRIBUTORS
4596/1-A, 11 Darya Ganj, New Delhi - 110 002 (India)
,~

1
First pUblbhed HI86
Hevisl.'d from the 1983 RUl'sian edition Preface

@> II3~aTe,lbClIlC' <d1hICIHaJI 111 KOJI3') , 1983


'ij; English translation. Mir Publishers. 19aiJ

The main idea behind this book is to amalgamate the de-


scription of the basic concepts of the theory and the practical
methods of solving problems in one book. Therefore, each
chapter contains first a description of the theory of the
First Indian Edition: 1994
subject being considered (illustrated by concrete examples)
Irette'" aasie La," and then a set of selected problems wi th solutions. The prob-
Reprint: 1996
lems are closely related to the text and often complement it.
Reprint: 1998 Hence they should be analYfed together with the text. In
Reprint: 1999 author's opinion, the selected problems should enable the
Reprint: 200I reader to att~jn a deeper understanding of many important
topics and /to' visualize (even without solving the problems
This edition has been published in IndIa by arrangement with but just by going through them) the wide range of applica-
Mir Publishers, Moscow tions of the ideas presented in this book.
In order to emphasize the most important laws of electro-
Copyright © English Translation, Mir Publishers, 1986 magnetism, and especially to clarify the most difficult top-
ics, the author has endeavoured to exclude the less impor-
All rights reserved. No part of this book may be reproduced or trans-
tant topics. In an attempt to describe the main ideas concise-
Iy, clearly and at the same time correctly, the text has been
mitted in any form or by any means, electronic or mechanical, includ-
kept free from superfluolls mathematical formulas, and the
ing photocopying, recording, or any information storage and retrieval main stress has been laid on the physical aspects of the phe-
system without permission, in writing, from the publisher. nomena. With the same end in view, various model repre-
sentations, simplifying factors, special cases, symmetry con-
Published by : siderations, etc. have been employed wherever possible.
Satish Kumar Jain for CBS Publishers & Distributors. SI units of IDtaSUrements are used throughout the book.
4596/I-A, II Darya Ganj, New Delhi - 110 002 (India) However, considering that the Gaussian system of units is
still widely used, we have included in Appendices 3 and 4
For sale in India only the tables of conversion of the most important quantities
and formulas from SI to Gaussian units.
Printed at: The most important statements and terms are giYen in
Nazia Printers, Delhi - 110 006 italics. More complicated material and problems involving
cumbersome mathematical calculations are set in brevier
ISBN: 81-239-0306-5
type. This material can be omitted on fIrst reading without
any loss of continuity. The brevier type is also used fot'
problems and examples.
fi Preface Contems
The book is intended as a textb k f
dents specializing in physics (i~ t~O f or undergraduate stu-
on general physics) It' c I e bramework of the course
teachers. . . an a so e used by university
TNhel~uthor is grateful to Prof. A.A. Detlaf
L . . \.aptsov who re' d I and Reader
5
numbet. of valuable VIewe t Ie manuscript and made a ..
comments and sllg'gestions. Preface ••.••••
List of Notations • . . . . . ..
. . .
..
to

I. Irodov it
t. Electrostatic Field in a Vaeuum
it
1.1. Electric Field • • • . . . t6
t,2. The Gauss Theorem . 19
1.3: Applications of the Gauss Theorem . . 23
t,4. Differential Form of the Gauss Theorem 25
1.5. Circulation of Vector E. Potential . . . 30
1.6. Relation Between Potential and Vector E 34
1.7. Electric Dipole . • • . . . • 39
Problems •..........

2. A Condudor in an Electl'08tatic Field 45


2.1. Field in a Substance . . . . . . . • 45
2.2. Fields Inside and Outside a Conductor . . • 47
2.3. Forces Acting on the Surface of a Conductor 49
2.4. Properties of a Closed Conducting Shell . 5t
2.5. General Problem of Electrostatics. Image Method 54
2.6. Capacitance. Capacitors 57
problems •....•. 60

3. Eledm Field in Dielectrics • 67


3.1. Polarization of Dielectrics 67
3.2. Polarization 70
3.3. Properties of the Field of P 72
3.4. Vector D . . . . . . . . 76
.3.5. Boundary Conditions . . . 80
3.6. Field in a HomogeneouS Dielectric 84
Problems •.•.• 86

4. EIlt'TgY of Electric Field • . . . • • • • 94


4.1. Electric Energy of a System of Charges . . . . . , 94
4.2. Energies of a Charged Conductor and a Charged·
Capacitor 99
4.3~ Energy of Electric Fidd . . . . . tOO
4.4. A System of Two Charged Bodies t05
4.5. Forces Acting in a Diplectric t06
Problems •............ t to
8
...... _~

Contents

9.7. Energy and Forces in Magnetic Field 240


5. Direct Current . . . • • . . . • • • . . . • . 116 244
Problems .
5.1. Curr~nt Density. Continuity Equation . . ft6
5.2. Ohm s Law for a Homogeneous Conductor 119 to. Muwell's Equations. Energy of Electromagnetic Field 253
5.3. Generalized Ohm's Law . . • . . . . . . 122
5:4. Branched Circuits. Kirchhoff's Laws . . . 126 10.1. Displacement Current . . . . . . . . 253
5.5. Joule'3 r qw . . . . . . . • • • . . . . 129 10.2. Maxwell's Equations. .... . . . . 251
5.6. Transient blCesses in a Capacitor Circuit 133 10.3. Properties of Maxwell's Equations . . . . 261
Problems •.......•..••••.• 136 10.4. Energy and Energy Flux. Poynting's Vector 264
10.5. Electromagnetic Field l'\foml'ntum 268
6. Magnetic Field in a Vaeuum 141 Problems . . . . . 271
6.1. Lorentz Force. Field B 141 277
6.2. The Biot-Savart Law . . . . '. 145 11. Etedric Oscillations
6.3. Basic Laws of Magnetic Field . • • • • . • . . . 148 277
6.4. Applications of the Theorem on Circulation of Vec- 11.1. Equation of an Oscillatory Circuit
11.2. Free Electric Oscillations . 280
tor B . . . . . . . . . . . • • • . . • . . . • 150 285
6.5. Diffe!e~tia! Forms of Basic Laws of Magnetic Field 154 11.3. Forced Electric Oscillations
11.4. Alternating Current 290
6.6. Ampere s Force . 155 Problems " 293
6.7. Torque Acting on a Current Loop • 159
6.8. Work Done upon Displacement of Current Loop 161
Problems ..... 163 Appendices it" • 299
1. Notations for Units of Measurement . . . 299
7. Magnetic Field in a Substance . • . • • • 172 2. Decimal Prefixes foI'" Units of Measurement . . . . . 299
7.1. Magnetization of a Substance. Magnetization Vec- 3. Units of Measurement of Electric and Magnetic Quan-
tities in SI and Gaussian Systems . 299
tor J . . . . . . . . • . • . . • . . . . . . . 172
7.2. Circulation of Vector J . . . • . 176 4. Basic Formulas of Electricity and Magnetism in SI
and Gaussian Systems . . 300
7.3. Vector H . . . 178 304
7.4. Boundary Conditions for Band H . 182 5. Some Physical Constants . .
3ubject Index . . . . . . . . . . 305-
7.5. Field in a Homogeneous Magnetic 185
7.6. Ferromagnetism .......• 188
Problems 192
8. Relative Nature of Electric and Magnetic Fields 198
"I, 8.1. EIElctromagnetic Field. Charge Invariance 198
8.2. Laws of Transformation for Fields E and B 200
8.3. Corollaries of the Laws of Field Transfo;m~tio~ 206
8.4. Electromagnetic Field Invariants 208
Problems '" . . . ••.... 209

9. Electromagnetic Induction 217


9.1. Faraday's Law of Electromagnetic Induction. Lenz's
Law ...•.............•.•• 217
9.2. Origin of Electromagnetic Induction 220
9.3. Self-induction 226
9.4. Mutual Induction . " ..••. 231
9.5. Magnetic Field Energy . . . . . • • . 234
9.6. Magnetic Energy of Two Current Loops 238
List of Notations (~rostatic Field in a Vacuum

1.1. Electric Field


Vectors are denoted by the bold-face upright letters
{e.g. r, E). The same letter in the standard-type print (r, E) Electric Charg~. At present it is knowntl~at ~iverse ~he­
denotes the magnitude of the. vector. nomena in nature are based on four fundamentallllteractl~ns
Average quantities are denoted by angle brackets ( >, among elementary particles, viz. strong, electromag,!etIc,
e.g. (v), (P). weak, and gravitational interactions. Ea~h. type of I~ter­
The symbols in front of the quantities denote: action is associated with a certain charactel'lstic of a particle.
A is the finite increment of the quantity, viz. the difference For example, the gravitational interaction. depends o.n t~e
between its final and initial values, e.g. AE = E z - E I , mass of particles, while the electromagnetic lllteractlOn IS
Acp = CPt - <fl; determined by electric charges. .
d is the differential "(infinitesimal increment), e.g. dE, dcp; The electric charge of a particle is one of its basl~ charac-
6 is the incremental value of a quantity, e.g. 6A is the ele- teristics. It has the following fundamental properties: .
mentary work. (1) electrlc charge exists in two forms, I.e. it can be pOSI-
Unit 'vectors: tive or negative; . .
ex, ell' e z (or i, j, k) are the unit vectors of Cartesian coordi-
(2) the algebraic sum of charges in any electl'lcally lllSU-
nates x, y, z; lated system does not change (this statement expresses ·the
ep , eq., e z are the unit vectors of cylindrical coordinates law of 'conservation of electric charge); .
p, (f'. z; (3) electric charge is a relativistic invariant: its maglll-
n is the unit vector of the normal to a surface element; tude does not depend on the reference system, in othe~ words,
't is the unit vector of the tangent to the contour or to an
it does not depend on whether the charge moves or IS fixed.
interface. I t will be shown later that these fundamental properties
Time derivative of an arbitrary function f is denoted by have far-reaching consequences. '. ,
Electric Field. In accordance with modern theory, lllter-
.af/at or by the dot above the letter denoting the function, f. acti,on among charges is accomplished through a field .. Any
Integrals of any multiplicity are denoted by a single sym- electric charge q alters in a certain way the .propertIes ~f
bol ) and differ only in the notation of the element of inte- the space surrounding it, I.e. creates an electnc fielri.. ThiS
means that another, "test" charge placed at some pomt of
gration: dll is the volume element, dS is the surface ele- the field experiences the action of a for~e. .
ment al;d dt is the element of length. Symbol 1, denotes the Experiments show that the force F actmg on a fixed test
integration over a closed contour or a closed surface. point charge q' can always be represented in the form
Vector operator V (nabla). The operations involving this F == q'E, (1.1)
operator are denoted as follows: vcr is the gradient of
cp (grad cp), v· E is the divergence of E (div E), and V X E where vector E is called the intensity of the electric iield at .
is the curl of E (curl E). a given point. Equation (1.1) shows that vector .E can be
Vector product is denoted la·: b], where a and bare defined as the force acting on a positive fixed HIllt charge.
vectors. Here we assume that the test charge q' is sufficiently small so
12 1. Blectrodctic Fidd in JI FUllUm 1.1. Electric Field 13

that it.s intr?du~tion does not noticeably distort the field where ri is the distance between the charge qi and the point
under mveStl~atlOn (as a result of possible redistribut' f under consideration.
charges creatmg the field). Ion 0 This statement is called the principle of superposition of
. The Field of 8 Point Charge. It follows directly from ex er
i.ment (~Qulomb's law) that the intensity of the field ~
1Jdxe~ pomt charge q at a ,distance r from it ean be repre.sent-
:f electric fields. It expresses one of the most remarkable prop-
erties of fields and allows us to calculate fIeld intensity of
any system of charges by representing this system 8!' an ag-
e 111 the form gregate of point charges whose individual contribut.ions are

E=_1 .!L e
41t8 r 2 T' 0
I (1,2)
given by formula (1.2).
Charge Distribution. In order to simplify mathematical
calculations, it is of~n convenient to ignore the fact that
charges have a discr te structure (electrons, nudei) and
assume that they are" eared" in a certain way in space. In
:~)er~l:~o, ~s the electric constant and e is the unit vector of other words, it is conv nient to replace the acturJl distribu-
e vector r drawn from the centre of the field
l"ad, l,:; h
T
I
tion of discrete point charges by a fictitious continuous
i~Je charge q i~ located, to the point we are interest:c't ~~e distribution. This makes it possible to simplify calculations
ormula (1.2) IS written in SI. Here the eoefficient . considerably without introducing any significant erNr.
1/41t1,o "'= 9 X 109 m/F , While going over to a continuous distribution, the concept
of charge dlWsity is introduced, viz. the volume density p,
t~le r d~a.rge q is measured in coulombs (C) and the field in ten- surface density 0, and linear density A., By definition,
SIt) E In, volts pe~ metre (Vim). Vector E is directed alon r dq • dq dq
or ~pposItely to it depending on the sign of the charueg P=dV' o=dS' A.=df' (1.4)
~o~mula (1.2) expresses Coulomb's law in the "field" q. t
It .IS nnpol'tan: that the intensity E of the field created ~rm~ where dq is the charg'e contained in the volume dV, on the
lJ?Int charge is inversely proportional to the square of tYh surface dS, and in the length dl respectively.
d l~tRn("e r All e t
. ~: I'd .' . .
xperJmen a I l ts indicate that this lawe
resu Taking these distributions into consideration we can repre-
IS HI 1 tor dIstances from 10- 13 cm to several kilometres sent formula (1.3) in a different form. For example, if the
a~dl thedre are no .grounds to expect that this law will ~ charge is distributed over the volume, we must replace qj
VIO ate f or larger distances. by dq == p dV and summation by integTation. This gives
It should also be noted that the force actin on a te
~harg~ in th~ field created by a fixed point charie does n~: E=....!- r pedl'
J
=_1_ \ prdV (1.5)
.
.ete~ on wether the test charge is at rest or moves. This
reLer~ t.? a system of fixed charges as well.
41t80 r' 4nE J o r' ,
where the integration is performed over the entire space with
(1 Prm.clplc of Superposition. Besides the law expressed b nonzero values of p.
th· )fi : ~ 81;0 follows from experiments that the intensity
2
e . e1 0 a system of fixed point charges is equal to the
J Thus, knowing the distribution of charges, we can complete-
ly solve the problem of fmding the electri y field intensity
vectOf sum of the intensities of the fields that would be by formula (1.3) if the distribution is discrete or by formu-
create d by each ('harge separately: la (1.5) or by a similar formula if it is continuous. In the
general case, the calculation involves certain difficulties
!
I E=~ (1.3)
(although they are not of principle nature). Indeed, in order
to find vector E, we must first calculate its projections E~,
Ell' and E z, which means that we must take three integrf!.ls

,1

1 I, I
,-

r l'
I, 1. Electro.tcttc Fteld In • Vacuum 1.1. Electric Field 15
'1
of the t.vpe (1.5). A8 a rul~, the problem becomes much problem: we must lind the.component dEx of the lield create~1 by the
simpler in cases when a system of charges has a certain sym- 1·lement dl of the filament, having the charge dq, and then llltel:l'rate
metry, Let us consider two examples. the result over all the elements or the tilament. In this ca~e
1 A dt
Example 1. The field on the axis of atbin uDiformly charged ring. dEx=dE
. cos ':'=-4- r-- cos ':t,
;leo - ,
A charge q > 0 is uniformly distributed over a thin ring of radius a.
Find the electric field intensity E on tbe axis 'Of tbe ring as a function where A = q/2t is the linear charf{e density. Let ~IS reduce this equation
of the distance z from itseentre. to the form convenient for lIltpgratlOn, Figure 1.2 shows that
It ean be easily shown that vector E in this case must be directed dl cos a = rda and r = x/cos a; consequently,
along the axis of the ring (Fig. 1. t). Let us isolate element dl on the 1 Ar Ib, A
dE x = - - · - - 2 - -=-=-4--. cos ada.
4.n:eo r lleox
E
This ·expressio.l can be easily integrated:
ao A
E = _f.._ 2
4.n:e ox Ji cos a da = -4·- - 2 sin a o_
;le x
I)
o

E where a o is the maximum value of the angle a, sin a o= 11}tr12 +x 2•


r
z Thus,
E':;" q/2l 2 I q
4.n:e ox Vl~-J-x2 4.n:e ox Jf~+X2
t:--..:;;!l--oo
In this case also E ~ qI4n. 8eX"s for x » 1 as the lield of a point charge.
Fig. 1.1 l"ig. 1.2 Geometrical Description of Electric Field. If we know
ring in the vicinity of pOInt A. We write the expression for the comp- vector E at each point, the electric field can be visually re-
onent dEL of the field created by this element at a point C: presented with the help of field lines, or lines of E. Such a
t Adl . line is drawn so that a tangent to it at each point coincides
dEL = - z - · - 1 - cos a, with the direction of vector E. The density of the lines,
..n.lle r
Le. the number of lines per unit area normal to the lines is
where A. = q/2rr.a. The values of r and a will be the same for all the proportional to the magnitude of vector E, Besides, the lines
elements of the ring, 'Ind hence the integration of this equation is sim-
ply reduced t{) the replacement of Adl by q. As a result, we obtain are directed like vector E'. This pattern gives the idea about
the configuration of a given electric field, Le. about the di-
E=-q- I
4n.eo (a l +zl)'fl rection and magnitude of vector E at each point of the field.
On the General Properties of Field E. The field E defined
It can be seen that for z~ a the field E ~ qf4rr.eoz2 , i.e. at large dis- above has two very important properties. The knowledge of
tances the system behaves as a point charge. '
Example 2. The field of a .. niformlyeharged straight filaDlellt. A these properties helped to deeper understand the very con-
thin straight filament of length 2l is Uniformly charged by a charge q. cept of the field and formulate its laws, and also made it pos-
Find ~he ~eld intensity E at a point separated by a distance x froIl1 sible to solve a number of problems in a simple and elegant
the mIdpomt of the filament and located symmetrically with respect way. These properties, viz. the Gauss theorem and the theo-
to its ends.
rem on circulation of vector E. are associated with two most
I t is clear from symmetry considerations that vector E must be important mathematical characteristics of all vector fields:
directed as shown in Fig. 1.2. This shows the way of solving this
the flux and the circulation. It will be shown below that in
.,I - '\

16
1.2. The Gauss Theorem
~--------
17
terms of these two concepts not only all the laws of electri-
city but also all the laws of magnetism can be describefJ,. Let The Gauss Theorem. The flux of E through an arbitrary
us go over to a systematic description of these properties. closed surfaco S has a remarkable property: it depends only
on the algebraic sum of the charges embraced by this sur-
face, i.e.
1.2. The Gauss Theorem
Flux of E. For the sake of clarity, we shall use the geomet··
II A'EdS·--Qln,
"_ 1 (1.7)
rica1 description of electric field (with the help of lines
I j eo
of E). Moreover, to simplify the where the circle on the integral symbol indicates that the
analysis, we shall assume that the integration is performed over a closed surface.
density of lines of field E is equal This expression is essentially the Gauss theurem: the flux of
E to the magnitude of vector E. Then
the number of lines piercing the
area element dS, the normal n- to
which forms angle ex with vector E,
Fig. 1.3 is determined as E ·dS cos a. (see
Fig. 1.3). This quantity is just the
flux d<1l of. E through the area element dS. In a more compaet
form, this can be written as
d<1l = En dS = E.dS, Fig. 1.'( Fig. 1.5
where En is the projection of vector E onto the normal n to
the area element dS, and dS is the vector whose magnitude E through a closed surface is equal to the algebraic sum of the
is equal to dS and the direction coincides with the direction charges enclosed by this surface, divided by eo.
of the normal. It should be noted that the choice of the di- Proof. Let us fIrst consider the field of a single point charge
rection of n (and hence of dS) is arbitrary. This vector could q. We enclose this charge by an arbitrary closed surface S
be directed oppositely. (Fig. 1:4) and find the flux of E through the area element dS:
If we have an arbitrary surface S, the flux of E through 1
it can be expressed as dlP = ~s-~ex__~= - ne4 . ..J." dS cos ex = - 4
q
neo dQ, (1.8)
o r·
(!J= Is E as. (1.6) where dQ is the solid angle resting on the area element dS
and having the vertex at the point where the charge q is lo-
cated. The integration of this expression over the entire
This is an algebraic quantity. since it depends not only on surface S is equivalent to the integration over the entire
the configuration of the field E but also on the choice of the solid angle, Le. to the replacement of dQ by 4n. Thus we
normal. If a surface is closed it is customary to direct the obtain <lJ = qleo, as is defined by formula (1.7).
normal n outside the region enveloped by this surface, i.e. It should be noted that for a more complicated shape of a
to choose the outward normal. Henceforth we shall always closed surface, the angles a may be greater thnn n12, and
assume that this is the case. . hence cos a hnd dQ in (1.8) generally assume eitlICr positive
II Although we considered here the flux of E, the concept of or negative vlJlues. Tlius, dQ is an algehraic quantity: if
flux is applicable to any \'Setor field as well. dn rests Oft the inner l'idp of the surface S, dQ > 0, while if
it rests on the outer side, dQ < O.
2-181
18 1. Electrostatic Field in a Vacuum
1.3. Applications of the Gauss Theorem 19
~:.--;.;...:..:-'.-::...:.~---_:..:.

In particular, this leads to the following conclusion' if


the charge q is located outside a closed surface S the flux stance: while the licld E itself depolld:-; Oil the mutual con-
figuration of all the charges, the [lux of E through an arbi-
of .E t.hr?ugh th~s surface is equal to zero. In order'to prove
tills, It IS suffiCient to draw through the charge q a conical trary closed surface S is determined by the algebraic sum
surface tangent to the closed surface S. Then the integration of the charges inside the surface S. This means that if we
o! Eq. (1.8) o~er the surface S is equivalent to tho integra- displace the charges, the fwld E willlJC changed everywhere,
tIon over Q (FIg. 1.5): the outer side of the surface S will be and in particular on the surface S. Generally, the flux of E
seen from the point q at an angle Q > O. while the inner side through the surface S will also change. However, if the
~t an angl~ -Q (the two angles being equal in magnitude): displacement of charges did not involve their crossing of
the surface S, the flux of E through this surface would re-
r~~ su~ IS equal to zero, and <1> 0, which" also agrees
=-c;

wltn (1./). In terms of fIeld lines or lines of E, this means main lJ,nchanged, although, we stress again, the field E itself
that the num~er of lines entering the volume enclosed by may change considerably. What a remarkable property of
th~ surface S IS equal to the number of lines emerging from electric field I
thIS surface.
Let us now consider the case when the electric field is
create~ by a system of point charges ql' Q2' . . . • In this 1.3. Applications of the Gauss Theorem
case, III accordance with the principle of superposition E =
+
= E1 E 2 + ..., where E 1 is the field created by the Since the p,eld E depends on the configuration of all charges,
charge qil etc. Then the flux of E can be written fh the form the Gauss t'heorem generally does not allow us to deter-
mine this field. However, in certain cases the Gauss theorem
~EdS=~(EI-' E2 + ... )ciS proves to be a very effective analytical instrument since it
gives answers to certain principle questions without solving
= J EidS + A;J £2 dS -'.- ... =ClJ i -I- (}J2 _!-
.k-. I I I •.••
the problem and allows us to determine the field E in a very
simple way. Let us consider some examples and then formu-
If- accordanc~ wi~h what was sa~d above, each integral on the late several general conclusions pbout the cases when appli-
rIght-hand SIde IS equal to qi/eo if the charge qi is inside cation of the Gauss theorem is the most expedient.
the closed surface S and is equal to zero -if it is outside the Example 1. On the impossibility 01 stable eqUilibrium 01 a charge
surf.ace S. Thus, the right-hand side will contain the alge- in an electric field. Suppose that we have in vacuum a system of fixed
braIC sum of only those charges that lie inside the surface S. point charges in equilibrium. Let us consider one of these charges,
To c?mplete the proof of the theorem, it remains for us e.g. a charge q. Can i~ be stable?
to consIder the case when the charges are distributed contin- In order to answer this question, let us envelop the charge q by a
uousl~ with the volume density depending on coordinates. small closed surface S (Fig. 1.6). For the sake of defmiteness, we as-
In thIS case, we may assume that each volume element dV sume that q > O. For the equilibrium of this charge to be stable, it is
necessary that the field E creatl'd by all the remaining charges of the
contains a "point" charge p dV. Then on the right-hand side system at all the points of the surface S he directed towards the charge
I of (1. 7) we have q. Only in this case any small displacement of the charge q from the
eqUilibrium position will gi ve rise to a restoring force, and the equilib-
Ii qln = ~ P dV, (1. 9) rium state will actually he stable. But such a configuration of the
field E around the charge q is in contradiction to the Gauss theorem:
the flux of E through the surface S is nl'gative, while in accordance
where the integration is performed only over the volume with the Gauss theorem it must he l'qual to zero since it is created by
contained within the closed surface S. charges lying outside the surface S. On tile other hand, the fact that
We must pay attention to the following important circum- E is equal to zero indicates that at S()Illl' p()ints of thl' surface S vl'clor
E is directed inside it and at some other lliJints it is llirect.ed outside.
1. iUectroltatlc Field In a VaclJlJm
1.3. Applications 01 the Gauss Theorem 21

to the field from the positively charged plane, while the lower arrows,
Hence it follows that in any electrostatic~lield_.acharge cannot be in to that from the negatively charged plane. In the space between the
stable equilibrium. planes the intensities of the fields bdng added have tlw same (Hrec-
Example 2. The field of a unHormly charged plane. Suppose that tion, hence the result (1.10) will be doubled, and the resultant field
the surface charge density is o. It is clear from the symmetry of the intensity \\ill be
problem that vector E can only be normal to the charged plane. More-
over, at points symmetric with respect ~o t~is pl.ane, vectors E ohvi- E=oh o' (1.11)
ously have the same magnitude but OPPOSite directiOns. Such a configu- where !J sLands for the magnitude of the surface charge density. It can
ration of the field indicates that a right cylinder should be chosen for be easily seen that outside this space the field is equal to zero. Thus,
the closed surface as shown in Fig. 1.7, where we assume that {1 > O.

o +
-- - ---
£=0

Fig. 1.6 Fig. 1.7


Fig. 1.8 Fig. 1.9
The flux through the lateral surface of this cylinder is equal to in the given case the field is located between the planes and is uni-
zero, and hence the total flux through the entire cylindrical surface form.
is 2E l!.S, where l!.S is the area of each endface. A charge (J AS is This result is approximately valid for the plates of finite dimen-
enclosed within the cylinder. According to the Gauss theorem, 2E AS = sions as well, if only the separation between the plates is considerably
= (J tJ.S/e o, whence E = 0/2e o' In a more exact forni, this expression smaller than their linear dimensions (parallel-plate capacitor). In this
must be written as case, noticeable deviations of the field from uniformity are observed.
En = 0/2eo' (1.10) only near the edges of the plates (these distortions are often ignored in
calculations).
where En is the projection of vector E onto the normal n to the charged F..xample 4. The field vf an infinite circular cylinder uniformly
plane the normal n being directed away from this plane. If (J > 0 charged ov('r th~ surface 1,0 that the charge A corresponds to its unit
then En > 0, and hence vector E is directed away from the charged ]en~th.
plane, as shown in Fig. 1.7. On the other hand, if {1 < 0 then En < ~, In this case, as follows from symmetry considerations, the field
and vector E is directed towards the charged plane. The fact thiit E IS is of a radial nature, i.e. vector E at eaen pomt is-perpendicular to
the same at any distance from the plane indicates thatJ the corre- the cylinder axis, and its magnitude depends only on the distance r
sponding electric field is uniform (both on the right and on the left of . from the cylinder axis to the point. This indicates that a closed sur-
the plane). face here should be taken in the form of a coaxial right cylinder
The obtained result is valid only for an infinite plane surface, (Fig. 1.9). Then the flux of E through the enrlfaces of the cylinder i~
since only in this case we can use the symmetry considerations dis- equal to zero, while the flux through the lateral surface is E T 2 mh
cussed above. However, this result is approximately valid for the region where E r is the projection of vPetor E onto the rad ius vector r coincid
near the middle of a finite uniformly charged plane surface far from ing with the normal n to the lateral surface of the cylinder of radim
its ends. r and height h. According to the Gauss theorem, 1'7 r'21trh =0 Altho fOJ
Example 3. The field of two parallel planes charged uniformly with > a, whence
densities (J and - ( J by unlike charges.
This fIeld can be easily found as superposition of the f,elds created ,.. - _A_ (r> a). (1.12
'r - 21teoT
by each plane separately (Fig. 1.8). Here the upper arrows correspond
22 1. Electrostatic Fteld tn a Vacuum

where E r is the projection of vector E onto the radius vector r coinciding


with the normal n to the surface at each of its points. The sign of the
charge q determines the sign of the projection E r in this case !is well.
Hence it determines the direction of vector E itself: either away from
the sphere (for q > 0) or towards it (for q < 0).
If r < a, the closed surface does not contain any charge and hence
within this reg-ion E = 0 everywhere. In other words, inside a uni-
formly charged spherical surface the electric field is absent. Outside
this surface the field decreases with the distance r in accordance with
the same law as for a point charge. .
Example 6. The field of a uniformly charged sphere. Suppose that
a charge g is uniformly distributed over a sphere of radius a. Obvious-
ly, the field of such a system is centrally symmetric, and hence for de-
termining the field we must take a concentric sphere as a closed sur-
face. It can be easily seen that for the field outside the sphere we obtain
the same result as in the previous example [see (1.13)1. However,
inside the sphere the expression for the field will be different. The
sphere of radius r < a encloses the charge q' = q (rla.)S since in our
case the ratio of charges is equal to the ratio of volumes and is pro-
portional to the radii to the third power. Hence, in accordance with
the Gauss theorem we have
E r ·4nri = 1- q (r}S
-
a

llo 1.4. Differential Form of the Gauss Theorem
whence
1 . q A remarkable proper~y of electri~ r~e~~s::fedsf~da ~rff:~:n?f~~:
E r =-4- .... r
nll o a
(r<a), (1.14)
theorem suggests thdat ~htls the~rb~ml'lt~s a~ an i~strument for analysis
which would broa en I s POSSI 1
andI~~~~~;~~~10 (1.7~ wticg is c~~:~r:: ~hi~h~:f:bii~~:~h~l ::l~~
i.e. inside a uniformly charged sphere the field intensity grows linear-
ly with the distance r from its centre. The curve representing the de-
pendence of E on r is shown in Fig. 1. to. the differential formlo t e h agU~ensity p' and the changes in the field
tion between the vo ume c ar e . .,
General ConcllJsions. The results obtained in the above intensity E in the vicinity of a gIven pOInt m space.
1. Electrostatic Field i-
••
a Tr
F aCllllm
1.5. Circulation of Vector E. Potential 25
For this purpose we first rep t h
enveloped by a clo~d surface S i':r: : e charge q in the volume V
only in combination with a scalar or vector function by which it is
is ~h~ volum~ charge ?ensity, averaged ~~ '1Vr
= I(p) V, Where {p)
~ sthl~uhte t.hlS expressIOn into Eq. (1 7) and d' e;;o bumeh
, w IC gives .
r
Then We
IVI e ot Its sides by
symbolically multiplied. For example, if we forin the scalar product
of vector V and vector E, we obtain
° °
+- +-
a
~. ~ E dS=(p)/Bo•
V·E=VxEx+VyEy..LVzEz=-
ax E x
I oy Ey 0% E z•
(1.15)
We now make the volume Y Je d . It follows from (1. i 7) that this is just the divergence of E.
point we are interested in. In this c~ to zero. by c0';1tracting it to the Thus, the divergence of the field E can be written as div E or V·E

t-
S:'
~iluhaneof P at the given point of the I~P) wJltohvlOusly tend to the
d side of Eq. (1.15) will tend toe I' an ence the ratio on the
(in both cases it is read as "the divergence of E"). We shall be using
the latter, more convenient notation. Then, for example, the Gauss
P Bo• theorem (1.18) will have the form
The quantity which is the limit of the ratio of t. E dS to
Is called the divergence of the fi ld E d ' 'j' Vas V--+-O (1.20)
hy defuIition. e an IS denoted by div E. Thus,
The Gauss theorem in the differential form is a local theorem:
the divergence of the field E at a given point depends only on the
div E= lim ..!.- k. E dS
v...o V ' j ' . (1.16) electric charge density p at this point. This is one of the remarkable
properties of electric field. For example, the field E of a point charge
The dive!'lfence of any other v to fi . is different at different points. Generally, this refers to the spatial
wa y• It· follows from definitioneC(1 16)eltdh ls de~rmined in a similar derivatives oE:dox, oEy/By, and oEz/o% as well. However. the Gauss
furi ctlon of coordinates. . at divergence is a scalar theorem states that the sum of these derivatives, which determines the
In order to obtain the ex ressi . divergence of Ef"turns out to be equal to zero at all points of the field
we m~t, in accordance with f116,o~ fkr the. divergence of the field E (outside the charge itself).
determine the flux of E thrO~ h 't a e an Infinitely small volume Y' At the points of the field where the divergence of E is positive, we
volu~e. and find the ratio of tfis f1~~ closed surface enveloping thi~
ohtaIned for the divergence will d :I
the volume. The expression
have the sources of the fieIa (positive charges), while at the points
where it is negative, we have Binks (negative charges). The field lines
dffl
te sys)tem (in different systems ~~o~~. th:e c~oice of the coordi-
erent. For example, in Cartesian coordi~~:e S'tl~ tU!D8 out to be
emerge from the field sources and terminate at the sinks.
. s 1 IS given by
div E = ~+ oE _L oE%
y
1.5. Circulation of Vector E. Potential
0:& oy I 0;-. (i.17)
Thus. we have found that as Y --+-' '. Theorem on Circulation of Vector E. It is known from
I
::nd,j. to plea, while the left-hand side O~~i1~5)d" Its nght.hand side mechanics that any stationary field of central forces is con-
e Ivergence of the field E' 1 d s IV E. Consequently
lame point through the equati~~ re ate to the charge density at th~ servative, Le. the work done by the forces of this field is
I independent of the path and depends only on the position

This equation expre


f divE=ple o'
-th-----
I (1.18)
of the initial and final points. This property is inherent in
the electrostatic field, viz. the field created by a system oj
The form of man e sses .e Gauss theorem in the differential f fixed charges. If we take a unit positive charge for the test
ira~IY simplified w~P[:::;~d::~i:~ir:;P~i~fftions.can be co~:r:t
il charge and carry it from point 1 of a given field E to point 2.
the elementary work of t he forces of the field done over thE
n artesian coordinates, the operator Vehasrth~ ;:;:tIal operator V.
distance dl is equal to E· dl, and the total work of the fiek
V=i.!-+
ox j
3-. a
oy + k 8i" ' (1.19)
forces over the distance between points 1 and 2 is defined a1
where I, J, and k are the unit vecto f th
operator V itself does not have any ~~anin~ X[-i
.
r-.
and Z-axes. The
ecomes meaniD~fu J
2
~ E dI. (1.21
t
26 1. Electrostatic Field tn a Vacuum 1.5. Ctrculation of Vector E. PotenUal 27

This integral is taken alon '. Indeed, if the oppollite were true and some lines of field E?lerJ!
therefore called the line inte:ra~ certam hne (path) and is closed, then taking the circulation of vector E along this line we would
We shall now show that fm ' h . immediately come to contradiction with theorem (1.22). This means
integral (1.21) of the path hrot t e mdependence of line that actually there are no closed lines of E in an electrostatic field:
that when taken along an b\ween two points it follows the lines emerge from positive charges
and terminate on negative ones (or go F!;::::j
gral is equal to zero. Integr:I (~r;f)Y closed path, this iute- to infinity). I • 'I
. . over a closed contour is Example 2. Is the configuration of an I I
called
. the cIrculation of vector E t..
an 'IS d.enoted by 'V'
d electrostatic field shown in Fig. 1.12
possible?
-4 . lr
I
Thus we state that cireul t' f .. t
~on 0 vector E in anv" electro-
static field is equal to ze ro,a I.e. No, itisnot. This immediately becomes
clear if we apply the theorem on circula- I lr I
tion of vector E to the closed contour L -.J
shown in the figure by the dashed line. ----
~Edl=O. (1.22)
The arrows on the contour indicate the
direction of circumvention. With such a

1248 special choice of the contour, the contri- Fig 1t2
This statement is' called th h bution to the circulation from its ver- . .
vector E. e t eorem on circulation of tical parts is equal to zero, since in
this case E .L dl and E ·dl = O. It remains for us to consider the
In order. to prove this th
. eorem.. we break an arbitrary closed two horizontal segments of equal length:,. The figure shows that the
contributions to t.lte circulation from these regions are opposite in
pa.th Into two parts la2 and 2bl sign, and unequal in magnitude (the contribution from the upper seg-
2 (FIg. 1.11). Since line integral (1.21) ment is larger since the field lines are denser, and hence the value of
i )
c:?
E is larger). Therefore, the ci'rculation of E differs from zero, which
(we denote it by does not contradicts to (1.22).
1 depend on the path be~~een 'points 1
(a) (b) Potential. Till now we considered the description of elec-
b and 2, we have
otb er h an,
f =
d it12 is
i '. On
• 12
the. tric field with the help of vector E. However, there exists
another adequate way of describing it by using potential cp
clear that (it should be noted at the very outset that there is a one-to-
Fig. 1.11 T (b) (b)
one correspondence between the two methods). It will be
• 112 - - \ ,where \ is the shown that the second method has a number of significant
Integral over the same segment b b ,21 • ., 21
advantages.
direction. Therefore ut taken In the opposite The fact that line integral (1.21) representing the work'of
(n) (b) (a) (b) the field forces done in the displacement of a unit positive
~ 12 L1
-I- ~
= 11 - L2 = 0, charge from point 1 to point 2 does not depend on the path
allows us to!state that for electric neld there exists a certain
Q.E.D. scalar function <p (r) of coordinates such that its decrease is
A field haVing property (1 22) . I given by
Hence, any electrostatic field . IS ca led .the potential field.
The theorem on' . lS a potentzal fleld.
to draw a numbe~I~Ctl?tIon of vector E ~akes it possible (1.23)
sorting to calculations ~mfortant ~oncluslOns without 1'e-
- . e us conSIder two examples.'
Example f • TI Ie rIe Id I ines of an electrostatic field E cannot be where <p 1 and <P2 are the values of the function <P at the
closed. points 1 and 2. The quantity <P (r) defined in this way is

I ~
28 1. Electrostattc Field in tJ Vacuum 2U

called the field potential. A comparison of (1.23) with the field of a lixed point charge: •
expression for the work done by the forces of the potential 1 q IJ Ii r
field (the work being equal to the decrease in the potential E.dt --
-
- - - (' r ·die
/trrf,o ,.2 ,l.'l:'o r:!.
energy of a particle in the field) leads to the conclusion
that the potential is the quantity numerically equal to the po- = -d (_1_ _.'L _i-const)
4Jlco r I ,
tential energy of a unit positive charge at a given point of the
field. where we took into account that cr dt =-= 1· (dl}r= dr, sillee
We can conditionally ascribe to an arbitrary point 0 of the the projection of dl onto en ~nd hence ?ll 1', is ~qual to ~he
field any value CPo of the potential. Then the potentials of increment of the magnitude of vector r, Le. dr. 'I he quantlty
all other points of the field will be unambiguously deter- appearing in the parentheses under the diffe~enti~l is exact-
mined by formula (1.23). If we change CPo by a certain value ly i:p (1'). Since the additive constant contamed III ~he fo~'­
Acp, the potentials of all other points of the field will change mula- does not play any physical role, it is usually omitted ill
by the same value. order to simplify the expression for (p. Thus, the potential
Thus, potential cp is determined to within an arbitrary of the field of a point charge is given by

I~
additive constant. The value of this constant does not play
any role, since all electric phenomena depend only on the = __
1 }L II (1.2:»
_ _ 4n~_'_
eleetric field strength. It is determined, as .will be"'shown
later, not by the potentia] at a given point but by the·poten- The absence'6f an additive constant in this expression
tial difference between neighbouring points of the field. indicates· that we conventionally assume that the potential
The unit of potential is the volt (V). is equal to zero at infinity (for r -+ 00).
Potential of the Field ola System of Charges. Let a system
Potential of the Field of a Point Charge. Formula (1.23) consist of fixed point charges ql' q2' .... In accordance
contains, in addition to the definition of potential cp, the with the principle of superposition, the field intensity at any
method of finding this function. For this purpose, it is suf- point of the field is given by E = E I + E 2 + ..., wh~re
ficient to evaluate the integral ~ E dl over any path be- E l is the field intensity from the charge ql' etc. By llsmg
formula (1.24)1 we can then write
tween two points and than represent the obtained result as
a decrease in a certain function which is just <p (r). We can E·dl = (E l + E 2 + ...) dl = EI·dl + E 2 ·dl + ...
make it even simpler. Let us use the fact that' formula = -dcpl - dCP2 - ... = --dcpl
(1.23) is valid not only for finite displacements butffor ele-
mentary displacements dl as well. Then, in accorda~ce with where cp = 2jcpf, Le. the principle of superposition t~rns out
this formula, the elementary decrease in the potential over to be valid for potential as well. Thus, the potential of a
this displacement is system of fixed point charges is given by
-dcp =--0 E·dl. (1.24) .- _1_ '" !!i- -I (1.26)
In other words, if we know the field E (r), then to find cp I cp - 4nco L..J r 'I
1. _i

we must represent E·dl (With the help of appropriate trans- where ri is the distance froID the point charge qj 10 the point
formations) as a decrease in a certain function. This function under consideration. Here we also omitted an arbitrary con-
willbo the potential cpo stant. This is in complete agreement witll the faet that any
Let us apply this method for finding the potential of the real system of charges is Lounded in space, and honce its
potential can bo taken equal to zero at inflIlity.
30 1. Electrostatic Field in a Vacuum 1.6. Relation Between Potential and Vector I!: 3t
-----------_._---- .-'-'-'=";~---

If the charges forming the system are distributed cUlItin- pression with formula (1.24) gives
uously, then, as hefore, we assume that each volume ele- Ex = - orp/vx, (1.29)
ment dV contains 3 "point" charge p dV, where p is the charge
. density in the volume dV. Taking this into consideration, wher~ the symbol of partial.derivative emphasizes that the
we call write formula (1.26) in a different form: functIOn rp ~x, y, z) must he differentiated only with respect
to x, assummg that y and z are constant in this case.
. In a similar wa~, w.e can obtain the corresponding expres-
( 1.27) 810.ns for the prOJectIOns E y and E z. Having determined
E", Ell' and E z, we can easily fmd vector E itself:
where the integration is performed either over the entire E= _ ( ocp
ox i + ~-
iJy
j + ~ k)
oz . (1.30)
space or over its part containing the charges. If the charges
are located only on the surface S, we can write T~e quantity in the par~Iltheses is the gradient of the po-
1 ~ adS
tentzaZ cp (grad cp or Vcp). We shall be using the latter more
<p = 4neo -r-' (1.28) convenient notation and will formally consider VCP 'as the
product of a symbolic vector V and the scalar cpo Then
where (J is the surface charge density and dS is the element Eq. (1.30) can be represented in the form
of the surface S. A similar expression corresponds to the
case when the charges have a linear distribution. (1.31)
Thus, if we know the charge distribution (discrete or con-
tinuous), we can, in principle, fmd the potential of any L~. the field inte~sity E'is equal to the potential gradient
system. With the miflUS SIgn. This is exactly the formula that can
1.6. Relation Between Potential and Vector E be used for reconstructing the field E if we know the
function <p (r).
It is known that electric fwld is completely described by
f E~ample. Find the field intensity. E if the field potential has the
vector function E (r). Knowing this function, we can find orm. (1~ ~ (x, y) = -axy, where. a IS a c~nstant; (2) cp (r) = -a'r,
the force acting on a charge under investigation at any point whe~ a Is.a constant vector and r IS the radlUs vector of a poiut under
of the neld, calculate the work of fwld forces for any displace- conslderahon.
ment of the charge, and so on. And what do we get by (1) By using formula (1.30), we obtain E = a (yi -1- xj).
_ (2) Let us first represent the function ip tls cp == --ax.x --. llll!! __
introducing potential? First of all, it turns out that if we izz, where ax, a y and az are cl)nstants. Then with the help of for-
know the potential cp (r) of a given electric fIeld, we can re- mthil! a (1.30) we find E = axi -+ ayj
..8 case the field E is uniform.
+ a k = a. It can be seen that in
construct the fwld E (r) quite easily. Let us consider this Z

question in greater detail.


The relation between rp and E can be established with tho Let ~s derive one more useful formula. We write the right-
help of Eq. (1.24). Let the displacement dl be parallel to hand sl~e of (1.24) in the form E·dl c=o R I dZ, where dl =
the X-axis; then dl = i dx, where i is the unit vector along . Id1 l IS an elementary displacement and E I is the projec-
the X-axis and dx is the increment of the coordinate x. In tIOn of vector E onto the displacement dl. Hence
this case
E·dl = E·i dx = Ex dx, IE l = -ocpI8Z, I (1.32)
where Ex is the projectioll of vector E onto the ullit vector i
(and not on the displacement dll). A comparison of this ex- Le. the projection of vector E onto the direction of the dis-
33
1.6. llelation Between Potential and Vector E
--------------~_. ----------~ ------
g2 1. Electrostatic Feeld in a Vacuum On the Advantages o~ Potential. 1l was noted earlier that
. electrostatic field is completely cltaracterizeu by vector
placement dl is equal to the d' • . .
p.otential (this is emphasized b;r~~~onal telflvfative .of the function E (r). Thea what is the use of introducing potential?
rivative). sym 0 0 partial de- There are several sound reasons for doing that. The concept
of potential is indeed very useful, and it is not by chance
Equipotential Surfaces Let . d
equipotential surface, viz. 'the s:;a~~t:~ :l~e t~etco~cept. of that this concept is widely used not only in physics but in
potentiallp has the same value. We shall sho~o~~ ~ 0 whIch engineering as well..
it. If we know the potential <P (r), we can easily calculate
:~ each point o.f the surface is directed along thea n::::.~r~ the work of field forces done in the displacement of a point
e eqUipotentIal surface and. towards the decrease in the charge q' from point 1 to point 2:
potential. Indeed, it follows A l2 ~ q' «PI - <pz), (1.33)
from formula (1.32) that the
projection of vector E onto where <PI and..<vz are the potentials at points 1 anu 2. This
any direction tangent to the means that the required work is equal to the decrease in the
eq,!ipo~entialsurface at a given potential energy of the charge q' upon its displacement from
pomt 18 equal to zero. This point 1 to 2. Calculation of the work of the lield forces with
means that vector E is norm- the help of formula (1.33) is not just very simple, but is
al to the given surface. Fur- in some cases the only possible resort.
ther, let us take a displaC&- Example. A df~rgc q is distributed over a thin ring of radius a.
ment dl along the normal to the Find the work of the lield forces dune in the displacement of a point
surface, towards decreasing charge q' from the centre of tpe ring to i:llinity. .
Fig. 1.13 p. Then Olp < 0, and accord- Since the distributiun o[ the charge q over the ring is unknown, we
mg to (1 32) E > 0 . cannot say anythiug detinite about the intensity E of the field created
v,ector E is. directed towards decreasing ~, ~r i~ the d;;;' by this charge. This means that we cannot calculate the work by eval-
tIon ?pposite. to ~hat of the vector Vlp. uating the integral ~ q'E dl in this case. This problem can be easily
It IS expedIent,.to draw equipotential surfaces in such a solved with the help of potential. Indeed, since all elements of the
way that the potential difference between two neighbourin ring are at the same distance a from the centre of the ring, the poten-
surfaces b~ t~e same. Then the density of e ui otentiJ tial of this point is Ipo ~= q/4Jv ou. And we know that cp = 0 at intinity.
s~aces WI.ll vIsually indicate the magnitudes fieYd inten- of Consequently, the work A = q'cpo = q'q/4nlO oa.
sttIes a~ dIfferent points. Field intensity. will be higher . 2. It turns out in many cases that in order to find electric
t e re~lOns ~h~re equipotential surfaces are denser '"th~ field intensity E, it is easier lirst to calculate the potential
pot~ntial rehef IS steeper"). \ " lp and then take its gradient than to calculate the value of E
Smce vector E ~s normal to an equipotential surface eve - directly. This is a considerable advantage of potential. In-
where, the field hnes are orthogonal to these surfaces. ry deed, for calculating <p, we must evaluate only one integral,
while for calculating E we must take three integrals (since
Th
lin:s ::
l
F~guhe df 3 shows a two-dimensional patt~rn of an electric field
h liDes eorres~o~d to equipoten~ial rlUrfaces, while thetiled MUd
it is a vector). Moreover, the integrals for calculating <P are
I. t. e mes of E. Such a representatIOn ean be easil v· usually simpler than those for Ex, By, and E z·
i~~~:t:yd\:tiYghhows
~
dthe hdirec.t!o.n of vector E, the regiorrs wi::e field
er an were lower as we'l's tIl . "\h
Let us note here that this does not apply to a comparative-
10 13 ly small number of problems with high symmetry, in
~~~~r steepness of the putential relief. SU(~ri a ;ai'tern ~~?':sed'to which the calculation of the lielu E directly or with the help
d. t' qilal~~i~lve answers to a number of .~lU. ,.stiOJl8 sU~.h as ';In'wha.t
lrec ,IOn WI "charge placed at a certain point m~veP Whe I> • of the Gauss theorem turns out to be much simpler.
iliagiutude o! the potential gradient hi~her? At which U:in~.;t.e1
e orce actmg on the charge be greater~" et.c. A 3-0181
r
I
34 1. Electrostatic Field in a Vacuum
----------'--'--'-.:-:..'-.:-:..~....:..::..:..::......:::..:......:=-:..=::.::::=-------
1.7. Electric Dipole
There are some other advantages in using potential which
will be discussed later. from the negative to the positive charge:
1.1. Electric Dipole p = ql, (1.35)

The Field of a Dipole. The electric dipole is a system of two where q > 0 and I is the vector directed as p.
equal in magnitude unlike charges +
g and - g, separated It can be seen from formula (1.34) that the dipole field
depends on its electric mom~nt p.. It will be shown below
by a certain distance l. When the dipole field is considered
. , that the behaviour of the dIpole III an external field .al~o
depends on p. Consequently, p is an important characterIstIc
p of the dipole. ' .
(b)
It should also be noted that the potential of the dI~)Qle field
decreases with the distance r faste~ than the ~otentlal of the
eb field of a point charge (in proportIOn to 1frz Illstead of 1fr).
!In order to find the dipole field, we shall use formula (1.32)
+q
a~d calculate the projections of vector E onto two mutually
perpendicular directions along the unit vectors er and ~
/ (Fig. 1. 14b):
q P
E aq> 1 2pcos~
Fig. 1.14 "",' r = - --ar = 4nll
o
-,-.- ,
(1.36)
it is assumed that the dipole itself is pointlike, i.e. the dis-
E aq> 1 p sin ~
t) = - r'~ = 4neo - - , . -
tance r from the dipole to the points under consideration is
assumed to be much greater than l. Hence, the modulus of vector E will be
The dipole field is axisymmetric. Therefore, in any plane
passing through the dipole axis the pattern of the field is J.: -?V-1-+-3-c-o-s
E=VE~+E6= 'tJ,80 r
z-tt. (1.37)
the same, vector E lying in this plane.
L~t us first find the potential of the dipole field and then In particular, for tt = 0 and tt = n/2 we obtain the ex-
its illtensity. According to (1.25), the potential of the dipole pressions for the field intensity on the dipole axis (E u) and
field at the point P (Fig. 1.14a) is defined as on the normal to it (E.d:

<.p ,-
1
4itl:: o
(lJ '1) =
7..- - -,:: 1
4ne
q(r_-r.)
r.r_ • Ell = ~80 2~, EJ. = 4~80 ~, (1.38)
o
Since r» l, it call 1e seen fro.:n Fig. 1.14a that r__ r + = Le. for the same r the intensity E u is twice as high as EJ.'
= 1 cos \} and r +r_ = r 2 , where r is the distance from the The Force Acting on a Dipole. Let us place a dipole into
point P to the dipole (it is pointlikel). Taking this into a nonuniform electric field. Suppose that E+ and E_ are
accoulJt, we get the intensities of the external field at the points where the
positive and negative dipole charges are located. Then the
(1.34) resultant force F acting on the dipole is (Fig. 1.i5a):
F = qE+ -qE_ = q (E+ - E_).
where p = ql is the electric moment oj the dipole. This quan-
tity corresponds to a vector directed along the dipole axis The difference E+ - E_ is the increment LiE of vector E
on the segment equal to the dipole length 1 in the direction
3*

J
1. Electro$tatlc Field In a Vacuum
f
1.7. Electric Dipole 37
I' of vector I. Since the length of this segment is small, we can
write
~E aE
where fJE ~/az is the derivative of the corresponding projec-
~E=E+-E_=-. l = az l. tion of vector E again onto the direction of vector I or p.
Let a dipole with moment p be oriented along the symme-
Substituting this expression into the formula for F, we try axis of a certain nonuniform field E. We take the positive
find that the force acting on the d'ipole is equal to direction of the X-axis, for example, F
as shown in Fig. 1.17. Since the incre- qe <= p.
I F=p"*, (1.39)
ment of the projection E ~ in the direc-
tion of vector p will be negative, F:r < O. ~F
and hence vector F is directed to the
where p = ql is the dipole electric moment. The derivative left, i.e. towards increasing field in- q...-----tp
appearing in this expression is called the directional deriva- tensity. If we rotate vector p shown in

(.~ ... tive of the vector. The symbol of


partial derivative indicates that it
is taken with respect to a certain
direction, viz. the direction coincid-
the figure through 90° so that the dipole
centre coincides with the symmetry
axis of the field, it can be easily seen
that in this position F:II: = O.
q-----~
FI

Fig. 1.16
ing with vector I or p. The Moment of Forces Acting on 8
~:-q~
Unfortunately, the simplicity of Dipole. Let<4Is consider behaviour of a dipole in an exter-

z:s.-
formula (1.39) is delusive: taking nal electric field in its centre-of-mass system and find out
~q~- the derivative fJE/fJl is a rather com- whether the dipole will rotate or not. For this purpose, we
plicated mathematical operation. must find the moment. of external forces with respect to the
We shall not discuss this question dipole centre of mass*.
<bJ in detail here but pay attention to ~By definition, the moment of forces F + = qE+ and F _ =
JL the esse\"~e of the obtained result. = -qE_ with respect to the centre of mass C (Fig. 1.18) is
Fig. 1.15
First of all, note that in a uniform equal to
field aEliJl = 0 and F = O. This
means that generally the force acts M = [r+ X F+I + [r_ X FJ=[r+ X qE+l -[r_ X qEJ,
on a dipole only in a nonuniform field. Next, i::l the gener- where r + and r _ are the radius vectors of the charges q +
al case the direction of F coincides neither with vector E and - q relative to the point C. For a sufficiently small di-
nor with vector p. Vector F coincides in direction only pole length, E+ ~ E_ and M = [(r+ - r_) X qEl. It re-
with the elementary increment of vector E, taken along the mains for us to take into account that r + - r _ = I and
direction of I or p (Fig. 1.15b). qI = p, which gives
Figure 1.16 shows the directions of the force F acting on a dipole
in the field of a point charge q for three different dipole orientations.
We suggest that the reader prove independently that it is really so.
IM= [p X EJ.! (1.41)

lf we are interested in the projection of force F onto a This moment of force tends to rotate the dipole so that
certain direction Xi, it is sufficient to write equation (1.39) its electric moment p is oriented along the external field E.
in terms of the projections onto this direction, and we get Such a position of the dipole is stable.
Thus, in a nonuniform electric field a dipole behaves as
Fx = P a:zx , (1.40)
• We take the moment with respect to the centre of mass in order
to eliminate the moment of inertial forces.
38 1. Electroltattc Field in a Vacuum
Problem' 39
follo,,":s: under the action of the moment of force (1.41),
the. dIpole tends to. get oriented along the field (p tt E), position, the moment of external forces will return the di-
whIle under the action of the resultant force (1.39) it is pole to the equilibrium position..

Problems
• t.t. A very thin disc is uniformly charged with sut'fa?e char~
+q density lJ > O. Find the electric field intensity E on the axiS of thIS
disc at the point from which the disc is seen at an angle .Q.

-q

Fig. 1.17 Fig. 1.18 £.


displaced towards the region where the field E has larger
magnitude. Both these motions are simultaneous.
The Energy of a Dipole in an External Field. We know
that the energy of a point charge q in an external field is
W. qcp, where cp is the fIeld potential at the point of lo- Fig. Lt9 Fig. 1.20
catIOn of the charge q. A dipole is the system of two charges
and hence its energy in an external field is ' Solu.io1l. It is clear from symmetry considerations th.at o~ the disc
axis vector E must coincide with the direction of this aXIS (FIg. 1.19).
W = q+cp+ + q_cp_ = q (cp+ _ cp_), Hence, it is safWentto find the component d!£z from the charg; of
the area e1emeBt tiS at the point A and th~n mtegrate the o.btamed
where CP+ and cp_ are the potentials of the external field at expression over the entire surface of lthe dISC. It can be easIly seen
the points of location of the charges +q and -q. To within that'
a quantity of the second order of smallness, we can write 1 lJdS
dE z =-4- - I cos 'It. (1'
alp JtEo r
cp+-cp-= az 1,
In our case (dS cos 'It)/rz = dO is the solid angle at which the area
,,:here acp/Ol is the derivative of the potential in the direc- element dS is seen from the point A, and expression (1) can be writ-
ten as
tIOn of the vector l. According to (1.32), acp/al = -E It
and henen '" + - '"_ ~ L E ,I - Ell.
from which we Il"l
1
dE z = -4- (J dO.
JtEo

IW = -p.E. (1.42)
Hence, the required quantity is
E=-4- aQ.
1
Jtto
It follows from this formula that the dipole has the mini-
mUt;D. energy (Wmin = -pE) in the position p tt E (the It should he noted that at large distances from the disc, Q = SIrS,
where S is the area of the disc and E = ql4nEors just as the field of the
positIOn of stable equilibrium). If it is displaced from this point charge q = lJS. In the immediate vicinity of the point O,the
solid angle g = 2a and E = a/2eo'
Problems
40 1. Electrostatic Field in a Vacuum

• f .2. A thin nonconducting ring of radius R is charged with a This gives


linear density A = Ao cos cp, where '-0 is a positive eoDStant and cp dEy = (A cos a da)/4ne oy and E y = A/4.tteoy·
is the azimuth angle. Find the electric field intensity E at the centre
of the ring. We have obtained an interesting result: Ex = E y independently of y,
Solution. The given charge distribution is shown in Fig. t,20.
The symmetry of this distribution implies that vector E at the point 0
is directed to the right, and its magnitude is equal to the sum of the
projections onto the direction of E of vectors dE from elementary
charges dq. The projection of vector dE onto vector B Is
1 dq
dE cos cp = -4- -R' cos cp, (1)
ne o
where dq = AR dlp = AoR cos cp dcp. IntegTating (1) over cp between 0
and 2n, we find the magnitude of the vector E:
2n a
E
.=
"-0 R
4ne
o
Jr cost d
cp cp =
"-0
4e R •
o
o Fig. 1.21
It should be noted that this Integral is evaluated in the most simple
way if we take into account that (cost cp) = 1/2. Then i.e. vector E is oriented at the angle of 45° to the filament. The modu-
lus of vector E is
2n
~ cos' cp dcp=(cos' cp) 2n=n. E= V E~+E~=j,. Y2i4rt8 oY •
o • 1.4. The Gauss theorem. The intensity of an electric field de-
pends only on the coordinates x and y as follows:
• 1.3. A semi-infinite straight uniformly charged filament has a
charge'" per unit length. Find the magnitude and the direction of the E = a (xi +
yj)/(x 2 +
y2),
field Intensity at the point separated from the filament by a distanC'-t! y
and lying on the normal to the filament, passing through its end. where a is a constant, and i and j are the unit vectors of the X-and
Y;-axes. Find the charge within a sphere of radius' R with the centre
Solution. The prohlem is reduced to finding Ex and E., viz. the at the origin.
projections of vector E (Fig. L21, where it is assumed fliat '" > 0). Solution. In accordance with the Gauss theorem, the required
Let us start with Ex. The contribution to Ex from the-charge element charge is equal to the flux of E through this sphere, divided by &0'
of the segment dz Is . In our case, we can determine the flux as follows. Since the field E is
1 Adx. axisymmetric (as the field of a uniformly charged filament), we arrive
dE x =-4- -t-sma. (1) at the conclusion that the flux through the sphere of radim(R is equal
neo r j to the flux through the lateral surface of a cylinder having the same
Let us reduce this expression to the form convenient for integration. radius and the height 2R. and arranged as shown in Fig. 1.22. Then
In our case, dx = r dalcos a, T = ylcos a. Then "
q=80 ~ E dS=8 oE rS,
dE x = - A
4 sin ex da.
neoy
where Er = aIR and S = 2nR·2R = 4nR2. Finally, we get
Integrating this expression over a between 0 and n/2, we find q = 4n8 oaR.
Ex = "'/4neoy· • 1.5. A system consists of a uniformly charged sphere of radius
In order to find the projectiou E y , it is sufficient to recall that R and a surrounding medium filled by a charge with the volume den-
dEli diffeq from dE~ in that sin a"ln (1) is simply replaced by C08 lx. sity p = aIr, where a is a positive constant and r is the distance from
42 1. Electroirtattc Field in a Vacuum
Problem. 43

the centre of the sphere F' d th h of the spheres and of the distance between their centres. In particular,
electric field intensity E; o~~ide ~hc arghe of. th~ dsphere for which the it is valid when one sphere is completely within the other or, in other
the value of E. e sp ere IS In ependent of r. Find words, when there is a spherical cavity in a sphere (Fig. 1.25).
Solution. Let the sought charge of the sphere be q. Then, Using the
• 1.7. Using the solution of the previous problem, find the field
intensity E inside the sphere over which a charge is distributed with

,I II·
1 lie

I Iii
Iii
I

Fig. 1.24 Fig. 1.25


Fig. 1.22 Fig. 1.23
Gauss theorem we can writ th f II . the surface density CT = CTo cos {Jo, where 0"0 is a constant and {Jo is
surf~ of radi~s r (outside :he :ph~r~~i~t ~h~r~h~ir~~ ~r a spherical the polar angle...'
r Solution. Let us consider two spheres of the same radius, having
E~ ·4n r 2 - q .t- 1 a uniformly distributed volume charges with the densities p and -po
- _ - \. -4nr2dr. Suppose that the centres of the spheres are sep-
80 £0. r
R
After integration, we transform this equation to
arated by the distance 1 (Fig. 1.26). Then, in
accordance with the solution of the previous
problem, the field in the region of intersection
tz
2
E ·4nr = (q ~ 2naR2)leo +
4nar 2/28 0 • • of these spheres will be uniform:
The intensity E does t d d E = (p/380 ) 1. (1)
theses is equal to ze~~. J!rt~~ on r when the expression in the paren-
In our case, the volume charge differs from zero
q =
2naR2 and E = a/280 ' only in the surface layer. For a very small l,
~ f .6. Find the electric fi ld . t · . we shall arrive at the concept of the surface

WI
sectIon of two spheres unifo In enslty .Ii: In the region of inter- charge density on the sphere. The thickness of
volume densities p and _p th c3~rged by unlike charges with the the charged layer at the points determined by
spheres is determined by v~ctor le(F::~af.r:3)~etween the centres of the angle -& (Fig. 1.26) is equal to 1 cos {Jo. Hence,
in this region the charge per unit area is
Solution. Using the Gauss th (J = pl cos {Jo = 0"0 cos a, where °0 = pl, and
electric field intensity Within eo~efm, lwe can easilY.lshow that the expression (1) can be represented in the form Fig. 1.26
a UD! orm Y charged spflere is
E = (p/38 0 ) r, E = -(0"0/380) k,
where r is the radius vector r I t' . where k is the unit vector of the Z-axis from which the angle it ii'J
can consider the field in the r e.a Ive .to the c~ntre of the sphere. We measured.
sUperposition of the fields of et~on of.;n\ersecthlOn of the spheres as the • 1.8. Potential. Tho potential of a certain electric field has the
an arbitrary point A (Fig 1 24)0 \lfntlh~r my.c arged spheres. Then at form Ip = ex (xy - z2). Find the projection of vector E onto thp. di-
" 0 IS regIOn we have
E = E+ + JL = P (r+ - r_V3e n = pI/3e .
rection of the vector a = i + 3k at the point M (2, 1, -3).
n Solution. Let us first find vector E:
Thus, in the region of intersectio f th h
form. This conclusion is valid retpardle o f etse sp ~rebs the field is uni- E = -Vip = -a (yl +xj - 2zk).
" ss 0 he ratio etween the radii
2.1. Field in a Substance 45
1. Electrodatlc Field In a Vacuam where dq is the charge contained between the spheres of radii rand
r + dr. Hence
The sought projection is
r:dE r +2rE.r dr=-pr
1 , d r, aE r +2. E_' r __ ...£...
E a =E . ..!..= -0: (yl-zj- 2lk)(i+3k). -0: (y-61) Eo ar r eo
. a }f1+31 JfiO Substituting (1) into the last equation, we obtain
At the point M we have p =-" -flEoa,

E. -0:~1t18) J~o 0:. i.e. the charge is distributed uniformly within the sphere.
• 10ft. Dipole. Find the force of interaction between two point
• f.9. Find the potential lp at the edge of a thin df!e of l'Jdius R dipoles with moments PI and P2, if the vectors PI anti P2 are directed
with a charge uniformly distributed over olle of its sides with the .... along the straight line connecting the dipoles and the distance be-
face density o. tween the dipoles is l.
. S~lut~on. ~y definition, the potential in the case of a surface charge Solution. According to (1.39), we have
dl8trl~ution IS defined by integral (1.28). In order to simplify in- F = PI I aElal I•
. tegratlOD, we shall choose the area element dS in the fonn of a part
of the ring of radius rand width!,dr (Fig. 1.27). Then dS = 2'&r dr, where E is the field intensity from the dipole P2' determined by the
r = 2R cos '6-, and dr = -2R sin '6- t:H}. After substituting these ex- first of formulas (1.38):
pressions into integral (1.28), we obtain the expression for cp at &be
point 0: E=_1_ 2P2
...' 4nEo za

lp=- oR
neo
Jo ~sin'6'd{;. Taking the derivative of this expression with respect to I and sub-
stituting it into the formull\. for fl, we obtain
n/2 F __ 1 _ 6PIP2
We integrate by parts, denoting ~ =• - 41tEo l4'
and sin t}- dt}- = dv:
It should be noted that the dipoles will be attracted when PI t t P2 and
repulsed when PI H P2'
) 'It sin t}- dt}- = - '6- cos '6-

Fig. 1.27 + \ cos'6-dtJo=-'6-cos{}+sinil'.


01
2. A Conductor in an Electrostatic Field
which gives :-1 after substituting the limits of integration. As a re-
sult, we obtam ~/
lp = oRfneo'
2.1. Field in a Substance
• 1.10. T.he potential of the field inside a charged sphere depends Micro- and Macroscopic Fields. The real electric field in
?nly on the dI.stance r from its. Cl'ntre to the point under consideration any substance (which is called the microscopic field) varies
I~ the foll?wI~g 'Yay: lp = ar' + b, where a and b are constants. abruptly both in space and in time. It is different at differ-
Fmd the dIstrIbution of the volume charge p (r) within the 'sphere. ent points of atoms and in the interstices. In order to find
Solution. Let us first find the field intensity. According to (1.32) the intensity E of a real field at a certain point at a given
we have •
instant, we should sum up the intensities of the fields of all
E r = -oq>/ar = -2ar. (1)
individual charged particles of the substance, viz. electrons
I I T~en we US? th~ Gauss theorem: 4nr'Er = q/eo • The differential of and nuclei. The solution of this problem is obviously not
thiS expressIOn IS
feasible. In any case, the result would be so complicated
1 i
4n d(rIE r) = - dq = - p·4nr' dr,
Ii 66 eJ
2.2. Fields Inside and Outside a Conductor 41
46 2. A Co..a~ctor·tn an Electrodatic Pield
bution of induced charges is mainly determined by the prop-
that i~ would ~e impossibl~ to use it. Moreover, the knowledge erties of the substance, i.e. its physical nature and the
of tillS field IS not reqUIred for the solution of macroscop- shape of the bodies. We shall have to consider these ques-
ic problems. In many cases it is sufficient to have a simpler tions in greater detail.
and rougher description which we shall be using henceforth.
Under the electric field E in a substance (which is called 2.2. Fields Inside and Outside a Conductor
the rruu:roscopic field) we shall understand the microscopic
field averaged over space (in this case time averaging is Inside a Conductor E = O. Let us place a metallic conduc-
superfluous). This averaging is performed over what is called tor into an external electrostatic field or impart a certain
a physically infinitesimal volume, viz. the volume contain- charge to it. In both cases, the electric field will act on all
ing a large number of atoms and having the dimensions the ch2.l.'ges of the conductor, and as a result all the negative
that are many times smaller than the dli!tances over which charges (electrons) will be displaced in the direction against
the macroscopic field noticeably changes. The averaging over the field. This displacement (current) will continue until
such vo~umes smooth~ns all i.rregular and rapidly varying (this practically takes a small fraction of a second) a cer-
fluctuatlOns of the mIcroscopIC field over the distances of tain charge distribution sets in, at which the electric fi.eld
the order of atomic ones, but retains smooth variations of at a~l the points inside the conductor vanishes. Thus, in the
the macroscopic field over macroscopic distances. statIC case the electric field inside a conductor is absent
Thus, the field in the substance is (E = 0).
Fu.rther, siw;e E = 0 everyWhere in the conductor, the
E=Emacro=(Emlcro). (2.f) densIty of excess (uncompensated) charges inside the conduc-
The Inftuence of a Substance on a Field. If any suhstance is tor is· also equal to zero at all points (p = 0). This can be
introduced into an electric field, the positive and negative easily explained with the help of the Gauss theorem. Indeed
charges (nuclei and electrons) are displaced, which in turn since inside the conductor E = 0, the flux of E through an;
leads to a parti.al separation· of these charges. In certain closed surface inside the conductor is also equal to zero.
regions of the suhstance, uncompensated charges of different And this means that there are no excess charges inside the
signs appear. This phenomenon is called the electrostatic conductor.
. / induction, while the charges appearing as a result of sepa- Excess charges appear only on the conductor surface with
ration are called induced charges. a certain density (J which is generally different for different
Induced charges create an additional electric field which points of the surface. It should he noted that the excess
s~face charge is located in a very thin surface layer (whose
in combination with the initial (external) field forms the
resultant field. Knowing the external field and the distribu- thIckness amounts to one or two interatomic distances).
tion of induced charges, we can forget about the presence of The absence of a field inside a conductor indicates in
the substance itself while calculating the reSultant iield accordance with (1.31), that potential cp in the cond~r
since the role of the substance has already been taken int~ has the same value for all its points, i.e. any conductor in
account with the help of induced dlarges an electrostatic field is an equipotential region, its surface
Thus, the resultant field in the presence of a substance is being an equipotential surface.
determined simply as the superposition of the external Bald The fact that the surface of a conductor is equipotential
and the field of .wauced charges. However, in many cases implies that in the immediate vicinity of this surface the
the situation is complicated by the fact that we do not kaow field E at each point is directed along the normal to the sur-
beforehand how all these charges are distributed in space, face. If the opposite were true, the tangential component of
and the problem turns out to be not as simple as it eooJd E would make the charges move over the surface of the
seem at first sight. It will be shown later that the distri-
2. A Condllctor in an Electrostatic Field
-------
conductor, Le. charge equilibrium would be impossible. 2.3. Forces Acting on the Surface of a Conductor 49
Example. Find the potential of an undlarged condudiug sphere
provided that a point charge q is located at a distance r from its centre shall take a small cylinder and arrange it as is shown in
(Fig. 2.1). Fig. 2.3. Then the flux of E through this surface will be
PotentialljJ is the same fur all points equal only to the flux through the "outer" endface of the
of the sphere. Thus we can calculate cylinder- (the fluxes through the lateral surf~ce and the
its value at the centre 0 of the sphere,
r because only for this point it can inner endface are equal to zero). Thus we obtam En /18 =
be calculated in the most simple way:
1 q., (1)
Ip= 4nco' r~t-fjl·
where the tJrst term is the potential of
Fig. 2.1 the charge q, while the second is the n
potential of the charges induced on
the surface of the sphere.Dut since aU induced charges are at the same
distance a from the point 0 and the total induced charge is equal to
zero, q;'=O as well. Thus, in this case the putential \if the sphere will
be determined only by the Erst term ill (1).

l"igure 2.2 shows the field and the charge distributions for
a system consisting of two conducting spheres one of which 4'
(left) is charged. As a result of electric induction, the ch?rges Fig. 2.2 Fig. 2.3
of the opposite sign appear on the surface of the right = 0/18/80' where En' is ..the projection of (vector E onto
uncharged sphere. The field _of these.eharges will in turn the outward normal n (with respect to the conductor), /18
cause a redistribution of charges on the surface of the left is"9the cross-sectional area of the cylinder and 0 is the local
sphere, and their surface distribution will become nonuni- s~face charge density of the conductor. Cancelling both
form. The solid lines in the figure are the lines of E, while sides of this expression by /18. we get
the dashed lines show the intersection of tlquipotential sur-
fa~s with the plane of the figure. As we move away from
I En = a/so· I (2.2)
this...system, the equipotential. surfaces become closer_and If 0 > 0, then En > 0, Le. vector E is directed from the
closer to spherical, and the field lines become closer to ra- conductor surface (coincides in direction with the normal n).
dial. The field itself in this case resembles more and more the If 0 < 0, then En < 0, and vector E is directed towards the
field of a point charge q, viz. the total charge of the given conductor surface.
system. Relation (2.2) may lead to the erroneous conclusion that
The Field Near aConductor Surface. We shall show that the field E in the vicinity of a conductor depends only on
the electric fwld intensity in the immediate vicinity of the the local charge density 0'. This is not so. The intensity E
surface of a conductor is connected with the local charge is determined by all the charges of the system under con-
density at the conductor surface through a simple relation. sideration as well as the value of a itself.
This relation can be established with the help of the Gauss
theorem . 2.3. Forces Acting on the Surface of a Conductor
. Suppose that the region of the conductor surface we are Let us consider the case when a charged region of the sur-
interested in borders on a vacuum. The field lines are nor- face of a conductor borders on a vacuum. The force acting
mal to the conductor surface. Hence for a closed surface we on a small area· ~S of the conductor surface is
~F = at.S .E o, (2.3)
\

50 2. A Conductor in an Electrostatic Field 2.4,., Properties of a Closed Conducting Shell 5i


2
where 0 f!S ;is the charge of this element and Eo is the field where we took into account that 0 = eoEn \ and E; = E •
created by all the other charges of the system "in the region The quantity F u is called the surface density of force. Equa-
where the charge 0 f!S is located. It should b~ noted at the tion (2.7) shows that regardless of the sign of 0, and hence
very outset that Eo is not equal to the field intensity E in of the direction of E, the force Fu is always directed outside
the vicinity of the given surface element of the cojductor, the conductor, tending to stretch it:
although there exists ~a certain relation between th~m. Let Example. Find the expression for the electric force acting in a
us find this relation, Le. express Eo through Be' vacuum on a conductor as a whole, assuming that the field intensities
Let E a be the intensity of the field created by the charge E are known at all points in the vicinity of the conductor surface.
on. the area element f!S at the points that are very close to Multiplying (2.7) by dS, we obtain the expression for the force dF
acting on the surface element dS:
thIS element. In this region, it behaves as an infinite uni-
formly charged plane. Then, in accordance with (1.10), E a = dF=+ eo£2 dS,
= 0/ 28 0'
The resultant field both inside and outside the conductor where dS = n dS. The resultant force acting on the entire conductor
(near the area element f!S) is the superposition of the fields can be found by integrating this equation over the entire conductor
Eo and Eo. On both sides of the area element f!S the field E lIUliace:
is p~actically ~he same, while the field Eo has opposite di~
ractIOns (see FIg. 2.4 where (for the sake of definiteness it is
assumed that C1 > 0). From 'the condition E = 0 inside the
conductor, it follows that Eo = Eo, and then outside the 2.4. Properties of a Closed Conduding Shell
conductor, near its surface, E = Eo +
Eo = 2Eo. Thus,
It was sho.wn that in equilibrium there.are no excess charges
Eo = E/2, (2.4) inside a conductor, viz. the material inside the conduc-
and Eq. (2.3) becomes tor is electrically neutral. Consequently ~ if the substance is
removed from a certain volume inside a conductor (a closed
1
AF=T aAS·E. (2.5) cavity is created), this does not change the field anywhere,
AS Le. does not affect the equilibrium distribution of charges.
Dividing both sides of this This means that the excess charge is distributed on a
equation by f!S, we obtain the conductor with a cavity in the same way as on a uniform
expression for the force acting conductor, viz. on its outer surface.
on unit surface of a conductor: Thus, in the absence of electric charges within the cavity
1 the electric field is equal to zero in it. External charges,
Fu-T.aE. (2.6) including the charges on the outer surface of the conductor, do
not create any electric field in the cavity inside the conductor.
Fig. 2.4 :- We can write this expression This forms the basis of electrostatic shielding, Le. the screening
in a different form since the of bodies, e.g. measuring instruments, from the influence
quantities a and E appearing in it are interconnected. Indeed, of external electrostatic fields. In practice, a solid conduct-
in accordance with (2.2), En = a/eo, or E = (a/eo) n, where ing shell can be replaced by a sufficiently dense metallic
n is the outward normal to the surface element at a given grating.
point of the conductor. Hence That there is no electric field inside an empty cavity can
be proved in a different way. Let us take a closed surface S
(2.7) enveloping the cavity and lying completely in the material
4*
52 2. A Conductor in an Electrostatic Field 2.4. Properties Df a Closed Conducting Shell 53

of the conductor. Since the field E is equal to zero inside the space outside the cavity, the field created by the charges
conductor, the flux of E through the turface S is also equal located inside the cavity.
to zero. Hence, in accordance with the Gauss theorem, the Since the conducting medium is electrically neutral every-
total charge inside S is equal to zero as well. This does not where it does not influence the electric field in any way.
exclude the situation depicted in Fig 2.5, when the surf/tge Therefore, if we remove the medium, leavi.ng only a conduc-
ting shell around the cavity, the field WIll not be ~hanged
anywhere and will remain equal to zero beyond thIS shell.
Thus the field of the charges surrounded by a conducting
shell a~d of the charges induced on the surface of the cavity
(on the inner surface of the shell) is equal to zero in the en-
p
tire outer space.
r We arrive at the following important conclusion: a closed
conducting shell divides the entire space into the inner .and
outer parts which are completely independent oj one another
in respect of electric fields. This must be i~te:preted as follows:
any arbitrary displacement of charges InSIde the shell does
Fig. 2.5 Fig. 2.6 not introduce any change in the field of the outer space,
and hence the ch.arge distribution on the outer surface of the
of the cavity itself contains equal quantities of positive shell remains unchanged. The same refers to the field inside
and negative charges. However, this assumption is prohibit- the cavity (if it contains cbarges) and to the ~istribution ?f
ed by another theorem, viz. the theorem on circulation· of charges induced on the cavity walls. They WIll also remam
vector E. Indeed, let the contour r cross the cavity along unchanged upon the displacement of charges outside the
one of the lines of E and be closed in the conductor material. shell. Naturally, the above arguments are applicable only
It is clear that the line integral of vector E along this contour in the framework of electrostatics.
differs from zero, which is in contradiction with the theorem
on circulation. Example. A point charge q is within an electrically neutral sh~ll
whose outer surface has spherical shape (Fig. 2.6). Find the potential
Let us now consider the case when the cavity is not empty cp at the point P lying outside the shell at a distance r from the
but contains a certain electric charge g (or several charges). centre 0 of the outer surface.
Suppose also that the entire external space is filled by a The field at the point P is determined only by charges induced on
conducting medium. In equilibrium, the field in this medium . the outer spherical surface since, a~ was shown ab«;lve, the field of the
is equal to zero, which means that the medium is electrical- point charge q and of the charges mdu.ced on the. mner surf~ee ?f the
ly neutral and contains no excess charges. sphere is equal to zero everywhere outsIde the caVIty. Next, III VIew of
symmetry, the charge on the outer surface of the shell is distributed
Since E = 0 inside the conductor, the field flux through a uniformly, and hence
closed surface surrounding the cavity is also equal to zero. 1 q
According to the Gauss theorem, this means that the alge- qJ = 4n: eo -,:-.
braic sum of the charges within this closed surface is equal to
zero as well. Thus, the algebraic sum of the charges induced An infinite conducting plane is a special case of a closed
on the cavity surface is equal in magnitude and opposite in conducting shell. The -space on one side of this plane is
sign to the algebraic sum of the charges inside the cavity. .electrically independent of the space on its other side.
In equilibrium the charges induced on the surface of the We shall repeatedly use this property of a closed con-
cavity are\arranged so as to compensate compleLely, in the ducting shell.
2.5. General Problem of Electrostatilcs 55
2. A Conductor in an El~ctrostatic Field

2.5. General Problem of Electrostatics. static case the charge is distributed over thlle surface of a
Image Method conductor in a unique way as well. Indeed, th'Iere is a one-to-
one correspondence (2.2) between the charges; on the conduc-
d . ~r~~ue~tlY~ we must solve problems in which the charge tor and the electric field in the vicinity of itts surface: (J =
t~:.rl ~tJon IS . unknown but the potentials of conductors
Jr s ape an d relative arrangement are given We must
find th~ pot~ntial cp (r) at any point of the field between the
~~a~d uc ors. t should be recalled that if we know the poten-
d th (r),. the field E (r) itself can be easily reconstructed
an f en Its value in the immediate vicinity of the condu~tor
s'!r a~es .can be used for determining the surface ch
dlstrlbutlOll for the conductors. arge
(a) (b) (e) l
The Poisson and I aplac t· L .
equation for the functi e equa .lOns. et ~s derIve the differential
into the left-hand sid~~( /r~t)n~hal). For t~IS p¥rpose: we substitute Fig. 2.7
i.e. E = -v<p. As a result . b .e expressIon o~ E In .terms of 'P,
for potential which is cali ':,(}tOh tapIn.the general dIfferentIal equation = EllEn. Hence it immediately follows that the uniqueness
, ell e Olsson equation:

. I V2<p= -P/eo,/ (2.8)


of the field E determines the uniqueness of the charge
distribution over the conductor surface.
The solution of Eqs. (2.8) aM (2.9) in the generail case is a compli-
7thh:~ r~eifo~ Lapl~ce operator (Laplacian). In Cartesian coordinates cated and laborious ·problem. The analytic solutions i of these equations
were obtained only for a few particular cases. As ffor the uniqueness
theorem, it simplifies the solution of a number of «electrostatic prob-
V2=~
+~
2
a
2 ax ay + az' 2 ' lems. If a solution of the problem satisfies the Lalplace (or Poisson)
equation and the boundary conditions, we can stat6e that it is correct
i.e.1:\the scalar product v·v [see (1.19)). and unique regardless of the methods by which itt was obtained (if
is tran~f~:ma:d ~~tcharg~s beltween th!! cond.uctors (p = 0), Eq. (2.8) only by guess).
Example. Prove that in an empty cavity of a cconductor the field
o a SImp er equatIOn, VIZ. the Laplace equation:

I V'<P = O. I (2.9)
is absent.
The potential <P must satisfy the Laplace equatiwn (2.9) inside the
cavity and acquire a certain value <Po at the'cavity's ,walls. The solution
of the Laplace equation satisfying this condition c~an immediately be
founrl. It is <p = <Po' In accordance with the uniquemess theorem, there
Eq.S:(2~h8)t~~(4~~ f~\h~t~~ii;;:s~a~~t ~:t~:e~u~h~i~~n~~~ :~~sfies can be no other solutions. Hence, E = -V<P = 10.
qUIres t e given values <Ph 'P2' ... on the surfaces of the conducto~~ Ima~e Method. This is an artificial metho,>d that makes it
It can ~e prove? theoretically that this problem has a uni- possible to calculate in a simple way the ,electric field in
que solutIOn. ThiS statement is called the uniqueness theo- some (unfortunately few) cases. Let us consiider this method
rerr;. Fro~ the physical point of view, this conclusion is by using a simple example of a point charge ~q near an infinite
q'!Ite obvIOUS: If there are more than one solution there conducting plane (Fig. 2.7a).
wIll be .several potential "reliefs", and hence the field E at , The idea of this method lies in that we mmst find another
each pom~ generally has not a single value. Thus we arrive problem which can be easily solved and whwse solution or a
at a phYSically absurd conclusion. part of it can be used in our problem. In ourr case such a sim-
Usm~ the uniqueness theorem, we can state that in a ple problem is the problem about two changes: q and -g.
----

56 2. A Conductor in an Electrostatic Field 2.6. C.pacUance. Capacitors 57

The field of this system is well known (its equipotential charges whose action on the charge q,is eq~ivalent to the action of all
surfaces and field lines are shown in Fig. 2.7b).
Let us make the conducting plane coincide with the mid-
cha~~\\~~deu~~dii~3:~~~etc~;l~l~~r::'chargesfor which the eqnipoten-
tial ~ul'face~ with cp = 0 wo~J!d (a) (~)
dle equipotential surface (its potential qJ = 0) and remove coinCide With the cond~c!Illg
the charge --g. According to the uniqueness theorem, the half-planes. One or two fI~tltIO~StL'q f
field in the upper half-space will remain unchanged. Indeed, charges are insuflicient III tillS I
qJ = 0 on the conducting plane and everywhere at infinity.
case' there should be three of -q I q
the~ (Fig. 2.8b). Only with such a
The point charge g can be considered to be the limiting case configuration of the ~:ystem of fOl~r I
of a small spherical conductor whose radius tends to zero charges we can realize the ft- .--l---
and potential to infinity. Thus, the boundary conditions for quired "trimming"', i.e. ensure 7, j
the potential in the upper half-space remain the same, and that the potential on the con- I
dueling" half-plane~ ~e equal to
hence the field in this region is also the same (Fig. 2.7c). zero. These three hctl tlOllS cl.Jar- q b---lf---<>- q
It should he noted that we can arrive at this conclusion ges create just the same held
proceeding from the properties of a closed conducting shell within the "right angle" as the Fig. 2.8
field of the charges induced on
[see Sec.2.1J. since hoth half-spaces separated by the con-
ducting plane are electrically independent of one another, the con~ucting plah.es. n alion of point chal'gl's {anolher proble~n),
HavIll\r.found I IS con 19l1b f tlH'r questions, for example, lmd
ili'e ~~~e~t~~{a~ldsfi~f/i~~~~~i~; ~n ~ny point within the "right angle"
and the removal of the charge -g will not affect the field
in the upper half-space.
or determine the fw-ce acting Oil the charge q.
Thus, in the case under consideration the field differs from
zero only in the upper half-space. In order to calculate this
field, it is sufficient to introduce a fictitious image charge 2.6. CapaCitance. Capacitors
g' = -g, opposite in sign to the charge q, by placing it on
the other side of the conducting plane at the same distance Capacitance of an Isolated Conductor. Let I~S fconSid~r a
as the distance from g to the plane. The fictitious charge g' solitarv conductor, i.e. the conductor .remove( r~m 0 101'
creates in the upper half-space the same field as that of tlie
.
conductors, 1)0 d'Ies". and charge"
, , , . Expenments
. show
. that
1 the
't
cl·r.r e of this conductor is directly propor~lOJ.la to I S
charges induced on the plane. This is precisely what is
meant when we say that the fictitious charge produces the same p~~e~tia\ <p (we assumed that at infin.ity potent~,aI IS eJual t~
"effect" as all the induced charges. We must only bear in zero): cp oc q. Consequently, the r~tlO g!cp ~oe.., n~t ~~~n y
on the charge q and has a certain value or eac so I r
mind that the "effect" of the fictitious charge extends only to
the half-space where the real charge q is located,. In another conductor. The quantity
half-space the field is absent. c ~'" g!cp (2.10)
Summing up, we can say that the image method is essen-
tially based on driving the potential to the boundary con- is called the electrostatic capacitance of an isolated conductor
ditions, I.e. we strive to find another problem (eonfignra- (Qr simply capacitance). It is numerically.equal to th~ charge
tion of charges) in which the field configuration in the region that must be supplied to the conductor III order to lIlcre~se
of space we are interested in is the same. The image method its potential by unity. The capacitance depends on the SIze
proves to be very effective if this·can be done with the help and shape of the conductor.
of sufficiently simple configurations. Let liS consider one Example. Find the capacit:mce of an isolated conductor which has
more example. the shape of a sphere of radIUS R.
Example. A point charge q is placed between two mutually perpen- It can he seen from formula (2.10) that for this jlurpose. we must
dicular half-planes (Fig. 2.8a). Find the location of fictitious point mentally chargre the conductor by a charge q and calculate Its potell-
58 2. A Conductor in an Electro8tattc Field 59
2.6. Capacitance. Capacitors
tial <po In accordance with (1.23), the potential of a sphere is
. the charges on the plates must be equal in magnitude
I.e. ( d )
<p=
J'
00

1
Erdr=-- q f
00

1 q
--2 dr = - - - .
and opposite in sign ~ :.n
The basic charactens lC 0 .
f!
~apacitor is its capacitance.
d onductor the capaci-
4nfo . r 4n8 o R I

R R Unlike the capacitance of an Isolate c t' f i'ts charge to


'tor is denned as t lle ra 10 0 •
Substituting this result into (2.10), we find tance 0 f a. cap~cl b t n the plates (lhis difference 18
the potentIal difference e wee
C = 4m oR. (2.U) called the voltage):
The unit 'ofcapacitance is the capacitance of a conductor (2.12)
whose potential changes by 1 V when a charge of 1 C is C=qlU.
\ \
supplied to it. This unit of capacitance is called the fa-
rad (F). The charge g ofa capacitor is the charge of its positively
The farad is a very large quantity. It corresponds to the
capacitance of an isolated sphere 9 X 106 km in radius, charged' plate. ., f a capacitor is also measured
Naturally, the capaCitance 0 ,
which is 1500 times the radius of the Earth (the capacitance
in farads. " t depends on its O'eometry
of the Earth is 0.7 mF). In actual practice, we encounter The capacitance of a capaCl or a between th~ plates,
capacitances between 1 ~F and 1 pF.
(size and shape. of i~S pl~\~s)th~leC;p~citor. Let us derive
Capacitors. If a conductor is not isolated, its capacitance
will considerably increase as other bodies approach it. This and the m~teqalf t ~~e capacitances of some capacitors
is due to the fact Jhat the field of the given conductor causes
a redistribution of charges on the surrounding bodies, Le.
the expreSSIOns or.
assuming that there is a rr
cuum between their plates.
I I te Capacitor. This capacitor
Capacitance of a Para e -p a d b a ap of width h.
induces charges on them. Let the charge of the conductor consists of two parallel pl~ttes ~epar~~:n :cco~ding to (1.11),
be g > O. Then negative induced charges will be nearer to If the charge of the capaCi or IS g, . '1 . E _ ale
the conductor than the positive charges. For this reason, the . h fi Id between Its pates IS - 0'
the intensity of t e .e h f each plate. Consequent-
potential of the conductor, which is the algebraic sum of the where a = glS and S IS t e area 0 .
potentials of its own charge and of the charges induced on ly, the voltage between the plates IS
other bodies will decrease when other uncharged bodies U = Eh = ghlfoS.
approach it. This means that its capacitance increases.
This circumstance made it possible to create the system Substituting this expression into (2.12), we obtain
of conductors, which has a considerably higher capacitance C = foS Ih . (2.13)
than that of an isolated conductor. Moreover, the capacitance
of this system does not depend on surrounding bodies. d 'thout taking into account
This calculation was rna e WI lates (edge effects).
field disto~tions n~ar thel e~~~e °1a~~~lor is determined ~Y
Such a system is called a capacitor. The simplest capacitor
consists' of two eonductors (plates) separated by a small
distance. The capaCItance 0 a rea p tely the smaller the gap h III
this formula the more. accura. nsions of the plates.
In order to exclude the effect of external bodies on the
capacitance of a eapacitor, its plates are arranged with re- compari~on with th; l~ne~r t~~acitor. Let the radii of thE
CapaCItance of a p. enca es be a and b respectively. II
speet to one another in such a way that the field created by inner and outer capaclt?r pl.at field intensity between thE
the charges accumulated on them is concentrated almost the charge of the capaCItor. IS g, .
completely inside the capaeHor. This means that the lines plates is determined by the Gauss theorem.
of E emerging on one plate must terminate on the other, 1 q
Er = into rt·
\'

60 2. A Conductor in an Electrostatic Field


Problems
The voltage of the capacitor is
charges on the outer surface. It follows from this expression that
u = J"r E r dr = _q_
4n80
( ..!.. __
a
1 ). _ _q_ (..i.. __ +..!..).
1
b cP - 4n80 r a b
a

It can be easily seen that the capacitance of a spherical


It should be noted that the potential.can. be found in such ~f~e
form only at the poin~ 0 s~nce all the. like mduced charges are a e
a
capacitor is given by same distance from this pomt and theIr
distribution (which is unknown to us)
C=4Jt£o ~
b-a . (2.14) does .not play any role.
• 2.2. A system consists of two
It is interesting that when the gap between the plates is concentric spheres, the inner sphere of
small, Le. when b - a «a (or b), this expression is re- radius R l having a charge ql' What cha-
duced to (2.13), viz. the expression for the capacitance of a rge q must be placed onto the outer
parallel-plate capacitor. spher~ of radius R 2 to make the poten-
tial of the inner sphere equal to zero?
Capacitance of a Cylindrical Capacitor. By using the same What will be the dependence of poten-
line of reasoning as in the case of a spherical capacitor, we tial cP on the distance r from the cen-
obtain tre of the system? Plot schematica~ly
the graph of this dependence, assummg
that ql < o. 4.' 'Fig. 2.9
c-·- In2n8 ol
(b/a) ,
(2.15)
Solution. We write the expressions. . h
for potentials outside the slstem (CPn) and In the regIOn between t e
where l is the capacitor's length, a and b are the radii of the spheres (CPI):
inner and outer cylindrical plates. Like in the previous 1 ql +q2
CPn=-- - - - ,
1 ql
CPI=-4-- - - CPo,
+
case, the obtained expression is reduced to (2.13) when the 4n80 r n80 r
gap between the plates is small.
where CPo is a c~r~ain constant. Its val~ can be easily found from the
The influence of the medium on the capacitance of a ca- boundary conditIOn: for r = R 2 , CPn - CPl, Hence
pacitor will be discussed in Sec. 3.7.
lfu = q2/4ncoR2'
From the condition <PI (R J ) = 0 we find that q2 = -ql R 2 IR l •
Problems The <P (r) dependence (Fig. 2.10) will have the form:
• 2.1. On the determination or potential. A point charge q is at
a distance r from the centre 0 of an uncharged spherical conducting
_ ql
<P II - 4n80
1-R 2/R J
r ,CPr
=.3..L
4n80
(_1r ~).
R1
layer, whose inner and outer radii are equal to a and b respectively.
Find the potential at the point 0 if r < a. • 2.3. The force acting on a surface charge. An. uncharged me-
tallic sphere of radius R is placed into an external umform. field'Js a
Solution. As a result of electrostatic induction, say, negative charges result of which an induced charge appears on the sphere Wit~ su ice
will be induced on the inner surface of the layer and positive char-
ges on its out{~r surface (Fig. 2.9). According to the principle of super-
°
density = 00 cos 'ft, where 00 is a positive consta.nt an~ ~ IS ~ po ar
angle. Find the magnitude of the resultant electnc force actmg on
position, the sought potential at the point 0 can be represented in like charges.
the form

(q- - -t- \.~' - -dST, '1) - -dS- )


Solution. According to (2.5), the force acting on the area element
q-J - -1-- 0_ 0+ dS is
4rtE o r • a :. b '
dF=-taEdS. (1)
where the first integ-ral is taken over all the charges induced on the . '"
inner surface of the layer, while the second integral, over all the It follows from symmetry considerations that t~e resultant force f
is directed along the Z-axis (Fig. 2.11), and henc0 it can be represented
2. A Conductor in an Electrostatic field Problem.
as the sum (integral) of the projections of elementary forces (1) onto where the minus sign indicates that the induced charge is opposite
the Z-axis: to sign to the point charge q.
dF z = dF cos {}. (2)
• 2.5. A point charge q is at a distance 1 from an infinite conduc-
It is expedient to take for the area element dS a spherical zone dS = ting plane. Find the work of the electric force acting on the charge q
done upon its slow removal to a very large distance from the plane.
f

1
o
-q F q

Eq x dx x
I

o , Fig. 2.12 Fig. 2.13

Fig. 2.10 Fig. 2.11 Solution. By definition, the work of this force done upon an ele-
mentary displacement dx (F;.ig. 2.13) is given by
= 2nR sin t}·R dt}. Considering that E = a/eo, we transform (2) as ql
follows: 6A = F % dx = - 4neo (2%)1 dx,
clFz = (na I R2/ eo ) sin t} cos {} d{} = - (na~R2/po) cos3 {} d (cos t}).
where the expression for the force is obtained with the help of the
Integrating this expression over the half-sphere (i.e. with respect to image method. Integrating this equation over x between land 00, we
cos if between 1 and 0), we obtain find
F = na8R2/4eo' ql r
00
dx ql
• 2.4. Image method. A point charge q is at a distance 1 from an A= - 16neo J 7= - 16ne ol •
infinite conducting plane. Find the density of surface charges induced I
on the plane as a function of the distance r from the base of the per- Remark. An attempt to solve this problem in a different way (through
pendicular dropped from the charge q onto the plane. potential) leads to an erroneous result ~h!ch differs from w~at
was obtained by us by a factor of two. ThIS IS because the re.latIOn
~olutton. Accord.ing to (2.2), the :surface charge:density on a conduc- A = q (IJ>l - <PI) is valid only for potential fields. However, In the
tor IS connected .wIth the electric field near its surface (in vacuum) reference system fixed to the conducting plane, the electric field of
through the relatIOn a = eoEn' Consequently, the problem is reduced induced charges is not a potential field: a displacem.ent o.f the charge
to determining the field E in the vicinity of the conducting plane. q leads to a redistribution of the induced charges, and theIr field turns
Using the image method, we find that the field at the point P out to be time-dependent.
(Fig. 2.12) which is at a distance r from the point 0 is • 2.6. A thin conducting ring of radius R, having a charge q, is
q 1 arranged 80 that it is parallel to an infinite conducting plane at a ~is­
E=2Eqcosa=2 - - - - - tance I, from it. Find (1) the surface charge density at a point of the
4neox2 x '
plane, which is symmetric with respect to the ring and (2) the electric
Hence field potential at the centre of the ring. .
Solutton. It can be easily seen that in accordance with the image
method, a fictitious charge -q must be located on a similar ring but
Problems 65
2. A Conductor in an Electrostatic Field

on the other side of the conducting plane (Fig. 2.14). Indeed, only in the required force (see Fig. 2.15b)
this case the potential of the midplane between these rings is equal to 2 V 2- 1 q2
zero, Le. it coincides with the potential of the conducting plane. Let F =c F 2 - F 1= ---'74-
neo
- 2a 2 ,
us now use the formulas we already know.
(1) In order to find {J at the point 0, we must, according to (2.2), and answer the second question.
find the field E at this point (Fig. 2.14). The expression for E on the • 2.8. Capacitance of parallel wir~s. :rwo long straight wires
with the same cross section are arranged In aIr parallel to one another.
R

1 2

1I
I
A B
-qC---l;---~
........ _---- (8) (b)

Fig. 2.14 Fig. 2.15


Fig. 2.16 Fig. 2.17

axis of a ring was obtained in Example 1 (see p. 14). In our case, this The distance between the wires is 11 times larger th~n the radi~s of the
expression must he doubled. As a result, we obtain wires' cross section. Find the capacitance of the WIres per umt length
ql provided that 11:;;}> 1.
{J= 21t (RI+12)'/1 Solution. Let us mentally charge the two wires by charg~s of the
(2) The potential at the centre of the ring is equal to the algebraic same magnitude and opposite signs so that t~e charge per um.t length
sum of the potentials at this point created by the charges q and -q: is equal to "'. Then, by defmition, the reqUIred capaCitance is
=
1 (q
if
q) Cu ",IU, (1)
ljl= 41t8o yR2+411 • and it remains for us to find the potential difference between the wi~es.
It follows from Fig. 2.16 showing the dependences of the potentl~ls
• 2.7. Three unlike point charges are arranged as shown in lp + and Ip_ on the distance between the plates that the sought potential
Fig. 2.15a, where AOB is the right angle formed by two conducting difference is
half-planes. The magnitude of each of the charges is I q I· and the dis-
tances between them are shown in the figure. Find (1) the total charge u= ILlcp+1 + ILlCP-l =2jLl(jl+I· (2)
induced on the conducting half-planes and (2) the force acting on the
charge -q. The intensity of the electric field created by one of the wires at a dis-
tance x from its axis can be easily found wi th ,the help of the Gauss
Solution. The half-planeD forming the angle AGE go to infinity, theorem: E = "'/2ne ox. Then
and hence their potential q; = O. It can be easily seen that a ·~Ylltem b-a
haVing equipotential surfaces with ljl = 0 coinciding with the conduc-
ting half-planes has the form sho1inl in Fig. 2.15b. Hence the action I 8(jl+ I = J\ '" b-a
E dx = 2neo In --a- , (3 )
of the charges induced on the conducting hali-planes Is equivalent to a
the action of the fictitious charge -q placed in the lower left comer of
the dashed sauare. where a is the radius of the wires' cross section and b is the separation
Thus we have already answered the first question: -qe between the axes of the wires.
By reducing the syst0m to iour point ch:trges, we can easily find
5-0181
., 3.1. Polarization 0/ Dielectrics 67
!i
'I' tili
-------
A Conductur
2. in an Electroltatlc Field
'I' difference between them is equal to zero. Consequently,
It follows from (1), (2) and (3) .nat
1,1., . E 1X l 1 + E 2x (l - 11) = 0,
'I' i Cu = neo/ln 1').
I ! where E 1x and E x are the projections of ve.ctor E onto the X-axis to
!
where we took into account that b :it> a. the left and to tte right of the plane P (Fig. 2.19b).
On the other hand, it is clear that
• 2.9. Four identical metallic plates are arranged in air at the
same distance h from each other. The outer plates are connected by a a = -(a1 + a.),
conductor. The area of each plate is equal to S. Find the capacitance where, in accordance with (2.2), a 1 = eOE1n1 = eOE 1X and a2 =
of this system (between points 1 and 2, Fig. 2.17).
= e E 2 = -eOE 2X (the minus sign indicates that the normal U2
Solution. Let us charge the plates 1 and 2 by charges qo and -qo. o n,
is directed .
oppositely to the umt vector 0 f t.he X,-axIs.
.) .
Under the action of the dissipation field appearing between these Eliminating E!X and E 2x from these equatIOns, we obtam
plates (edge effect), a charge will move in the connecting wire, after
which the plate A will be charged negatively while the plate B will a1 = -a (I - 11)/1, a2 = -al 1/l·
acquire a positive charge. An electric field appears in the gaps be- The formulas for charges q1 and q2 in terms ~f q have a si~ilar form.
tween the plates, accompanied by the corresponding distribution of It would be difficult, however, to solve thiS problem WIth the help
potential cp (Fig. 2.18). It should be noted that as follows from the
symmetry of the system, the potentials at the Imiddle of the system as p
well as on its outer plates are equal to zero. -
By defini tion, the capacitance of the system in this case is -
r------
C = qo/V . (1) r--
r--
where U is the required potential difference between the points 1 B
-=- x
and 2. Figure 2.18 shows that the putential difference V between the A ~
01" _02
inner plates is twice as large as the potential difference between the -
outside pair of plates (both on the right and on the left). This also r--
refers to the field intensity: 1 r- a ;r
E = 2E'. (2)
(a) (b)
And since E ex a, we can state that according to (2) the charge qo on
plate 1 is divided into two parts: qo/3 on the left side of the plate 1
and 2qo/3 on its right side. Hence Fig. 2.1st= Fig. 2.19
V = Eh = ahlE o = 2qoh/3e oS,
of the image method, since it would require an infmite series of fIcti-
and the capacitance of the system (between points 1 and 2) is tious charges arranged on both sides of the charge q, and to find the
field of such a system is a complicated problem.
C= 3eoS
2h •

• 2.10. Distribution 01 an induced charge. A point charge q is


placed between two large parallel conducting plates 1 and 2 sepa- 3. Electric Field in Dielectrics
rated by a distance 1. Find the tutal charges q1 and q2 induced on each
plate, if the plates are connected by a wire and the charge q is located 3.1. Polarization of Dielectrics
at a distance 11 from the left plate 1 (Fig. 2.19a).
Dielectrics. Dielectrics (or insulators) are substances that
Solution. Let us use the superposition principle. We mentally practically do not conduct electric current.. This ~neans that
place somewhere on a plane P the same charge q. Clearly, this will in contrast, for example, to conductors dielectriCS do not
double the surface charge on ~ach plate. If we now distribute uniformly
on the surface P a certain cnarge with surface density a, the electric contain charcrcs
t>
that can move over considerable distances
field can be easily calculated (Fig. 2.19b). and create elcctric current.
The plates are connected by the wire, and hence the potential
~
'
,"~i)
3.1. Polarization of Dielectrics 69
68 3. Electric Field in Dielectrics
ably lower than the intensities of internal electric fields in
When even a neutral dielectric is introduced into an exter- the molecules.
nal electric field, appreciable changes are observed in the Bulk and Surface Bound Charges. As a result of polariza-
field and in the dielectric itself. This is because the dielec- tion, uncompensated charges appear on the dielectric surf!"ce
~ric is acted up?n. by a force, the capacitance of a capacitor as well as in its bulk. To understand better the mechamsm
Increases when It IS filled by a dielectric, and so on. £=0
In order to understand the nature of these phenomena we .!.-
must take into consideration that dielectrics consist either p~
p~ ~I p~ It
of neutral molecules or of charged ions located at the sites
. of a crystal lattice (ionic crystals, for example, of the NaCI
7~ I , I p,+
I I I
type). The molecules can be either polar or nonpolar. In a I I I:
I I I.
polar molecule, the centre of "mass" of the negative charge I I I.
is displaced relative to the centre of "mass" of the positive
charge. As a result, the molecule acquires an intrinsic dipole X X X
moment p. Nonpolar molecules do not have intrinsic dipole
mo~ents, since the "centres of mass" of the positive and ne- (a) (b) (c)
gatIve charges in them coincide. Fig. 3.1
Polarization. Under the action of an external electric A.~

field, dielectric is polarized. This phenomenon consists in


the following. If a dielectric is made up by nonpolar molecu- of emergence of these charges (and especially bulk charges),
les, the positive cll~rge. in each molecule is shifted along the let us consider the following model: Suppose that we have a
field and the negatIve, In the opposite direction. If a dielec- plate made of a neutral inhomogeneous dielectric (Fig. 3.~a)
tric consists of polar molecules, then in the absence of the whose density increases with the coordinate x accordIng
field their d~pole moments are oriented at random (due to to a certain law. We denote by p~ and p~ the magnitudes of
thermal motIOn). L nder the action of an external field the the volume densities of the positive and negative charges in
dipole moments acquire predominant orientation in' the the material (these charges are associated with:nuclei and
direction of the external field. Finally, in dielectric crystals electrons).
In the absence of an external field, p~ = p~ at each point
?f the NaCI ty~e, an external field displaces all the positive
IOns along the field and the negative ions, against the field. * of the dielectric, since the dielectric is electrically neutral.
Thus, the m~chanism of polarization depends on the struc- However, p~ as well as p~ increase with x due to inhomog~­
ture of a dielectric. For further discussion it is only impor- neity of the dielectric (Fig. 3.1b). This figure shows that III
, the absence of external field, these two distributions ex-
tant that regardless of the polarization mechanism all the
actly coincide (the distribution of p~(x) is shown by the
positive ~harges duri~g this process are displaced aiong the
field, whIle the negatIve charges, against the field. It should solid line, while that of p~ (x), by the dashed line).
Switching on of the external field leads to a displacement
be noted that under normal conditions the displacements of
of the positive charges along the field and of the negative
charges are very small even in comparison with the dimen-
charges against the field, and the two distributions will be
sions of the molecules. This is due to the fact that the inten-
shifted relative to one another (Fig. 3.1c). As a result, un-
sity of the external field acting on the dielectric is consider- compensated charges will appear on the dielectric surface as
well as in the bulk (in Fig. 3.1 an uncompensated negative
* There exist ionic crystals polarized even in the absence of an charge appears in the bulk). It should be noted that the re-
external field. This property is inherent in dielectrics which are
called electrets (they resemble permanent magnets).
3.2. Polarization 7f
70 3. Electric Field in Dielectric,
dipole moments of the molecules in this volume and write
version of the field direction changes the sign of all these the ratio
charges. It can be easily seen that in the case of a plate made (3.2)
of a homogeneous dielectric, the distributions p' (x) and
p~ (x) would be IT-shaped, and only uncompensated surface Vector P defined in this way is called the polarization of a
charges would appear upon their relative displacement in
dielectric. This vector is numerically equal to the dipole mo-
the field E.
Uncompensated charges appearing as a result of polariza- ment of a unit volume of the substance.
There are two more useful representations of vector P.
tion of a dielectric are called polarization, or bound, charges. Let a volume !'1 V contain !'1N dipoles. We multiply and di-
The latter term emphasizes that the displacements of these vide the right-hand side of (3.2) by !'1N. Then we can write
charges are limited. They can move only within electrically
n~utr~l m?lecules. We shall denote bound charges by a prime P = n (p), (3.3)
(q , p , (J).
Thus, in the general case the polarization of a dielectric where n = !'1NI!'1V is the concentration of molecules (their
!ea~s to the appearance of surface and bulk bound charges number in a unit volume) and (p) = ('T.PI)!!'1N is the mean
III It. dipole moment of a molecule.
We shall call the charges that do not constitute dielectric Another expression for P corresponds to the model of
molecules the extraneous charges. * These charges may be a dielectric as a mixture of positive and negative "fluids".
located both inside and outside the dielectric. Let us isolate a very small volume !'1 V inside the dielectric.
The Field in a Dielectric. The field E in a dielectric is the Upon polarization, the positive charge p+!'1 V contai~ed
term applied to the superposition of the field E of extra- in this volume will he displaced relative to the negative
neous charges and the field E' of bound charges: 0 charge by a distance I, and these charges will acquire the
dipole moment !'1p = p~ !'1 V·1. Dividing both sides of this
E = Eo + E', (3.1) formula by !'1 V, we obtain the expression for the dipole mo-
where Eo and E' are macroscopic fields, Le. the microscopic ment of a unit volume, i.e. vector P:
fields of extraneous and bound charges, averaged over a P = pi.!. (3.4)
physically infinitesimal volume. Clearly, the field E in the
dielectric defined in this way is also a macroscopic field. The unit of polarization P is the coulomb per square meter
(elm').
Relation Between P and E. Experiments show that for
3.2. Polarization a large number of dielectrics and a broad class of phenomena,
'Polarization P linearly depends on the field E in a dielectric.
Definition. It is natural to describe polarization of a dielec- For an isotropic dielectric and for not very large E, there
tric with the help of the dipole moment of a unit volume.
If an .ext~rnal field or a dielectric (or both) are nonuniform, exists a relation
polarl.zatlOn. turns out to be different at different points of P = 'X8 oE J (3.5)
t~e dlel~ctrlc. In order to characterize the polarization at a where 'X is a dimensionless quantity called the dielectric
given POIllt, we must mentally isolate an infinitesimal volume susceptibility of a substance. This quantity is independent
!'1 V containing this point and then find the vector sum of the of E and characterizes the properties of the dielectric itself.
'X is alwavs greater than zero.
• ~xtraneous c~arg~s are frequently called free charges, but this Hencef~rth, if the opposite is not stipulated, we shall
term IS not convement In some cases since extraneous charges may be
not free.

, I

'1
1,1 '1'
i i!
','II
72 3.3. Properties 0/ the Field 0/ P 73
3. Electric Field in Dielectrics

consider only isotropic dielectrics for which relation (3.5) of the positive and negative bound charges as a result of
is valid. polarization. Then it is clear that the positive charge
H~wever, there exist dielectrics for which (3.5) is not p:Z+dS cos a inclosed in the "inner"part of the oblique
applIcable. These are some ionic crystals (see footnote on cylinder will pass through the area element dS from the
page oR) and ferroeZectrics. The relation between P and E surface S outwards (Fig. 3.2b). Besides, the negative charge
for ferr~electr~cs .is nonlinear and depends on the history p:tdS cos a enclosed in the "outer" part of the oblique cyl-
of the dIeI?ctnc, l.e. on the previous values of E;
(this'phe- inder will enter the surface S through the area element dS.
But be know that the transport of a negative charge in a
nomenon IS called hysteresIs). -
certain direction is equivalent to the transport of the posi-
tive charge in the opposite direction. Taking this into account,
3.3. Properties of the Field of P we can write the expression for the total bound charge pass-
The Gauss Theorem for th£' Field of P. We shall show ing through the. area element dS of the surfa~e S in the out-
that the field of P has the following remarkable and impor- ward direction:
tant property. It turns out that the flux of P through an dq' = p:Z+ dS cos a + Ip:IL dS cos a.
p Since I r: I = p~ we have
dq' = P+ (l+ + L) dS cos a = p+l dS cos a,
(3.7)
where l = l+ "+
L is the relative displacement of positive
and negative bound chaJ;ges in the dielectric during polari-
(a) zation.
( b)
Next, according to (3.4), p:l = P and dq' = P dS cos at
Fig. 3.2
or
ar~itrary closed sur~ace S is equal to the excess bound charge
dq' = P n dS = P·dS. (3.8)
(wIth the reverse sIgn) of the dielectric in the volume en- Integrating this expression over the entire closed surface
. closed by the surface S, i.e. S, we find the total charge that left the volume enclosed
by the surface S upon polarization. This charge is equal to
~ PdS= -qint. (3.6) ~ P.dS. As a result, a certain excess bound charge will ?'
be left inside the surface S. Clearly, the charge leavlllg the
This equation expresses the Gauss theorem for vector P. volume must be equal to the excess bound charge remaining
Proof of the theorem. Let an arbitrary closed surface S within the surface S, taken with the opposite sign. Thus, we
envelope a part of a dielectric (Fig. 3.2a, the dielectric is arrive at (3.6).
hatched). When an external electric field is switched on Differential form of Eq. (3.6). Equation (3.6), viz. the Gauss
the ~ielectric is polarized-its positive charges are displaced theorem for the field of vector P, can be written in the differential form
relative to the negative charges. Let us find the charge which as follows:
passes through an element dS of the closed surface S in the
outward direction (Fig. 3.2b).
Let 1+ and C be vectors characterizing the displacement
I V·P=-p',
(3.9)
c:

75
3.3. Properties of the Field of P
74 3. Electric Field in Dielectrics
such a dielectric there is no excess bound bulk1ch~rge tn~
i.e. the divergence of the field of vector P is equal to the volume density only a surface bound charge appears as a resu t 0 po arl-
of the excess bound charge at the same point, but taken with the oppO-
zat~~·us find the relation between polarizatio? P a nd ~he
site sign. This equation can be obtained from (3.6) in the same manner
as the similar expression for vector E was obtained (see p. 24). For
this purpose, it is sufficient to replace E by P and p by p' .. d 'ty a' of the bound charge at the lOterf ace e-
surf ace ensl . 1 t s use property
When Is p' Equal to Zero in a Dielectric? We shall show tween the dielectrics. For this purpose, e u '
that the volume density of excess bound charges in a dielec- (3.6) of the field of vector~. We
tric is equal to zero if two conditions are simultaneously choose the closed surface lO the D
satisfied: (1) the dielectric is homogeneous and (2) there form of a flat cylinder whose end- 2~J~~~t---
are no extraneous charges within it (p = 0). faces are on different sides of the 1
Indeed, it follows from the main property (3.6) of the interface (Fig. 3.3). We shall as- I
field of vector P that in the case of a homogeneous dielectric, sume that the height of the cyl- • n'
we can substitute xeoE for P in accordance with (3.5), take inder is negligibly small and the F' 3 3
x out of the integral, and write area !1S of each endface is so small Ig. .
that vector P is the same at all . d'
x &eoE·dS .. -q'. . ts of each endface (thiS also refers to the surface ensl~Y
oJ
~?I~f the bound charge). Let 11 be the common normal to t e
The remaining integral is just the algebraic sum of all the interface at jl. given point. We shall always draw vector n
froDm. dielecd~riC ~h~o g~~e~}ri~ ~hrOugh the lateral surface
charges-extraneous and bound-inside the closed surface
S under consideration, Le. it is equal to q + q'. Hence,
x (q + q') = - q', from which we obtain Isregar Illg d 'th (3 6)'
of the cylinder, we ca'h write, in accor ance WI .,
q' = - 1~X q. (3.10)
P
2n
!1S + Pin' t1S = - a' t1S,
h P and P , are the projections of vector P in d ielec-
This relation between the excess bound charge q' and w. er; on2to the n~~al n and in dielectric 1 onto the normal
the extraneous charge q is valid for any volume inside the t~I(p' 3 3) Considering that the projection of. vector Ponto
dielectric, in particular, for a physically infinitesimal vol- n Ig. . 1 ~, is equal to the projection of thIS vector o~to
ume, when q' __ dq' = p'dV and q -+- dq = pl1V. Then, after . t h e norma. ( m n) normal n taken with the opposIte
cancelling out dV, Eq. (3.10) becomes t~e o~pos~te ~m ~ we can ~ite the previous equation
sIgn, l.e. tn' - - In' 11' t1S)'
, x • in the following form (after cance mg .
p = - 1+x p. (J.H)
P2n - Pin = - a'. (3.12)
Hence it follows that in a homogeneous dielectric p' = 0 This means that at the interface bet~een ~iel~ctrics the
when p = O.
Thus, if we place a homogeneous isotropic dielectric of normal component of vector P ha~ a dlsc.ofntIDdu!ty, ;~ose
. d d d a' In partIcular, I me mID IS a
any shape into an arbitrary electric field, we can be sure magmtu e epen s on . d d't' (3 12) acquires a
that its polarization will give rise only to the surface bound vacuum, then P 2n = 0, an con I Ion .
charge, while the bulk excess bound charge will be zero at simpler form: (3.13)
all points of such a dielectric,
Boundary Conditions for Vector P. Let us consider the where P n is the projection of vector ~ onto th~ outward nor-
behaviour of vector P at the interface between two homo- mal to the surface of a given dielectrIC. The SIgn of the pro-
geneous isotropic dielectrics.. We have just shown that in
('

76
3. Electric Field in Dielectrics
a.4. Vector D 77
jection P n determines the sign of the surface bound charge
a' at a given point. Formula (3.13) can be written in a diHer-
ent form. In accordance with (3.5), we can write expression (3.15) C an he transformed as follows:,
~ (eoE + P) dS = qlnt. (3.1.6)
(3.14)
where En is the projection of vector E (inside the dielectric The quantity in the parentheses in t~~ integrand ~ denoted
and in the vicinity of its surface) onto the outward normal. by D. Thus, we have defined an auxIlIary vector
Here, too, the sign of En determines the sign of a'.
A Remark about th(' Field of Vector P. Relations (3.6)
and (3.13) may lead to the erroneous conclusion that the
ID=eoE+P, I (3.17)
field of vector P depends only on the hound charge. Actually. h fl through an arbitrary dosed surface is equal.to the
this is not true. The field of vector P, as well as the field of E, w ose uxsum of extraneous c1larges enclosed hy this sur-
'algebraic
depends on all the charges, both hou~d and extraneous. face:
This can he proved if only by the fact that vectors P and
E are connected through the relation P = xeoE. The hound
charge determines the flux of vector P through a closed sur-
I~ DdS = qlnt.\ (3.18)
face S rather than the field of P. Moreover, this flux is de- .- This statement is called the ~auss thearer- for fiel~e~
termined not hy the whole hound charge hut by its part
enclosed hy the surface S. I t should h~. n ote? ~h~t ve~or D ISpth~~~~~ist~Za~~~P it is
ly different quantities. eo a~. d have an' deep
inde~d an aux~liary veft~~rW~~: ;::~e~t~ of the leld of
physICal meamng. Howe 't. (3 18) justifies the intro-
3.4. Vedor D vdectt~r Do'f etxhf:~:~~or~~ne~ua~~o~ase~ it ~onsiderably simpli-
uc IOn . d' I t ' *
The Gauss Theorem for Field D. Since the sources of an fies the ~naly(s3iS107f) thedfle(idH~n) a:: ~can~sfor any dielectric,
electric field E are all the electric charges-extraneous and RelatIOns . an '. ,
hound, we can write the Gauss theorem for the field E in both isotr?pic (~nf7)anS\~~~~P;~~t the dimensions o.f vect~r
~~~r~:~lOs~me'
the following form:
D as those of vector P. The 2quantlty D IS
~ eoE dS = (q + q')lnt, (3.15) measured in coulombs per square metre (elm ).
Differential form of Eq. (3.18) is
where q and q' are the extraneous and hound charges en-
closed by the surface S. The appearance of the hound charge
q' complicates the analysis, and formula (3.15) turns out I V·D=p, I (3.19)
to be of little use for finding the field E in a dielectric even .
Le. the divergence 0 f th e field D is equal to the volume densi ty of an
oint
in the case of a "sufficiently good" symmetry. Indeed, this
formula expresses the properties of unknown field E in terms extran~ous ch~rge .at the obat~i~fJ fro~ (3.18) in the same way as it
of the bound charge q' which in turn is determined hy un- wasThis
doneequatiOn can Ee,\see p. 24\" It suffices to replace E by D and
for the field
known field E. take into account only extraneous charges.
This difficulty, however, can he overcome hy expressing
. '. ftPJ] called dielectric displacement! or electro-
We Dh~~llo
s a no i b e u~ing
the charge q' in terms of the flux of P hy formula (3.6). Then * The ql;'antity this Dterm, however, III order to
static inductton. ~ .
emphasize the auxiliary nature of \~etor .
78 3. Electric field in Dielectrics 3.4. Vector lJ 79

At the points where the divergence of vector D is positive we have


the sources of the field D (p > 0), while at the points where the diver- Let us illustrate what was said above by several examples.
gence is negative, the sinks of the lield D (p < 0).
Example t. An extrallPUUS 'poil~t charge '1. is loc.atc(~ at th~ cenl~e
Relation Between Vectors D and .E. In the case of isotrop- of a sphere of radius a, made 01 an IsotropIC dIelectrrc WIth a dlelectrrc
ic dielectrics, polarization P = xeoE. Substituting this
expression into (3.17), we obtain D = 8 0 (1 +
x) E, or

I D=eoeE, I (3.20)

where e is the dielectric constant of a substance:


8=1+x. (3.21)
r
The dielectric constant e (as well as x) is the basic electric
characteristic of a dielectric. For materials 8> 1, while
for vacuum 8 = 1. The value of 8 depends on the nature of Fig. ~L4 Fig. ;LS
the dielectric and varies between the values slightly differ-
ing from unity (for gases) and several thousands (for some constant e. Find"'ihe projectiun 1,', of Iield intensity E as a functiun of
ceramics). The value of e for water is rather high (8 = 81). the distance r frolll the centre of this sphere.
Formula (3.20) shows that in isotropic dielectrics vector The syIllllletry uf the system allows us to use the Gauss th~orem
D is collinear to vector E. For anisotropic dielectrics, these for vector D for sol ving the 'problem (we cannut usc here the srn.l1lar
theorem for vector E, since the bound char~e is unlwo.wn to us). l' or a
vectors are generally noncollinear. sphere of radius r with the centre at the POlllt of locatIOn of the charge
The field D can be graphically represented by the lines q we can write the following relation: 4;r.r"lJ r . = q. Hence we can lind
of vector D, whose direction and density are determined in lj, and then, usillg formula (3.20), the required quantity t r :
the same way as for vector E. The lines of E may emerge and 1 '/ E ) 1 q
terminate on extraneous as well as bound charges. We say E r (r < a) = 4ne"; <:/:2' r (r > a =="4ne o ~.
that any charges may be the sources and sinks of vector E.
The sIJurces and sinks of field D, however, are only extraneous Figure 3.4 shows the curves lJ (r) and E (r).
Example 2. Suppose that a system consists of apoint ~harg.e q >.0
charges, since only on these charges the lines of D emerge and and an arbitrary sample 01 a homogelleou~. IsotropIc dielectrrc
terminate. The lines of D pass without discontinuities (Fig. 3.5), where S isa certain closed surface. h~d ?ut what Will hap-
through the regions of the field containing bound charges. pen to the ileIds of vectors E and D (and to theIr fluxes through the
A Remark about the Field of Vector D. The field of vector surface .) if the dielectric is removed.
The field E at any point of space is determined by the charge q
D generally depends on extraneous as well as bound charges and by bound charges of the polarized diele~tr.ic. Since in o.ur case
(just as the field of vector E). This follows if only from the D = e eE this refers to the lield D as well: It IS also determIned by
relation D = 8 oeE. .However, in certain cases the field of the exrra~eous charge q and by the bound char~es of the dielectric.
The removal of the dielectric will change the held E, and hence the
vector D is determined only by extraneous charges. It is lield D. The flux' of vector E through the surface S will also cha~ge,
just the cases for which vector D is especially useful. At since negative bound charges will vanish from inside. this s~rface.
the same time, this may lead to the erroneous conclusion However the flux of vector D through the surface SWill remalll the
that vector D always depends only on extraneous charges same in ;pite of the change in the lield D. . .
. Example 3. Let us consider a system C?ntallllllg no extraneo.us
and to au incorrect interpretation of the laws (3.18) and charges and having only bound charges. :::;uc~ a ~ystem can, for
(a.1!J). These laws express only a certain property of field example, be a sphere made 01 an electret (see p. 6b). 1, I~ure 3.?a shows
D but do not determine this field proper. the field E of this sy::;tem. What can we say about the fIeld Dt
RO 3, Electric Field in Dielectrics 3.5. Boundary Conditions 81

First of all, t~e abs~nee of extraneous charges means that the field
J) has ~o sou:(;~s, the Imes of D d.o IIOt emerge or terminate anywhE're. where the projections of vector E are taken on the direction
lJowever, the 11I~ld J) PXISts
_ 'md.IS.sllown I'n I"I'g . 3 .u.
( "b 'fl}'
Ie Illes 0 l' LL' of the circumvention of the contour, shown by arrows in
the figure. If in the lower region of the contour we take the
n
n

..
!------I ---:'
I c I~
Lines ofE Lines of D 2 [ ~ J
~ '"'r' •
(a) Cb)
Fig. 3.7 Fig. 3.8
Fig, 3.G
projection of vector E not onto the unit vector 't' bu~ onto
and ~ coi~cid~ outsi?e the sphere, but inside the sphere they have the common unit vector 't, then En' = - E H , and It fol-
01.PdPoslte dlfectlOns, smce here the relation D = 2 eE is no longer va- lows from the above equation that
l and D = FoE + P. 0 ' Ii:

(3.22)
3.5. Boundary Conditions
I.e. the tangential component of vector E turns out to be
Let ~s first consider the behaviour of vectors E and D the same on both sides of the interface (it does no.t have a
at. the mterface between two homogeneous isotropic dielec-
discontinuity). .
trics. Suppose that, for greater generality, an extraneous Boundary Condition for Vector D. ~et us ta~e a cylInder
su:face charge exists at tl16 interface between these dielec- of a very small height and arrange it at the lll~erface be-
trICS. The required conditions cari be easily obtained with tween two dielectrics (Fig. 3.8). The cross sectIOn of the
the help of two theorems: the theorem on circulation of cylinder must be such that vector D is the same within each
vector E and the Gauss theorem for vector D: of its endfaces. Then, in accordance'with the Gauss theorem
~E dI = 0 and ~ n dS =--= glnt- for vector D, we have
D 2n .!1S +
DIn' ·!1S = (JAS,
. Boundary Condition for Veclor E. Let the field near the where (J is the surface density of the extraneous charge at
lDterface be E 1 in dielectric 1 and E 2 in dielectric 2. We the interface. Taking both projections of vector D onto the
choose ~ sm~ll ?l(~_ngateu rectangular contour and orient it as common normal n (which is directed from dielectric 1 to
~hown In Fig, .L'. The sides of the contour parallel to the dielectric 2), we obtain Din' = - DIn' and the previous
lllterfac~ must have slIch a length that the fwld E over this
equation can be reduced to the form
!,eng.th ,,1Tl each dielectric can be assumed constant. The
height of th~ contour must be negligibly small. Then, in
accordance With the tllf'orem on circulation of vector E \ D 2n - Din = a. (3.23)
we IJave '
It follows from this relation that the normal component
of vector D generally has a discontinuity when passing
6-0181
82 3. Electric Field in Dielectrics 9.5. Boundary Conditions 83

throngh tllO inlt'l'[a(;('. If, however, tltel'O aro /10 extraneous pouents o[ vectors D are equal leads to the conclusion that LJ 2 > LJ 1
charges at the interface (u = 0), wo ohtain in magnitude, Le. the lines of D must be denser in dielectric 2.

I Din fl~1l" I
In this case the normal components of vector l) do not have
a discontinuity and tUI'll out to he tlte same on different
sides of the interfaGo.
Thus, in the absence of extraneous charges at the inter-
face hetween two homogeneous isotropic dielectrics, the
components E, anfl J)n vary continuollsly dnring a transi- FieldE FieldD
tion through this interface, while the components En and
D, have discontinuities. Fig 3.9 Fig. 3.1U
Hdl'action o[ E and D Lines. The boundary conditions
which we obtained for the components of vectors E and D We see that in the case under consideration, the lines of E are
at the interface between two dielectrics indicate (as will refracted and undergo discontinuities (due to the presence of bound
be shown later) that these vectors have a break at this in- charges), whileihe lines of D are only refracted, WI ~hou t thscon ttnu-
terface, Le. are refracted (Fig. 3.9). Let us flIld the relation ities (since there are no extraneous charges at the Illterlace).
between the angles a l and a 2 • Boundary Condition 'on the Conductor-Dielectric Inter-
In the absence of extraneous charges at the interface, face. If medium 1 is a conductor and medium 2 is a dielectric
we have, in accordancp- with (3.22) and (3.24), E 2 , = E l , (see Fig. 3.8), it follows from formula (3.23) that
and 8 2 E 2n = 8 r E ln • Figure 3.9 shows that
tan Cl2 E H /E 2n (3.26)
tan Cll E u /E l1l •

Taking into aceount the above conditions, we obtain the where n is the conductor's outward normal (we omitted the
law of refraction of lines E, and hence of lines D: . subscript 2 :;ince it is inessential in the given case). Let us
~ tan Cl2 E2
(;3.2!'i)
verify formula (~).26). In equilibriulll, the electric fteld
tan (;(, Et inside a conductor is E =,c 0, and hence the polarization
This means that lines of D and E will form a larger angle P = O. This means, according to (;3.17), that vector D =~ 0
with the normal to the interface in the dielectric with a inside the conductor, Le. in the notations of formula (:L~3)
larger value of 8 (in Fig. ;3.9, 8 2 > 8 1), D1 = 0 and DIn = O. IIenceD 2n = a.
Bound Charge at the Conductor Surface. If a homogeneous
Example. Let us represent graphically the fields E and D at the dielectric adjoins a charged region of the surface of a COIl-
i II ["ri'ace hetween two homogeneous dielectrics 1 and 2, assuming that ductor, bound charges of a certain r1en:;ity a' appeal' at the
awl llli~re is 110 extraneous charge on this surface.
:.'2> 1-'1 conductor-dielectric interface (recall that the volume den-
Since 1'2 > ;'n in accordance with (3.25) Cl 2 > at (Fig. 3.10). sity of bound charges p' = 0 for a homogeneous dielectric).
Considering that the tangential component of vector E remains un-
changed anrl using Fig. ;·1.9, We can easily show that F 2 < Hl in Let us now apply the Gauss theorem to vector E in the same
magnitude, i.e. the lines of E ill dielectric 1 must be denser than in way as it was done while deriving formula (2.2). Consider-
dielectric 2, as is shown in Fig. 3.10. The fact t~at. the normal corn- ing that there are both bound and extraneous charges (a
0*
3.6. Field in /l Homogeneous Dielectric 85

84 3. Electric Field in Dielectrics a' are unambiguously connected with the extraneous charges
a on the surface of the conductor.
and G') at the conductor-dielectric interface we arrive at As before, the,re will be no field inside the conductor (E =
the following expression: E" = (G + a')le o' On the other = 0). This means that the distribution of surface charges
hantl, according to (3.2G) E" = Dn/ee,o = alee o' Combin- (extraneous charges a and bound charges a') at the conductor-
ing these two equations, we obtain Gle = G + a', whence dielectric interface will be similar to the previous distri-
bution of extraneous charges (G), and the configuration of

I '0 = - -8-1
8- 0 . I (3.27)
the resultant field E in the dielectric will remain the same
as in the absence of the dielectric. Only the magnitude of
the field at each point will be different.
I t can be seen that the surface density G' of the bound In accordance with the Gauss theorem, a + G' = eDEn,
charge in the dielectric is unambiguously connected with where En = Dnlee o = a/eeo' and hence
G + a' = Gle.
the surface density G of the extraneous charge on the con- (3.28)
ductor, the signs of these charges being opposite.
But if the charges creating the field have decreased by a factor
3.6. Field in a Homogeneous Dielectric of e everywhere at the interface, the field E itself has become
less than the field Eo by the same factor:
It was noted in Sec. 2.1 that the determination of the
E = Eo/e. (3.29)
resultant field E in a substance is associated with consider-
able difficulties, since the distribution of induced charges in Multiplying both sides of this equation by eeo, we obtain
the substance is not known beforehand. It is only clear
that the distribution of these charges depends on tho nature • D = Do, (3.30)
III
and shape of the substance as well as on the configuration Le. the field of vector D does not change in this case.
II' I
II
of the external field Eo. It turns out that formulas (3.29) and (3.30) are also valid
:1 1

Consequently, in the general case, while solving the prob- in a more general case when a homogeneous dielectric fills
I' lem about the resultant field E in a dielectric, we encounter the volume enclosed between the equipotential sur1aces
serious difficulties: determination of the macroscopic field of the field Eo of extraneous charges (or of an external field).
E' of bound charges in each specific case is generally a com- In this caBe also E = Eo/e and D = Do inside the dielectric.
plicated independent problem, since unfortunately there In the eases indicated above, the intensity E of the field
is no universal formula for finding E'. of bound charges is connected by a simple relation with
An exception is the case when the entire space where there the polarization P of the dielectric, namely,
is a field Eo is filled by a homogeneous isotropic dielectric. E' = -Ple{}. (3.3t)
Let us consider this case in greater detail. Suppose that we
have a charged conductor (or several conductors) in a va- This relation can be easily obtained from the formula E =
cuum. Normally, extraneous charges are located on conduc- = Eo + E' ifl we take into account diat Eo = eE and
tors. As we already know, in equilibrium the field E inside P = xeoE. .
the conductor is zero, which corresponds to a certain As was mentioned above, in other cases the situation
unique distribution of the surface charge G. Let the fIeld is much more complicated, and forml1las~ (3.29)-(3.3t) are
created in the space surrounding the conductor be Eo. inapplicable.
Let us now fill the entire space of the field with a homo- CornU.Fie. Thus, if a homogeneous dielectric fills the
geneous dielectric. As a result of polarization, only surface entire space oeeupied by a field, the intensity E of the field
bound charges G will appear in this dielectric at the inter-
face with the conductor. According to (3.27) the charges
87
Problems

. h' I
9. Electric Field in Dtl!lectrics
Here dq' is the bound c~arg~ ill th \ ~n,
radii rand r + dr. Considermg a q
a;:erp'4Jtr2
between the spheres with
dr, we transform (1)
will be lower than the intensity Eo of the field of the same as follows:
extraneous charges, but in the absence of dielectric, by a , 2 d
r 2 dP r +2rP r dr=-pr r,
factor of e. Hence it follows that potential cp at all points
will also decrease by a factor of e: whence
(2)
cp = cpo/e, (3.32)
where CPo is the field potential in the ahrnce 01. the dielectric. In the case under consideration we have
The same applies to the potential difference:
e-l 8-1 q
U = Uoh, (3.33) Pr=xeoE r = -e D r =-;- 4n:r 2

where U0 is the potential difference in a vacuum, in the . (2) will have the form
and after certain transformation expressIOn
absence of dielectric. 1 q
In the simplest case, when a homogeneous dielectric p'=~ ?"
fills the entire space between the plates of a capacitor, the
potential difference U between its plates will he by a factor which is just the required result.
of e less than that in the absence of dielectric (naturally, f vector D An infmitely large pla~e
at the same magnitude of the charge q on the plates). And • 3.2. The Gauss thedo~i t~~c with th~ dielectric constant e IS
since it is so, the capacitance e = ql U of the capacitor made of a hBtnogeneous Ie ee 1
uniformly charged by an. extraneous
filled by dielectric will increase e times charge with volume densIt~ P > O. Ex I{J
e' = ee, (3.34) The thickness of the platt! IS 2a.
Find the magnitude of vector E an.
(ll 1~h1lii-l~
l;; E?,
where e is the capacitance of the capacitor in. the absence the potential cp as functions of the dI-
stance 1 from the middl~ of. the pl a~e
of dielectric. It should be noted that this formula is valid (assume that the poten~Ial IS ~ero n h
when the entire space between the plates is filled and edge
effects are ignored.
the middle of the plate), ChOOSlll
X-axis perpendicular to the pal'.
y\ I'

Plot schematic curves for the pro-


. . n E (x) of vector E and the po-
~~~t:fal ~(x). (2) Find the surface and
Problems volume densities of the bound charge. /
. .3.1. Polarization of a dielectric and the bound charge. An extra- Solution. (1) From symmetry co~- .
neous point charge q is at the centre of a spherical layer of a heteroge- siderations it is clear that ~'I= ~;ii FIg. 3.11
neous isotropic dielectric whose dielectric constant varies only in the the middle of the plate, w 1 I' a d-'
radial direction as 8 = air, where a is a constant and r is the distance other points vectors E f a~h pef P:: In order to determine E, w,e s~bIl
from the centre of the system. Find the volume density p' of a bound icular to the surface 0 I' P a . D (since we know the dIstrr u-
charge as a fuuction of r within the layer. 'use the Gauss theorem for vector W take for the closed su~face a
tion of only extran~ous charges)! hose endfaces coincid~s WIth the
Solution. We shall use Eq. (3.6), taking a sphere of radius r as right cylinder of helghthl, one 0 ~'onal area of this cylInder be S.
the closed surface, the centre of the sphere coinciding with the centre midplane (x = 0). Let t I' cross-sec 1
of the system. Then
Then
4Jtr2 • P r = -q' (r),
DS = pSI, D = pi,
E = pl/ee:o (1 ~ a),
where q' (r) is the bound charge inside the sphere. Let us take the E = paleo (l ~ a).
differential of this expression: DS = pSa, D = pu,
4Jt d(rl·p r) = - dq'. (1)
89
Problems
88 3. Electric Field in Dielectrics
where P is the volume density of the extraneous charge. Hence
The graphs of the funtions Ex (x) and <p (x) are shown in Fig. 3.11. IJ p r3 -a'
It is useful to verify that the graph of Ex (x) corresponds to the de- E-- -=-- --,-
rivative -alp/ax. - eEo 38 08 r
(2) In accordance with (3.13), the surface density of the bound
charge is The corresponding curves for E (r) and cp (r) are shown in Fig.· 3.12b.

, e-1
fJ =Pn="X8 0 E n =(e-1)pa/e=--. pa>O.
e
This result is valid for Il0th sides of the plate. Thus, if the extraneous
charge p > 0, the bound charges appearing on both surfaces of the
plate are also positive.
In order to find the volume density of the bound charge, we use
Eq. (3.9) which in our case will have a simpler form:

o\---'---!---~, 0 a
p'=_ ap x
ax
=_.!.-
ax
( e-1 px)=-- e-1 p.
e e
a b
(a) (b)
It can be seen that the hound charge is uniformly distributed over Fig. 3.12
the bulk and has the sign opposite to that of the extraneous charge.
• 3.3. A homogeneous dielectric has the shape of a spherical . ·f I d· tributed with the volumc
.3.4. Extraneous ~argerUd·lu~r:\:ad~of a homogeneous dielec-
layer whose inner and outer radii are a and b. Plot schematically the
curves of intensity E and potential <p of the electric field as functions
of the distance r from the centre of the system, if the dielectric is
deMity p > 0 over ~ sp ~:e 0 ~~nd (1) the magnitude of vector E as
tric . with the pet:nntttvI y 8. he centre of the sphere and plot the
a
charged by a positive extraneous charge distributed uniformly (1) function of the dl!~tanceEr(f)ro~d tm (r). (2) the surface and volume den-
over the inner surface of the layer, (2) over the layer's hulk. curves of the functions raT '
sitie5 of bound charge.
Solution. (1) We use the Gauss theorem for vector D, taking d . E we shall use the Gauss thea-
for the closed surface a sphere of radius r: Solution. (1) In .order to ketcr~h~e di~tribution of only extraneous
rem for vector D smce we now
4nr'D = q, c~: D p
where q is the extraneous charge within this sphere. Hence it follows 4 , D P r E=-= - r,
r<a, 4nr3IJ=Tnr p, =7' 3888100 0
that
D (r < a) = 0, D (r > a) = q/4nr 2 • 2 4, D_ Pa3 E=!!-=pa3~.
r;;:' a, 4nr D =3 na p, - 3r' ' eo 3eo r
The required intensity is
E (r < a) = 0, E (r > a) = D/eeo. The curves for the functions E (r) and ~ (r~are ~hown in Fig. 3.13.
(2) The surface density of the boun c arge IS
The curve for E (r) is shown in Fig. 3.12a. The curve for <p (r) is also e-1 pa
llhown in this figure. The curve <p (r) must have such a shape that the fJ'=Pn=-e- T·
derivative 8<plar taken with the opposite sign ~orresponds to the curve
of the function E (r). Besides, we must take into account the normali- d ·t of bound charge it is sufficient to
zation condition: <p -+ 0 as r -+ 00. In order to find the volume enSI y f I (3 11) ~nd we get
It should be noted that the curve corresponding to the function repeat the reasoning that led us to ormu a . ,
cp (r) is continuous. At the points where the function E (r) has finite 8--1 (1)
discontinuities, the function <p (r) is only broken. . p' =c - --;;- fl·
(2) In this case, in accordance with the Gauss theorem, we have
4 'rh••• __It can be obtaincd ill a different way, viZ· . b Y 1I~(i.llg
4nr2IJ=Tlt (r 3 -a') p, n:;:KU E d x doc~ not depend on coor(I rna t es m-
Eq. (3.9). Since P = "XfO an
90
3. Electric Field in Dielectrics
Problems 91
side the sphere), we obtain
p' = -V'p = -XEllV.E where we assume that the charge of the inner plate is q > O. Let us
find E with the help of the Gauss theorem for vector D:
where EoV'E = P + p' H' '
mula (1). . ence p = -x (p +
p'), which gives for- D 1 q 1 q
E = EEo = 4nEo ~ ~ 4'1Eo r;;:.
2
. 4nr D = q;
• 3.5. Capacitance of a cond t .
spherical conductor of radius a, su~~o~~de~lIbd the capacitance of a Substituting the latter expression into (1) and integrating, we find
y a layer of a homoge-
q b 4ne oa
U = 4neoa In a' C = In (bla) •
• 3.7. The Ga_ theorem and the principle of SU,ertlOSitiOD.
Suppose that we have a dielectric sphere which retains polarization

a
r
·Fig. 3.13
Fig. 3.14
Ileous diplectric and hav· tI
stant f. Plot approximat~r:fur~:sf~~Pf radius b and the dielectric con-
tan ce from the centre of the sphere Y)t~nd <fJh(r), ~here r is the dis-
VI' l y. ,I e sp ere IS charged positi-
Solution. Dy definition th .
potential <fJ of the conducior eS~;~I~~~agnce C =11~!<fJ· Let us find ,the Fig. 3.15 Fig. 3.16
00
, b menta y a charge -
q to ·t.
1 .
"1 after an external electric field is switched off. If the sphere is. polar-
<fJ= \ E r dr= _
. 4n£o ~.\ £r 2 T 4
r
-.!L dr-' _1 \ q
-
00

-- dr. ized uniformly, the field intensity inside it is E' = - P/SEo, where P
a n8. ., r20 is the polarization. (1) Derive this formula assuming that the sphere is
I h polarized as a result of a small displacement of all positive charges of
ntcgrating this expression, we obtain the dielectric with respect to all its negative charges. (2) Using this
formula, find the intensity Eo of the field in the spherical cavity inside
<fJ=~4
q (_1_ j-~)., 4nE oEa an infinite homogeneous dielectric with the permittivity f if the field
nEoE a b ,L- =
1-c,~--"'-,--_ intensity in the dielectric away from the cavity is E.
p -HE-1) a!b
Ill' curves for E (r) and ()
<fJ r are shown in Fig. 3.14. Solution. (1) Let us represent this sphere as a combination of two
spheres of the same radii, bearing uniformly distributed charges with
h • 3.6. Capacitance of a ca ac·t
t e radii of the plates a and b \\~er~ or:-A ~pherical .capacitor with volume densities p and -po Suppose that as a result of a small shift,
heterogeneous dielectric who' _ . a. ;- b, IS filled wIth an isotropic the centres of the spheres are displaced relative to one another by a
from the centre of the svste~ea;er~ttl\'!ty depends on the distance r distance I (Fig. 3.15). Then at an arbitrary point A inside the sphere
the capacitance of the capacitor. f - air, where a is a constant. Find we have
So/ution. In accordance w'th th d .. E' =E~+E~=-3P(r+-r_)= __ .J:!.,
capacltor(C= !U) th I ~ efillltlOnofthecapacitanceof eo 3eo
potential djffere~ce 'u f~lrob~em jreduced to determination of th:
IS where we used the fact that field intensity inside a uniformly charged
a gIven c large q:
b
sphere is E = pr/3E o, which directly follows from the Gauss theorem.

u= J
a
Edr,
(1)
It remains for us to take into account that in accordance with (3.4),
pI ~ P. ,
(2) The creation of a spherical cavity in a dielectric is equivalent
to the removal from a sphere of a ball made of a polarized material.

~
..
i' ,
-;;-'-'-.'-. '.
"
.•

,
m
IIl
,.

II
',I

II'.1.[ 92
3. Electric Field in Dielectrics
1

1. '/ Consequently, in a('cordance with the principle of superposition, the


Ii field E inside the dielectric can he represented as the sum E = E' + Problems 93
+ Eo· Hence
1'1
Eo = E - E' = E + P/3E O
' Here
a'
h t cos t}
we took into account. t ,a t t
-+- 0, i.e. if the charge q IS JUS a
th;inte~face
- lIr As 1 -+- 0 the quantity
, there is no surface
Considering that P = (e:- 1) EoE, we obtain
'II charge on the plane. ' f a CI' a thin rinO' with the centre at
Eo = (2 + e) E/3. (2) Let us imagine at the lllt~h t the inner a;' outer radii of this
the point 0 (Fig. 3.17). Suppose ,a are r' and r' + dr'. The surface
• 3.8. Boundary conditions. In the vicinity of point A (Fig. 3.16) rmg
belonging to a dielectric-vacuum interface, the electric field intensity bound charge WI. thm . th' . g is
,IS rID
in vacuum is equal to Eo, and the vector Eo forms the angle ao with dq'=o' • 2:rtr' dr'. It can be ~:
the normal to the interface at the given point. The dielectric permittiv~ n from the figure that r 2 = 12 r • +
ity is e. Find the ratio E/Eo, where E is the intensity of the field in~ whence r dr = r' dr', an~ the exp.re~~
side the dielectric in the Vicinity of point A. sion for dq' combined WIth (1) glV~S
Solution. The field intensity inside the dielectric is e-1 dr
dq'=-e'+1 qlrz·
R= V -E~-I-E~.
(1) Integrating this equatio~ over r be-
UsilJJ{
we fmdconditions (3.22) and (3.21) at the interface between dielectrics, tween land 00, we obtam
Fig. 3.17 q'= - e-1 q.
e+1
E-r; = Eo sin a o, En = IJnlce o = Eonle = Eo cos aole,

• 3.~O: ~ p(),lIlt charge q dl'~I~ctric with the dil~lectric p~rmlttlvl-


when> Eon is the normal component of the vector Eo in a vacuum. , , n the plane separating a va.cu~I!1
Substituting these expressions into (1), we obtain
from an F' mtimte fiomo~eneousf
d the magmtudes I
0 vectors
D and E in the entIre space.
ty e. m . II from the continuity of the normal
Solution. In this case, It fa o~ eE ,Only the surface cha.r~e .a'
i.e. E < Eo. component of vector D thatf2 n n~;:t of veo;Lor E in the vlclmty
willthe
of contribute to the
point under n?rm
consl a t"c~P~ence
d era 10 •
the above equality can be
• 3.9. A point charge q is in a vacuum at a distanc\: 1 from the written in the form
plane surface of a homogeneous dielectric filling the half-space below
the plane. The dielectric's permittivity is e. Find (1) the surface den- 0'/2eo = e (-0'/2eo).
sity of the bound charge as a function of the distance r from the point
charge '1 and analyse the obtained result; (2) the total hound charge on We immediately und that ~; = ~. no bound surface charge (with the
the surface of the dielectric. Thus, in the giv~n ca~e I~re:s 'ontact with the extraneous point
exception o~ ~hc pomt~hl~ ~lh~e~le~tric licld iu the surrounding spa~e
Solutioll. Let liS Use the continuity of the normal component 0' charge q). 'I hiS mea~s a , and b' depends only on the dls-
vector D at the dielectric-vacuum interface (Fig. 3.17): is the field of the, pomt char~e ( ~eq ~harge q' is unknown, and hence
D 2n = tance r from thiS cha~ge'l u m for vector D. Taking fo~ the ~lose~
theofG~uds.s
ra 1U,St rlCo:'~h
DIn' E 2n = eE1n
or we musta use
surface sphere \\1
the centre at the point of locatIOn o.
the charge q, we can write
---1 - --
q2 cos t}+--=e
4n:f o r
(J'
2e o
( 1 -cos
-_ q (J' )
\'t--._
4:rte o r 2
,
, 2e
2:rtr 2 D o + 2rr.r 2
J) = q,
o I II ""uit'ltll'S of H'l'tor D at the disLa,nce r
o
where the u'rm a' /2£0 is the component of the electric field created where Do hand J) are . tht lC V"CUUIn
lu . 'I I'll tJ'e
all(
";I'eleclric reS}lech"eiy,
,. • to
Dear the region of the plane under consideration, where the surface fromBesides,
the C arge
from qlHI
t Ie conet WUI
· <. 'tvJ of lhe tanrrential
5
component of vee r
charge density is a'. From the last equality it follows that
E it follows that J) = d'o.
0,=_e-1 ~ (1)
e+ 1 2ltr 3 '
Combining these two conditions, we find
q eq
Do ~ 2lt (1+e) r 2 ' j)oc= 2lt <1+1;) r 2 •
94
4. Energy 01 Electric Field
4.1. Electric Energy 0/ a System of Charges 95
and the ele&tric field intensity in the entire space is
de. Then the corresponding work of these forces is
E = .!.!J!. = 'I
80 2n (t +e) for 2 6/1 1• z = FI·rll l + F 2 ·dl z·
It ean ~ seen that for 8 = 1 these formulas are reduced to the al- Considering that F 2 = - 1'\ (according to the N.ewton
ready famlhar expressions for D and E of the point charge in a vac-
uum. third law), we can write this expression in the form
The obtained results are represented graphically in Fig. 3.18. It 61'11.2= F I (dll-dl l )·
The quantity in the parentheses is the dis~la~ement .of
charge 1 relative to charge 2. In other words, It IS ,the ~IS­
placement of charge 1 in the system of. referen.ce 1! '
whIch
is rigidly lixell to charge 2 and accomplishes wIth It a trallS-
lational motion relative to the initial reference system K.
Indeed the displacement dl l of charge 1 in system K can
be rep;esented as the displacement dl 2 of .system K' plus
the displacement dli of charge 1 relatIve to system
. Field E K': dl l = dl z +dlj. Hence, dl l - dl z = dli, and
FieldD
Fig. 3.18 I{ 6At,2==Ft.dl~.
Thus, it turns out that, the sum of the elementary works
should be noled that the field n in this case is not determined by the done by two charges in an arbitrary system of reference K
extranC?us charge alone (otherwise it would have the form of the field
of a pOlllt charge). is always equal to the elementary work done by the~or~e
acting on one charge in another reference system (/\ ) III
which the other charge is at rest. In other words, the work
6A l.Z does not depend on the choice of the initial system of
reference.
4. Energy of Electric Field The force F acting on charge 1 from charge 2 is conserva-
tive (as a cellt~al force). Consequently, the work of the given
4.1. Electric Energy of a System of Charges force in the displacement dli can be represented as the de-
~nergy .Approach to Interaction. The energy approach crease in the potential energy of interaction between the
to lllteractlOn between electric charges is, as will be shown pair of charges under consideration:
rather fruitful in respect of its applications. Besides, thi~ 6.111. 2 - dW l2 ,
approach makes it possible to consider the electric field
from a different point of view. where W l2 is the quantity depending only on the distance
between these charges.
First of all, let us find out how we can arrive at the con-
cept of the energy of interaction in a system of charges. 2. Let us now go over to a system of three point charges
1. Let us first consider a system of two point charges 1 (the result obtained for this case can be easily g~neralizerl
and 2. We shall [lOd the algebraic sum of the elementary for a system of an arbitrary nllmber oE charges). fhe war\..
works of the forces F I and F 2 of interaction between the performed by all forces oE interaction during elementary
charges. Suppose" that ill a certain system of reference K displacements of all the charges can be represented as the
the charges were displaced by dl l and dl 2 during the time sum of the works of three pairs of interactions, i.e. 6.1
= l'lA1. 2 + 6A1.3 + 6.1 2 ,3' But as it has just been shown,
4J. Electric Energy of a System of Charges 97
96 4. Energy of Electric Field
obviously docs 1I0t depend on the number ~f charg?s con-
for each pair of interactions 6A i, k = - dW ik , and hence stituting the system. Thus, the energy of mteractlOn for
a system of point charges is
6A = - d (W12 + W13 + W 23 ) = - dW,
where W is the energy of interaction for the given system of t4.2)
charges:
W = W 12 + W13 + W 23 • Considering that WI =c= qi(Pio where qi is the ith cha:ge of
tho system and (ei is the potential create~ ~t the pomt of
Each term of this sum depends on the distance betwoon location of the ith charge by all the remaunn~ charg~s, we
corresponding charges, and hence the energy W of the given obtain the final expression for the energy of mteractlOn of
system of charges is a function of its configuration. the system of point charges:
Similar arguments are obviously valid for a system of
any number of charges. Consequently, we can state that to
each configuration of an arbitrary system of charges, there II! --- -- - 21~ g.(D·
I riO
(4.3)
corresponds a certain value of energy W, and the work of
all the forces of interaction upon a change in this configu-
ration is equal to the decrease in the energy W Example. Four similar point ch,~rges q ar~local('d at th: ve:tice~
of a tet.rdbedron w.ith an edge a (I'lg. 4.1). I·tnd the energ) of tnter
6A = - dW. (4.1) ac:tinll of charges in this syst.em.
Energy of Interaction. Let us find the expression for the The energy of interaction fn.. each pair of charges of t.his system i~
energy W. We again first consider a system of three point t.he same and equal to 1V I = q2 /4m oa. It. can be seen from the figure
that. the total number of interacting pairs is six, a~d hence the energy
charges, for which we have found that W = W 12 + WllI + of interaction of all point charges of the system IS
+ W 23 • This sum can be transformed as follows. We
W = 6 WI = 6 q2/4ne oa. q
represent each term W ik in a symmetric form: Wilt =
Another approach to the solution of this
= ~ (W lk + W kl ), since W ik = W k !. Then problem is based on formula. (4.3). The p,0t.e-
ntial q> at the point of 10catlOII of one of the
charges. created by the Held of all the other
W = ; (W t2 + W21 + W 13 + W31 + W 23 + W 32 ). charges. is q> = 3q/4ne oa. lienee q q

Let us group the terms with similar first indices: q


W =+ [(W 12 + W (3) + (TV21 + W23 ) + (W:11 + W32 )]. Fig. 4.1

Each sum in the parentheses is the energy WI of interaction Total Energy of Interaction. If the charges are arranged
between the ith charge with all the remaining charges. continuously, then, representing the system of charges. as
Hence the latter expression can be written in the form a combination of elementary charges dq = p dV and gOlQg
over in (4.:1) from summation to integration, we obtain
W = ; (W t + W 2 + W s) =+ ~ i=l
3

Wj'
w· -~- ~ plpdV, I
This expression can be generalized to a system of an ubi-
trary number of charges, since the above line of reasoning 7-0181
4. Energy of Electric Field 4.2, Charged Conductor and Capacitor Energies !lO
98

where (jJ is the potential created by all the charges of the corresponds only to the energy W I2 , ~hile c~lcllla.tion b~
system in the volume element dV. A similar expression can formula (4.4) gives the total energy o( lllteractwn: III a~dl­
be written, for example, for a surface distribution of charges. tion to W 12 , it gives intrinsic energies WI and W 2 • DIsre-
For this pmpose, we must replace in (4.4) p by 0' and dV hy gard of this circumstance is frequently a cause of gross
dS. errors.
Expression (4.4) can be erroneously interpreted (and We shall return to this question in Sec. 4.4. Now, we
this often leads to misunderstanding) as a modification of shall use formula (itA) for obtaining several important re-
expression (4.3), corresponding to the replacement of the sults.
concept of point charges by that of a continuously distrib-
uted charge. Actually, this is not so since the two expres-
sions differ essentially. The origin of this difference is in
different meanings of the potential (jJ appearing in these ex- 4.2. Energies of a Charged Conductor and
pressions. Let us explain this difference with the help of a Charged Capacitor
the following example.j Energy of an Isolated Conduetor. Let a conductor. have
Suppose a system consists of two small balls having a charge q and a potential (jJ. Si?ce the value of (jJ IS the
charges qi and q2' Thedistance between the balls is consi- same at all points where charge IS located,w,e .can. take (jJ
derably larger than their dimensions, hence qi and t}z can he in formula {4.4),:outof the integral. The remallllllg !ntegral
assumed to be point charges. Let us find the energy W of is just the charge q on the conductor, and we oLtam
the given system by using both formulas.
According to (4.3), we have
1
W ="'2 (qt(jJt + q2(jJ2) = qt(jJl = q2(jJ2,
~
. W .. -* = 0f -f-. \
= (1t.G)

These three expressions are written assuming that C = q/(jJ.


where <PI {(jJ2) is the potential created by charge q2 (ql) at Energy of a Capacitor. Let q and (jJ + be the charge. and
the point where charge ql (q2) is located. potential of the positively charged plate. o~ a capaCitor,.
On. the other hand, according to formula (4.4) we must According to (4.4), the integral can be splIt lllto two parts
.split the charge of each ball into infinitely small elements (for the two plates). Then
p dV and multiply each of them by the potential <P created
by not only the charge elements of another ball hut by the
charge elements of this ball as well. Clearly, the result will
W = + (q+(jJ+ + q-(jJ-).
be completely different: Since q_ = - q+, we have
1 1
W = WI + W 2 + W 12 , (4.5) W =2 q+ «(jJ+ -<p-) =2 qU,
where WI is the energy of interaction of the charge elements where q = q+ is the charge of the capacitor ~nd .U is the
I of the first ball with each other, W 2 the same but for the potential difference across its plates. Consldermg that
II second ball, and W 12 the energy of interaction between the C = q/ U, we obtain the following expression for the energy
'I cjJ.arge elements of the first ball and the charge elements of of the capacitor:
the second ball. The energies WI and W 2 are called the ----------\
Iii
I!!
intrinsic energies of charges ql and q2' while W 12 is the energy
of interaction between charge ql and charge q2'
Thus, we see that the energy W calculated by formula (4.3)
I w~5'{-~!f- f,. (4.7)

7*
100
4. Energy oj Electric Field 4.3. Energy of Electric Field iOt

J I :,;hulIJd Le, I~uled he~e thai these formulas determine the the expression C = eeoS/h, we obtain
lutal cllergy 01 lIltel'aCllOn, viz. not only the energy of in-
teractIOII Lot \\ een the charges of one plate and those of the - cUt
W -2- --'- tlloSU' _ £Ilo
2 --2 h
(Y )2 Sh •
ut.he~ plate, but also the energy of interaction of charges
wlthlll each plate.
And since U1h = E and Sh = V (the volume between the
And Wh~t if We Have a Dielectric? We shall show that capacitor plates), we get
formula,s (4. fJ) and (4.7) are valid in the presence of a dielectric.
For t!llS purpose, we consider the process of charging a W =+eoeE2V. (4.8)
capacitor a" a transport of small portions of charge (d ')
from one plate to the other. q This formula is valid for the uniform field which fills the
. Th~ elen~entary .\I'ork performed against the forces of the volume V.
f1eld 111 th IS case IS
In the general theory, it is p~oved that the en~rgy ':v
bLl == l./' dq' = (q'IC) dq', can be expressed in terms of E (m the case of an £SOtrop~c
where l./' i::> dielectric) through the formula
the potential tltfference between the plates at
the moment
transferred.
whelJ the next portion of charge dq' is being
W= ~ llo~E' dV= j E;D dV. (4~9)
Integruting this expression over q'
obtain between 0 and q, we 4/
The integrand in this equation has the meaning of the ?nergy
A = q2/2C, contained in the volume dV. This leads us to a very Impor-
tant and fruitful physic'al idea about localization ot energy
which ~oilJcides with the expression for the total energy of in the field. This assumption was confirmed in experiments
a capacitor. ~onsequel~tly, the work done against the forces with fields varying in time. It is the domain where we en-
of the eleclnc lleld IS completely spent for accumulating counter phenomena that can be explained with the help. of
t~Je en~rg~ W of the charged capacitor. Moreover, the expres- the notion of energy localization in the field. These vary~ng
sion o.DtalD~d for the work A is also valid in the case when fields mav exist independently of electric charges WhICh
there IS a dIelectric between the plates of a capacitor. Thus,
have generated them and may propaga~e in space in the
w.e hav~ proved the validity of (4.7) in the presence of a form of electromagnetic waves. Expenments show that
dlOlectl'lc.
electromagnetic waves carry energy. This circumstance con-
Obviously, all this applies to (4.6) as well.
firms the idea that the field itself is a carrier of energy.
The last two formulas show that the electric energy is
4.3. Energy of Electric Field distributed in space with the volume density
On Loc?lizatioll of Energy. Formula (4.4) defines electric footE' E·D
energy n' of any system in terms of charge and potential. W=-2-=-2- (4.10)
It turns out, however, t?at energy W can also be expressed
t}~rough another quantity characterizing the field itself It should be noted that this formula is valid only in the
Vl~. throu~h field intensity E. Let us at first demonstrat~ case of isotropic dielectrics for which the relation P = x€oE
thIS .by u~lDg the simple example of a parallel-plate capaci-
tor, Igno.nn g field distortions near the edges of the plates holds. l'
For anisotropic dielectrics the situation is more comp lcat-
(edge effect). Substituting into the formula W = CU 2/2
ed.

f
102
4. Energy 01 Electric Field
4.3. Energy of Electric Field 103
A~other substantiation for fo
~hr:gl of an i~olated charged cond~:~~~s(~9~ It is known that the layer is given by
energ;r{:;ufhe Ifield rrect , proceeding from th;J~~2~fel·otcuaSI~hOt~ thaft
L ' Iza IOn 0 BOllEI 4 2d
et us consider an arbitrar .. dW=-Z- itl" r,
denbtallly isolate a tube of infinit~ .PosItIvely charged conductor. We
I' y mes of E (Fig. 4.2), and tak::'i~.cross section, which is bound- where E = q2/4nB oBr 2, Integrating this expression over r between a
=dS dt T~·an ellementary volume dV= and b, we obtain
. IS vo ume contains the energy
dt ED DdS . q2
W= 8nE oB
(1' 1) .
a -T
"2 dSdt =~ Edt.
. Example 2. Find the work that must be done against the electric
Let us now find th forces in order to remove a dielectric plate from a parallel-plate char-
th~d in the entire isol:te~n~~?elo~al­ ~d capacitor. I t is assumed that the charge q of the capacitor remains
IS purpose, .we !ntegrate the l~st I' o~ unchanged and that the dielectric fills the entire space between the
pressIOn, conSldermg that th d x capacitor plates. The capacitance of the capacitor in the abseace of
D dS is the sa . I
m I' pro uct
of the tube andeh a 1. cross sections the dielectric is C.
out of the i~tegral~nce It can be taken The work against the electric forces in this system.is equal to the
increment of the electric energy of the system:
Fig. 4.2 D dS ~
dW=-2- \
•.
Edt=~
2 cp,
A = 6.W = W2 - WlJ

A where WI is the energy of the field between the capacitor plates in


wh ere A is a point at the b . . '. the presence of thi!'Qielectric and W 2 is the same quantity in the absence
It remains for us to make ~1lllDlng of t~e Imagmary tube. of the dielectric. Bearing in mind that the magnitude of vector D
fix£ression over all the tubes and Ja~t th , I.e. integrate the obtained
e tep will not change as a result of the removal of the plate, 1.1'. that D 2 =
h d. Considering that pote~tial n. the energy localized in the entire
t ~ tubes (since they originate
write
0:~h esrme at the end faces of all
I' sur ace of the conductor), we
= D1 = (J, we can write

A=W2 -W I = \/~
2B .

__ ~)
2B B
V = ~
2C
(1-~),
B
o o

W= i~ DdS,
where V = Sh and C= BoS/h, S being the area of each plate and h
the distance bl'tween them.
w~ere the integration is performed ov
h lth one of the equipotential surf fr a closed surlace coinciding
t eorem, this integral is equal to ~chs. hn accordance with the Gauss
The Work of the Field During Polarization 01 a Dielectric.
An analysis of formula (,'1.10) for the volume energy density
we finally get I' c arge q of the conductor, and
reveals that for the same value of E, the quantity w is E
W = qrp/2, times greater in the presence of a dielectric than when it is
Q.E.D. absent. At first glance this may seem strange: field intensity
in both cases is maintained the same. As a matter of fact.
Let us consider two exam l ' I l . when a field is induced i'l a dielectric, it does an additional
we get by using the ide~ of ~n~r 1 ulstral~Ing. the. advantages work associated with polarizatioll. Therefore, under the
gy oca lZatlOn In the field.
Example 1. A point char I' . energy of the field in the dielectric we must understand the
!"'lade of a homogeneous diel:ctr1 IS ~t the ce?tre of a spherical layer -sum of the electric energy proper and an additional work
lI~ner and outer radii of the la eCr ~~th
e
the dielectric constant E. The which is accomplished during polarization of the dielectric.
Fmd the electric energy contaiKed in thlJuadl. tlo a ~nd b respectively. In order to prove this, let lIS substitute into (4.10) the
IS Ie ectrIC layer.
We mentally isolate in the d' 1 . quantity EoE + P for D, which gives
ical layer of radius from r to Ie +ect~lc ~hvery thin concentric spher- l'oE2+ E . P
r r. I' energy localized in this w=--
2
--
2'
(4.11)
104
4. Energy of Electric Field
4.4. ..1 S,/stem of Tu'o Charged Bod/.-s 10;:;
The first term on the ri ht-ha d ' '. -------_...::......- ----._------
energy density of the fiel~ E . n side comcldes with the
4.4. A System of Two Charged Bodies
us to verify that the "addit'o I~" a vacuufI.l. It remains for
with polarization I na energy E·P/2 is associated Suppose that we have a system of two clwrged bodies in a
Le~ us calculate' the work done b . vacuum. Let one body create in the surrounding space the
polarIzation of a unit volume of th YI ~hle el~tr~e .field for field E I , while the other body, the field E 2 • The resultant
e ( Ie ectnc, I.e. for the field is E = E) -I- E 2 , and the square of this quantity is

dJ - J- I dl E2=E;+E~ + 2E j ·E2 •
- .....- - - - 1 = 0
:
===~==:
1-1.- L
+
..-: - +

Therefore, according ,to (4.9), the total energy W of the


given system is equal to the sum of three integrals:
Fig. 4.3
lV --
- r
.1 -.,--
FoFt
- (r
Il" Eof~ dl .,I
) -2-
'
JeoE j' E ~ c.il'. , (4.14)
disRlacements of char es' , .
agamst the field upon g p+ and ~- I'espectIvely along and
which coincides with formula (4.5) and reveals the field
E to E + dE. Neglect~~ I~~rease m the field intensity from
meaning of the terms appearing in this sum. The first two
g e second-order terms we write
6A=p~E.dl++r'E.dJ ' integrals in (4.14) are the intrinsic energies of the first and
second chaJitred bodies (WI and W 2 ), while the last integral
where dJ + and dl are add' . -. - is the energy of their interaction (W I2 ).
increase in the fi""eld b dIEtlO(nFa.1 dIsplacements due to an
The following impprtant circumstances should be men-
P"- = -- P+, we obtain Y Ig . 4 . 3) . ConSI' d '
ermg that tioned in connection with formula (4.14).
6..1 = p: (dl+ - dL). E ~= p~ dl. E,
1. The intrinsic energy of each charged body is an essen-
tially positive quantity. The total energy (1.9) is also always
where dl = dl + - dl is h . .
~he positive charges relative\ e t~ddItlon~1 displacement of positive. This can be readily seen from the fact that the in--
tegrand contains essentially positive quantities. However,
mg to (3.4) p' dl _ dP d o e negatIve charges. Aceord-
, + - an we get the energy of interaction can be either positive or ne~ative.
6A = E.dP. 2. The intrinsic energy of hod ies remains constant upon
Since P E (4.12) all possible displacements that do not. change the configu-
= xe o , we have ration of charges on each body, and consequently this energy
can be assumed to he an additive constant in the expression
oA =E'xe o dE= d ()(e~E2) = d (E~P). for the total energy W. In such cases, the changes in Ware
completely determined only by the changes in the interac-
Hence, the total work f tion energy W)2' In particular, this is just the mode of be-
unit volume of a dielectric ~~ent or the polarization of a
haviour of the energy of a system consisting of two point
charges upon a change in the distance between them.
. . A = E·P/2, (413) 3. Unlike vector E, the energy of the electric field is
wlndl coincides w'th th .
Thus, the volu~e enee:econd t~rm in formula (4.11). not an additive quantity, Le. the energy of a field E which
the energy eoE 2 /2 of the lY
densIty w = E· D/2 includes
associated with the 'p I . eld. proper and the energy E.P/2
is the sum of fields E) and E 2 is generally not equal to the
sum of the energies of these fields in view of the presence of
o aflzatlOn of the substance. the interaction energy lV12 . In particular, if E increases n
times everywhere, the energy of the field increases n2 times.
~
I;:

106
4. Energy oj Electric Field 4.5, Forces A cUng in a Dielectric 107

4.5. Forces ~cting in a Dielectric expense of a decrease in the electric energy W of the system
(or its field). Here we assume that these displacements do
Electr~striction. E~periments show that a dielectric in not cause the transformation of electric energy into other
an electnc field expenences the action of forces (sometimes kinds of energy. To be mOre precise, it is assumed that such
these forces are called ponderomotive). These forces appear transformations are negligibly small. Thus, for infinitesimal
when the diel.ectric is neutral as a whole. Ponderomotive displacements we can w.rite
forces appa.ar In the lo.ng run due to the action of a nonuni-
fo:m ~lec.tnc field on dIpole molecules of the polarized dielec- ~A= -dW\q, (4.15)
tnc ~It IS known that a dipole in a nonuniform electric where the symbol q emphasizes that the decrease in the
field IS acte~l upon by a force direcled towards the increasing energy of the system must be calculated ,when charges. on
field). In thIS case, the forces Ilre cansed by the nonuniformi- the conductors are constant.
~YI of not only ~he ~acroscopic fIeld hut the microscopic Equation (4.15) is the initial equation for determining
e d as well, whIch IS created mainly by the nearest mole- the forces acting on conductors and dielectrics in the electric
cules of the polarized dielectric.
field. This can be done as follows. Suppose that we are in-
. Unde~ t~e action of these electric forces, the polarized terested in the force acting on a given body (a conductor or
dI~le~tnc IS deformed. This phenomenon is called electro- a dielectric). Let us displace this body by an infinitesimal
stnctwn: As a r~sult of electrostriction, mechanical stresses distance dx in the direction X we are interested in. Then
appear In the dIelectric.
the work of the required force F over the distance dx is
( 0,wing to electrostriction, not only the electric force ~A = F xdx, -Where F x is the projection of the force F onto
WhlC~l depe;'1ds on the charges) acts on a conductor in a the positive direction of the X-axis, Substituting this ex-
polanzed dIelectric, but also an additional mechanical pression for ~A into (4. t'5) and dividing both parts of (4.15)
f~rce c.aused .by the dielectric. In the general case, the effect by dx, we obtain
o a dIelectrIc on. the resultant force acting on a conductor
cannot be taken Into account by any simple relations and
the ~I:oblem of calcul.ating the forces with simulta~eous (4.16)
aJlaJ~~Is of th,e mechamsm of their appearance is, as a rule,
rather complIcated. However, in many cases these forces We must pay attention to the following circumstance.
can .be clllcnlat:ed in a sufficienLly simple way without a It is well known that the force depends only on the position
~et~I1ed :malysls of their origin by using the law of conser- of bodies and on the distribution of charges at a given in-
vatIOn of energy.
stant. It cannot depend on how the energy process will deve-
Energy Method for Calculating Forces. This method is lope if the system starts to move under the action of forces.
~,he most general. It allows us to take into account automat- And this means that in order to calculate F x by formula
~cally' all force interactions (both electric and mechanical) (4.16), we do not have to select conditions un~er which all
IgiorIng thei~ origin, and hence leads to a correct result. the charges of the conductor are necessarIly constant
I e~ us consIder the essence of the energy method for cal- (q = const). We must simply find the increment dW under
c~ atmg forces. The simplest case corresponds to a situa- the condition that q = const, which is a purely mathemati-
tion when charged conductors are disconnected from the cal operation.
powe~ supply. In this case, the charges on the conductors It should be noted that if a displacement is performed at
rem?lll unchanged, and we may state that the work A of constant potential on the conductors, the corresponding
all mternnl forces of tII:e syst~m upon slow displacements calculation leads to another:expression for the force: Fx =
of the conductors and dIelectncs is done completely at the +
= aw/ax 11jO' However (and it is important!) the result

~
.,; !.~ ".' ".'
... :..L ...
•.'
. .' .;'
i08
4. Energy of Electric Field 4.5. Forces Acting in a Dielectric 109

of the. calculation of F:c with the help of this formula or on them) decrease by a factor of e:
(4.16) IS the same, as should be expected. Therefore, hence-
forth we shall confine ourselves to the application of only F = Fo/e. (4.17)
formula (4.16) and will use it for any conditions, including Hence it follows that two point charges ql and q2 separat-
those where q =1= const upon small displacements. We must ed by a distance r and placed into an infinite liquid or gas-
not be confus.ed: the derivative aw/ox will be cakulated eous dielectric interact with the force
at q = const 10 such cases as well.
F --
-- _1 -~,
4ne o er 2
(4.18)
Example: Fin? the !orce acting on one of the plates of a II 1-
plate ~apaCltor III a .hquid dielectric, jf the distance het~a the
an
:te
~lateJ h , tre capac~tance. of !he capacitor under given conditions is
vo tage U IS malOtamed across its plates.
which is smaller than the force in a vacuum by a factor of
e. This formula expresses the Coulomb law for point charges
II?- this case, if we mentally move the plates ap1l.rl the volta e U in an infinite dielectric.
full:~~f:::h~~el~~~: ~h~c;~f~I~ ~;i::~/tli:~i~~sh:iin::;j:~ We must pay a special attention to the fact that point
charges in this law are extraneous charges concentrated on
X ~e ~orce u;nder t.he assumption that q = macroscopic bodies whose dimensions are small in comparison
- const, I.e. WIth the help of formula
x'+dx (1..16). Here the mo.st convenient expres- with the distance between them. Th'.1s, the law (4.18) has,
-q---- -----
x
Sion for the energy of the capacitor is unlike the Coulomb law in vacuum, a very narrow field
c F _ ~ ~
of applicationi!<-the dielectric must be homogeneous, infinite,
+q----<J--__ W- -n- CC= _ _ :r liquid or gaseous, while the interacting bodies must be
Q "toG 2ee oS '
pointlike in the macros~opic sense.
Fig. 4.4 w~ere ~ is the permittivity of the dielec- I t is interesting to note that the electric field intensity
triC, ~ IS the area of each plate, and :r is E as well as the· forceF acting on a point charge q in a
th~ .dlsta;llce ~etween them (x = h).
. ~ext, let us cho~se the posItIve dIrection of the X -axis as is sho\vn h~mogeneous liquid or gaseous dielectric filling the entire
In Fthlg. 4.4. :'-CCO~dlllg to (4.16), the force acting on the upper plate space of the field, are by a factor of e smaller than the values
of e capaci tor IS
of Eo and F 0 in the absence of dielectric. This means that
F,,=- oW
ax
I=--~
q 2ee oS •
(1) the force F acting on the point charge q in this case is de-
termined by the same formula as in vacuum:
The minus sign i!I this formula indicates that vector F is directed F = qE, (4.19)
~owards the neg~tIv~ values on the X-axis, Le. the force is attractive
y nature. Consldenng that q =
we transform (1) to
as = Ds = ee ES and E = Ulh
0 ,
where E is the field intensity in the dielectric at the point
where the extraneous charge q is placed. Only in this case
F" = -GUt/2h. formula (4.19) makes it possible to determine the field E
in the dielectric from the known force F. It should be noted
Forces in a Liquid Dieleetric. Formula (1) of the last that Gnother field differing from the field in the dielectric
example shows that the force of interaction between the will be acting on the extraneous charge itself (which is
plates of a parallel-plate capacitor in a liquid dielectric located on some small body). Nevertheless, formula (4.19)
IS smaller t~an the corresponding force in a vacuum by a gives the correct result, strange as it may seem..
factor of e (Ill vacuum e = 1). Experiment shows that this Surface Density of a Force. We shall be speakmg of the
resul~ can be gener~liz~d: if the entire space occupied by a force acting on a unit surface area of a charged conductor
~eld IS ?lled by a lIqUId or gaseous dielectric, the forces of in a liquid or gaseous dielectric. For this. pu.rpos?, let .us
mteractlOn between charged conductors (at constant charaes consider a parallel-plate capacitor in a hqUld dielectnc.
'"
110 4. Energy of Electric Field Problems 111

Suppose that the capacitor is charged and then disconnected S l t' In accordance with (4.6), the intrinsic energy of each
h It u e/qo:~l to qm/2 where <F is the potential of a shell created o!lly
• '0 R ' .where f{ is the shell rad1U~.
from the power supply to maintain the charge and the field e c harge q on'r I't', I' .e. (p -- q/4Jt£
sbye t hiS
E of the capacitor constant when the plates are moved apart. Thus, the intrinsic energy of each shell IS
Let us consider once again Fig. 4.4. The energy of the
capacitor is the energy of the field within it. In accordance 1 q:. 2

with (4.9), this energy is W = (1/2) EDSx, where S is the W 12 =; 4..-1:£0 2R I- : ; '
surface area of each plate and x is the distance between them The energy of interaction between the charged shells is equhl tt: the
(Sx is the volume occupied by the field). By formula (4.16), char e of one shell multiplied by the po.tential <F created b~ t e c_arge
the force acting on the upper plate is of th~ Zther shell in the point of locatIOn of the charge q. W I2 . - q(p.
In our case (R 2 > R), we have
1
Fx=c -8W/8x l q = -"2 EDS, 1 q2 1 q I q2
W 12 ~~ qt 4n£0 R z =--= 4)1£0 ~.
whence the surface density of the force is
The total electric energy of the system is

W=WI+Wz-j-It'IZ= 4ne
1 (qI '
-2H t·
'1~
28
: qlqz)
R •
(4.20) o t 2
'
z

We have obtained an interesting and important result • 4 3 Two small metallic balls of radii R I and H 2 are in vacuum
at a di~t~nce cl'1hsiderably exceeding their dimensions and ha;e I a
of a general nature (for a liquid or gaseous dielectric). It certain total charge. Find the ratio ql /q2. bet:-v~en the chafl~es o. t l~
turns out that the surface density of the force acting on a balls at which the energy of.the sys.tem ~s. mlmmaI. What IS the po
conductor is equal to the volume density of the electric tential difference between the balls III thiS case?
energy near the surface. This force is directed always out- Solutiun. The elecLric energy of this system is
ward along the normal to the surface of the conductor (tend-
ing to stretch it) regardless of the sign of the surface charge.
1
W=WI-IWz-!-W IZ = 4ne
(qI -iL1_qlqZ)
2R -+- 2H 2 ' l '
o I

where Wand Ware the intrinsic electric energies of the b~IIs ~qljJ~~~,
Problems W is th~ energ) of their interaction (ql<F2 or qz<FI),. anhd l 11;-
ta~~e between the balls. Since q2 =, q - ql' where q IS t e to a c arge
IU h
• 4.1. Energy of interaction. A point charge q is at a distance l of the system, we have
from an infinite conducting plane. Find the energy W of interaction
between this charge and the charges induced on the plane.
Solution. Let us mentally "freeze" the charge distributed over the
W _ _1_
_. 4neo
[-..1L
2R I
+- (q_q,)2 -+-
2H 2
ql (qL-qtl
l
J.
plane and displace under these conditions the point charge q to in-
finity. In this case the charge q will move in the potential field which The energy W is minimal when aWlaql ,= O. IIence
is 'equivalent to the field of a fixed fictitious point charge __q, located R Rz
I
at a fixed distance l on the other side of the plane. We can write q ~q and qz""'q RI.J.R '
straightaway I~ Rt+R z I z
1
W = - -_ q2
_ where we took into account that R I and R 2 are consideraIlly smaller
4..-1:80 'U' than land
• 4.2. Intrinsic, mutual, and total energies. A system consists q\lq2 = HI/Hzo
of two concentric metallic shells of radii R I and R 2 with charges ql
and q2 respectively. Find intrinsic energies WI and It' 2 of each shell, The potential of each hall (lIwy can he. considered isob ted) ,is (p ~ q!,R:
Hence it follows (rolll tlip ahove I'dallOll that '.I' . : qe, I.e, lhe I'0ll n
l2
the energy W of interaction between the shells, and the total elec-
tric energy 0 the system W, if R 2 > R I • tial difference is ('qllal to zp['o lor SIlI·,II a d,sll'lh"llOlI. ,

I',
112 4. Energy of Electric Field Problems 113

• 4.4. Energy localization in the field. A charge q is uniformly As a result of integration, we obtain
distrtbuCed inside a sphere of radius R. Assuming that the dielectric
constant is equal to unity everywhere, find the intrinsic electric energy ..1-- q(qo+q/2) (_1 1_)
of the sphere and the ratio of the energy Wt localized inside the sphere -- 4JH'o RI R z .•
to the energy W z in the surrounding space. Remark. If we try to calculate the work in terms o[ the potential
Solution. Let us first find the fields inside and outside the sphere as A = q (<PI - <P2), where <p is the potential createll by the ch.argo q(l
with the help of the Gauss theorem: at the point of location of the charge q, the answer would be (II [ferent
. q 1
Et = 4 q 3 r (r ~ R), E z =-4---- (r;?R).
neoR neo r Z

We can now calculate the intrinsic electric energy of the sphere:


a 00

W=Wt+Wz=) e~Ef 4nrz dr +) e~E~ 4nr 3 dr= 8n~:R (--}+1).


o a
Hence, it follows that Fig. 4.6
Fig. 4.5
1 3q z
W=----
4neo 5R ' and incorrect. T~ is due to the fact that this approac~ does not take
into account the additional work pNformerl by electnc forces upon
the change in the configuratiOI) of the charge q located on the l'Xpand-
It is interesting to note that the ratio W/W z does not depend on ing shell.
the radios of the sphere. • ~.6. A point charge q is at the centr~. of a sphlfical unch~rged
• 4.5. A spherical shell is uniformly charged by a charge q. conducting layer whose inner .and oute~ radl.l are a and b respectl vcl~ .
A point charge qp is at its centre. Find the work of electric forces upon Find the work done by electnc forces III thiS system upon the remo\-
the expansion of the shell from radius R t to R z• al of the charge q from its original position through a small hole
(Fig. 4.6) to a very large distance from the spherical layer.
Solution. The work of electric forces ia equal to the decrease in
Solution. We shall proceed from the fact that the work of electric
the electric energy of the system: forces is equal to the decrease in. the .electric ene.rgy of the system. As
is well known, the latter is locahzed III the field.ltself. ',fhus, the prob-
A = Wt - W2• lem is reduced to determination of the change III the lleld as a result
of this process. . .
In order to find the difference WI - Wz, we note that upon the expan- It can be easily seen that the field around the charge q wtll change
sion of the shell (Fig. 4.5), the electric field, and hence the energy only within the spherical layer with the inner radius a and the outer
localized in it, changed only in the hatched spherical layer. Conse- radius b. Indeed, in the initial positiono.f .the charg~; there ~as no
quently, field in this region, while in the fmal. POSI twn there IS a certalll field

W t - W z=
rJ T
Z
eo
(Ei- En 4m z dr,
(since the conducting spherical layer IS far from the charge q). Conse-
quently, the required work is
b
V
at A=O-Wl= - J\' -EoEz
2- d .
where E t and E z are the field intensities (in the hatched region at a a
distance r from the centre of the system) before and after the expan-
sion of the shell. By using the Gauss theorem, we find Considering that R = q/4nEor2 and dV = 4m 2 dr, and integrating,
we obtain
E =_1_ q+qo ~ _ _1_ k qZ a-b
I 4ne o rZ 2- 4neo r2 ' , 1 = - - - - <n.
. 8neo IIlJ

8-0181
H4 . 4_,_Energy 01 Electric Field Problems H5

• 4.7. The work done upon moving capacitor plates apart pl~l'manent voltage U. F.inu the force /' acting on a unit surface of the
A parallel-plate air capacitor has the plates of area S eacb. Find th~ plate from the dielectrIC.
work A' against the electric forces, done to increase the di8tance bc-
t,we~n the plates from x 1.to ~2' if (1) the charge q of the capacitor and Sol~tion. The resultant force / acting per unit area of each plate
(2) It~ voltage U ,fire mallltallled constant. Find the increml'nt.."l of thl; can be represented as
elpclnc energy or the capacitor in ,he two cases.
1=/o-/', (1 )
Snlution. (1) The required work is
where /0 is the electric force acting per,unit area of a plate from the
other plate (it is just the force per umt area when the dielectric is
absent). In our case, we have
I = lole, 10 = uE = f.12/2eo, (2)

whpl'l', A\ is th~ intensity of the l1eld createJ by one plate (E = u/2£.0). where E is the field intensity in the region occu~ied.by one plate, creat-
,It !S In tillS, fJeld that the charge located on the other plate moves. ed by the charges of the other plate. ConsJderwg that a c= D =
fillS work IS completely spent for increasing the f'lectric energy'
= eeoUlh and substituting (2) into (1), we obtain
AW-- A', .
/' = 10 (1 - lie) = 10 (10 - 1) f.oU2/2h 2•
(2) III this case, the force acting on each capacitor pJat~ will de-
(lpnd on the, oistanc~ between the plat!'s. Let us write the elementary For example, for U = 500 V, h = 1.0 mm and 10 = 81 (water), we get
work 01 the force actlllg on a pla1.l~ during its displacement over a dis- l' = 7 kPa (0.07 atm).
tane(' d.1' relative to the other plate:
• 4.9. The force ~cti~g on a dielect~ic. A cylindri~,al,layer of a
homogeneous di~lectrlc. WIth the dielectrIC c~nstant c IS wtro,duced
IiA'= qE1.dx= eoSU~~ into a cylindridH capaCItor so that the layer hll~ the gap of WIdth d
2 x2 '
between the plates. The mean radius of the plates IS Il sHch that R ~ cl.
The capacitor is connect~d tp ~ s~ur~e of a perman.ent voltage U. Fwu
the force pulling the dlelectl'lc wSlde the capaCItor.
where we took into account that q = eu, £1 = U!2;r, and C c= eoS/x.
Aft,,!, integration, we obtain Solution. Using the formula W = q2!2C for thf) energy of a capac-
itor, we fmd that, in accordance with (4.10), the required force is

Fx = - a;: Iq = ~ O~2()X =~~ ~~ . (1)

Since d ,,<:: R the capacitance of the given capacitor can b~ calcul~ted


The ill('J'eIlH'lIt of thp el"('lrie energy of the capacitor is by the formula for a paraIJel-plate capacitor.. Therefore, .If the diele-
ctric is introducer! to a depth x ann the capacitor length IS l, we have

Cc= f.f.ox~2nR + Eo (l-;>.2nR eo.~nR (ex-j-l-x), (2)

It should be noted that AW = -A', Substituting (2) into (1), we obtain


~hus, by movi,ng the plates apart, we perform a positive work
(a~amst the e!eetI'lc forces), The energy of the rapacitor deaeases in Fx = eo (e - 1) nRU2!d.
tillS case. III ordrl' to understand this, we musl ronsider a SOJ'rce main-
!?i!ling the potentwl diff?rence of the capacitor at a constant value. • 4.10. A capacitor consists of two fixed plates in the form of a
J hiS source also accomplIshes the work A S' According to the law of semicirck of radius R and a movable plate of thiekness h made of a
dielectric with the dielectric constant 10, placed between them. The
conserva,tion ,')f energy, A
= ---2.'1 ~__ 0, + A' = AlV, whence As = A W _ A' = latter plate can freely rotate about. th(' axis 0 (Fig. 4.7) and practica~­
ly fills the entire gap between the l1xed plat!~s. A constant voltage f! IS
(It !j .8. ro~ccs ~ct~!I~ betwee~ conductors in a dieledric. A paral- maintained between the plates. Find the moment M about the aXIs 0
l~l-pl~te capacJ~or IS Im!!H'rsed, 111 the horizontal position, into a liq- of forces ading 011 the movable pIa te wllPn it is placed as shown in the
Uid !helectr1c WJth the flIelectric constant 8, filling the gap of width h figure.
between the plates. Then thf' capacitor is f'onnected to a SOur('(' of SolutitJrt. The work performed by the moment of forces M upon
8*
116
5. Direct Current
5.1. Current Density. Continuity Equation 1t7
thc rotation of thc platc through an ". I ., I .'
tit'crcnsc in the electric cnergy of the ..gtc"e ctlllcnt da IS equal to thc is zero. However, when an electric field is applied, an orde-
. sys em a q = const [see (4.16»):
_'\;/ z da= -dWl q , red motion with a certain average veloeity u is imposed on
where W = q2 /2C. Hence the chaotic motion of the carriers. and a current flows
through the surface S. Thus, an electric current is essentially
M z = - oW /. =L~
iJa q 2 C2 • (1) an ordered transfer of electric charges.
In the case under consideration, C = C + C h The quantitative measure of electric current is intensity
dr:l~~frfcap~~tances of the parts ?f the cap~citor ~ith a~d ;;\h~~~ tf: I, defined as the charge transferred across the surface S in
. e area 0 a sector wl~.h an angle a is determined as S = a unit time:
= aR2'12, and hence
1= dQ/dt.
C =~ EoaR2/2h + EEo (n - a) R2/2h.
Differentiating with respect to a we find The unit of current is the ampere (A).
ac,liJa = (EoR2/2h) (1-E). SUbstitutin Current Density. Electric. current may be distributed non-
t~IS ~xpression into formula (1) and con~ uniformly over the surface through which it passes. Hence,
sIderlllg that C = qlU, we obtain in order to characterize the current in greater detail, current
U2 eoRI density 'vector j is introduced. The magnitude of this vector
M Z =-2- 2h (1-e) is equal to the ratio of the current dI through a surface
Fig. 4.7 element perpen,dicular to the direction of motion of charge
= -(8-1) 8oR2U2 < 0
'4h . carriers to th€" area dS J of this element: j =ceo dI/dS J. For
The ncgative sign of M indicat th h the direction of vector j we take the direction of velocity
acting clockwise (oppositelyZ to the eps 't~t t de. mOI?ent of the force is
see Fig 4 7) Th' , OSI Ive IrectlOn of the angle a' vector u of the ordered'motion of positive carriers (or the
pacitor: " IS moment tends to pull the dielectric inside the ca~ direction opposite to that of the velocity vector of the or-
~n :~;i~fb;i~~~ ~~:~ ~I~ fh:n~ependent of~he angl~
AJz a .. However,
dered motion of negative carriers). If the current consists of
positive and negative charges, its density is derIDed by the
IS due to the fact that for smail vaIu oment M Z - o..ThIS dIscrepancy
a
as was done in the solut:on of tb' es of we cannot Ignore edge eflects formula
, ' I S pro b l em.
i = P+U+ p_u_,+ (5.1)
5. Direct Current where p+ and p_ are the volume densities of positive and
5.1. Current Density. ContinUity Equation negative charge carriers respectively, and u + and u_ are the
velocities of their ordered motion. In conductors,where the
Electric Current. In this chapter we sb~ll co ~ charge is carried only by electrons (p_ < 0 and" u+ = 0),
selves to l' f .,. nlme our-
d' . an a~a YSI~ 0 conduct JOn CUl'l'ent in a conducting the current density is given by
me HIm, espeCIally m metals. I t is well known that ele t . j = p_u_. (5.2)
~ur(rent iSh the transfe~ of charge through a certain sU~f:~~
say, t e cro~s sectJOn of a conductor). The field of vector i can be graphically represented wnh
In a. conductmg medium, current can be c . d b the help of lines of current (lines of vector j), which are
trons/Ulhmetals), ions (in electrolytes), or sOr:;~fl:the/p:~~~= drawn in the same way as for vector E.
~l~as~ti~ ~ \ ~bsenc~ of electric field, curren t carriers perform Knowing the current density vector at each point of the
f' 0. JOn all( on average the some number of carriers surface S under consideration, we can also fLlld the C'Irrent
~ur eIther sign passes tllrough each side of any imaginal' passing through th is fiurfflce f1S the f1nx of vector j:
face S. Thus, the current passing through S in this 'cas~
I = J j·dS, (d)
118
5. Direct Current
5.2. Ohm's Law for a Homogeneous Condnctor 1 t!J
. Current I is a scalar al ebra" --
from formula (5.a) that b~side;co{luanttty . It can be seen Procl'edin~ in the same way as for the flux of vector E in Sec. 1.4, we
the current is determined b th 1 I~r factors, the sign of, lind that the divergence of vector j at a certain point is equal to the
the normal at each point 0 YtJ e cflolce of the direction of decrease in the charge density per unit time at the same point:
of tlI e d"ll"eetlOn of vectorsn dSlie Sur
If hace S. "
i e: b Y tl Ie ch'
vectors dS are reversed th . t e dIrectIOns of all the
Olce
r V·j= -ap/at. I (5.6)
Continuitv Equation' L e curr~nt I changes sign.
. .J' • e t us Imagine a I se rl This lcatls to thc steady-state condition (when iJp/DI = 0):
III ? conllucting medium throu ,h wI . c.o . sUl'face S
It IS customary to choose the ~ut nch a. cUl~ent IS passing. I V·j=o.j (5.7)
normal to closed surfaces I h ward dIrectIOn for vectors
, aD( ence for ve t dS . This Illeam; that Ior dircct current the field of vector j does not have
sequently, the integralh . d .,' COl'S. Con-
sources.
the volume V (' I J'] S gl\ es the charge leaving
A enve oped by the su f S)
. ccording to the law of 'conse t' I' ace per unit time. 5.2. Ohm's Law for a Homogeneous Conductor
IS ?qu~l to the decrease in tl rva ~o~ of .ch?rge, this integral
unIt tIme: 18 c alge IllSHle volume V per Obm's law, which was discovered experimentally, states:
the current passing through a lwmogeneous conductor is pro-

I~idL -*.) (.5.4)


portional to the potential ditJerence across its terminals (0['
to the l.101taf'!.' .ftfr
l)
T

,.

Tllis relation is called the


pression for the law of
'.
. .contl~ulty equation. I t is an ex-
JI=UiR, l (5.8)
In the case of a t dcollservatI~n of charge. where Jl is the electric resistance of the conductor.
· t 'b
d IS s ea v-state (du'CCt) The unit of resistance is the ohm (Q).
n lItion in Sp'lce . 't ." current, the charge
W I' I 'C mu~ remalll unch I I Resistance R depends on the shape and size of the conduct-
. Of( S, 011 t IS right-hand side of E allge(. II other
sequently, for direct C\lr' t I q. (5.4) dq/dt = O. COIl- or, on its material and temperature, and above all on the
, " Tell we lave. confIguration (distribution) of the current through the con-
~j dS= O.
(5.5)
ductor. The meaning of resistance is quite clear when we
are dealing with a wire. In a more general case of the volume
This means that in this . distribution of current, it is meaningless to speak of re-
start or terminate any' I case, the lInes of vector j tlo not sistance until we know either the position of the leads at-
to have no sources in tl:-'" ~ere. The. fteld of vector j is said tached to the eonductor or the current configuration.
,e case of dIrect current.
In the simplest case of a homogeneous eylindrical eOll-
5 ' Differential form or the conti . t ductor, resistance is given by
( JI) and (5.5) in the diITt'n'ntial forn~ul~ y ell~~ation. Let us write Eq~.
. j' .
or t liS purpose, we express the
ch Ulo'e
__
...
q u.s p d V
JiJ
.
J I
aUf tit' right··hand ~ide of Eq (5 4)" d J .
' . as --at P dl ~,
' R = Ps'
l
(:i.!l)

-- -- P d I' J where l is the length of the conductor, S is its cross-sectional


. 1ii'. "lere, we have taken the partial time derivative of area, and p is the resistivity, which depends on the material
SineI' r can dpPl'llll 011 lim. , . P.
C .IS wl'I1 as on coordinatp< TI
c HIS,
'-0
of the conductor and its temperature. Resistivity is measured
in ohm-metres (Q·m).
(r,;Y j ciS - _ I'
J
-.±
at
(,Iii
. Resistivity of the best conductors like copper and alumi-
nium is oC_the order of 10-8 Q·m at room temperature.
121
5.2. Ohm's Law for a Homogeneous Conductor
120 5. Direct Current
in the form (5.10) we can rewrite (5.5) as follows:
Differential Form of Ohl:!.'s Law. Let us establish a re-
lation between the current density j and the field E at a l~ aEdS=O,
point of a conducting medium. We shall confine ourselves u
to an isotropic conductor in which the directions of vectors where the integration is performed over any closed surface
j and E coincide. S inside the conductor. For a homogeneous conductor, the
Le~ us me~tally iso!ate. around a certain point in a con- quantity a can be taken out of the integral:
ductIng medIUm a cylIndrICal volume element with .genera-
trices parallel to vector j, and hence to vector E. If the cylind- a~ EdS=O. '
er has a cr?ss-sectional area dS and length dl, we can write
on the basIs of (5.8) and (5.9) the following expression for According to the Gauss theorem, the remaining i?te.gral is
such cylindrical element: proportional to the algebraic sum of the charges mSlde the
closed surface S, Le. proportional to
]'dS = E dl
pdlldS • the excess charge in this surface. How- En
ever it can be directly seen from the
After cancelling out common terms we obtain the following last' equation that this integral is
equation in vector form: equal to zero (since a =1= 0). This means

Ij~fE~.E, I (5.10)
that the excejEl charge is also equal to
. zero. Since the surface S is chosen ar-
bitrarily, the excess char,ge 'under th~se
conditions is always equal to zero m-
= side a conductor. ,
wh?re a 1/p is the conductivity of the medium. The unit The excess charge can appear only Fi~. 5.1
reClpro~al to an ohm is called a mho (Siemens). Consequently,
the UOlt .for the measurement of a is mho per metre. at the surface of a homogeneous con-
ductor at places where it comes in C,I}lildlL with other
EquatIOn (5.10) is the differential form' of Ohm's law conductors, as well as in the regions wher8 the conductor
It does not .contai~ any diff~rentials (derivatives), but i~
called the dIfferentIal form SInce it establishes a relation has inhomogeneities.
Electric Field of a Current-carryillgConductor. Thus,
between quantities corresponding to the same point in a when current flows, an excess charge appears on the surf~ce
conductor. In other words, Eq. (5.10) is an expression for of a conductor (inhomogeneity region). In accordance WIth
Ohm's law at a point. (2.2), this means that the normal component of vector E
Methods for Calculating Resistance (R). There are several appears at the outer surface of the conductor .. Further, from
methods for measuring resistance and all of them are ulti- the continuity of the tangential component ?f vector E,
mately ba~ed on the application of relations (5.8)-(5.10). we conclude that a tangential component of thIS vector also.
The expedIence of using a particular method is determined exists near the surface of the conductor. Thus, near the. sur-
by the formulation of the problem and by its symmetry. face of the conductor, vector E forms (in the ~resence ofthe
The practical applications of these methods are described in current) a nonzero angle (J. with the vertical (FIg. 5.1). When
Problems 5.1-5.3 and 5.6.
.Charge in a Current-carrying Conductor. If the current is the current is absent a = O. . . .
If the current flows in a steady state, the dlstl'lbutlOn
dlrect (constant) the excess charge inside a homogeneous of electric eharges in the conducting medium (generally
~Qndu~tor is ~qual to zero everywhere. Indeed, Eq. (5.5) speaking, inhomogeneous) does not change in time, although
IS valId for dIrect current. Taking into account Ohm's law
122 5. Direct Current 5.3. Generalized Oh!!'.m~·s~L~a~w~ -_-1_2~3
-----------
the charges are in motion: at each point, new charges con- cells, accumulators) or eondllctors at different temperatures
tinuously replace the departing ones. These moving charges (thermocouples). .
create a Coulomb field which is identieal to tbe one created · d Ohm's Law In order to deSCrIbe extraneous
GIenera Il z e · f e fJ el d
by stationary charges of the same configuration. This means forces quantitatively, the concepts of extrane?us orc. I .
that the electric fIeld of stationary currents is a potential and its strength E* are introduced. The lat~er IS .n~mencallY
field. equal to the extraneous foree acting on a umt posltl\:e C~I~~~
At the same time, electric fIeld in the case of stationary Let us now turn to current density. If an ~lec~r~ e it
currents differs sig-nineantly from electrostatic, or Coulomb, generates in a conductor a current of d.enslty l - oE, E
field of fixed charges. If charges are in equilibrium, the is obvious that under the combined actlOn of the, field '11
electrostatic fwld inside a conductor is always equal to zero. and the extraneous force field E*, the current denSIty WI
The electric field of stationary currents is also an electro- be given by
static (Coulomb) field, although the charges inducing this
field are in motion. Hence, the field E of steady-state currents
exists also inside a current-carrying conductor.
t j=o(E+E*). \ (;).11)

This equation generalizes the law ~5.10) to, the ~ase ~;


inhomogeneolls regions of a conductlllg medI~lm, and
5.3. Generalized Ohm's Law
called the generalized Ohm's law in the ~iffe~entIaI fOfl~l.
Exlrant'ous Forces. If all the forces acting all charge Ohm's La,,' for a Nonun iform Subcll'cml.. Nonumform
curriers wcre reduced to electrostatic forces, the positive subcil'cuits are characterized by the presence of extraneOIlS
carriers would move under the action of these forces from forces in t h e m , ' ,., t t
regions with a higher potential to those with a lower poten- Let us consider a special, but practICally qUIte.lmpor an
tial, while the negative carriers would move in the opposite case when a current flows along a thin wire. In thIS ~ase, t~Ie
direction. TIds would lead to the equalization of potentials, d'il'ec.tion of the current will coincide with the aXIs of t 10
and the potentials of all mutually connected conductors wire and the current density j can be assum~d to be th~ sa~~
would become the same, thus terminating the current. In at all points on the cross section of the WIre, Suppose th ..
other words, lmder the action of Coulomb forces alone, a ~he cross-sectional area of the wire is equal t~ 8 (8 need not
stationary licld must be a static field. be the same over the entire length of the WIre): _1- .
In order to prevent this, a direct CUl'l'cut circuit must Dividing- both sides of Eq. (5.1~) by ~, formIIlg a SC,l HI '.
contain, besides the subcircuits in whieh positive carriers prodnc.t of the resulting expresSIOn ":Ith the element. dl
move in the direction of deereasing potential q;, subcircuits of the wire along its axis from erosS s~c.tIOn .~ to ~ross sect I?"
ill whieh positive carriers move ill the direction of increas- 2 (this direction is taken as the P?SItlve dIrectIOn) and r~~~
ing If' or against the electric field forces. In these subcircuits, tegrating over the length of the WIre between the two c c'
the transfer of carriers is possible only through forces that sections, we obtain
are not electrostatic. vVe shall call such forces extraneous. 2 2 2
Thus, a direct current can be sustained only witlt the \1:1=~Edl+~E*dI. (;).12)
help of extraneous forces whicu are applied either to cer- j 1 1
tain parts of the cirellit, or to the entire eircuit. The
physieal naturo of oxl I'illH'OliS forcos may be ql1ite diverse. Let lIS transform the integrand in the, fl.rst integr,al: .we
Snell fOl'ecs may bo ('!"pated by physieal or chemical inho- re.p]oc(' (J hv iff) and J. ill by jidl, where JI IS the pl'OJectIOll.
.. ~ . .' " . . II F·t! we IIOt('
mo,gclloili(~s ill a ('olldllclor, rot· O\,llllplc, roreps l'esllltin,g of vee tor j onto the (hrcdlOII of vecLor {, III \Cr, .' .~
from a cO/llact ol" rondllctors or different types (galvanic that jt is an algebraic quantity and depends OIl the Ollenta
124 5. Direct Current 5.3. Generalized Ohm's Law 125

tion of vector j relative to dl: if j tt dl then j 1 > 0; if j # dI, I


Since the potential decreases over the segment R fr0l!! .left t? rig~t,
>
O. This means that the current flows in the posItIve dIrectIOn
then j 1 < O. Finally, we replace j 1 by liS, where I the (from 1 to 2). In the present case! Ipl < .lp2' but th~ curre~t .f1ows f~om
current, is also an algebraic quantity (like j I)' Since' I is point 1 to point 2, Le. towards .Ific~easII~g P?te~tlal. ThIs IS p~ssIb~e
the same in all sections of the circuit when we are dealing only becausp.an e.mJ. ,g is actIng Ifi this CIrcUIt from 1 to 2, I.e. m
with direct current, this quantity can be taken out of the the' positive direction.
integral. Cosequently, we get
Let us consider Eq. (5.15). It follows from thi~ eq.uat~on
2 2
that points 1 and 2 are identic.al for a . closed . cIrcUIt, I.e:
fjdl=If ~
J (J .IPs· (5.13) «PI = «PI' In this case, the equatwn acqUIres a sImpler form.
1 1
RI = ~, (5.16)
The expression P dll S defines the resistance of the sub-
circuit of length dl, while the integral of this expression is where R is the total resistance of the closed cir~uit, and ~
the total resistance R of the circuit between cross sections is the algebraic sum of all the .e.m:f. 's in t?~ circuit. .
1 and 2. Next we consider the subClrcUIt contallllllg the source
Let us now consider the right-hand side of (5.12). Here, of the ~.mJ. between terminals 1 and 2. Then, R in (5.15)
will be the internal resistance of the e.m.f. source in t.he
'0 R the first integral is the poteo- subcircuit under consideration, while 'PI - 'P2 the potential
o---::.j~ . tial difference 'PI - 'Pz, while difference acro~ its terminals. If the source is disconnected,
1 2 the second integral is the elec- I '-= 0 and l= 'P2 - 'Pl' In other words, the e.m,f. of

r£~
tromotive force (e.mJ.) ~, act- the source can be defmec! as the potential difference across
"'2 iog in the given subcircuit: its terminals in the open circuit.
~ 2 The potential difference across the terminals of an e.m.f.
~t2 = ) E*dl. (5.14) source connected to an external resistance is always less
1 2 1 than its e.m.f. and depends on the load.
Fig. 5.2 Like the current I, the electro- Example. The external resistance of ~ circuit is TJ times higher th,an
.. . .. motive force is an algebraic the internal resistance of the source. Fmd the ratIO of the potential
quantity: If e.m.f. facIlItates the motion of positive carriers dillerence across the terminals of the source to its e.m.f.
in a certain direction, then ifIll> 0; if, the e.m.f. hinders Let R be the internal resistance of the source and R a the external
this motion t"12 < O. resistance' of the circuit. According to (5, i5), '!P2 - !PI = ,g - R,! I
As a result of all the transformations described above
while according to (5.16), (R j + R a ) ! =,g. From these two equa-
tions, we get
(5.12) assumes the follOWing form: '
1p2-Ip, = 1- Ri! = 1- Rj Ra TI
IRI='Pl-'P2+~t2' , (5.15) If r!J Rd-R a Ri+R a = 1+TI •

This equation is the integral form of Ohm's law for a n01t- It follows from here that the higher the value of TI, the closer the ~o­
tential difl'crence across the current terminals to its e,m.f., and VIce
uniform subcircuit [ef. Eq. (5.11) which describes the same versa.
law in the differential formJ. .
. Example. Co~sider the subcircuit shown in Fig. 5.2. The resistance Concludin a this section, let us consider a picture illus-
IS nonz?ro. only. III the .segment H. The lower part of the tigure shows trating the 'how of direct current in a closed circuit. Fig-
the variatIOn of potential !P over this region. Let us analyse the course
of events in this circuit. . ure 5.:3 shows the distribution of potential along a closed
126 5. Direct Current

circuit containing the e.m.f. source on segment AB. For III order to pl'OVl' Ihi~ law. il i~ ~ul'l'J('il'ul to eOllsidl'" the
the sake of clarity of representation, potential <p is plotted C,lse when lhe circuil cOllsists of three su!lcireuils (Fig. :i.:l)_
'P along the generatrices of a cylindri- LeI liS assunH' Ihe dirl'dioll of cil'CllmVl'lIlioll to he clockwise,
A cal surface resting on the current con-
tour. Points A and B correspond to
the positive and negative terminals
of the SOUice. It can be seen from the
figure that the flow of current can be
described as follows: the positive
charge carriers "slide" along the in-
· 5 .3
F19. dined "chute" from point lOA
't'
to point
<P B along the external suhcircuit
while inside the source, carriers "rise" from point qJ B to qJ , Fig. 5.4 Fig-. 5.;>
with the help of extraneous force shown by an arrow. A
as shown ill lhe figure. Applying Ohm's Jaw (;j.1S) 10 cach
of the three suhcircuits. we obtain
11 R I = qJ2-- cps + 'f l ,
5.4. Branched Circuits. Kirchhotrs Laws 12 Hz = cps - CfJI -t- 'f 2 ,
Calculations of branched circuits, for example, determina-
tion of current in individual branches, can be considerably
I sRI. = qJl - IP2 + '(s·
simplified by using the following two Kirchhoff's laws. Adding these equations and cancelling all potentials, we
Kirchhoff's Fircit Law pertains to the junctions, Le. arrive at formula (5.18), i.e. Kirchhoff's second law.
branch points in a circuit, and states that the algebraic Thus, Eq. (5.18) is a consequence of Ohm's law for 1l011-
sum of the currents meeting at a junction is equal to zero: lIniforIll subcireuils.
Setting up of a System of Equations. In each specifIC case,
~Ik = O. (5.17) 1\ irchhoff's laws lead 10 a complete syslem of alg-cbrait:
~fere, currents converging at a junction and diverging from equaliollswhich call he used, say, for l'ilHlilig all the llll-
It are supposed to have opposite signs; for example, we can known currents in the circllit.
assu~e the forme~ t? ~e positi~e and .the latter. to be negative The number of equations of the form (:'>.17) and (5.18)
(or VIce versa, tillS IS ImmaterIal): When applIed to Fig. 5.4, Illust he equal to the Ilumber of unknown quan! ilies. Care
Eq. (5.17) assumes the form II _.- 1 2 + 1 3 0-== O. should be taken to ensure that Ilone of the equiltions is iI
Equation (5.17) is a consequence of the steady-state con- corollary of allY olher equalioll ill lhe system:
a
dition (5.7); otherwise, the charge at junction would cllilnge (1) if a branchell circuit has N jllnctiolls, illdejwllt!I'nt
equations of lype (5.17) can he set up only for N 1 JUIII'-
and the currents would not be in a steady state.
Kirchhoff's Second Law. This law is applicahle to any lions, and the equation for lhe last jUliet ion will he iI r'ornl-
closed contour in a brandied eirenit dnd states that the lary of tile preceding equations;
algebraic sum of the products of cwnfit and resistance in each (:2) if a D!'i111clied cirelli! containC' seve!'al ell'S!"! Inops.
part of a network is equal tv the algebraic sum of e.m.f. 's in tlw indepc'nl[pill eqllil1ions o[ t~'pe C,.iS) Coli! 1)(' ~e! up only
the circuit: ' fol' loops which eal!lIo( Ill' "htainpd hy !'u!lL'rilililosillg"
the 100Jls con:,idl'l'ed heron'_ 1"01' p\amplp, "111'11 r-ljll,l-
(5.18) I iO/lS will Lt, independl'lil for loops 7.'24 and 234 oj' the
5.5. loule'lJ Law 129
128 5. Direct Current
direction of circumvention, the:corresponding Iterm IR .in
circuit shown in Fig. j,G. Equation fot' tho loop 1234 (5.t8) should be taken with the positive sign; in the OPPosl~e
will follow from the two preceding olles. It is possible to case, the minus sign should be used. The sam~ pr?cedure ~s
set up autonomous equations for two other loops, say, applicable to lJ: if an e.m.f. increases po.tentIaI 1Il th~.dl­
124 and 1234. Theil the equation for contour 284 will be a rection of circumvention, it should be assIgned the pOSItive
sign; the negative sign should be used in the opposite case.
3
Example. Find the ma~nit~de and d.irec~ion of the cur~ent passing
through resistor R in the CIrCUIt shown m Fig. 5.8. All reSIstances and
e.m.f.'s are assumed to be known. R
r----~4 There are three subcircuits, and hence,
three unknown currents I, I).. and 1 2 in
this circuit. We mark (arbitrarily) the sup-
1 posed directions of these currents by ar-
rows (at the right jU!lction). . . I
Fig. 5.6 Fig. 5.7 The circuit contams N = 2 JunctIOns.
This means that there is only one inde- 12
consequence of these two equations. The number of auto- pendent equation of type (5.17):
nomous eql..\ations of type (5.18) will be equal to the smallest
number of discontinuities which must be created in a cir- I + II + 1 2 = O.
cuit . in order to break all the loops. This number is Let us now ,Met up equations of type lS2
equal to the number of subcircuits bounded by conductors (5.18). According to the number of sub-
circuits there should be two of them. Let F' 58
if the circuit can be drawn in a plane without intersections. us con;ider the loop co.taining Rand Ig. .
I:or example, it is necessary to set up three equations of R I and the loop w~th R an~ R 2 • Taki.ng
type (5.17) and three of type (5.18) for a circuit (Fig. 5.7) the clockwise directIOn for clrcumventmg each of these loops, we
containing four junctions. This is so because the number of can write
discontinuities (marked by crosses in the circuit) breaking -IR + I 1R 1 = -1!!1> -IR -+ I 2R 2 = 1!!2'
all the loops is three (the number of subcircuits 'is also We can verify that the corresponding equation fo.r the lo~p with
equal to three in this case). If we assume the currents to Rand R can be obtained from these two equatIOns. Solvmg the
be unknown the number of discontinuities will be six in I
system of 2three equatIOns,

we 0 btam
.
accordance with the number of subcircuits between junctions, 1- -R I I!!2+ R 21!!1
which corresponds to the number of independent equations. - R I R 2+RR I +RR 2 '
The following procedure should be adopted while set-
ting up equations of type (5.17) and (5.18). If we fmd that as a result of substitution of numerical .valu~s into
this equation I > 0 the current actually flows as shown III Fig. 5.8.
1. The directions of currents are marked hypothetically If I < 0, the cllrre~t flows in the opposite direction.
by arrows, without 'caring for the direction of these arrows.
. If a certain current turns out to be positive as a result of
calculations, this means that the direction chosen for it is 5.5. Joule's Law
correct. If, however, the current is found to be negative, its
actual direction will be opposite to the one pointed by the The passage of current through a conductor having a
arrow. resistance is invariably accompanied by liberation of heat
2. Having selected an arbitrary dosed loop, we cir- (heating of conductors). Our task is to find the quanti~y
cumvent its sections in one direction, say l clockwise. If of heat liberated per unit time in a certain part of the mr-
the assumed direction for any current coincides with the \I-OIM I

Iii
ii

'Ii
130._-------=:.:..=..::..:..::.:...::..::::..:.;:.:::..--------
5. Direct Current
5.5. Joule's Law 131
------------_. ~~:.::.::::.::-.::...:.=~---------
cuit. We shall consider two cases which are possible, viz.
uniform and nonuniform subcircuits. The problem is consid- Since in accordanee with Ohm's law CPI - lJl2 = RI,
ered on the basis of the law of conservation of energy and we get
Ohm's law. (5.19)
Uniform Subcircuit, Suppose that we are interested in
the region between cross sections 1 and 2 of a conductor
(Fig. 5.9). Let us find the work This is the expression for the well-known Joule's law.
2 done, by the field during a time We shall now derive an expression for the differential
interval dt in the region 12. form of this law, eharacterizing the liberation of heat at
If the cuuent through the con- different parts of a conducting medium. For this purpose,
ductor is equal to I, a charge we isolate a volume element of this medium in the form of
dq = I dt will pass through each a cylinder with its generatrices parallel to vector j, viz.
. cross 'section of the conductor the current density at the given point. Let dS and .dllbe the
. FIg, 5.9 during the time dt. In particular, cross-sectional area and length of the small cylmder re-
this charge dq will enter cross spectively. The amount of~eat liberatedin .this volu~e durin~
s~ction 1 and the tsame charge will leave cross section 2. the time dt will be given, m accordance WIth Joule s law, by
~lDce ~harge distribution in the conductor remains unchanged
~n th~s case (the current is dire~t), the whole process fJQ =;,RI2 dt = ~~l (j dS)2 dt = pj2 dV dt,
IS eqUIvalent to a transfer of charge dq from section 1 to
section 2 with potentials CPI and CPI respectively. where dV = dS dl is the volume of the small cylinder. Di-
Hence the work done by the field in such a charge trans- viding the last equation' by dV dt, ':Ie .obta.in a fo 7mula for
fer is the amonnt of heat liberated per umt tIme m a umt volume
6A =dq (CPI - CPI) = I (CPI - CPa) dt. of the conducting medium, or the thermal power density of
the currtmt:
According to the law of cons9l'vation of energy, an equiv-
alent amount of energy must be liberated in another form. (5.20)
If the conductor is stationary and no chemical transforma-
tions take place in it, this energy must be liberated in the This formula expresses Joule's law in the differential form:
form of internal (thermal) energy.. Consequently, the con- the thermal power density of current at any point is p'rop~r­
ductor gets heated. The mechanism of this transformation tional to the square of the current density and to the res£stw£ty
is quite si~ple: as a result of the work done by the field,
charge . c~rrlers . (fo~example, electrons in metals) acquire of the medium at that point. .
Equation (5.20) is the most. general. form, of l.oule's law,
an ~ddItI.onal.kmetIcenergy which is then spent on exciting applicable to all conductors -IrrespectIve of thel~ sh~pe or
lattIce VIbratIOns due to collisions of the carriers with the
lattice sites' atoms. homogeneity, as well as the nature of .forces WhIC~ mduce
Thus, in accordance with the law of conservation of ener- the electric current. If the charge earners are subJected to
electric forces only, we can write, on the basis of Ohm's law
gy, the elementary work 6A = 'Q dt, where Q is the heat (5.10),
liberated per unit time (thermal power). A comparison of
this equation with the preceding one gives Qd=j.E=aE2. (5.21)
This equation is of a less general n~tur~ than (~.20).
Nonuniform Subcircuit. If a subClrcUIt contams a source
132 5. Direct C', rent .5,6. Transient Processes in a Capacitor Circllit
----
of c.m.f., the charge carriers will be subjected not only to . an d COIlSH
pJ'2•,,-~•
. Ior tl 13 t lJ,." Q"d (see (5.20».
c.c-=
1/p 311d
electric forces, but to extraneous forces as wel1. In accordance ~he thermal power density of curront 1Il an mllOffiogeneous
with the law of cOllservatioD of energy, the amount of heat medium can then he written in the form
liberated in this case will be equal to the algebraic sum of
the works done by the electric and extraneous forces. The (5.24)
same applies to the corresponding powers: the thermal power
must be equal to the algebraic sum of the powers due to
electric and extraneous forces. This can be easily verified
by multiplying (5.15) by /: 5.6. Transient Processes in a Capacitor Circuit

0
R/2 = (<pt - 'P2) / + W,/. (5.22) Transient Processes. These are th~ processes i?vol.ving
The left-hand side of this expression is the t,hermal power a transition from one stationary regIme of the CircUlt to
Q liberated in the subcircuit. In the presence of extraneous another. An example of such a process- I
is the charging and dischar~ing of.a
forces, the value of Qis determined by the same formula capacitor, which will be conSIdered III
(5.19) which is applied to a uniform subcircuit. The last detail in this section. . C q± R
term on the right-hand side is th..! power generated by ex- So far, we have considered only (h-
traneous forces in the subcircuit uNIHf consideration. It rect current#It turns out, h~wev~r, that , K
should also be noted that the last H'1'ill (ll) is an algebraic most of the laws obtained lfl thIS case
quantity and, unlike R/2, it roverSt~;~ its sign wh!m the di- are also applic~ble to .alterna~ing c~lr- Fig. 5.10
rection of the current / is revt)f<;ed. . rents. This applIes to aU cases lD Wlll.ch
Thus, (5.22) indicates that tho heat liberated in the re- the variation ofeurrent is not too rapId. . )I
gion between points 1 and 2 is equnl, to the algebraic sum of The 'instantaneous value of current will then practlca y
the electric and extraneous powers. The sum of these powers, be the same at all cross sections of the eircuit. SUC~I cll~Tents
Le. the right-hand side of this equation, is called the electric aIllI el1e ld s aSSOCIa
. ted' W'\·tli
' them
. are called quaslstatzonary . d
power developed in the subcircuit. It can then be stated (a more exact eriterion for quasi~tationary currents an
that for a stationary subcircuit, the thermal power evolved fields is given in Sec. 11.1). . I • I ws
is equal to the electric power developed in the subcircuit Quasistationary currents can be deSCribed. by t 10 a ,
by the current. obtained 'for constant currents by applying these laws to
Applying (5.22) to the entire unbranched circuit ('PI = instantaneous values of quan~ities... . . r f a
= <P2), we get • Let us now caIlsidet' the dlschargmg and ChalgIIlg 0
capacitor, assumwg- . that t he 'curren t s in these
' processes
Q='f:l. (5.23) are quasistationan. I I
In other words, the total] oule heat liberated per unit time Discharging of ~ Capacitor. If the plates of a c larg~(
in the entire circuit is equal to the power developed by c,apacitor wilh eapacitance C are connected t~lI'ougILa r~~Hs­
extraneous forces only. This means that heat is generated tor R a current will flow through the resistOl'. et ,q
only by extraneous forces. The role of the electric field and U be the instantaneous values ~f th~ current, ~harge
is reduced to redistribution of this heat over different parts of the positive plate, ana the potential lhfference bet.ween
of the circuit.
We shall now derive Eq. (5.22) in the differential form.
the plates (voltage). . ' .\ '
Assuming that the CIIlTent I is p~sitIve whel~. It
from the positive plate to the negatIve plate (F Ig. . ,
df;)s
For this purpose, we multiply both sides of Eq. (5.11) by

~
~
_:i-"._
,-,'o',;,-

,
134 5. Direct Current C ·t· Circuit 135
--------- 5.6. Transient Processes i.n a apacl or .~----

we can write I = - dqldt. According to Ohm's law, we


obtain the following expression for the external subcircuit '. t 0 tl1e loop 1'f,R2, we get
having resistance R: . RI =
.
qJI - qJ2
.
~,+of the circuit, includlllg
.
RI = U. where R is the total reslstalnce f source. Considering
the internal resistance of t le e.m..
COJisidering that I = - dqldt and U = giG, we can
transform the above equation as follows:
dq If
dt + RC ~oc O. (5.25)

After separating the variables in this differential equation


and integrating, we get
q =., qoe- Itt, (5.26)
o
Fig. 5.12
where go is the initial charge on the capacitor and T is a Fig. 5.11
constant having dimensions of time: U .ccco qlC, we can rewrite
that I = dq/j.t and 'P2 - qJI =
T = RG. (5.27) the last eq u:ation in the form
This constant is called relaxation time. It can be seen from •
dq 8-q/C
(5.26) that T is the time during which the capacitor charge dt"= R .
decreases to 1Ie of its initial value.
Differentiating (5.26) with respect to time, we obtain
the law of variation of current: Separating the variables, we get
dq
1= -(it=/oe- tlt , (5.28) Rdq =dt. o
8-Q/C
Fig. 5.13
where 10 = qo/T is the current at the instant t = O.
Figure 5.11 shows the dependence of the capacitor charge Integrating this equation under __ 0 we obtain
the initial COli d 1't'1011 q -- 0 at t -- ,
q on time t. The / vs. t dependence also has the same form.
Charging of a Capacitor. Consider a circuit in which
a capacitor G, a resistor R and a source of e.m.f. '0 are
RG In( 1 -;c ) = - t,

connected in series (Fig. 5.12). To begin with, the capacitor whence _ (1 _e-t1t). . (5.29)
is not charged (key K is open). At the instant t == 0 the q- qm · · t 'rh'\rge
_ ",C is the limitillg value of the capHl~1 or ' (
key is closed, and a current starts to flow through the circuit, Hero qm - _ RC .
thus charging the capacitor. The increasing charge on the (for t -+ 00), HIIlI .~=-
The current vallCS WI 1.
'tl' time according to the followlIlg
capacitor plates will obstruct the passage of the cunent
and gradually decrease it. law: (5.30)
The ClllTent in the circllit will now be assllmed positive 1= ~~ = loe- tlt ,
if it flows toward the positive plate of the cnpacitor: I
= dqldt. Applying Ohm's law for a non\luifonll sHbcircuit where 1 0 = 'f,1ll. f d I on t are shown in Fig. 5.13.
The depemlences 0 q an
'i·
~

~
;~:"""
._' ,;i_
';'i,

,
136 5. Direct Current Problem' 137
Problems conductors and C is the mutual capacitance of the conductors in the
.• 5.~. Resistance of a conducting medium. A metallic s here of presence of the medium.
~adlUs a IS surrounded by a concentric thin metallic shell of d' bl Solution. Let us mentally supply the charges +q and -q to the
fhe space bet,,:een the~ two ele~tr?des is filled with a homoa e~~~u~ conductors. Since the medium between them is poorly conduc.ting.
poorly conductIng medIUm of reSIstIVity p. Find the resistanc: of th the surfaces of the conductors are equipotential, and the neld configu-
gap between the two electrodes. e ration is the same as in the absence of the medium.
d ~olution. Let us ~nentally isolate a thin spherical layer between Let us surround, for example, the positively charged conductol' by
ra 11. rand r t dr. Lmes of current at all points of this layer are r- a closed surface S directly adjoining the conductor and calc:ulat.e R
pe~dlcular to It, and therefore such a layer can be treated as a cy\'fn- and C separately:
drIca! conductor of length dr and a cross-sectional area 4 . Z U'
formula (5.9), we can write nr. SIng
U u u
R=T-
(J~EndS '
dr
dR=p -4- q ee o ~ E" dS
nr Z C=(f U
Integrating this expression with respect to r between a and b, we get
where the integrals are taken over the given surface S. While calcu.-
R =..i!..-
4n:
(1.._1.)
a b •
luting R, we used Ohm's law in the form j = (JE, and C was calcula-
ted with the help of the Gauss theorem.
Multiplying these expressions, we obtain
• 5.2. Two identical metallic balls of radius a are placed in a
hOI.nogeneous boorly conducting medium with resistiVity p F'nd th ",' RC = eoe/a = eoEp.
~hSlsJ~~~e of bt e medium b~tween the balls under tbe conditi~n tba~ • 5.4. Conditions on the bounda~y of a conductor. A conductor
e IS "nce etween them IS much larger than their size. with resistivity p borders OJ! a dielectric whose dielectric coustant is
. Solution. Let us mentally impart charges +q and _ to th b II e. The electric induction at a certain point A at the surface of the con-
Smce ~he ba}ls are at a large distance from one anothe/ elect~c fletd ductor is equal to D, vectorD being directed away from the conductor
nhar t e sh" ace of each ball is practically determined only by the and forming angle a with the normal to the surface. Find the surface
c ~rge 0 f t ~ n~arest sphere, and its charge can be considered to be charge density on th.e conductor and the current density near the
uIlIformly dIstrIbuted over the surface. Surrounding the ositiv I point A.
~hargedball. by a concentric sphere adjoining directly the Call'~ s~l­ Solution. The surface charge density on the conductor is given by
face, we WrIte the expression for the current through this sphere;
(J = Dn = D cos a.
1= 4lta 2j,
The current density can be found with the help of Ohm's law: j =
where j is the current density. Using Ohm's law (/' = Elp) and the = E/p. It follows from the continuity equation (5.5) that the normal
formula E = q/4neoa2, we obtain
components of vector j are equal, and since in the dielectric ill = 0
1= qleop. (there is no current), in the conductor we also have in = O. HeDt:e,
vector j in the conductor is tangent to its surface. The same applies
Let us now flIld the potential difference between the balls: to vector E inside the conductor.
U = lp + - lp_ ~ 2ql4nE oa. On the other hand, it follows from the theorem on circulation
of vector E that its tangential components on. difIerent sides of tb;e
The sought resistance is given by interface are equal, and hence E = E t = J) SIn alee., where E", 15
the tangential component of vector E in the dielectric. Taking tJieBe
R = UII = p/2na. arguments into account, we obtain
:thiStreSfUtlht is vadl~d regardless of the magnitude of the dielectric con- E D sina
"an 0 erne mm . j =p eoep
. . ., 5.3. Two conrl~ctors of arbitrary shape are placed into an in- • 5.5. The gap between the plates of a parallel-plate capacitor is
~~dtet~~od?~e~e~~s slIghtly cond.ucting medium with the resistivity p filled consecutively by two dielectric layers 1 and 2 with thicknesses
t.hi' w ec~n: cons~~nt e. FJ!Id the value of the product RC for 11 and 12 , dielectric constants CI and E2' an~ rcsi8tiviti~s PI and f,,'
Ii system. \It here R lli the rClillitunce of the medium between the
The capacitor is under a permanent voltage (;, the electrIC field bemg
t38 139
5. Direct Current Problem.

directed from layer 1 to 2 F' d h taining resistors Rand Ro, we can write
charges at the interface b~tw::n tthe sd'!rlfacte. delnsit y of extraneous
.
S oI utJon. The required f
.
h
e Ie ec ric ayers. (R +
R o) I = It - ~o,
sur ace c arge density is given by where the positive direction is chosen clockwise. On the other hand, for
a = D s n - ,D In-- 8 02
8 E 2 - e 8 1 E l'
0 (1) the nonuniform subcircuit aRb of the circuit we have
In order to determine E and E h I + It,
7'
since h = ii' it follows lhat E w~ sEall make use .of two conditions:
= U. Solving the two e a ' I PI - 2 P2' and besides EIII + E 1 _
RI = CPa - CPb

values into (t), we obtii~ tIOns, we find Eland E 2 • SUbstituting lh~; -while lor the suhcircuit aCb we have
~ + CPI - CPI = CPb - CPa·
a= B 2P2- eIPI U
PIll +
P2 12 eo • The joint solution of these equations
gives
Hence it follows that a = Of,or _
, 8 l PI - e2P2'
• 5.6. Nonhomogeneous c d t I Fig. 5.14
cular cross section of area S . on DC or. A ong conductor of a cir-
dc !1ds only on the distance ~ f~~ethf a dterial w~lOse resistivity
pe
where a is a constant Th d e cpn uctor aXIS as p = a/r2 The charge on plate 1 is determined by the formula ql = C (CPI - CPI)'
intensity E in the co~duct~rCon uctor came~ current I. Find (1) field Therefore, the final result is
of the conductor. and (2) the resistance of the unit length
RC
Solution. (1) In accordance w'th Oh ' '. ...' ql= R+R (~-t8'o)'
o
related to current density i wh·l . . m s dlaw , field lDtensity E is
l
can write· , I e I IS rel ate to current I. Hence we It can be seen that ql > 0 f~r It > Ito and vice versa .
1= J j21tr dr= J (Elp) 2nr dr.
• 5.8. The work of an e.mJ. source. A glass plate completely
fills the gap between the plates of a parallel-plate capacitor whose
capacitance is equal to C. when the plate is absent. The capacitor is
The field intensity E is the same II' connected to a source of permanent voltage U. Find the mechanical
th!! given conductor, Le. is ind at d
pOI ts of the cross section of work which must be done against electric forces for extracting the
:::.: :,t:;;e:.:~~~ :?:~f!e~l~. :!;~u~n:n
and then applying to this con\th, f~h
O{h:' C':;d~~~re~~ilih:t~~~
exahmple, the conductor's axis
plate out of the capacitor.
Solution. According to the law of conservation of 'energy, we can
vector E. our e t eorem on circulation of write
Thus, E can be taken out f th . Am + As = ~W, (1)
gration we obtain 0 e lDtegral, and as a result of inte- where Am is the mechanical work accomplished by extraneous forces
against electric forces, A s is the work of the voltage source in this
E = 2naI182. process, and ~ W is the corresponding increment in the energy of the
(2) The resistance of a u 't I h capacitor (we assume that contributions of other forms of energy to
wHhthe help of the formul~1 R e~gt of t~e .c~nductor can be found the change in the energy of the system is negligibly small).
equation by length lof the sectio;;oF/{ DlvdldlDg both sides of this Let us find ~ Wand A s. It follows from the formula W = CU2/2 =
R and voltage U, We find e con uctor, haVing resistance = qU!2 for the energy of a capacitor that for U = const
R u = Ell = 2na182. ~W = ~C U2/2 = t!.q U/2. (2)
Since the capacitance of the capacitor decreases upon the removal of
• 5.7. Ohm's law for no
shown in Fig. 5.14 the emf' nunl
'f rm
d
suhcircuit. In the circuit the plate (~C < 0), the charge of the capacitor also decr~ases (t'..q <
Raland .R" and the capacita~~esc~ot~ Iffo?{ sourckes, the resistances < 0). This means that the charge has passed through the source against
n resistances of sources are ne l"blcapaci or are nown. The inter- the direction of the 1i.ction of extraneous forces, and the source has
plate 1 of the capacitor: g 19l Y small. Find the charge on done negative work
Solution. In accordance ~itli ali ' I f As = !'"q'U,
m s aw or,a closed circuit con-
II1I
1'['

1.

II
1
'I;

6.1. Lorentz Force. Field B 141


1

1'1
140 :S. Direct Current
I'III Comparing formulas (3) and (2), we obtain
SoluUlIn. The required amount of heat is given by
I
II As
Suh~t.itution of this expression into (1) gives
= 2.1 W. Q= JllP dt, (1)
" U

Am = -.1W 1 (e Irom which it follows that first of all we must find the time dPpendence
or A III = 2" - 1) Co1I 2 • I (t). For this purpose, we shall use Ohm's law for the part JR2 of .
t.he circuit (Fig. 5.16):
f Thus, extracting the plate out of th . RI = fill - lJl2 = 1I,
o~ces) do a positivework(a<Ja't 1• .e capaCitor, we (extraneous
thIs case accomplishes a n~g:d~~ e
tor decreases:
Lrlc ('k
fdrces) .. The e.mJ. source in
wor , an the energy of the capaci-
or
RI = q/C. (2)
Let. us differentiate (2) with respect to
Am > 0, As < 0, .1 W < O.
t.ime:
• 5.9. Transient proccSse A . . dE dt
s~lUrce of e.m.f. tE, and a re~is~~r CircUIt co,!sists of a permanent R~=_1 1 T-' RC'
nes. The internal resistance 'of th R and. c~paclto~ ~ connected in se- dt C'
m?ment t = 0, the capacitance' e source Is.negliglbly small. At the Integrating the last equation, we obtain
WIse) decreased by a factor of 11 °t,~hd r-hPacltor w~s abruptly (jump- It -IIRC Fig. 5.16
functi~n ~f t·t e current III the circuit as a ln = RC' 1=100 , (3)
1
..1
. Ime.
Solution. We write Ohm's law
mhomogeneous part ltER2 f th fo~ t~e
1o
where I. is deterqrincd by condition (2) for q = qo, Le. 10 = qolRC.
& 1 .!.l2 (Fig. 5.15); 0 e CircUit
Substituting (3) into (1) and intewating over time, we get
2
Q=...!!!... (1_0- 21 / RC ).
T~ RI = <PI - <P2 - {! = 1I - €]. 2C
Considering that U = qlC' where C' -_
R = Cit], we obtain '
Fig. 5.15 RI = t]q/C _ Ji'
6. Magnetic Field in a Vacuum
. . '" . (1)
~Ve dd'Ierentillte this equation with r p . 6. t. Lorentz Force. Field B
IIIour case (q decreases) dq/dt = _l;es ect to tune, considering that
R dE TJ dl Lorentz Force. Experiments show that the forc.e F acting
([t=-C I , -r=T=- R~ dt. on a point charge q generally depends not only on the posi-
Integration of this equation gives tion of this charge but also on its velocity v. Accordingly,
the force F is decomposed into two components, viz. the
ln~=
1
-~
RC '
I 1
=. oe
-Tjl/RC
,
electric component Fe (which does not depend on the motion
0
of the charge) and the magnetic component F m (which
where J 0 is determined by condition (1) • I 11 dee d , we can wri te depends on the charge veloc.ity). The direction and magn i-
tude of the magnetic force -at any point of space depend
RIo = TJqo/C - e,
on the velocity v of the charge, this force being always
where go = ,ffC is the char e f h . perpendicular to vector v. Besides, at any point, the magne-
has changed. Therefore, g o t e capacitor hefore its capacitance
tic force is perpend icnlar to the direction specifiCd at this
1 0 = (11 - 1) ~/R. point. Finally, the magnitude of this force is proportional
~ .5.10. A charge qo was su to the velocity component which is perpendicular to thi~
C, which at the moment t =
lied t O
lP . ,.' .
h a cal,antor with capacitance
the amount of heat liberated in J:eas s .unted hy resistor R. Hnd direction.
resIstor as a function ot time I.
143
6.1. Lorentz Force. F£eld B

142 6. Magnetic Field in a Vacuum


of the (inertial) reference system. On the other hand, the
magnetic component of the Lorontz force varies upon a tran-
f .
byAll tltc!o'(' propcrtic'
introdll;;ill r the CO~IC~ magnetIc f~lI'e? can be described sition from one frame of reference to another (because of v).
this field by v~ctOl" B wili~~ °II ;n~f:n.etl~ fteld. CI~al"acterizilJg Therefore, the electric component qE must also change.
at each pomt of sp'lce W(e el1ll\n~s the specIfic direction Hence it follows that the decomposition of the total force F
magnetic force ,in 'tl:e f~rme can WrIte the expression for (the Lorentz force) into the electric and magnetic compo-
nents depends on the choice of the reference system. Without
Fm = q [v )( BI. (6.1) specifying the reference system, such a decomposition is
meaningless.
Tl.wn Magnetic Field of a Uniformly Moving Charge. Experi-
wIll bethe. totalbyelectromagnetic
gIven . force acting on charge q
ments shoW that magnetic field .is generated by moving
C1harges (currents). As a result of the generalization of expe-
I F=qE+q Iv x RI· I (6.2) rimental results, the elementary law defining the field B
of a point charge q moving at a constant nonrelativistic-
It is called the Lorentz for E· . ' . . velocity v was obtained. This law is written in the form*
it is valid for constant as c~~ll xpr~sslOn «(~.2)g IS un.i versa I:
magnctic fIelds for any velo 't as orf vlarYIn electric and B=.fQ.. q[vx3 r ] \ (6.3)
. . CI Y V 0 t lC charge 411: r ,
Th e actIOn of the Lorentz fo .
ciple be used for determinin tl~~e on a. charge can in prin-
\
If.:

of vectors E and B. Hence gth magmt.udes and directions wltere ~o is the magnetic constant, the coefficient
force can be considered a~ e expre~s~on for the Lorentz ~o/41t = '10- 7 HIm,
magnetic fields (as was do t.he t~CfimtlOn of electric and
It should be emphasizednet}~~t m~;::7iCoffi:~c~ric field). * and r is the radius vector from the point charge q to the
on an electric charge at rest I }. oes not act point of observation. The tail of the radius vector r is
essentially differs from el~ct;i 1 ~\~eS~Cl; m~gnetic field fixed in a given frame of refe- B
only on moving charges. c e. agnetIc fIeld acts rence, while its tip moves with
Vector velocity v (Fig. 6.1). Conse-
fIeld on aBmovin
character'·
~zes tl lC foree actlllg
. due to magnetic quently, vector B in the given
an analog of vecro~ ~r~~, and l~e~ce in this respect it is reference frame depends not
to electric field. aracteflZIng the force acting due only on the position of the
point of observation but on
perpendicular to the veioci:;n~eI~ ore; that it is always
A distinctive feature of m t' f .
If time as well.
Fig. 6.1
fore, no work is done over the c or 0 t. Ie charge. There- In accordance with formu-
energy of a charged particle ch:zrge: TIllS means that the la (6.3), vector B is directed
tic field alwa , . . movmg In a permanent magne- normally to the plane containing vectors v and r, the rota-
motion of the y;ar~~~:.ws unchanged h'respective of the tion around vector v in the direction of vector B forming
In the no 1 l' . .
(6.2), like a;~~e:tl~~~s~~~c::~[~:Si~~ii~~pe\~~~~r~~zcl~~~~: • Formula (6.3) is also valid in the case when the charge moves
with an acceleration, hut only at suffIciently small distances r from
the charge (so small that the velocity v of the charge does not noti-
ceably change during the time ric).
the •fmal
Several methods
analysis, thevofall a b g fiIe ld B h ave been worked out. In
measurin
describeu by Eq. (G.2). • re ased on the phenomena that can be
6. Magnetic Field in a VOC1l1Jm
~------------
6.2. The Biot-Savart Law 145
the rig-ht-hallrlcll SystClll with vector v (Fig. li.1). It should
be nQtHd that vector B is Hxial (pseutlovector). First, we have to deal with beams of particles moving
The quantity B is called maKfictic induction. at velocities close to the velocity of light, for which this
Mag:nctic induction is measmetl in teslas (1'). "correction" and the electric force become comparable (it
The electric field of a point charge q moving at a non- should be noted that relation (6.5) is also valid for relati-
relativistic velo(~ity is described by the same law (1.2). vistic velocities).
Hence expression «U~) cnn be written in the form Second, during the motion, say, of electrons, along wires,
their directional velocity amounts to several tens of a mil-
B =, Eo!-to [v >< E]c= [v X Ellc\ (6.4) limeter per second at normal densities, while the ratio
(v/C)2 ~ 10-24 • It is indeed a negligible correction to the
where c is t.he electrollynamic constant (c -~, 1/V eo!-to), equal electric force! But as a matter of fact, in this case the magne-
tie force is practieally the only force since electric forces
disappeared as a result of an almost ideal balance (much

L-1,
1
Fig. 6.2
Z
more perfect than 10- 24 ) of negative and positive charges
in the wires. The participation of a vast number of charges
in creating current compensates for the smallness of this
term.
In other words, the excess charges on the wires are negli-
gibly small in"comparison with the total charge of carriers.
For this reason, magnetic forces in this ease eonsiderably
to the velocity of light in vacuum (this coincidence is not exceed the electric forcgs acting on excess charges of the
accidental).
wires.
EKample. Comparison of forces of magnetic and electric interaction
between moving charg~. Let two point charges q of a sufftciently large
mass move in parallel to onc another with thc same nonrelativistic
velocity v as is shown in Fig. 6.2. Find the ratio betwecn the magnetic
Fm and electric Fe forces acting, for example, from charge 1 Oil 6.2. The Biot-Savart Law
cli8rge 2.
The Principle of Superposition. Experiments show that
According to (6.2), Fm = quB and Fe = qE where v is the velocity magnetic fields, as well as eleetric fields, obey the principle
of charge 2 and Band E are the induction of the magnetic and the
intensity of the electric fielns created by charge 1 at the point of loca- of superposition: the magnetic field created by several
tion of charge 2. moving charges or currents is equal to the vector sum of the
The ratio Fm/Fe -= vB/E. According to (6.4), in our case B = magnetic fields created by each charge or current separately:
= IJEIc'l.. and hence
(6.5) (6.6)
Even at sufficiently high velocities, e.g. u = 300 km/s, this ratio The Biot-Savart Law. Let us consider the problem
is equal to 10-a, Le. the magnetic component of the force is a milli-
onth fraction of the electric component and constitutes a negligible of determining the magnetic field created by a direct electric
correction to the electric force. current. We shall solve this problem on the basis of law (6.3)
This example may give rise to the following question: determining the induction B of the fwld of it uniformly
Are snch forces wOI,th investigating? It turns ont that they moving point charge. We substitute into (6')~) the charge
are, and theI'e are two sound reasons hehin(l this. pdF for q (where dV is the volume element and p is the
, volume charge density) and take into account that ill accord-
f.
10-0181
6.2. The Biot-Savart Law 147
146 6, Magnetic Field in a Vacuum

anre with (5.2) pv = j. Then formula (6.3) becomes 'It is clear from the ligure that dl cos a = r da and r = b/cos a. Hence
'B--~ I clIsada
I dB ~ {;;- Ii X,:I'1' .1 (6.7)
/ -- I.n b

Integrating this expression over all current elem~nts, which ~s equiva-


lent to the integration over a between -n/2 and n/2, we lind -
If the current I flows along a thin wire with the cross-
sectional area t!S, we have B=~.Y:!- (IU1)
4n b
j dV = j t!S dl = I dl, Example 2. Magnetic field at the axis of circular current. Figu-
where dl is an element of the length of the wire. Intl'O- re 6.4 shows vector dB Ihlln the current element I dl located to the
ducing vedor dt ill the direction of the current I, we can
write this expression as follows: B

j dV = I dt. (6.8) dBz - dB


, Vectors j dV and I dt are called volume and linear current dl A
elemen is resj)ecti vely. Replacing in formula (6.7) the vol-
ume current element by the linear one, we obtain

I dB= * I[~~ xrl.

Formulas (6.7) and (6.9) express the Biot-Sauart law.


I (6.9) I
b
I;t!

A
dB,B
z

0
R
r

In accordance with the principle of superposition the I


total field B is found as a result of integration of Eq. (6.7) Fig. 6.3 Fig. 6.4
or (6.9) over all current elements:
right. All curre~t deIlll'nts will form thl' cone of vectors dB, and it can
Bc=_,fA:.lL \' [jXrldV B=~~ I[dlxr] (6.10) be easily seen that the resultant \"I'dol' ,B at point A will be directed
4n. r8 ' 411: 'j' r3 • upwards along the Z-axis. This II1l'allS lhat in order to fmd th~ magni-
tude of vcdOl" H, it is sul'ricil'lIl 10 SUIH up the compollPnts 01 vectors
Generally, 'calculation of the ;.nagnetic induction of dH along lhe Z-axis. Each such pl'ojl'c.liol1 has the form
a current of ,111 arbitrary configuration by these formulas flo I dl
is complicated. However, the calculations can be consider- dR. = dB cos
4
f-l C~ -,-
'1:1
-2-
r
cos ~,
ably simplifIed if CllI'l'ent distribution has a certain sym-
metry. Let us consider several simple examples of drtel'- where we took into account that the angl~ between the element dl
and the radius ve/'lor I' is equal to n12, and hrllce the sine is equal to
min i Ilg' the ma~netic ind uctio 11 of CllI'ren t. ' IInity. lnterrrat ing this px pression over dl (whkh gives 2rrR) and taking
EXaJpple f. Magnetic field or the line current. i.e. the c\Il"rent into' accolI~t that {'os ~ ~- nl,. and r ~ (z~ : R2)1/2, we get
flowing along a thin straight wire of infinite length (Fig. 6.3). Ilo 2nR21
In accordance with (u.9), vectors dB from all current rlements have B=.Im (Z2-, R2)8;2 • (ti.12)
at a .poillt A the ,same direction, :,iz. are directed behind the plane of
the hgure. There!orp, lhe summatlOlI of vectors dB can hp r(!placed by Hence it follows that the rnagnitudp or v('etor B at the centre of HIP
the summation of their magnitudes dB, where enrrpnt ring (z = 0) and at a distance z » R is given by
dB=c ~ I dl cos a ..l':!'... 2JlI B ~~ ~ (' 13)
4n r2 B z ,~'o 1
m R z?/H 4n z3 0 .•

10*
148 6. Magnetic Field in a· Vacuum 6.3. Basic Laws 0/ Magnetic Field

6.3. Basic Laws of Magnetic Field terminate. In other words, magnetic field has no sources.
Like electric field, magnetic field has two very important in contrast to electric fwld. .
properties. These properties, which are also related to the Theorem Oft Circulation of Vector B (for till' magnellc
flux and circulation of a vector field, express the basic laws field of a direct. current in a vacllum). Circulation oj uector H
of magnetic field. around an arbitrary contour r is equal to the product oj ~~o
Before analysing these laws, we should consider tIle by the algehraic sum oj the currel/ts enl'eZoped by the contour 1 :
graphic representation of field B. Just as any vector field
fi.eld B can be visually represented with the help of th~ (6.15 )
h~es of vector B. They are drawn in a conventional way,
VIZ. so that the tangent to these lines at any Doint coincides
with the direction of vector B, and the ~density of the where I ~I,,, and I,. are algebraic 4I1alltitie~. The CUI'-
lin.es is proportional to the magnitude of vector B at' a given rent is assumed positive if its direction is eOllllecled with
pOlDt. the direction of the circumven-
The geometrical pattern obtaiped in this way makes it tion of the contoui· through the
possibl~ to easily judge about the configuration ofa given right-hand serew rille. The cur-
magnetic field and considerably simplifies the analysis of rent having the opposite direc-
some situations. tiOIl is considerc9 to be negat ive.
. Let us now consider the basic laws of magnetic field, This rule is illustrated. in Fig.
I.e. the Gauss theorem and the theorem on circulation. 6.5: here currents I] and {a are
The Gauss Theorem for Field B. The flux of B through positive sinctl their directions are
any closed surface is equal to zero: . 'connected with the direction of /1> 0
/2" 0
cont.Olir circumvention through
the right-hand screw rule, while . Fig. 6.5
(6.14)
current I 2 is negative.
The theorem on circulation (f).i5) can he proved Oil the
This theorem is essentially a generalization of experience. basis of the Biot-Savart Jaw. In the general case of arbitra-
It expresses in the form of a postulate the experimental ry currents, the proof is rather cumbersome and ~il1 not
result that the lines of vector B have neither beginning be considered here. We shall treat statement (h.i5) as
nor end .. Therefore, the number of lines of vector B, emerg- a postulate verified experimentally. .' _ .
ing from any volume bounded by a closed surface S, is Let us make one more remark. If CIlrrent I 1I1 (b.L)) IS
always equal to the number of lines entering this volume. distl'ibllted over the volume where contour r is located,
Hence follows an important corollary which will be repeat- it can l,e represented in the form
Iii, edly used below: the flux of B through a closed surface S
I = \ j dS. (f).Hi)
Iii,' bounded by a certain contour does not depend on the shape
oj the surface S. This can be easily grasped with the help
of the concept of the lines of vector B. Since· these lines In this expression, the integral is taken over an arbitrary
are not discontinued anywhere, their number through the surface S stretched on contour r. Current density j in
surface S bounded by a given contour (i.e. the flux of B) the integrand corresponds to the point where the area
indeed must be independent of the shape of the surface S. element. dS is located, vector dS forming the right-handed
Law (6.14) also expresses the fact that there are no magne- system with the direetion of the contour circumventioll.
tic charges in nature, on which the lines of vector B begin Thus, in the general case Eq. (6.15) can be written a~
151
6.4. Theorem on Circulation of,Vector B
150 6. Magnetic Field in a Vacuum
. 1 t'on of vector B and then ~ee whcthc!' t II is
rem Oil Circu a I
follows: method is universal.
. fi Id' f·a strai~ht current. Direct current 1
~Bdl=~IO f jdS=f.to.\ indS. (6.17) • Example t •. Ma~netlc Ie 0; ht ~'irp wi th a circular cross sedion
flows along a~ mftmtely long. st~all 'nduction B outside and inside the
TIIC fact that cil'c,dati ;,1 of vector B gene.r'llly differs of radius a. Fmd the magnetIc lie ( I
from zero illdicates thalii! contr<lst to electrostatic field, wire. t of the probl~m t Ilat t he I'Illes 0 f
field B is not a potential field. SUcll fields are called It follows ~rom the s y mll1e r y f 'ircles with the centre at the
vortex. 01' solenoidal fiel!!",;,
, Since circulation of vector B is proportional to the cur-
rent 1 enveloped by the contom, in general we cannot
ascribe to magnetic field a scalar potential wllich would be
related to vector B through an expression similar to E =
1/
/'
-- "
vector B in thiS case have t he form 0 c

.......
r
B

= -Vip. This potential would not be single-valued: upon


{
each circumvention of the current and return to the initial f,\
point, this potential would acquire an incl'ement equal to \
""0/. However, magnetic potential lflm can be introduced and
effectively used in the regi-on of space whel'e the currents
" ........
.fit
..-...._/ / a

Fig. 6.7
r

are absent. . Fig. 6.6


The Role of the Theorem on Circulation of Vector B. . d !Sf vector B must be the same at all points
This theorem p.Jays aJinost the same role as the Gauss wire axis. The magm~u e" f the wire Therefore in accordance with
at a distance r from.t e ax!s 0 . 'B for a ~ircular contour r l
theorem for vectol's E and D. It is well known that field B
the theorem on nrBcu2latlO~ of /e~~:nce it follows that outside the
is detel'milled by all CUlTellts, while circulation of vector B, (Fig. 6.6), we have . nr -- flO ,
only by the currents enveloped by the given contour. In wire (6.18)
spite of this, in certain cases (in the presence of a special B = (flo/2n) Ilr (r> a).
symmetry) the theorem on circulation proves to be quite It should be noted that a direct solution of this pr~~I.emt (d'ith the help
effective since it allows us to determine B in a very simple of the Biot-Savart law) turns out ~I bet:~:ei~~~iro~:th;t inside the
way. K From· the same symmetry cons". era I d' to the theorem' on
This can be done in the cases when the calculation of \,:ire the. lines of vec~ornBf~~e :Is~i;~~l~~s. ~oC~t~l:nh (see Fig. 6"6~,
circulatiou of ve('{or B call be reduced, by an appropriate circulatIOn of vcchto I = l ( la)2 is the current enveloped by the gl-
choice of the contour, to the product of B (or B I) by the B '2nr = flo l , were r r " d th .'
ven contour. r Whence we I'mrl that illSI e e wIre
leugth of the contoul' 01' its part. Othel'\vise, field B mw;t (6.19)
B ~.~ (~lo/2n) lr/a
2
(r <. a)
be calculated hy some other methods, for example. with
the help or the Diot-Savar! law or by solving the cor- B ( ~ is shown in Fig. 6.7. "d 't'
The dep~ndence r f tube the induction B outSI e I ~s
respondiug differeutial equations, and the caJculatiu/I be- If the wire has the s ape 0 a. . ' 'de the tube magnetic field IS
('.omes much mOl"e d iffienlt. determined by formula (6.1~1)' Wl~111e mS-\h the help of the theorem on
absent. This can also be easl y s own WI
circulation of vector ~. f' Id of a solenoid. Let current I flow al?n a
Example 2: Magne Ie Ie h surface of a cylinder, Such cyhn ~r a
6.4. Applications of the Theorem conductor hehcally wound o~ tell d a solenoid Suppose that a umt
on Circulation of Vector B streamlined by the ~urrent !S ca e f h c~nductor. If the pitch
length of the solenOid contams n turns 0 f'the solenoid can be appro x-
Let us consider several practically i~po:tant examples of the helix is sufficientl smdall, each~r~hall also assume that the con-
y
illustrating' the effectiveness of the applIcatIOn of the theQ- imately replaced by a cl ose oop. ,
152 6. Magnetic Field in a Vacuum 6.4. Theorem on Circulation of Vector B 153

ductor cross-sectiollal "rea is so small that the CUITent in tIll' solenoid to infmity (nt a constant toroid cross section), we obtain in the limit
can be considered flowing over its surface. the expression ((i.20) for the magnetic field of an infmitely long so-
h 1xperiments and (,;lleulations show that the longer thl' solenoid lenoid. .
t ~ ess the n!aWll'tic induction outside it. For an inlinitelv long sole~ If the chosen circular contour passes outside the toroid, it does
nOId, ~agnetlc field ontside it is absent at all. . . * not envelop any currents, and hence B ·2J1r = 0 for such a contonr.
: .It IS clear from symmetry considerations that the lines of Vl'ctor B This means that outside the toroid magnetic field is absent.
l~slde the solenoid are. directed along its axis, vector B forming the In the above cOJl!'ic1erations it was assumed that the lines of cur-
1'Jght-handed system \nth the direction of the cnrrent in the soleIloid. rent lie iUlIleridional plaues, i.l'. the planes passing "through the ·,axis
00' of the toroid. In'real toroids, the lines of cur-
rent (turns) rlo not. lie' strictly in these planes, and
hence therl' is a current component arounrl
the 00' axis. This component creates an addi-
I B
, - ..... --,
.......•.......... ~
:! I:
. tional field similar to the field of a circular cur-
rent.
Example 4. Magnetie field of a current-car·
-: L 4_ :oJ r~'ing plane. Let us consider an infmite conduct-
:w ....lL ing plane with unc1irectional current uniformly
distribute!1 over its surface. Figure 6.10 shows
..........................

t.he cross section of such a plane when the cur-
rent flows behind the plane of the figure (this
Fig. G.S is marked BV crosses). Let us introduce the con-
Fig. tUJ cept of lille~r clI!tTent density i directed along the
lines of current. The malPlitude of this vector
'. Even the aho~'e consideratioIls abollt the magnetic fwld configura- is the current per unit length which plays the
Fig. 6.10
tIOn of the. sole,nOld indicate that we must choose a rectangular contour role of the "cToss-sectional <1l'ea". •
as shown In FIg. (i.H. Circulation of vector n around this contour is l\lentally dividing the current-carrying plane into. thin ~urrent
equal to BI, and the l'l!ntour envelops the current nlI. According t~1 tilament::;, we can easily see that the resnltant field B wlll be thrected
the theorem or~ clrculatlOlI, BI "'" ~lonlI, whence it foHows that inside parallel to the plane, downwards to the right of the plan.e and up~ards
a long solenuld to the left of it (Fif.{, 6.10) .. These directions can be easily estabhshed
B = ~lonI, (6.20) with the help of thl' right-hand screw rule.
In orc1l'r to determine induetinn B of the field, we shall use the
i.e. the. Iield il.ls!d~ a long s()lenoi~1 is uniform (With the exception of theorem on circulation of vector n. Knowing- the arrangement of the
~he regIOns. ildJOIll!lIg the solenoid s endfaces, which is often ignored lines of vector B, we shall choose the contour in the form o! rectaJ;gle
III caleul~tIOns). 11le product liT is called the Ilumber 0/ ampere-turns. 1234 (Fig. 6.10). Then, in accordance with the theor.em on CIrculatIon,
For !/ ~ 20()(I turns/m and 1 = 2 A, the magnetic field inside the so- 2Bl ~, !Joil, where I is the lenl!th of the contour SIde parallel to the
lenOId IS equal to 5 mT. conducting plane. This g-ives
Exnmple 3. l\fa~netie field of a torOid. A toroid is a wire wound 1 . (6.22)
arollnd a torus (hg. 6.9). . B =~ 2~tOI.
. It can he easily see,n from symmetry considerations that the Jines This formula shows that the magnetic field is uniform on both sides
of v?ctor B mus~ be CIrcles whose centres lie on the axis 00' of the of the plane. This result is also valid for a bounded current-carrying
t?rold. Hence It IS c1t'ar that for the contour we must take one of SUcll plane, but only for the points lying uear the plane and f~r from its
CIrcles.
ends.
.If a contour !iI'S in~ide I,h(~ toroid, it envelops the ('urrent N 1 wher!'
!V IS th~ number of turns in tIle toroidal coil and 1 is the ~urrent General Consideralicms. The results obtained in the above
III t~e wIre: Let the contour radius be r; then, according to the theorem examples could be found 'directly with the help of the
on clrculatlOn B '2:rrr 1'01\'1, whence it follow!' that inside the toroid Biot-Savart law. However, the theorem on circulation
B = (/to/2n) N /h.. (6.21) makes it possible to obtain "these results much more simply
II compa.rison. ofyi.21) alltl (6.1H) shows that the lIIaglll'tie licit! illsidp and quickly.
til(' torOJ.d cOJflCldes with the magnetic lield fIT J of tl](' straight i'lll'- However, the simplicity with which the field was cal-
rent flowmg along the 00' axis. Tellding N and radius H uf the toroid culated in these examples must not produce an erroneous
155
6.6. Ampere"s Force
154 6. Magnetic Field in II Vacuum
where Oil the right-hand side we have the projection of curl H onto
impression about the potentialities of the method based normal n. . . f Id B can be characterized by curl B
on the application of the theorem on circulation. Just as lIence, ea~h POlllt of vedor I~ I' determined by the propertie.s of
in the case of the Gauss theorem for electric field, the num- whose directlOll and. IIwglll~u~e1~1 lC. I'rection of curl B is detl'rllllllClI
ber of problems that can be easily solved by using the theo- t'
the field !tself at a given 1}010 tl: . ~~rface element S, for whi?h t.he
rem on circulation of vector B is quite limited. It is suffi- by thtetl~Il('~c~z)n ~hf~l~md~ti~esothee J;laguitude of curl H, attainS Its
quail I ) . ,
cient to say that even for such a symmetric current configu- maximum value: .1 B .~ obtained in coordinate represen~-
ration as the current ring, the theorem on circulation be- In mathematiCs, cur k therfact is more importallt: It
comes helpless. D88pitean apparently high symmetry, the tion. However" for our purpos~sa:b~ considered as the cross pl'Oduct
turns out that formally curl BB' 'tV X B. Wc shall he using the
magnetic field configuration does not allow us, however, of the operator 'tV ~y vecto~ r 'n I~~~ce it allows us to represcllt the
to find a simple contour required for calculations, and the latter, morc c~nven~nt. ~htth~Ohelp of the determinant:

~~x a~Zy iJ~~z


problem has to be solved by other, much more cumber-
some, methods. . cross product . X 'tV\: B = \ a \' (Ii .25)

Bx By Bz .
6.5. Differential Forms of Basic laws • r' the unit vectors of Cartesian coordinates. ThiS
where ~x' ell' eZj.j~. th curl of not only field B, but of ,IllY other
of Magnetic Field expressIOn I~ va I 01'.1' f f 11 E
Divergence of field B. The differential form of the Gauss theorem vector field a./rIso, iI!-IP:lrtthIC~:::;~r~lI1 ~~ ~irc~latioll of vector B. Accorlt-
Let us now CoUSH 1'1 e I' tl f I
(6.14) for field B can be written as :
illg I.'q. (6 . 17) call bc representee ill Ie orn
to. (u . 24) , c
, j H dt .
(6.23) lim ---c;- = f!oJr\'
s-u .
Le. the divergence of field B is equal to zero everywhere. As was men-
tioned above, this means that magnetic field has no sources (magnetic or (V X Bl" = l.l.oill' Hence
charges). Magnetic field is generated by electric currents and not by
magnetic charges which do not exist in nature. . \-'V-X-B-=-Il-oj-~ \ (fj.~6)
Law (6.23) is of fundamental nature: it is valid not only for con- . . of the theorem on ~irculatlOn
~f~~~o~r~~i~t;i~~l:~~~de d~~~~J~F:~i~~~i~,c~~fl~~h::;:i~~d~
stant but for varying fields as well.
Curl of field B. The important property of magnetic field expressed
br the theorem on circulation of vector B motivates the representation or viz. the current ensl y .
o this theorem in the differential form which broadens its potential- ve t cJ'. 1 t Ii j E . al to zero
ities as a tool for investigations and calculations. of V X B is equa ? rfio id circulation of vector IS equ •
For this purpose, we consider the ratio of circulation of vector B In an electrostatIc e ,
to the area S bounded by a contour. It turns out that this ratio tends and hence
to a certain limit as S -.. O. This limit depends on the contour orienta- \ 'V X E=O. \ (fj.~7)
tion at a given point of space. The contour orientation is specified by
vector n (the normal to the plane of the contour around which the . I t zero everywhere is a potentIal
circulation is calculated) the direction of n being connected with tlte A vector field whose curl IS equa?d l field Consequently, eleetro-
direction of contour circumvention through the right-hand screw rule. Iield. Otherwise it is ~aned the,~?f:n~~ganet'c field is a solenoidal J1~ld.
The limit obtained as a result of this operation is a scalar quantity static field is a potential field, II: I .
which behaves as the projection of a certain vector onto the direction
of the normal n to the planp, of the contour around which the circula- 6.6. Am~re's Force
tion is taken. This vector is caller! the curl of field B and is denoted as
curl B. Thus, we can write , h 1 I' e carrier experiences the
Ampere's Law.. Eac c~~egaction of this force is tr~llS-
. I Bdl action of a magnetIc forc~. h which charges are lIlovmg.
IHn'-'--=(curl H)n, (6.Y.)
8-0 S [nitted to a conductor t roug
156 6.6. Amp~re's Force 157
6. Magnetic Field in a Vacuum
Finally, it can he easily seen that currents of the same
~:s : resul~,
magnetic field acts with a certain force on a cur- direction are attracted, while the opposite currents repell
~ -carryIng conductor. Let us find this force
each other. Here we speak only about a magnetic force.
th~~;~~i:~t~~t the volume density of charges ~hich are It should be recalled, however, that besides magnetic forces
to .)'( W ~ cliIre~t (e.g. electrons in a metal) is equal there are electric forces, i.e. the forces due to excess charges
con~~lctore m1tenta tY ' Isolate a volume element dV in the on the surface of conductors. Consequently, if we speak aho~t
, . . con aInS the charcre ( t .
to pdF. The tl P. f . «,., curren car:ner) equal
>' the total force of interaction between the wires, it can either
f 'h , n , L orce actlllg on the volume element dV be attractive or repulsive, depending on the ratio between
o t. e conductor can be' written by formula (6.1) as follows: the magnetic and electric components of the total force
dF = p [u X Bl dV. (see Problem 6.7). .
Since i = pu, we have The Force Acting on a Current Loop. The resultant Ampe-
re's force acting on a current loop in a magnetic field is
IdF = [j X B) dv·1 (6.28)
dermed, in accordance with (6.29), by
F = I ~ rdl X B], (H.31)
If. the current flows along a thin
cordmg to (6.8), j dV = I dl, and conductor, then, ac- where the integration is performed along the given loop

I dF = I fdl X BI, I (6.29)


with current [.
If the m~netic field is uniform, vector Bean be taken
out of the integral, and the problem is reduced to the calcu-
where dJ is. the vector coinciding in direction with the
cUITdent and characterizing an element of length of the thO lation of the integral ~~ dl. This integral is a dosed chain
con uctor. , ' I n
of elementary vectorsdl and hence is equal to zero. Conse-
,. F~rmtdas (6.28) a~d (6.29) express Ampere's law. 1nte- quently, F = 0, i.e. the resultant Ampere's force in a uni-
~~af!ng th)ese expreSSIOns over the curren. t elements (vol ume form magnetic fIeld is equal to zero.
meal', we can find the rna t' f If, however, the magnetic lield is nonuniform, the resul-
tain vo] ume ~f d gne I.C o~'ee acting on a cer-
Th e f a . can uctor or on Its lmear part. tant force (6.31) generally differs from zero and ill each
called A(),rce~. ~ctfmg on currents in a magnetic field are concrete case is determined with the help of (6.31). The
npel e s orces. most interesting case for further analysis is that of a plane
Example. The force of 'nt t· b ". current loop of sufficiently small size. Such a current loop
Find the Arrwere's force with wh.ah l?~ .etw~en parallel currents. is called elementary.
?urrents /1 and I. interact in IC LW? JllfillJ~ely long- wires with The behaviour of an elementary current loop can be
IS b. Calculate the force per ~n~tcluumt'hlf rtheh distance between them
eng 0 t e system. conveniently described with the help of magnetic llwrnent Pin'
. Each current element of I is in th . .. By delin ition, magnetic moment Pm is given by
I.e. aeeording to (6.19), the 2 field B e~agn~t1C field of ~he current 1 I,
t.ween the element of current / . d I - (flI/4:r~ 2f1/b. fhe angle be- Pm = ISn, (6.:32)
from fonnula (6.29) that the io~~e v:~.tor B1 IS 1(/2. Hence, it follows where [ is tJw cUlTent, S is the areil hounded hy lhe loop
ductor with current I I'S F _ laB> mg per unIt length of the con-
2 u -- 2 It or and n is the normal to the loop whose direction is eonnected
Fu=~ 2/ 1 12 with the direction of the current in .tho !f;op through the
4rt b (6.30) rig-lit-hand screw rule (Fig. 1).11). In terms of magnetic lipid,
the clemenU;' \' loop is eompletely charaeteT'iz,~d by its
Naturally, the same expression . b. b ' .
per unit leDgth of the d car~ e 0 tallied lor the force actin/.( magnl.'lic moment Pm'
con uctor With current fl'
iSS 6. Magnetic Field in a Vacuum 6.7. Torque Acting on a Current Loop 159

1\n j~volved calculation with the help of formula (6.31), (0.3:1) in terms of the projections onto this direction. This
taklIlg 1I1to accoullt the smaJl size of the loop, y ields th~ gives
foHowing expression for the force acting Oil all elementary iJB x
F x= Pm an- (6.34)
cnrrent loop in a nOlluniform magnetic field: '
where iJlJxiiJn is the derivative of 'the corresponding pro-
IF= Pm"* ' (6.33) jection of vector B again with re-
spect to normal n (or with respect
where Pm is the magnitude of the magnetic moment of the to Pm) to the loop.
loop, and oBion is the derivative of vector B along the Example. An elementary current loop
having a magnetic moment Pm is ar-
ranged perpendicularly to the symmetry
10 0 F~~Pm .axis of a nonuniform magnetic field, vec-
tor Pm being directed along vector B. Let
us choose the positive direction of the
X-axis as is shown in Fig. 6.13. Since
the increment of projection B x will be Fig. 6.13
10 G).----~ Pm negative in the direction of vector Pm.

b
F < O. Hence .yector F is directed to
F~ tlfe left, i.e. to .fli.e side where B is greater. If we rotate the cur-
rent loop (and vector Pm) through 90° so that the loop centre -coin-
cides with the symmetry axil\- of field B, in this position F x = 0, aIlji
10 €lfolo'---~O Pill vector F is directed perpendicularly to the X-axis and to the same
1 'side as vector Pm·
F=O
Fig. 6.11 Fig. 6.12 6.7. Torque Acting on a Current Loop
direction of normal n or along the direction of vector p . Let us consider a plane loop with a current I in a uniform
This formula is similar to expression (1.39) for the for~e magnetic field B. It was shown above (see p. 157) th?t
acting on an electric dipole in an electric field. the resultant force (5.31) acting on the current loop In
It follows from forml.da (6.33) that, like in the case of a uniform magnetic field is equal to zero. It is known from
electric dipole, mechanics that if the resultant of the forces acting on any
(1) in a uniform: magnetic field F = 0 since oBian = 0; system is equal to zero, the resultant moment of these for?es
(2) the direction of vector F g-enerally does not coincide does not depend on the position of point 0 relativ~ to wInch
with vector B or with vector Pm; vector F.coincides in the tot'ques are determined. Therefore, we can speak about
direction only with the elementary increment of vector B the resultant moment of Ampere's forces in our case.
taken in the direction of vector Pm at the locus of the loop. ny deflllitinll, the resultant moment of Ampere's forces
This is illustrated by Fig. 6.12, where three arrangements is gin~n by
of the loop in the magnetic field of straight current To
are shown. The vector of the resultant force F actillg on the M = (~ !r:< dFl, (6.35)
loop in each case is abo shown in the flgllre (it is useflll
to verify independently that it is really so). where dF is defined by formula (5.29). If we perform cal-
If we are interested in the projection of force ..~ onto culation by formula (6.35) (it is cumbersome and hardly
a certain dit'ectioll X, it is sufficient to write expression interesting, hence we omit it here), it turns out that the
16,) 6. Magnetic Field in a Vacuum
6.8. Work Done upon Displacement of Current Loop 161
moment of fOl'ces for an arbitrary shape of the loop can
be represented in the form ' Considering that ab is the area bounded by the loop and Iba = Pm'
we obtain

IM = {Pm >: B], I (6.36)


M = PmB sin a,
which in the vector form is written as (6.36).
where Pm is the magnetic moment of the current loop (Pm = Concluding this section, we note that expression (6.36)
B = ISn for a phme loop). * is valid for nonuniform magnetic fields as well. The only
It is clear from ((j.36) that the IlWln- requirement is that the size of the current loop must be
F. ent M of Ampere's forces acting on .sufftciently small. Then the effect of nonuniformity on the
a current loop in 11 uniform magne- rotational moment (torque) M can be ignored. This pre-
tic field is perpendicular both to vec- cisely applies to the elementary current loop.
tor Pm and to vector B. The mag- An elementary current loop behaves in a nonuniform
nitude of vector 1\1 is M' = PmB sin ct, magneti(~ field in the same way as an electric ~dipole in
where ct is the angle between vec- an external nonuniform electric field: it will be rotated
F tors Pm and B. When Pm tt B, M ~= 0 towards the position of stable equilibrium (for which
and it is not difficult to see that the Pm tt B) and, besides, under the action of the resultant
Fig. 6.14 position of the loop is stable. When force F it will be pulled into the region where induc~ion B
Pm H B, 1\'1 also eq uals zel'O but such is larger. {t'

a position of the loop is unstable: t he slightest deviation


from it leads to the appearance of the moment of force that
tends to deviate the loop from the ill itial po,'.:ition still 6.8. Work Done upon'Displacement of Current Loop
further. When a current ioop is in an external magnetic field (we
shall assume that this field is constant), certain elements
.Example. Let liS verify the validity of formula (6.36) by using of the loop are acted upon by Ampere's forces which hence
a Simple example of a rectangular current loop (Fig. 6.14).
. It ca.n be seen from the figure that the fo~oes <Jcting on sides a are will do work during the displacement of the loop. We shall
perpendicular to them and to vector B. Hence these forces <JfP directed show here that the work done by Ampere's forces upon an
along the hOrIzontal (they are not sh~':Vn in, the figure) and only strive elementary displacement of the
to stretch (or compress) the loop. S,'1es 0 are perpendicular to H I
and hence each of them is acted upon by the force ' loop with current I is given by
F c= !hB. 6A = Id<1J, (6,37)
@
F
~hese forces tend to rota te the' loop so tl,a t vedol' Pm becomes (hreeted where d<D is the increment of the B,n
lr ke vee tor B: Hence, the loop is acted npon by II couple of forces magnetic flux through the loop
whose torque J~ t'9'ual to the p;'oduct of the arm Ii sin 1'1, of the couplr
by the force P, I e. upon the given displacement.
We shall prove this formula Fig. 6.15
M = lbBa 8i:' Ct.
in three stages.
1. Let us first consider a particular case: a circuit (Fig. 6.15)
with a movable jumper of length l is in a uniform magnetic
* If till' l00p .is ::ot plane, its ma:'ne,j,: rnO'lwnt
<~-
i ... IIfil
u
= l' ';'d<:.....
~t
field perpendicular to the circuit plane and directed behind
where the integnt! is LuLen OVer the slHIael' S s\.rr'tehed over th,; cur- the plane of the figure. In accordance with (6.29), the jumper
rent loop. This intf'g'l'dl does nM depend 011 the choice' of til(> surfacl: S is acted upon by Ampere's force F c;= IlB. Displacing the
and depends only on the loop over which it is stretched.
jumper to the right by dx, this force accomplishes positive
11-0181
Problem,
162 6. Magnettc Field In II Vacuum
(6.37), where d<l> is the increment of the magnetic flux
work , through the entire circuit.
6A = F dx = IBZ dx = IB dS, (6.38) In order to find the work done by Ampere's forces upon
the total displacement of the current loop from the initial
where dS is the increment of the area bounded by the con- position 1 to the final position 2, it is sufficient to integrate
tour.
In order to determine the sign of magnetic flux <I>, we expression (6.37):
2
shall agree to take t~e normal n to the surface bounded by
t~e c~ntour always In such a way that it forms with the
A= ) 1 d<l>. (6.39)
dl~ectlOn of the c~rrent in the circuit a right-handed system 1
~Ig. 6.15): In thIS case, the current I will always be a posi- ! If the current 1 is maintained constant during this dis-
tIV~ ~uantIty. On .the other hand, the flux <I> may be either
", placement we obtain
posItIve or negatIve. In our case, however, both <I> and
dCf> = B dS are positive quantities (if the field B were ': A = 1(<1>2 - <1>1)' (6.40)
dIrected towards us or the jumper were displaced to the where <1>1 and <1>2 are magnetic fluxes through the contour
left, d<I> < 0). In any of these cases expression (6.38) can in the initial and final positions. Thus, the work of Ampere's
be represented in the form (6.37). forces in this c;ase is equal to the product of the current
2. The obtained result is valid for an arbitrary direction by the incremei1t of the magnetic flux through the circuit.
of the fi.eld B as well. To prove this, let us decompose vec- Expression (6.40) gives not only the magnitude but also
tor B Into thr~e components: B ~o Bn + Bz + B x . The the sign of the accompli~hed work.
compon~nt .Bz dIrected along, the jumper is parallel to the
Example. A plane loop with current I is rotated in magnetic
~urrent In It and does not produce any force acting on the field B from the position in which n H B to the position in which
Jumper.. The component B x (along the displacement) is n tt B, 0 being the normal to the loop (it spould be recalled that the
res~onsible for the force perpendicular to the displacement, direction of the normal is connected with the direction of the current
whIch does not accomplish any work. The only remaining through the right-hand screw rule). The area bounded by the loop is S.
I) Find the work of Ampere's forces upon such a displacement, assuming
comJilonent is Bn which is normal to the plane in which that the current I is maintained constant.
'I, the Jumper moves. Hence, in formula (6.38) instead of B
In accordance with (6.40), we have
I we. must take only B n • But B n dS = d<l>, and we again
al'nve at (6.37). A = I [BS - (-BS)) = 2IBS.
3.. Let. us now ~onsider any current loop which is arbi- In the given casa, the work A > 0, while upon the reverse rotation
trarIly dIsplaced I? a constant non uniform magnetic field A <0.
It should be noted that work (6.40) is accomplished not at the
(t~e contour of tIus loop may be arbitrarily deformed in expense of the energy of the external magnetic field (which does not
thIS process). We mentally divide the given loop into infinite- change) but at the expense of the e.m.f. source maintaining the cu~re~t
l~ small current elements and consider their infinitesimal in the loop. (This question will be considered in greater detaIl III
?Ispla~ements. Under these conditions, the magnetic field Chap. 9.)
10 whICh .each element of current is displaced can be as-
sumed uOIform. For such a displacement, we can apply to Problems
each element of current the expression dA = I d'(p for the • 6.1. Direct calculation of induction B. Current I flows along
ele~entary work, where .d'<I> expresses the contribution (If a thfn conductor bent as is shown in Fig. 6.16. Find the magnetic
a gIven element of current to the increment of the flux induction B at point O. The required data are presented in the figure.
through the contour. Summing up elementary. works for 1; Solution. The required quantity B = B _ +B~, where B - is the
all the elements of the loop, we again obtain expression ,~t'

I
·~·.,
ft ..

" ,..•.,'
"".: .....•....••.

,
Y;;
164 8. Magnetic Pield in a Vacuum Problems 165

m a(l'netic ~eld created by the rectilin a r
by Its curvilinear part. According t th
on p. 146), we have 0
B
. p~rt of the loop and B ,
e IOt- avart law (see Examplef
Substituting (1) and (3) into (2) and integrating the result over r
between a and b, we obtain
B= !-to
IN In.!:
!-tol cos a da !-tol 2 (b-a) a
4na cos a o 2rui"' tg a"
(2) The magnetic moment of a turn of radius r is Pml = Irrr 2 •
B~= ~ I (2n- 2a o) a _ !-tol and of all the turns, Pm = ~ Pml dN, here dN is defined by formula
4n a2 - 2na (n-ao).
(3). The integration gives
As a result, we obtain
__ nIN bS - as _ nUl ( 2 + ab + b2) .
B = (n - ao +tg ao) !-toI /2na. Pm - b _ a -3- - 3 a
It is interesting to show that f
eXI,ression (6.13). or ao - 0, we arrive at the familiar • 6.3. Current I flows in a long straight conductor having th~
shape of a groove with a cross section in the form of a thin half-ring

Fig. 6.18 Fig. 6.19

of radius R (Fig. 6.18). The current is directed from the reader behind
the plane of the drawing. Find the magnetic induction B on the axis O.
Fig. 6.16 Solution. First of all, let us determine the direction of vector B
Fig. 6.17 at point O. For this purpose, we mentally' divide the entire conductor
into elementary filaments with currents dI. Then it is clear that the
• 6.2. A thin isolated wire fo I .
a large number N of closely packed ~s \ [ ane spIral consisting of sum of any two symmetric filaments gives vector dB directeli to the
rent I is flowing. The radii of th . urns I rough which a direct cur- . right (Fig. 6.19). Consequently, vector B will also be directed to the
and b (Fig. 6.17). Find (1) the r::.aIn er.na. and ~xternal loops are a right.
of the spiral (point 0) and (2) th gnebc .InductIOn B at the centre Therefore, for calculating the field B at point 0 it is suffIcient to
the given current.' e magnetIc moment of the spiral for find the sum of the projections of elementary vectors dB from each
current filament onto the direction of vector B:
Solution. (1) According to (6 13) h
radius r to the magnetic I'nduct: ~ t e colntribution of one turn of
IOn IS equa to B=.\dBsincp (1)

and of all the turns, B 1 = f.to Il2r, (1) In accordance with (6.11), we have
dB = /lo III /2rrR, (2)
B=JBldN,
(2) where dI ","-C (IIn) dq; (see Fig. 6.19). Substituting (2) into (1), we get
where dN is the number of turns in the interval (r, r+ drl,

dN=~dr.
h-n
166
6. Magnetic Field in a Vacuum
Problems
..• 6.4. Theorem on circulati
lIOn.. ' nsid.e a homogeneous Ion ,on .of 8 ~nd pr.!ncipl(. of sUpt"rposi_ • 6.5. Principle of superposition. Current I flows through a long
jhere IS a .clrcular cylindrical c; s.~ralg~t wlre?f ~lrcular cross section solenoid whose cross-sectional area is S and the number of turns n.
~ent of deusl ty j flows along th 0.1
i b
uctor aXIs ~nd displaced relati:~ ~t se aXI~ IS parallel to the con~
Y.a dIstance I. A direct cur-
Find th~ magnetic flux through the endface of this solenoid.
tnsldp the cavity. I.' wire. Ftnd magnetic induction B Solution. Let the nux of 8 through the solenoid endface be <1>. If
we place another similar solenoid in contact with the endface of our
S.olution. In accordance with h' . .
required quantity can be t e prmclple of superposition the
represented as follows: '
B = 80 - B'
where 8 0 is the magnetic induction of th~ conductor
(1 )
without cavity,

(a) (b)

Fig. 6.22 Fig. 6.23


solenoid, the flux through the contacting endfaees will be lI> +
lI> =
= lI>0. where ~ is the flux through the solenoid's cross section far
from its cndface. Then we have
Fig. 6.20
Fig. 6.21 11> ~ $0/2 = lLon!S /2.
while B' is the magnetic induction f h . By the way, we must pay attention to the following propertie-
to the current flOWing through th 0 ttl' ~elt at the same point due of the field B at the endfacc of a long solenoid.
be
. cn removed in order to create th par ?f. t e conductor which has 1. The lines of B are arranged as shown in Fig. 6.22. This can be
. Thus, the problem im lie fi e cavity. easily shown with the help of superposition principle: if we place on
0/
In~uction 8 inside the sO~id ~O:d~ all the ~alculation of magnetic
~Stng the theorem on circulation w~ or at ~ distance r from its axis.
the right one more solenoid, field B outside the thus formed solenoid
must vanish, which is pcssible only with the field configuration shown
Fi; ~~~~ ~~I'jih;h~~c~~¥rf~~~~'can cb~n;:~~~s::[~ ;ir~~~~' ~~~n~f in the figure.
2. From the principle of superposition, it follows that the normal
component B n will he the same over the area of the end face, since when
1
B="2f!o[jxr).
we form a composite solenoid, B n + B n = Bo, where HI, is the field
inside the solenoid away from its endfaces. At the centre of the elldface
B = B n , and we obtain B = Bo/2.
Using now this formula for B o and B' w fi d' h' .
, e n t elf dJilerence (1): • 6.6. The field Gf 8 solenoid. The winding of a long solenoid of
8_ lLO ['
- 2 J X r] - 2
lLo· 1L
[J X r']= 20 (j X (r-r')).
radius a is 11 thin conducting band of width h, wound in one layer prac-
tically without gaps. Direct current I flows along the band. Find
Figure 6.21 shoWs that r = I +'r , h
w ence r - r' = I, and
magnetic field 8 inside and outside the solenoid as a funetion of dis-
tance r from its axis.
1
B ="2 . Solution. The vector of linear current density i can be represented
flo [J :< I) . as the sum of two components:
Thus, in Our case the magnetic field B . . i=i 1 + ill"
If the current is flOWing towards us . In the cavity is uniform and
plane of the lIgure and is dl'r t d (Fig. 6.21), the Held B lies i~ the The meaning of vectors i.L and ill is clear from Fig. 6.23b. In our
ec e upwards.
case, the magnitudes of these vectors can be found with the help of
168
6. Magnetic Field in a Vacuum
Problems 169
Fig. 6.23a by the formulas
of the theorem on cirulation of vector ~:
i.l =/Cosa=1 V1-sin 2 a=(I/h) V1-(h/2na)2.
Fm = (f!0/4n) 212ll,
i lt = i sin a = Il2na. where I is the current in the wire. .
The magnetic induction B' .d h It should be noted that the two forces, electric and magnetic, are
accordance with (6.20) by th mSI.e t .1.' sole~oid is determined in directed oppositely. The electric force is resp'onsible for the. attraction
by I,,: , I . ' quantIty l.L' whIle outside the solen~id, between the wires, while the magnetic force causes their repulsion.
Let us find the ratio of these forces:
B i = f!o/.l = (f!oI/h) Vi - (h/2na)2 (r < a), FmlF e = eOf!0/2/A 2. (1)

B a = f!oilla/r = f!0I/2nr (r > a), There is a certain relation between the quantities / and t.. (see
Problem 2.8):
Where we used the theore . I . '
side the solenoid' 2nrB ~ on2cIr~u atlOn while calculating B out- '),.=C1U= Ine o U, (2)
Th . a - f!b ltal a nl]
W
.. us, h~~ing represented the current' h .
poslt!,on of transverse" and "Ion 't d' i~ t I.' solenoId as the super- where U = RI. Hence it follows from relation (2) that
the conclusion that onl th I gl. U I.na components, we arrive at /1'),. = In TJineoR.
e;Xist~ inside such a sole~oidea:dn~I~ldlilial component of the field B (3)
SIde It (as in the case of a strai~ht y I.' transverse component out- Substituting (3) into (1), we obtain
Beside 'f' d current).
d . S, I we ecrease the and 'dth . " Fm f!o In 2 l]
eDSlty unchanged, 1-+0 as h 0 hI Imamtammg the current ----- 2 - (4)
~.nldYal~?e field inside the solenoidre:Uai~~ I.h =thconstl' In. this case, ,,' Fe 80 n R2'
I I.' ,I.e. I.' so enold becomes
The resultant force of interaction vanishes when this ratio is equal
to unity. This is possible w~en R = Ro, where
~ ..6.7. Interaction of parallel
neghgtble resistance are shunted- at ~hr~ntB'd Tbwo lo~g wires with
eIr en s y reSIstor R and at Ro= .. /f!o Inl] =360Q.

:
the other 'Cnds are connected to a V eo n
sou~ce of constant voltage. The If R < RTJ then F m > Fe, and the wires repel each other. If, on
ra.dlU~ of the cross section of each the contrary, R > Ro, F m < Fe, and the wires attract each other.
WIre IS smaller than the di~tance This can be observed experimentally.
be~een t~eir axes by a factor of Thus, the statement that current-carrying wires attract each other
TJ. - 20. Fmd the value of the re- is true only in the case when the electric component of the interac-
SIstance !i at which the resultant tion can be neglected, Le. for a sufficiently small resistance R in the
fo:ce of I~teraction between the circuit shown in Fig. 6.24.
Fig. WIres vamshes.
Besides, by measuring the force of interaction between the wires
. ' Solution. There are excess sur- (which is always resultant), we generally cannot determine current I.
tIVe of whether or not the . face .charges on each wire (irrespec- This should be borne in mind to avoid confusion.
~ence, in.addition to the mC::~~~~If flowkng through them) (Fig. 6.24). • 6.8. The moment of A-mpere's forces. A loop with current /
t I.' e.lee-trlC force Fe. Suppose that orce m, we must take into account
a unIt length of the wi~e Th an exces~ charge A corresponds to i8 in the field of a long straight wire with current 1 0 (Fig. 6.25). The
length of the wire by the' other en. the elebctrlc force -exerted per unit plane of the loop is perpendicular to the straight wire. Find the moment
Gauss theorem: WIre can I.' found with the help of the of Ampere's forces acting on this loop. The required dimensions of the
system are given in the figure.
Fe=AE=A ~ E:.-_
2"-2 Solution. Ampere's forces acting on curvilinear pllrts of the loop
are equal to zero. On the other hand, the forces acting on the rectilinear
4m,0 l - - 4m l '
where I is the distance betwe h 0
parts form a couple of forces. We must calculate the torque of this
force acting per unit length :Pt~ee a~es of tbhe wires. The magnetic couple. ,
WIre can e found with the help Let us isolate two small elements of the loop (Fig. 6.26). It can be
seen from the figure that the torque of the couple of forces correspond-
r
I, j

170
6. Magnetic Field in a Vacuum

iug' to these elements is


Problems 171

dM = 2z tan cp dF, Since aB/az < 0, the projection of the force Fz < 0, Le. vector F
(t) is directed towards the loop with current /. The obtained result can
where the elementary Ampere's force is given by be represented in the vector form as follows:
dF = I diB.
(2)
The dependence of the magnetic induction
B on the distance r
It should be noted that if Pm (and hence
Z-axis as well) were directed oppositely, then
B z = -Band oBz/oz > O,and hence Fz>O
and vector Fwould be directed to the right, Fig. 6.27

--
X
Le. again oppositely to the direction of Pm.
HeBee, the obtained direction for F is valid for both orientations of Pm
• 6.10. Current I flows in a long thin-walled circular cylinder
of radius R. Find the pressure exerted on the cylinder walls.
I!'ig. 6.26 Solution. Let us consider a surface current element idS, where
i is the linear current density and dS is the surface element. We shall
from the ~traight \ . b f
circulation: vlre can I' ound with the help of the theorem on

B = !-Lo[/2nr. (3) ~ ,,'


Let us now sub~titute (3) . t (2) h 'b./ ./
ing that· it = dr ;'1d x ~= r c~~ 0 . ' t en (2) into (1) and, consicle- 08
over r between a and b. This gi~~smtegrate the obtained expression
Dr B'
M = (fto/n) II 0 (b - a) sin lp, ~ dS ~'!m
vector M being- directed to the left (I!'ig. 6.26).
l/
~B'
Bi
~ 6.9. A small coil with current h . C1 •
Pm, IS placed on the axis of a circul 'I avm"'f the magnetic moment Fig. 6.28 Fig. 6.29
current I is flowing. Find th f ce ar ~op 0 radius R, alo~ which
from the centre of the loop
in Fig. 6.27.
ft
F actlllg on the coil if its' distance
s an d vector Pm is oriented as is shown lind the relation between the surface and volume elements of the cur-
rent:
S alution According t (6 33) h
. 0 . , t e required force is defined as j dV = j6h·6b dl = i as.
F= Pm aB/an
where B is the magnetic indo ' . (t) The meaning of the lJuantities appearing in this relation is clarified
at the Iscus of the coil. Let Us ~~~:n ,o!. ~~~. field cr~ate~f by the loop in Fig. 6.28. In the vector form, we can write
tor Pm' Then the projection of (1) t Zt atXII~ III t,he d.lrechon of the vec- jdV=idS. (1)
on 0 118. aXIs wIll be
Fz = Pm aB/az = Pm ilB/az, The Ampere's force acting on the surface current element" in thiS
case is determined by the formula obtained from (6.28) with the help
II ~here w~ took into account that B = . of substitution (1):
(~rr~nt !II the loop. The magnet· .Z d B. for t~le given direction of
i (h.12), whenn' . IC III IIctlOn B I~ defmcd by formula dF = Ii X D'] dS, (2)
I!
I I
aB __ 3 f.LoRz/1
az ----Z(lZ+RZ)6/Z.
where B' is the magnetic induction of the field at the point of lo<:ation
of the given current element, but created by all other current elements
III
excluding the given one. In order to find B', we proceed as for cal-
'I
" culatin~ electric force (Sec. 2.3). Let B, be the magnetic induction
172
7. Magnetic Field in a Substance 7.1. MagneUzation of a Substance. Vector J 173
of the field created by the surface 1 '
to its surface (see Fig 6 29 wh e ~f~nt Itself at a point very close The field B', like the field Eo of conduction currents,
flowing from ~s). Acc~rding' to (~~~2) ~. a~u(T/ed th?t the current is has no sources (magnetic charges). Ilence, for the resultant
Further, USIng the theorem . i. l 2) ~Ol. field B in the presence of a magnetic, the Gaus.~ theorem
considerations we can easH OUCIrcu atwn of vector B and symmetry
field outside the cYlinder n!ars~et that the .magnetic induction of the is applicable:
I s surface IS equal to
. . . B = ~oI/21tR,
whIle InsIde the cylinder the field' b
The latter fact indicates that th fis ld s~nt.
rent elements at points 1 and 2 e e B from the remaining cur-
(3)
I fBdS~o·1 (7.2)

Fig. 6.29~ must be the same and ;:~fsf~l~h }O 11he ~ylinder. s!lrfac.e (see This means that the field lines of B remain continuous
and outsIde the cylinder surface: e 0 OWIng condItIOns mside everywhere in the presence of 11 substance.
B' = Bt and B = B' + B t = 2B'. Mechanism of Magnetization. At present, it is estab-
Hence it follows that . lished that molecules of many substances have intrinsic
magnetic moments due to the motion of intrinsic charges
. . B' = B/2. (4 in them. Each magnetic moment corresponds to a circular
SubstItutIng this result into (2) b' ) current creating a magnetic field in the surrounding space.
for the required pressure: ' we 0 tam the following expression
In the absence of external magnetic field, the magnetic
dF
p=- =
2B'
iB' = - - B' -
B2 moments of molecules are oriented at random, and hence
dS ~o - 2~ . the resultant' magnetic field. due to these moments is equal
Taking into account (3), we finally get 0 to zero, as well as the total magnetic moment of the snb-
stance. This also appHes to the substances that have no
p = ~oI2/81t2R2.
magnetic moments in· the absence of an external field.
It is clear from formula (2) th t h l' If a substance is placed into an external magnetic field,
pression. a t e cy mder experiences lateral com-
under the action of this field the magnetic moments of the
molecules acquire a predominant orientation, and the sub-
7. Magnetic Field in a Substance stance is magnetized, viz. its resultant magnetic moment
becomes other than zero. In this case, the magnetic mo-
7.1. Magnetization of a Substance ments of individual molecules no longer c.ompensate each
Magnetization Vector J • other, and as a result the field B' appears.
Field in Magnetics If a t ' Magnetization of a substance' whose molecules have no
into a magnetic field form~~r ~m sUb.stanc~ is introduced magnetic moments in the absence of external fIeld proceeds
the field will change. This is I y. c~r~ents m conductors, differently. When such materials are introduced into an
substance is a magnetic . eX~t~le y theJact that each external field elementary circular currents are induced
a magnetic moment) unde~·eih I, IS. magnetIzed (acquires in the molecules, and the entire substanc.e acquires a mag-
A magnetized substance creat e .;ctlOn of mag~Ietic field. netic moment, which also results in the generation of the
which forms, together with t~ I s .own magnetIc field B', field B'.
by conduction currents, the re=uft:~n:a~~l:eld B o created Most of materials are weakly magnetized when introduced
inlo a' magnetic. field. Only ferromagnetic materials such
B = Bo + B'. (7.1) as iron, nickel, cobalt and their many alloys have clearly
H~re B~ an~ B stand for the fields pronounced magnetic. properties.
physIcal mfimtely small volume. averaged over a Magnetization. The degree of magnetization of a magnet-
ic is characterized by the magnetic moment of a unit
115
1.1. M41,.etnatlon 01 4 Sub,tance. Vector J
114 7. Magnetic Field in a $ubdance
- . . he oint:- of their contact have op-
volume. This quantity is called magnetization and denoted adjacent mol~cules at t p tach othel'. The Oil Iy uncom-
. posite directiOns and comp~nsa_ etriOSe emerging 011 the lateral
by J. By de~nition, .
(7.3)
;1
pensated molecular curren ale currents form macroscopic
surface of the. cyl.inrler. ItesJ' circulating over the lateral
IUTjace magnetization curren.
where L\ V is a physical infmitely small volume surround- Y4 I'
ing a given point and Pm is the magnetic moment of an

'I
individual molecule. The summation is performed over
all molecules in volume L\ V.
By anillogy with what was done for polarization P
(see (3.3)1, magnetization can be represented in the form
J = n (Pm), (7.4) ___. . l'
~

where n is molecular concentration and (Pm) is the average


mllgnetic moment of a molecule. This formula shows that
-- x
vector J is collinear precisely to the average vector (Pm)' Fig. 7.2
Hence for further analysis it is sufficient to know the behav- Fit. 7.1
iour of vector (Pm) and assume that all the molecules rent]' induces the same mac-
within the volume L\ V have the same magnetic moment surface of the cyl.inder-lhe c~~at of all molecular currents
(Pm)' This will considerably simplify the understanding of roscopic magnetic fiel as
questions associated with the phenomenon of magnetiza- taken together.. er case' a magnetized magnetic
tion. For example, an increase in magnetization J of a mate- Let us now conSider anot~ t . tile molecular currents
Let for msance, .' k
rial implies the corresponding increase in vector (Pm): if is nonhomogeneous. : F' ~ 2 where the Ime thiC 'ness
J = 0, (Pal) = 0 as well. be arranged as sho~n Ill. Ig~f ~oieclllar clll'rents. It follows
If vector J is the same at each point of a substance, it is corresponds to the mtenslty J' directed behind the plane
said that the substance is uniformly magnetized. from the figure th.at vector. ~a nitude with the coord i-
Magnetization Currents 1'. As was mentioned above, mag- of the figure and Illcrea~st Irhe ;olecular cnrrents are not
netization of a substance is caused by the preferential orienta- nate x. It can be seen t a h eneous magnetic, aIllI as
tion of magnetic moments of individual molecules in one compensated inside a I?-0n omog rna ~etization current l'
cl irection. The same can be said about the elementary circular a result, th~ macrosc.OPIC vO:~~ve d1rection of the Y-axi~;
cmrents associated with each molecule and called "molecular appears, which flows III t~ Pb t the linear l' and surface ~
currents. It will he shown that such a behaviour of molecular Accordingly, we can spea a ou I I'n Aim and j' in A/m~.
., ., being measurec h t
cllrrents leads to the appearance of macroscopic currents J' current denSItIeS, t. • M netic. It can be stated t ~
called magnetization currents. Ordinary currents flowing in Calculation of Field B 10 a f ~g a magnetized magnetIC
the conductors are associated with the motion of charge the contribution to field. B t~O~ would be created by the
carriers in a substance and are called conduction currents (I). is equal to the contribution ~" a vacuum. In other
In order to understand how magnetization currents appear, same distribution o~ curre~tsd' tl':~ution of magnetization
let us first imagine II cylinder made of a homogeneous mag- words, having established t e _t~S the help of the Biot-
net ic whose magnetizatiou J is uniform and directed along currents 1', we can find, WI I'ng to them and thell
h field B' corresponc I '
the axis. Molecular cuneuts in the magnetized mag-netic Savart law, t e fi Id B by formula (7.1).
lIre oriented as shown in Fig. 7.1. Molecular CIIITcnts of caIc.ulate the resultant e
t76
7. Magnettc Fteld in a Subl1tance
7.2. Circulation of Vector J t77
U Iifortunately, . the distribut' 0
not only on the confi uratio 1 n of currents /' depends the contour r intersect the sUl'face S only once. Such molec-
J
netic but on the field as lY ~nd ~~e properties of a mag- ular currents create a macroscopic magnetization current ]'
case the problem of findin;~: or t IS re~son, in the general piercing the surface S.
directly. I t remains for us to ~n a magnetic cannot be solved r Let each molecular current be equal to 1 m and the area
~o the solution of this problem
IS the establ ishment of
iand find a~othe.r approach
. ' he first step III Hus direction
I embraced by it be Sm' Then, as is shown in Fig. 7.4, the
..' an Important reI at . b element dt of the contour r is wound by those molecular
n etIzatlOH current /' a I . IOn etween mag- currents whose centres get into the oblique cylinder with
viz. its circulation. nt a certam property of vector J, the volume dll c= Sill cos a dl, where a is the angle between
the contour element dl and the direction of vector J at the
point under consideration. All these molecular currents
7.2. Circulation of Vector J intersect the surface S once, and their contribution to the
It tUl'1lS out that for a statio . magnetization current is dI' = I mn dV, where n is the molec-
magnetization J around bJl?IY case the circulation of ular concentration. Substituting into this formula the
. an ar Itrary contour r is equal expression for dV, we obtain
dl dI' = l rn S mn cos a dl = J cos a dl = J dl,
II
I

I I
I
&6W r
Fig. 7.3 F' _
r
where we took into account that I mS 111 = Pm is the magnetic
moment of 1m individual molecular current, and I mS mn
is the magnetic moment of a unit volume of the material.
Integrating this expression over the entire contour r,
we obtain (7.5). The theorem is proved.
It remains for us to note that if a magnetic is nonhomo-
19. i.4 geneous, magnetization current I' generally pierces the
to the algebraic sum of . '. entire surface (see Fig. 7.3) and not only the region near its
by the contour r: magnetIzatIOn currents I' enveloped
boundary, adjoining the coutour 1'. This is why we can use
I
where l' - r j' dS
ItJdJ~I'·1
d'
(7.5)
the expression I' c= j' as, where integration is performed
over the entire surface S, bounded by the contour r. In
the above proof, we managed as if to "drive" the entire
cmrent I' to the boundary of the surface S. The only goal
.. - J an mtegration is performed Ov
arhltrary surface stretched on th er an of this method is to simplify the calculation of the current I'.
In order to prove this theo e contour r. Differential form of Eq.(7.5):
algebraic sum of molecular cu ,rem, we shall calculate the
Let us stretch an arbl"raI'y nenfts enSvelo ped by contour r.
(F'Ig. 7'.3). It can be seen '.
t sur ace
from tJ f
0 tl
. n Ie contour r
IvxJ=r, I (7.6)
currents intersect the surface S Ie /gure that, some molecular Le. the curl of magnetization J is equal to the magnetization current
and for the second time in 'the t~ ,ce (~H1ce .In o~le direction density at the same point of space.
such currents make no crnt 'b .pposlte directIOn). Hence A Remark about the Field of J. The properties of the
IIe t'IzatlOn
. current throllgl ) rI1I lItlOn to the,::;nant
reo It mag- field of vector J, expressed by Eqs. (7.5) and (7.6), do not
Howe. th . .d Ie surface S
\er, e molecular currents that are', imply at all that the field J itself is determined only by the
wound around cmrents I'. The field of J (which is hounded only by the
t 2-0 181
Ii

179
7.8. Vector H
~!! u 7._ Magnetic Field in a Substance .
lated to the circulation of the magnetization:
S}?ltial
,." l'jlregionthfilled by . magnet,'c)
. depends on all currents,
',Z,]l.' 1 on ,0 mag~letlZatlOn currents l' and the conduc- ~ J dl =1'. (7.9)
tl?ll currents I. However, ill some cases
wIth a definite symmetry the situation Assuming that the cirenlation of vectors Band J is taken
is such as if the field of vector J were vel' the same contour r, we express l' in Eq. (7.8) throngh
determined only by currents 1'.
ormula (7.9). Then
.Example. Fi.nd the surface magnetization
c"!ll"ent per umt length of a cylinder made
~ (:u -J) dl=I. (7.10)
?f ~ ho~ogeneous magnetic, if its magnet-
IzatIOn IS J, vector J being directed every- Let us denote the integrand of this expression by H.
where along the cylinder axis. Thus, we have found an auxiliary vector H,
Let us .apply Eq. (7.5) to the contour

\H~;'-J'\
ch D.sen as I,S shown in Fig. 7.5. It can be
Fig', 7.5
eaSIly see? that the circulation of vector J
around thl.S contour is equal to the product
(7.11)
(l. In thIS case, the magnetization current
" ' _, ., IS the surface ~urrent. If we denote its linear whose circulation around an arbitrary contour r is equal
J.rnslty ly ! ,the,contour under consideration embraces the ma ne-
tlzatlOll current i l. It follows from the equality It = i'l that g , to the algebraiO'tsum of the eonduction currents I embraced
if = J. (7.7) by this contour:
It should be noted, by the way that vectors i' 8:111 J are mutually (7.12)
perpend iClllar: i' 1. J. '

7.3. Vector H This formula expresses the theor{!m on circulation of vec-


tor H: the circulation of vector H around an arbitrary closed
T~eorem on Circulation of Vector H (for magnetic fields contour is equal to the algebraic sum of the conduction c'urrcnts
of dlre~t currents). In magnetics placed into an external embraced by this contour.
~agnetI~ fiel~l magnetization currents are induced. Hence The sign rule for curr~nts is the same as ill the case of
cl~c~latlOn .0;'
vector B will now be de.termined not only drculation of yector B (see p. 149).
b) conductIOn currents but by magnetIzation currents as It should be noted that vector H is a combination of two
well: quite different quantities, B//l o and J. Hence, vector H
is indeed an auxiliary vector which does not haye any deep
~Bdl=/lo(I+I'), (7.8) physical meaning. * However, the important property of
vector H expressed in the theorem on its circulation justifIOs
where I and l' are the conduction .and magnetization CUI'- the introduction of this vector: in many cases, it consider-
ren~s embraced by a given contour r. ably simplifies the inyestigation of the field in magnetics.
. SInce the .determination of currents l' in the general case Relations (7.11) and (7.12) are val id for any magnetics,
IS. a c0m.-phcated. problem, formula (7.8) becomes inap- induding anisotropic ones.
plIcable In practlcal respect. It turns out, however that
w~ can find a. certain auxiliary vector whose circuiation
wIll be determIned only by the conduction currents embracfJd * The quantity II is often called magnetic field intensity, hut we
shall not be using tlJis term to emphasize again its auxiliary nature.
by the contour r. Indeed, as we know, the current l' 'is
12*
('

180
7. Magnetic Field in a Sllhstaw'(,
7.3. Vector B 18t
It follows from formula (7 12) I .
vector H has. .the cli' menslOUS
. wI, lhe IIl:\<rnitude of
of. t II,, "' For paramagneties ~t > 1, while for diamagnetics fL < 1.
Hence, H is measured i l l ' Ie CUlrent pCl' !wIt ll'fI!!'tll. It should be noted that in both cases fL differs from unity
, amperes per metre (Aim).
only slightly, Le. the magnetic properties of these magnetics
Differential rorm or the tlIeO ' , are manifested very weakly.
rem on cIrculatwn or \'eCtor H:
A Remark about the Field of H. Let us consider a question
I'
I
I
V x H=j, I (7.13)
which often involves misunderstanding: which currents
determine the fIeld of veetor H? Generally, the fwld of H
i.e. the curl of vector H . I -,
current at the same pointlsofeqtuha tO tthe density of the conduction (just as the field of B) depends on all currents, both con-
e su b s ance. duction and magnetization. This can be seen even from
Relation Between Vectors J d (7.i5). However, in some cases the field H is determined
that magnetization J d d an H. It was shown above only by conduction enrrents. Vector H is very helpful for
·
a t a gIven epen' bs on the lllag ne I'IC 111'
point of the . d
uction B these cases in particular. At the same time, this makes
vect or J'IS related not withsuB stance . , Howev"C!, customarily
grounds for the erroneous conclusion that the field of H
ourselves to the analvsi" of flit WIth H. \\e slwll coniine always lkpends on conduction currents and for incorrect
the depeudence bet~~enU J . on YHt~lOS~ magnetics fOl' which interpretations of the theorem on circulation of vector H
cl1l d IS hllecu', viz.
and Eq. (7.13). This theorem expresses only a certain prop-
J = XH erty of tite lIeltl,of H without deHning the ~field itself.
where . h . ' (7.14) <It
. X IS t e magnetic susce tibil't I ' , , ' . Example. A system consists of a long straight wire with current I
quantIty typical of eacl p . l y. t IS a dllnenSlOnless
f·01'I ows from the fact tl I' t magnetIc (tl t
.
. J'
.' Ia X IS hlmension1ess
and an arbitrary piece of a papmagnetic (Fig. 7.6). Let us lind the
changes in the lieilis of Vl'ctors B and If
· . 0 f H and J aI'e tl la
SIOns
·
, accol'lllllg'
,e S<:.me ) . to (7 "11) t I LC ( f'1men- ami in the circulation of vector II arour.d
Un 1II~e electric sllsceptibilit' " . a certain fixed contour r upon the
magnetIc susceptibility X h
x ~\ Illch IS always positive
Accordingly the mag t~nay _ e .eltI~er positive hI' negative'
removal of the magnetic.
The field B at each point of space is
"d . ., ne ICS ,.,atlsf\"l f 1, ~ . • determined by the conduction current I
d IVI ed mtD paramagnetics ( fig . orIllU ,1 (i .14) are as well as by the magnetization currents
In paramagnetics, J tt H X ~ .0) a.JlII d~amagndit's (z < 0). in the paramagnetic. Since in our case,
It should be noted' that' iI~ ~~e. I.n (hamp'{!If'hl: J H H. in accordance with (7.15), If = B/l!l!o'
there exist ferromagnetic~ for' 'I ~f~tlOr to tll(',' !!lag-netics,
this also applies to the Held of vector
II, i.e. it also depends on the conduction
~laS a rather complicated' fol'!~ ,w i ~I~.' t le ~epeHde',cD J (H) current I and the magnetization cur- Fig. 7.6

. a hysteresi~" J' ,". J d epen d' sons tnot


It has rents.
n
It 1111ear, I .
and besides-,
The removal of t.he piece of paramagnetic will lead to a change
netIc. (Ferromarrnetic" will b I .. e pre IIstory of it mag- in the lield B, ami hence in the lield ll. The circulation of vector B
in Sec. 7.6.) ,., ~ e (eSenbell in greatel' dptail
around the contour r will also change, since the surface st.retched on
Relation Between Band H F the contour r will no longer be pierced by the magnetization currents.
expression (7 11) . • or magnetics olw,Xilw U·l1), Only the conduction current will remain. However, the circulation
H ence . , Z II h B/~o.
acqllJres the forlll (1 I.,) of v(!ctor II around the contour r will remain unchanged in spite of
the variation of the tield II itself.
B = p~loH, (7.1:) ) When r = 0 Inside a Magnetic? We shall show now that
where ~l is lhe !Jermeahilt·t.t/ ()f I magonetization ellrren'ls inside a magnetic are absent if (1)
. lie medilllll. tlte magnetic is homogeneolls nnd (2) there nrc no condllction
~ = 1 .:- Z. ClllTellt.S in it (j - 0). I II this case. fot' any shape of a 1l111g-
(7.1lj) netic mill allY conliglll'ation of the magnetic Held, we call be
7,4. Boundary Conditions for 8 and B
182 7. Magnetic Field in a Substance
in Fig. 7.7. Then the flux of B in the outward direction (we
ignore the flux through the lateral surface) can be written
and only surface magnetizat:~~ currents are e9
sure that volume magnetizat' ual
to zero,
in the form
In order to prove this we . currents remam.
c,II1atioll of vector J ar~ d shall use the theorem on cir-
+-
B 2n /).8 BIn' /).8 = O.
pletely lying inside the ~~gn:~itr~itrary contour r com- Taking the two projections of vector P onto the normal
geneous magnetic we can : ~ the H case of a homo- n to the interface, we obtain BIn' = -13]11' and after can-
ill Eq. (7.5) out ~f the i~teSgurbaSltItu~/ng
anu WrIte .X for J, take 'X. celling /).8, the previous equation becomes
(7.20)
l' =X~ Hdl.
i.e. the normal component of vector B turns out to he the
In accordance with (7.12.) h ' .. .
to the algebraic SUIll of th~ ~oe ~~m?mlIlg mtegral is equal

-~-_.~----
by the contour r. Hence f . n ~,ctlOn currents I embraced
have 01 a lOmogeneous magnetic, we
2
1
'1'1 " ,1"
l' = 'X/
. (7.17) ,
any contour inside the llHlgll~~·Clll~ClltsI. and I is valid for
. us Ie atlOllslup lJetweell tl ,. ' '
,,'t n'
~~nall ~ontour, when l' :-+ ~C:, III p.artlcular, for a. very
dI' Fig. 7.7
,
Fig. '7.8
Ih01l l' d8 ~ . dS' ln d8 and I -+dI - ] d8
" , r: ,,- ~ Xln ,and aftet' cancellinO' dS
L~ __ XllI.IJus eqnation holds f). n .. same on both sides of the interface. This quantity does not
t : : ' .we ohtam
small contour, Le. for an' d" (.t any orwntatIOn of the have a discontinuity.
Condition for Vector H. We shall assume, for higher
lIcllce the vectors i" au/' t~rect~oJIl, of the normal n to it. generality that a surface conduction current with linear
tile same equation: J temse \ es are related through
dens'iy i flows over the interfaeial surface of the magnetics.
. j' = xi. (7.18) Let us apply the theorem on circulation of vector H to
a very small rectangular contour whose height is negligibly
'I'll
if J'liS it0 [olio
P Ews that'III a IlOtllogcucous maguetie J" ce_' 0 small in comparison with its length l (the arrangement of
, -(. '.D. the contour is shown in Fig. 7.8). Neglecting the contribu-
tions from smaller sides of the contoUl' to the circulation,
7.4. Boundarv Conditions for Band H we can write for the entire contour
We slwll analyse tli> C d"
+-
Il2~l Hi~,l = iNl,
the interface between \ onhltlOns for vectors nand H at
conditions will be obtai w~ .omoge!IOOIlS magnetics. These
where iN is the projection of vector i onto the normal N
to the ('oQutOllI' (vector N forms a right-handed systom with
with tlte Iw.lp of the C' n~~ 'Illist as III the case of uielectrics
cnlatio/l Wo' l'ol'"lll 11 "ltllf!:iS t leOl'ern ctlt.d the theorem on
or vocto .. B alit I H these theorems
cir~ the direction of eontour circumvention). Taking the two
projections of vector H onto the comm'on unit vec.tor of tlte
have llw. form , . lil .
tangent. T (in IUf\l\illlU 2), we obtain TT lt , ,== -lfl~' C·an-
' I:>
celling l Ollt of tlwpreviolls eqllation, we get
1,BdS'O, ~Hdl~l. (7.19)
(7.21)
Condition
located dt tlte il t >'f" I
for V('dol' B I·,f'.t liS" I III [I g'1ne
..
[I very low cylinder
J et ,ICC Jt'tw('nll (wl)"lllaglletics as is shown
184
7. Magnetic Field in a SUbstance'
i.e. the tangential co - 7,/;, Firld in (7 Ilomo~eneous Magnetic t85
a discontinuity u on mpo~ent. ?f vector H general1
which is due to thPe prea tIansltlOn through the l'nteYI'f has :Taking these reilltio/ls into Ilecount. we obtllin the lllw of
If } sence of c d ' . Ilee, ~refraction of the lines of vector B (Ilnd hence of vector H
, IOwever, there are no o~ uctlOn Cllfl'ents.
f~ce between magnetics (i ~O~d) uctthlo currents Ilt the inter-
n " . weB) similllr to (2.25):
o vector H turns 0 t . - , e tangentilll co tana 2 ~
interface' u to be the same on botl 'd mponeJlt ---
(7.23)
. I Sl es of the tan a l III

1~2T=HIT·1
Fignre 7.10 depicts the fields of vector's B HIIII H near the
(7.22) illterface between two mllgnctics (in the absellce of con-
Thus, if there is no d . dut:tion currents). Here ~12 > ~ll' A compllrison of the den-
between two homo COli lICt,lOn Cllnent sities of the lines shows that lJ 2 >R 1 • while H 2 < HI'
geneons magnetics, the at the intcrfare
I I components B n The lines of B do not hllve a ~liscontinuity upon U transition
B2T
1,1
r--- through the interface, while the H lines do (due to the sur-
f fare magnetizlltion clIrrents).
I a2 The refraction of magnetic field lines is used in magnetic protection,
2 If, for example, a closed iron shell (layer) is introduced into an exter-
1 nal ma~n~tic tield. the lield lim's will be concentrated (condensed)
mainly on th~ shell itself. Inside th(' shell (in the cavity) the ma~netic
field turns out to!d>e considerably weakened in comparison wi th the
~xtern:ll lield. In other words, the iron shell ael;; like a screen. This
is used in order to protect sensitive devices from ('xtt'rnal magnetic
fields. ,
BIT
Fig. 7.9 FieJdB FieJdH
7.5. Field ill a Homogeneous ~.lgneIic
Fig. 7.10
Ilnd H T Vll ry contlJl
. lIOllSI ( . I . It was mentioned in Sec. 7.1 thllt determination of the
1hrough this interface. 6n ~~tel~~~ Il Jump) upon tl tl'ullsition resultant magnetic field in the pl'esence of arhitral'y mag-
T t n d , Hn in this case have d' er ~lln~1 '. the components
. I s h ould be noted that IsContlnlllttes.
netics is generally a complicated problem. Indeed, for this
purpose, in Ilccordance with (7.1), the fIeld B o of conduction
1I~ ;ect~r D, while vect::ci!rb ~ behav~s at the interface currents mnst be supplemented by the macroscopic fwld B'
e raellon of Lines of B T . e laves lIke E. createfl by magnetization c,ul'rents. The prohlem is that we
at the interface between 'twhe lInes of. B undergo refraction do not know beforehllnrl the configuration of magnetization
the case of dielectrics W h fl mllgnetlCs (Fig. 7 9) A. . currents. We can only state that the distribution of these
of angles (Xl and (X2=' e s Il find the ratio of th~ t~ng:IJ~~ cnrrents depends on 'the nat1ll'e and configllration of the
magnetic IlS well as on the configuration of the externlll
tan a - B 2TIB2n
----.!. lield B o, viz. the field of conduction C1ll'rents. And since we
tana l -~/B • do not know the distribution of magnetization currents, we
W I I IT In
e ~ la I confine ourselves to callnot calculate the fIeld B'.
~u~~)on .cllrre!!t at the interf:~: c~e w~e,n there is no con- The only exception is the case when the entire space
• ,lJI thiS Cllse we lillve . CCOr IJIg to (7.22) and occnpiell by the fwld B is fdled by a homogeneolls isotropic
dielectric. Let liS (",ollsider this ease in ~rp:ltel' dptail. Bllt
B 2T /1l2 = BfT/1l1 and B B first of all, we shall lIlIalyse the phenomenll obsPI'vpd whell
2n= In'
COlHlllclion Cllrrent llows along a hOlllogenpolls conduetol'
186 7.5. Field in a Homogeneous Magnetic 187

in a vacuum. Since each conductor is a magnetic, magneti- of the conduction current I pr~ctically h . coincide
d tion (the
B' ofwires
.the
zation currents also flow through it, viz. volume currents .) . d I '''ce at all pomts t e In HC
given by (7.18) and surface currents. Let us take a contour are thm ,an le......
ld f tl e magnetIzatIOll CII
".1 s differs from the indue-
fI~ BOfl ·tl field of the conuuction currents only In mag-
1 " •
embracing our current-carrying conductor. In accordance I··

with the theorem on circulation of vector J (7.5), the alge- I tIOn ° . .. 10vectors being. 1'0 1ate d'In the same way as the
de 0 the<:e
nl·t·u.,
braic sum of magnetization (volume and surface) currents is corresponding currents, VIZ. (7.24)
equal to zero everywhere since J = 0 at all points of the B' =--0 XBo.
contour, i.e. l' = I~ +I~ = O. Hence I~ = -I~, i.e. the
. ()f t.lle resultant field B B o -i' B' =
volume and surface magnetization currents are equal in Theu the inductlOll .
magnitude and opposite in direction. =(1 +X) B o 01'
Thus, it can be stated that in an ordinary case, when cur- n ,--~ fJB o. (i.25)
rents flow along sufficiently thin wires, the magnetic field
in the surrounding space (in a vacuum) is determined only I ti 'e space is filled with a homo-
This means that. when. t Ie en SI . times. In other words, the
by conductioH currents, since magnetization currents c.om- geneous magnotlc, B Increase .tJ the magnetic iielrl B upon
pensate each other (except fbI', perhaps, the points lying 't -lows the increase In . .
quantI y ft S. I. . d 1 y the f18ld with a magnetlc.
very close to the wire). fIlling the entll'o spaI· ce OCCt·\lP~e (7 )25) by tJ-f.\. we obtain
Let us now fill the space surrounding the conductor by If we divide bot 1 par s o . 0' f. ' ) '

a homogeneous nonconducting magnetic (for the sake of "l H --


- H0 \7.",6)
definiteness, we assume that it is a paramagnetic, X > 0).
. .
(in the case :lllder 50nsHleratlOn, fIe ld II. tUfllS• out to be
At the interface between this magnetic and the wire, surface
magnetization current l' will appear. It can be easily seen the same as 1II a vacllum). I valid when a homogeneous
that this CUlTent has the same direction as the eonduction Form. \lIas (7.24)-(~.26) lare abso d d by thp s~rfaces formed
CUlTent I (when X >0). t' fill' the entlre /..'0 ume oun e v . . I ')
magne lC. l S . (I field of the conduction current. .n
As a result, we shall have the conduction current 1, the o
by the lmes of B t le .. . d tio·n B inside the magnetic
surface and volume magnetization currents in the conductor this case too the magnetic 1Il uc
(tho' magnetic fIelds of these currents compensate one another . t' larger than B o· . 1 . 13'
IS tJ- IIDes. .I 1. hove the maanetie IIll uctlOn
alld hence can be disregarded), and the surfac.e magnetiza- In the cases ~orrsl( e:~~ .at · I ~lIrrents '\s connected with
tioll current l' on the nonconducting magnetic. For sufficient- of the .jiel~l ofJ ~a~:~e :;~~~~iC through the simple relation
ly thin wires, the magnetic field B in the magnetic will be magnetlzatlOlI 0 ., (7.27)
determined as the field of the eUI'l'ont I I'. + B = 1l0J.
Thus, the problem is reduced to finding the current 1'.
. b en<:ilv obtained from the formula
For this pnrpose, we surround tho conductor by a contour This c:xpresslOll can 0 L~. J l' t H' -- X and
arranged in the surface layer of the nonconducting magnetic. -- B'-~- B' if we take into account t Ja _B 0
B--- 0 ; ., _._ J/.
Let tlte plane of tho contour he perpendicular to the wire B ~-~ tJ-tJ-o , where H --
H ~. . I . > in other cases the
ax i,~, i.e. to tlte magnetization currents. ThOll, taking (7.7) As has <~.ll'oadY.1 been,~Il~ntlOpY~cl;t'~~O\~;ld forrilult;s (7.24)-
ant! (7.11) into consideration, we can write situatioll IS lilliC I mouo c o m , .
(7.27) become ,ilwppl~cablei' II" ro))';ider two c;imple exam-
l' = \~ i' dl = ~ .r dl = X If II dl. Concluding iilO :'('('.11011, ec , . .
ple,.
Acc(mlillg" 10 (7.12), it follows that r = If.
. I 'd A solenoid having nl amp,'re-
TrIO C()llji~lIl'ati()IlS of 1111' mag·llotizatioll current l' 'llld Example 1: Field H In a so \~~ltOl: ; ho'lllogene(Jus magnetic of the
turn~ per unl t length is HIler! "
~~_ 7.6. F rf' ')m.:-a::cg"..:n~e:..::.t~is~m~ , 1_8~!)
188 7. Magnetic Field in a Subs/ane,·
magnetics) and strongly Illilgnetic (fel'l'olllagnetics). It is
permeability
the magTletic.J..l > 1. F'In d th e magnetic
. mduction
. P ot' tJ\,' field in well known that in the absence of magnetic lield, para-
and diamagnetics are not magnetized and are characterized
mduction B in the solenoid is a ie~ce of mag'netJc the magnetic
. According to (6.20), in the b . by a olle-to-one cOITespol\(lence between magnetization J
t~e entIre space where t.he fi ld e~r'W 0 lonI.
o Since th.e magnetic fills and vector H.
effects>, th!! magnet.ic induc~ion ~ ers
ffi
mus trob e zero. Ignore the edge
(we larger:
/.L tImes
Ferrumagl/etics are the subst;lllces (solids) that Illay possess
spontaneous magnetization, Le. which arc Illagnetizell even
. B = flP-onI. (7.28)
In thIs
of the cast', t.he
magnetic, i.e.field
H = of vJ~.1.or II remallls
. ' , the S,;:!)n as III
. the absence
J B
The change in field B is caused b th .
current~ fl~)wing over the surface of the e appearance of magnetization
magnetJ~ IU the same direction as the
c~nd!Jcholl currents in the solenoid
wlDdl.ngw~en /.L > 1. If, however, P-< 1
the dl~ctIons of these currents will b~
opposIte.
h The obtained results are also valid
w en the magnetic has the form of a H
ve!y long rod arranged inside the sole- Fig. 7.13
no~d ,so th~t it is parallel to the sole-
,,'Fig. 7.12
nOId saxIs. a r
Exa"!ple 2. The field of straight. in the absence of eKtemal magnetic field. Typical ferro-
CUneDt III the presenee of a magnetic magnetics are iron, eobalt, allil many of their alloys. .
Su~pose that a magnetic fills a long Basic Magnetization Curve. A typical featlll'e of felTo-
~ .. cyl~nder of radius a along whose axis Fig. 7.11 magnetics is the complex nonlinear dependence J (H) or
a.g.lven current I flows. The permea-
I
blhty of the magnetic 11 > 1 Find th B (H). Figure 7.12 shows the magnetization curve for a
ferromagnetic whose magnetization for H ~= 0 is also zero.
I
magnet'
der aXi~~ In'd uc t'IOn B .as. a. function
. e the distance r from the cylin-
of
This curve is called the Imsic maKnetization curve. Even at
sinc';;~~:;::~t~~ ~~e directly the theorem on circulation of vector B comparatively small values of H, magnetization .I attains
is very helpful~ t~nc~~::;i::.s ar? ud~now!1' ~n thlis situation, vector II saturation (.I"). Magnetic induction B = [lo (ll I) also +
rents, For a circle of radiusIOn IS he errn me , on y byhconduction cur-
r, we ave 2nrH = I, w ence increases with H. After attaining saturation, lJ continues
ll
B = llP- oH = IJ.lloIl2rrr. to grow with II accorcing to the linear law B= ilo +
magnetic i~'d~a~~l IOn Bthroul~ the magnetic-vacuum interface, the
Upon a t T + const, where canst = [loIs. Figure 7.13 represents the basic
I (Fig, 7,11). c Jon ,un Jke H. undergoes a discontinuity magnetization cllrve on the lJ-H diagram.
In view of the nonlinear dependence B (II), the permeabil-
tionAn increase in
currents . e the m~gn~tIc!s
ThB insid . . c~used by surJace
. magnetiza- ity [.t for ferromagnetics cannot be denned as a constant
in the wire o~ h. ese. ~.urrents cOIUClde ,,, d:rect.ion with the current I clwracterizing tlw magnetic properties of J specific felTo-
On the other \l:ndxl~~is~~e sYhtem a.nll hence "amplify" this current. magnetic. However, as before, ills aswmed that j.l= BllloH,
current is liirectl'd' oppos\t:lyt e cil~~dder the surface magnetization
on field B in th ' .' ' .b11 I oes not produce ally effect bllt here ~l is a function of II (Fig. 7.14), The value i1m:.x
of both current: cmomagpneent!Ct' OutSIde thhe magnetic, the magnetic fields of permeability for ferromagnetics may lIe very large. For
. " sa e one anot er.
example, for pure iron pmax ~ 5,000. wilde for sHpennalloy
7.6. Ferromagnetism [.tmax = bOO/lOO.
beFerromagneties. In, Il.w g netic ,respecl, all suh~tanc(>s call it should bl~ lIoted that the concept of pCl"Iueability is
divided into weakly magnetic (para magnetics and dia-
too __?- Magnetic Field in a Substance 7.6. Ferromagnettsm 191

applieahlt) only tu till'basic lllili[lIetizatioll curve since, as The val lies of lJ r and II c for II ifferen t fl'lTomagnetics var.y
will ()I) SI'OWII, Lite n (ll) depcndcncc is nOlltllliq Ill'. over oroad ranges. Fot' transformel' steel, the hysteresis
Magnetic Hysteresis. Besides the lIonlinear d~ependence loop is narrow (He is small), while for fe.rro.magl.tetics use.d
B (Il) or.J ([j), fCiTnmn!~'netics also exh ibit magnetic hystere- for manufacturing permanent magnets it IS Wide (He IS
sis: tltr rrlal iOIl hd,\Vef>11 f] and II or J and II turns out to large). For eXilmple, for alnico alloy, He = ;')0,000 Aim
and B T = 0.9 T.
These peculiarities of magnetization curves are u.sed in a conven!ent
practical method for demagnetizing fei:romagnetlcs. ~ magnetlze.d
sample is placed into a coil through whIch an alternatIng cu~ent IS
passed, its amplitude being gradually ~educed to zero. In thIS c~se.
the ferromagnetic is subjected to multIple cyclIc reverse J.llagnehza-
tions in which hysteresis loops gradually decrease, contractIng to the
point where magn~tization is zero. .
Experiments show that a ferromagnetic i~ he~ted upon
reverse magnetization. It can be shown that til thiS case the
1

,'1,I amount of heat Qu liberated per unit volume of the ferro mal?-
Fig. 7,14 Fig. 7.15 netic is numerically equal to the "area" Sn of the hysteresIs
loop: ",'
be ambiguous and is determined by the history of the ferro-
(7.29)
magnetic's. magnetization. If we magnetize an initially
nomnaglletlZed ferromagnetic by increasing II from zero
Curie Point. As the temperature increases, the ability ?f
to tho valuo at which saturation sets in (point 1 in Fig. 7.15),
ferromagnetics to get magnetized becomes weaker, and 111
and then reduce H from +H 1 to --HI' the ma<rnetization
c~lI've will not follow path ]0, hut will take pdth 1 234. particular, saturation magnetization becomes lower. At.1l
certain temperature called the Curie point, ferromagnetIc
If we th?!! c!wnge II from -HI to +H1 the magnetization
clIrve wil] he below, and take path 4561. properties of the material disappear.
At temperatures above the Curie point, a ferromagnetic
., The ohtained closed. curve is calletl the hysteresis loop.
. becomes a paramagnetic. .
I hc ma:nmllm hysteresIs loop is ohtained whe!' ,;atlll'atioll
On the Theory of Ferromagnetism. The phYSical nat1ll'~
is attaincd ilt points] awl 4. If, however, there is no satu-
of ferromagnetism could be explained only with the help of
l:atioJl at extreme points, the hysteresis loops have similar
quantum mechanics. The so-called exchange forces may
form bnt smaller size, as if inscribed into tbe maximum
It yster'esis loop. appear in crystals under certain condi~ions. These forces
orient parallel to each bther the magnetic moments of elec-
Fig-lin' 7.15 shows that for H =-, 0 magnetization does not trons. As a result, regions (1-10 !lm in size) of spontaneolls
Yilllisit (poiut.2) and is characterized by the quantity lJ
magnetization, which are called domains, appear il.l c~'ystals.
(',,/It'd the reSidual induction. It corresponds to the residualr
Within the limits of each domain, a ferromagnetIc IS mag-
II/I/;;mtizatio,., Jr· .The existence of permanent magnets is
netized to saturation and has a certain magnetic lllomcn t.
il.'-'so.claled wl.th thiS residual magnetization. The quantity B The directions of these moments are different for rlifferent
YillllShes (polllt .'1) only under the action of the field H
domains. Hence, in the absence of external fIeld, the resultant
",hiell has the dil'Pction opposite to that of the field causing
magnetic moment of the sample is equal to zero,. and the
I Ill' lllagiletization. The qnantity Tl e is called the coercive
Jorce. s<'lmple as a whole is macroscopically TlonmagnetlZed.
When an external magnetic field is switche(l on, tllO
II

If12
7, Magnetic Field in a Suhstance

dO/llailiS ol'ipliled a/olll{ II . f' f I Problems


,'. Ie Ie t I{I'OW al tit
t I O/lldIlIS. ')/'I('lllpd £lg-aillsl Ihe fipld (' ~XPCllse f!f
g'l'Owlh IS ,·p\'P!'sih/(. 1,1 ' I ' : 111 weak i1e]tls, thIs radial direction, We shall use the theorem on circulation of magneti~­
. s rOllger' the/ds tI ' 1 ation vector J, taking as a ,con,tonr a smal.l rec.tangl? w~~se plane IS
l'l'ol'iPlilalioll or III 'lg'lIl'l,'(' I" , Ie SI/IIII lalleOIlS
' l I l O J l I P I I S wilhill II I' I perpendicular to the magnetIzatIOn current III thiS regIOn. Ihe ~rr~nge­
la,psI plaep This pl' '. . IC I'll 1['(' f omaill ment of this contour is shown in Fig. 7. i8, where the crosses llldIc~te
IIj',slel'esis alld'. oepss IS IITp veJ'si hie I' I 1
I'('sidll't1 'll't" I' . . ., W lie I {'xp aills the direction of the surface magnetization current. From the equalIty
, , ',.,lIl· lzallOlI.
Jl = i'l we get i' = J. ,
Then we can write J = XH, where H can be rletermmed from the
J>roblcllls circulation of vector II around the circle of radius r with the centre
• 7.1. Contlitions at the interface I
(Fig. 7,10) on a III (' • II the Vicinity of point A
i1gone Ic-vacuum inferfilcl', thl' magnl'tic inductjon

Fig, 7.18 Fig. 7.i9


Fig. 7.16
Fig'. 7.17
in the vacuum is B vector B f . on the conductor axis: 2nrH = I (it is clear from symmetry consider-
to the ,interfan'. Th'e p('rmeab)iIi~~JI~fg'ti1I,1 angle a~ w,ith the normal ations that the' lines of II lIlust be circles lying in the planes per-
mag-netIc illrfuction B ill tI " . ,Il nla/:rJletlc IS /I' Find Ull' pendicular to the conductor with the current l). ThiR gives
. If' m"g'nl'tlc lJl till' vidnity or '
Sullittun. The requirerl quantity , flomt A. t' = (~- i) J/2nr•

• 7.3. Circulation of vector H, A long thin conductor with cur-


11= V-B~ +/Jl rent I lies in the plane separating the space fIlled with a nonconduct-
" 1; , (t)
Considerin/:r conditions (7.20) and (7.22) ,'tt. ing magnetic of permeability ~ from a vacuum. Find the magnetic
the inll'rfacf', we lind induction B in the entire space as a function of the distance r from
Bn = B o cos ao, the conductor. Bear in mind that the lines of B are circles with centres
IJT=-'llflolfT~llItllf{ --'til '- B . on the conductor axis.
. • 0' t 0" - II 0 sIn a o,
~here H 0-, IS the tangential component o' Solution. The lines of H are clearly also circles. Unlike vector B,
Substituting" these expression~ . t (1) j veefo~ U o in the vacuum. vector H undergoes a discontinuity at the vacuum-magnetic interface.
~ lJl 0 ,We obtam
We denote by Hand H o magnetic fIelds in the magnetic and vacuum
B~B- l·/ c 2 a o T' ft"
oOS . sin 2 a , respectively. Then, in accordance with the theorem on circultaion
o of vector H around the contour in the form of a circle of radius r with
. • 7.2. Surfacemagnelizationf'urrenl " , the centre on the conductor axis, we have
JIlg conductor with current 1 " ' . A lOng' thIn currenl-carr.·_
vacu ' " 1~ al'ran<~ed
. UJn-IIl,lgn('tH' mtprfare IF'" ., 1- ~ 1'1 perpI'n I'
I
I 'j J
.1 JCU 111 .\ to a plaoe nrH + nrHo = I. (1)
netIc IS II. Find the lllleal' eu'r J,.,./ 'f' 11. \I' jlr'rnleabiU I" 01 the malY-
'
t lOll , . n'll (ell-If\, i' ()' th f' " Besides, B = B o at the interface, or
current 011 thiE in(.prJ'ac' 's .- ,,' ,', -c. .' (' sur ace magtl:ctiza-
rOllduetor. l' <I. a Hll.ctlUlI vi the Ihstallce r fruJIl Hie
~H = lI o, (2)
,";olt:tiu/I Let n" Hrst ('()/ISIIII'I' the COl j'" . ,. Solving Eqs. (1) and (2) together, we obtain
netlZatwlI current It j. 'j ,_ . f .. I 1.,~ratIOn o[ the surfac~ HlCig-
, ' .. (Colt rOIll 1- 19, 7,!, lIwt current has tltt'
t1li:s
[!~oI

.J
II = I H= I t 1I =
-j' (i-j-ft)nr' Ifo (i+t.t)nr·
13-01RI
" ~

"',
.
.
}:

:!tlh .
,,.
195
Problems
194 7. Magnetic Field in a Substance
- . I tI d ·t· width. Hence
The configurations of ttl(: lields n anti II in this case are shown in where I is the contour hClg It an r I S
l·'ig. 7.1!J. We advise the reader to convince himself that for ~t = 1, dJ '2lLH o
we arrive aL the well-known formulas for B and II in a vacuum. j;'''-~ --ar=--~ r.

• 7.4. Circulation of vectors Hand J. L\ 'direct current I flows • .' . ,clor ' f is direeted agai~st t!le norm.a1
along a long homogeneous cylindrical wire with a circular cross section The minus sIgn wdlcates t~hat 'h.
d J tern with the (IIrecllOn of Clf-
vector n which forms the flght- an sys
of radius R. The wire is made of a paramagnetic with the susceptibil-
ity X. Find (1) fIeld B as a function of the distance r from the axis ",.- ..,::::J
of the wire and (2) magnetization current density j' inside the wire.

ro~,
Solution. (1) From circulation of vector H around the circle of
radius r with the centre at the wire axis, it follows that
r < R, 211rH = I (rIR)2 (H ex: r), - ..--,

r> R, 2nrH = I (H ex: l/r).


Figure 7.20 shows the plots of the dependences H (r) and B (r).
(2) Let us use the theorem on circulation of magnetization J I--- ~
arouh'l the circle of radius r (see Fig. 7.20): 211rJ = r, where I' is r
dr
the magnetization current embraced by this contour. We find the
differential of this expression (going over from r to r + dr): -~
2n d (rJ) = dI'. Fig. 7.21
I' Fig. 7.20
I
Since dI' ~~, j'2nr dr, the above equation can bl' transforml'd as
follows: • h. ds j' is directed toward" us
cumvenLion of the c~ntour. In ot.~rth:ofi u're, i.e. volumr JIlagnetiza-
j' = .!...r +!:!...
dr
. in the region of the rJ~hht con~our~ the le!h-hand system.
II tion currents lorm Wit vec or J
I
Let us now consider that J = XH = (XI12nR2) r. This gives h s th hape of a ring with a narrow
I'
= i' 7.6. A (Jermanent magn~tdl: dia~~ter of the ring is d. The ,!?ap
j' xI/nR2.
gap between the poles. :rh~ ~j'd . f the field in the gap is n. F 111<1
It can be easily seen that this current has the same direction as the width is b, the magnetic wHuctHin/inside the magnet, ignoring the
conduction current (unlike the surface magnetization current which th magnitude" of vectors an< h
di~sipation of 'the field at the edges of t e gap.
has the opposite direction).
. ; ." on circulation of vector II around th.~
• 7.5. A long solenoid is filled wi th a nonhomogeneous isotropic Solutwn. USIll~ the the~re(r 7 22) and considering that there IS
paramagnetic whose susceptibility depends only on the distance r dashed eirele of dwmeter 19':t
from the s()lenoid axis as X = ar2 , where a is a constant. The magnetic no conduction current, we can Wfl e
induction on the solenoid axis is B o• Find (1) magnetization J and (2) (nd - b) l/-r; + bB//!o = 0,
magnetization current density j' as functions of r. ., , I I onto the direclion of cont?ur
Solution. (1) J = XH. In our case, H is independent of r (this di- where 1I-r; is the pI:oJe~honko f veet~hat it coincides with the (liredlOn
rectly follows from circulation of vector II around the contour shown circumvention (which IS ta en so •
in the left part of Fig. 7.21). Hence H = Ho on the solenoid's axis, of vector n in the gap). Hence
and we obtain bB bB (1 )
J = ar 2Ho = ar2B o//!o. Il-r;= - tJ-o(rtd-b) ;::::: - tJ-o nd .
. .' h h direction of vector II inside the
(2) From the theorem on circulation of magnetization J aroun d The minus sign mdlcates ~ at t .he \. 't'on of vector B at the same
an inflllitc1y narrow contour shown in Fig. 7.21 on the right, it follows . t·.
magnetic ma erld IS I' OpPOSI te to t eb ( IO
rcc Il l __ 0 as we 11 .
that point. It should be noted t~wLt'as J 7:10 'be found by formula (7.11),
Jl - (J + dJ) I = j;'l dr, The magnitude of magnetJzalon
13*
_.Hili .~ 1f' lid Ie
,_.._'_'2.:' . .
FlCld ill II SII!Jsfanrr Problems 197

taliiug into account (1): Solulion. (1) Let, for the sake of defmiteriess, vector B on the axis
be oirecteo upwards and coincide with the X-axis. Then, in accordance
J II/I'll II with (6.34), Fx 0= I'm aB/ax, where we took into account that the
-'1'---':"'1'-,/"-n-d- ~ -,I-II
IDagnetic mompnt Pm is directed as vector B (for paramagnetics) ant!
. .The relationship. IlI'tween Vt'r1ors B/IlO, II and J at h('nc(' replaced an by ax.
Inside the magnet IS shown in Fig. 7.23.' any point Next, since I'm 0= IV =-XllV and
eIB/ax = _2aBoxe- ax ', we have
Fx = - Axe-2ax2 , (1)

where A = 2aB:! XV/'IllO-


Having cal;ulated the derivative
dF,,/dx and equating it to zero, we ob-
tain the following equation for deter-
mining x m : 1 - 4ax 2 = 0, whence
B/I-Jo X m = 1/2 y~. (2)
(2) Substituting (2) into (1), we
Fig. 7.22 find
Fig. 7.23 ___ ,toF III .. /,c Fig. 7.24
. • 7.7. i\ Winding contailliw' "or t u
In the form of a torlls with the m~1 'II,e r
flls' ' , . I
It .
"dOUI\( on an 11'011 core
X- BltI V a'
slot of wid I h b (Firr 7 2?) I' .. t . (t , (lallle er : A narrow transversI' where we took into account that f1 ~ 1 for paramagnetics.
.... . w S CII 11\ Ie core \Vh'l t I . .
I
Ilrough Ihe winding, Ihl' magnptic illdu.t: ,I' ~hurrlen .. IS pas...o;ed
the permeability of iroll under the ( IOJI 111 e sol IS B. Find • 7.9. A long thin cylim{rical rod made of a paramagnetic with
siJ)ation of tIl!' field at the edrres o l'setI CO~II(httIOIIS, neglecting the dis- the magnetic susceptibility X and cross-sectional area S is arranged
b ICso·.
Sululio/!. In accordance with tI 't1 .
arollnd the circle of diaml'ter I (It ·FI~orelll.?1l CIrculation of v('Ctor n
. (see 'Ig. 7.22), we have
(rrd - !J) HlblIo~' NI, x dx
whl're JI and /10 are the magnitud f II' . .
that
I
spectivPly. Blsides, the nbsence of /i~l r .. In tl~on and In the slot n.',
I I Isst!'n 1011 at the edges lll('ans

B B o• 0

Fig. 7.25
lJsillg th",.:" two I'qll<ItiOiIS ,md tal;i",'
Illloll alld ,,«: d, we ohl:1I11 ... illto accollnt that B =

It dB
along the axis of a cllI'!'l'ntearrying coil. One end of the rod is at the
Jl
cenlre of the (,oil, whl'!'l' IhI' luagnetic fIeld is B, while the olh~r end
J!oN l---bB lies in the regioll when' IhI' lIlagnetic fwld is practically absent. Fint!
,. • 7.k. The foree adin« on a g t·
I· Ig. 7.24 is used for 1Il('~'l1ri~" (w' I 'j3 c Ie"
TI
t .
It' deVice shown in
which a small par:JIl1a~~eticgsph~~ I t I{ 1C1p 01 a b:~lanee) a force with
the force with which the coil acts 011 the rod.
Solutiol/. Let us IIll'ntally isolate all element of the rod of length d;T
pole of magnet .1/. The Ill~g' n r. ~ OJ v~ I1me ~. IS attracted to the (Fi£:,_ 7.25). The force acting o!,! this elemellt is given by
' e Ie IllI uctlOn on the axis of the rv I
I
Sloe I
(l'pends 011 Ihe height as B ~ B ,-"x~ . ,., 1',1 e
stants. Find (1) tlip h,.j ht ~!, whue En aud a arc con- d"~d
. ',. Pm aB
x
an .
so tha I thp allr;lcliVt' fcrrce l:"at ,,:llJch the sphere TIlust be placed
eeptibijily of lhe !"ll'alll,w;".t7c l\\~~~mum a.nd (2) the m~g-lletic SIlS- ~lIflPoSC that vector B on the axis of the roil is directed to the right
IS F , , . , . Ie lIlaXllIllI1ll forcC' ot atlr:lctioll
m (see the figure), toward::; positive valu!'s of x. Then B x = B, an = ax,
I ....

198
8. Relative Natllre of El t.
ec riC and Magnetic Fields
8.1. Electromagnetic Field. Charge Invariance 19!1
and since dpm = J S dx _
- XHS dx, we get
them. 'I'll is could be done only because the two fields were
dFx=XHSdx~_ XS B
ax -~ dB. static. In other cases, however, it is impo~sible.
Having integrated h It will be shown that electric and magnetic fields must
t is equation, we obtain
()
nlways be considered together as a single total electromagnetic
flCld. In othel' words, it turns out that electric and mag-netie
Fx = - XS \' B dH- XSB2 fields are in a certain sense the components of a single physi-
/l/lo C' - - 2/l,J'
Th . B 0 cal object which we call the electromagnetic field.
e !llmus sign indicates th The division of the electromagnetic field into electric
rod IS attracted to the cu::e:~tor ~ is dir~cted to the left, i. th
arrymg coIl ,.c. e and magnetic fields is of relative nature since it depends to
• 7.10. A small s h . a very large extent on the reference system in which the
magnetic susceptih T P ere of volume V, made of
carrying coil from thity ~, was displaced along th par~magnetic with phenomena are considered. The field wldch is constant ill
region wher th e pomt where the m ' . e aXIS of a C1lITent- one reference frame in the general case is found to vary in
against the emag~~i~gfetic field is practf:~i~cab~~~~t~~ dis B to the another reference frame.
S I . orees accomplished in th' . In the work Let us consider some examples.
o utlOn. Let us d' h IS case.
the elementar k Irect t eX-axis alon th .
placement of ill(7~rph donbe agaiI!st the ma~net~cafis o~ the coil. Then A charge moves in an inertial system K at a constant velocity v.
ere y dx IS equal to orces Upon the dis- In this system, we shall observe both the electric and magnetic fields
of this ch.u·ge, the two flelds varying with time. If, however, we go
6A= -F dx-. iJB x over to an iner~ial system K' moving together with the charge, the
x --Pm-d x
Where F . h an ' (1) charge will be'1it rest in this' system, and we shall observe only the
Th . x IS! e projection of the . electric field.
leS~;;~:eSlih~fn~~~:~~s ~hat thr~~~kt;~ ~~~: ~~~~~s~~t?
tll e X-axis
. Suppose two identical chatges move in the system K towards each
other at the same velocity'v. In this system, we shall observe both
:a ues of x. Then B = Bon dt e axis is directed tow IS orce... electric and magnetic fields, both these fwlds varying. In this case it
vn = -ax' h x . an an = a ( t h ' ard s posItIve
i~ ~h~tf~r~~'~otn~1:~I;:tfh:/:'~/:Z~~is~I~tI1?~~do~Xth; dir~ct~~~
is im:rossible to find such a system K' where only one of the lields
woul be observed.
X ,we rewrIte Eq. (1) In the system K, there exists a permanent nonuniform magnetic
field (e.g., the field of a fixed permanent magnet). Then in the system
6A= ~XHV!!!!... d _ XV K' moving relative to the system K we ~hall ob~crve a varying mag-
I. a x - - - H dB netic field and, as will be shown below, an ele<;tric fleld as well.
ntegratmg this equation bet x /l/lo .
. ween
(J
Band 0 we
, 0 ,am
hI .
Thus, it is clear that in different reference frames different
A=_~o
rr
j' B2V
BdB=-J--
relations are observed between electric and magnetic fields.
B 2/l/lo . Before considering the main points of this chapter, viz.
the laws of transformation of fields upon a transition from
one reference system to another, we must answer the
8. Relative Nature of Electric question which is important for further discussion: how do
an electric charge q itself and the Gauss theorem for vector E
and Magnetic Fields behave upon such transitions?
Charge Invariance. At present, there is exhaustive evi-
8. f. Electromagnetic Field Ch . dence that the total charge of an isolated system does not
So far . arge JAvallance
se ' we have considered ele . change with the change in the motion of c.harge carriers.
parately, without establishi . ctnc and magnetic field~ This maybe proved by the neutrality of a gas consisting
ng any clear reIat·IOn 1>etween of hydrogen molecules. Electrons in these molecules move
at much higher velocities than protons. Therefore, if the
---~\~

201
8.2. Laws of Transformation for Fields E and B
200 8. Relative Nature of Electric and Magnetic Fields
. ,;t blished in the special theory
charge depended on the velocity, the charges of the elec- of this transformatIon are e,. a}' ted way For this reason,
trons and protons would not compensate each other and the of relativity in a rather comp IC:es ondi~g derivations and
gas would be charged. However, no charge (to 20 decimal we shall not present h,ere ~het~~r co~tent of these laws, the
places!) has been observed in hydrogen gas. will pay mam at~entI~n 0 th m and the application of
Another proof can be obtained while observing the heat- corollaries followm~ rom e : e~ific problems,
ing of a piece of a substance. Since the electron mass is these laws for solvlJl~ s~;n §uppose we have two inertial
considembly smaller than the mass of nuclei, the velocity Formulation of the ro em., K nd the system K' moving .
of electrons upon heating must increase more than that of systems of reference: the system a 1 't v We know the
. th fi st system at a ve OCI Y 0'
nuclei. And if the charge depended on the velocity, the sub- relatIve to e r, ld E d B at a certain point in space
stance would become charged upon heating, Nothing ·of magn~tud~s of th~ fie s K a~hat are the magnitudes of the
this kind '.vas ever observed. and tIme m th~ systfm ' o'nt in space and time in the
'Further, if the electron charge depended on its velocity, .fields E' and B at t Ie same P a~e oint in space and time
then the total charge of a substance would change as a result system K'? (Recall tl~t t~e s d trme in the two reference
of chemical reactions, since the average velocities of electrons is a point whose COOl' lila es ::e Lorentz transformations~)
in a substance depend on its chemical composition. Cal- systems arc related through to this question is
culations show that even a weak dependance of the charge As was mentioned above,. t~Ie answer} . 1 't follows that
. h th O'y of relatIvItv from w llC I I I
on the velocity would result in extremely strong electric given III t e e I . f fi'elds are expressed by tIe
fields even in the simplest chemical reactions, But nothing the laws of transformatwn 0
of this kind was observed in this case either. formulas _ _"'-"---.----:;::----;;-;----:;:;:-----\
Finally, the design and operation of all modem particle
Ell = Ell, BII - BI" \ (8.1)
accelerators are based on the assumption that a particle's B - [voxEl/c z
charge is invariable when its velocity is changed.
Thus, we arrive at the conclusion that the charge of any
particle is a relativistic invariant quantity and does not
I' E.L+[voXBl
E .L = V -pz'
I
1
B'
.L =

i I
~.L_=
V1-~z .
"k the IOn'fitudinal and trans-
depend on the"particle's velocity or on the choice of the Here the s~Jllb()ls II ~ll( J - ~nal com ol~ents of the elec.tric
reference system. verse (relative to the vectol/ v o~ d I?s t,lle velocity of light
Invariance of the Gauss Theorem for Field E. It tlll'llS and mag-ne t ·IC c.ne I (\s , Rp ---- V 0',c ' an C I
out, as follows from the generalization of experimental in a vacuum (c 2 = 1/eof!o)'
results, that the Gauss theorem ~ E dS = qle o is valid " th
Written in terms of prOJectIOns, ese
formulas have the form

\
E~=EX' B~=B;.,:+(!ORZIC2
not only for fIXed charges but for moving charges also. I Tl
the latter case, the s1ll'face integral must be taken for a def- \
inite instant of tirnl' in a given reference system. " E'I- voE z 1'>' , (8,2)
Besides, since different inerti,ll reference systems ilre E'l""
'
.1
y 1-(IZ
J ' Ju~ 1!1_~Z

\
2
llz- v OE y /c
physically eq uivalent (ill accordaJICo with 1110' relati vi ty E Z ',l-l1 o lJ y B'
V1-~I' V1-~2
7' .,"
principle), we can state that the (;aIl8s theorelll is valid EI.-c. z =
for all inertial systern~ of reference. d X'-axes of coordinates are directed
where it is assumed that t h e X - all
8.2. Laws of Transformation for Fields E a'nd B t - xvo/c 2
t/~·
\Vltile going over from olle referellce system to anotller, V 1-(00/C)2
fields E and B are transformed in a certain .way. The laws
202
R. Relative Nature of EI('ctric and Magnetic Ficlds
8.2. Laws of Transformation for Fields E and n 203
along the vector va, the Y'-axis is parallel to th y_,' I h "
to the Z-axis. . e aXIS aUt t e Z-axIs It should be noted that the first formula in (8.1) can he
directly obtained in a very simple wa.y. Let a charge q
E' It f~II~~s. fromEqs. (8.1) alld (8.2) that each of the vectors in the system K have a velocity v o at a certain instant t.
. (I Il I. IS expressed both ill terms of E alld 8. This i~ The Lorentz force actin~ on this charge is F = qE I-
dll eVIdence of Il common nature of "],, t "
rIe-I I Is. 'I' aI
(On
·
separately each of tll"8e
"fiellls
1
<cc llC 1l1l1 magnetIc
I b I
. + q [v o X 81. We go over to the inertial system K', movilll{
. . ' . ".. taS 110 a so lite relative to the system K at the salUe velocity as the charge q
.rnealllllg:
1 ' 1speaking of eloctl'ic (lUll m'IO'II"tl'C(,.., \.' !'W I II S we JIlust
. at the instant t, viz. the velocity v o' At this instant, the
HI( I~ate t Ie system of reference in which these '6 Jd' charge q is at rest in the system K', and the force actiJl~
conSIdered. Ie s are
on the immobile ehurge is purely electric: F' ..co gE'. If
It sh?llld be emphasized that the properties of electro- Vo «c (as in the case under eonsiderlltioll), this {orec is
magnetic fwld expressed by the laws of its trllllsformation illvariant (F' = F), whellce we ol)taill the Ihst fOl'lIlula ill
a~e lo~al; ~he values of E' ,alld 8' at a certain point ill ~pace
:t d
i
tire HI he system K are uniqnely determined only by
. Ietlva IICS 0 E and 8 at the same point in space and time
(8.4). .
However, the transformation fOl'llIulas fol' magnetic Held
canbe obtained only with the help of the theory of relativity
III Ie system K.
as a result of rather cllmbersome calculations.
The following- features of the laws of field t.. ' f ' . Let us consider a simple example of the application of
al'e also . .
very Important. IclllS OJlllatlOn
formulas (8,1).
1. Unlike the transverse components of E· I 8 I' I Example. A.Jal'ge metallic plate moVl'S at a constant nonrelativistic
ch'lJlge 0 . t " , .tli( ,w IIC I
( U~.II a .ran~ItlOn hom Olle referollce system to Rn- velocity v in a'unifol'In magnetic Held 1J (Fig. 8.1). Fillll tlIP surface
°t]thor, theu. longtludmal components do not change and l'emaill density of charges iIHluced on the surfaces of the plate as a resultof
Ie same HI all systems. its motion. •
Let us go over to the reference sys-
.2. Vectors E and B in different systems are COJiliccted
WIth each other through highly symmetric relations TI'.
tem fixed to the plate. In accordance Bt
with the first of formulas (8.4), in this,
can be. seen most clearly ill the form of the Jaws of 'trm;~~ system of reference a constant uni-
formatlOll ,f~r tlte pro~ections.of the fields (see (8.2)1. form electric field \'

3. In. o.I del' to obtalll the fOJ'llltlla~


" ()f tll(", In v,er<;c t 1"1JI s- E' = Iv X Bl
t' .
ormatIOn (from K' to K), it is sufficient to I'eplac~ ill 'fo~'- will be observed. It is :directl'd towards
/Ill/las (8.1). and (8.2) all pl'imed quantities by llnprimod liS. Under the action of this external
ones (and VIce versa) as well as t.he sian of " field, the charges will be displaced so Fig. 8.1
S . I C "0'
ase of Field Transformation (u '~c) If tl _
b
peCta that positive charges will appear on
system K' moves relative to tl1 svst"ln K O a"t' . 1 'tIC
0
the front surface of the plate and negative charges, on the lJinrl sur-
( , a ve OCI y face.
L o «c, the rad lCals III tire denominators of fonn uhs (8 1')
, ' . J' " " C

The surface density a of these charges will be such that the Held
call be replaced by IInity, alld we obtain (, . created by these charges inside the plate completely con!pe.nsat~s ~he
external field E', since in equilibrium the resultant electrIc held IIIslde
Eir=E", Bir = 8", the plate must be equal to zeru. Taking into acconnt relation (1.11),
we ubtain '
E~ = E.l + (vo X B}, B~=B.l-(voxEl/c2. (8.3) a = foE' = fovR.
Hence it follows that It should be noted that while solving this problem, we coulll follow
another line of reasoning, viz. from Ihe point of vil'W of the reference
IE' ~ E+ (va X Bl, B' = B- [vo X E}/c 2 • I (8.4)
frame in whieh the plate J1loves at the velocity v. III this system there
will be an electric jidd inside the plnte. It appears as a result of the
J action of the magnetic cOlllponent of the Lorentz force causing the
205
204 fi. Rell/tive Nature of Electric and Magnetic Fields 8.2. Laws of Transformation for Fields E and n
. the fields E aud B lUust he
displacement of all the electrons in the plate behind the plane of
Fi;t. 8.1. As a result, the front surface of the plate will be char~ed
to an.other refcrence, sys~em,. '11 the eoneept of the
. tl 'u .. J:< 01' Iustance, eve .
positively and the hind surface, negatively, and the electric field treaLe(! Wi i C ie. . " char re IJy a muving magnetIC-
appearing- in the plate will be such that the electric force qE compen- force exerted. Ull a mov lU", . ~. The force is deter-
sates the magnetic component of the Lorentz force, q [v X Bl, whence field tioes not have any preCise m~a.llln~. and B at the point
E = - [ y X BI. This field is connected with the surface charge density mined by the vah~es of the q¥:ntIt~e:esult of motion of the
through the same formula lJ = BovB.
, Both approaches to the· solution of the given problem are equally wllere the charge ISElo~~t;\heira~;lue~ at this point change,
justified. . sources of th~ nellis a, well Otherwise the motion of the
the force Will ch,mge as e ~laO'nitude of the lorce. .
Relativistic Nature of Magnetism. Formulas (8.1) and ,;\' . sources does not a~ect th ob"'lem about the force actlllg
(8.2) for the transformation of fields lead to a remarkable ~t
,;, :,
Thus, while solvmg the E Pr 1 B at the point of location
conclusion: "the appearance of the magnetic field is a purely on a cIlarge, we must know . an( 't'
1 . 't v All these quantI les
relativistic effect following from the existence in nature i'·
,\>
of the charge as ~~~l a~ 1~~I:~,~e~~i~l ;eference frame we are
'~.~'.;, ~~I~:~e~~e~t~~I.l
of the limiting velocity c equal to the velocity of light in
a vacuum. .j.•.: ,._: ,: re a lve 0 K'I
If this velocity (and hence the velocity of propagation of The expression "nlOY - K B I
interactions) were infinite, no' magnetism would exist at all. ing field" (if it is lised) t -v-o..........
...
Indeed, let us consider a free electric charge. Only electric should be int~rpreted .as "
field exists in the system K where this charge is at rest. Con- a convenient way of the •q I
• t'OIl of. ~.\
sequently, according to (8.1), no magnetic field would verlJal ucscnp
,I
I L _
appear in any other system K' if c tended to infinity. This varying neh! unllor C('l-
field appears only due to the fact that c is finite, i.e. in the tain conditions, anll noth- Fig. 8.2
long run due to the relativistic effect. ing more. .
The relativistic nature of magnetism is a universal physical The following silllple tthat upon a transition
fact, and its origin is associated with the absence of mag- example will illllstraf.t.c the fa~n~ther the nel(l must be
netic charges. from one refcrcnc,c [ame to
Unlike most of relativistic effects, magnetism can be t1'oatell with care.
easily observed in many cases, for example, the magnetic .. t between the poles of a mag-
Example A charge(l partlde IS at res to the system K' that moves
field of a current-carrying conductor. The reason behind net lixed in 'the system K. Let us gOt?~~~ic velocity Yo relative to the
such favourable circumstances is that the magnetic field to the right (Fig. 8.2) at a t~n[~l~harged particle moves i~ the n~ar
may be created by a very large number of moving electric system K. (t) Can we stat~~ ,~a(2) I~ind the force acting on thiS partlc e
charges under the conditions of the almost complete vanish- netic tielll in the system .
ing of the electric field due to practically ideal balance of
the numbers of electrons and protons in cond Hctors. I TJ
\ . the system K'. . elic field. It should be no tell ,
m (t) Yes, the particle ~lUves In the rn~~'\ield and not relative to ~he
however, that it moves m. the magne • tlwt a particle moves relatIve
this case the magnetic interaction is predominant. magnetic field. It is meaIllngful to rS~iher objects. How:~ver,:~e cannot
The almost complete compensation of electric charges to the reference system, a magnlti'~e to the magnetic held. lhe ~at~:r
state that a particle I.novles ~e :: I All this refrrs not 'lIlly to magnl' IC
made it possible for physicists to study relativistic effects statement has no physlca ml'unmg.
(Le. magnetism) and discover correct laws. For this reaSOJl, but to electric tields as we}l. must take into aceount that t~l
(2) In order to lind the °lrc ! weted towards us (Fig. 8.2), WI
the laws' of electromagnetism, unlike Newton's laws, have
not<been refined after the creation of the theory of relntivity.
l
electric lield E' =- l vD.'>; B ~ ~recstem the charge will move to t ~Po
in the svst"1ll I\. . III thIS sy . ' 'n occur in crossL',ld<'CtrH'
Field Varies Rather than Moves. Since the relations he- appear I '. , lI,l this mottOll W I '
left at the ve O~·,!l.i ·_~o~ ' \ ' I of llefmiteness, we suppose •
th·.t the
twoon electric and magnetir. aelds vary upon a transition and magnetic ileitis. l' 01 t Ie sa <e
206 8. Relative Nature of Electric and lUagnetic Fields 8.9. Field Transformation Laws. Corollaries 207
~harge of the particle is q > O. Then the Lorentz force in the system K'
IS tions can be easily applied to the fields varying in space and
time. An illustrative example is the field of a uniformly
F' =qE' +q [-v o X B]=q [v o X B]-q[vo X B]=O
moving point charge.
which, h.~~v~,:er, dire~tly fo,nows from the invariance of the force upon Field of Freely Moving Relativistic Charge. The formulas
nonrelatr ,Jst1C transformatIOns from one reference frame to the other. for field transformation are interesting above all by that
they express remarkable properties of electromagnetic field.
8.3. Corollaries of the Laws of Field Transformation Besides, they are important from a
purely practical point of view since
. Some Simple Corollaries. Several simple and useful rela- they allow us to solve some problems
tIOns follow in some cases from transformation formulas in a simpler way.
(8.1). For example, the problem of deter-
'. If the ~lectric field E alone is present in the system K mining the field of a uniformly mov-
(the magnetIc field B = 0), the following relation exists v
ing point charge can be solved. ,as a
hetween the fields E' and B' in the system K': result of transformation of a purely
B' = -[Yo X E'lIc 2 • (8.5) COUlomb field which is observed in the
reference system fixed to the charge.
Indeed, if B = 0, then E~= El.lV 1 - ~2 and B' = ° Calculations sh9w (see Problem 8.10)
B~ c=. -[Yo X El/c 2 V 1 - ~~ c-= -[v o X 'E'1/c 2 , wh;re w~ that the lines 61 the field E created by
t~ok mto account that in the cross product we can write a freely moving charge q have the form
elthe~ E .01' El. (the ,same applies to the primed quantities). shown in Fig. 8.3, where'v is the ve- Fig. 8.3
I I Consldermg that B = Bj, + B~ = B~, we arrive at for- locity of the charge. This pattern cor-
mula (8.5). responds to the instant "picture" of the electric field config-
2•. If the magnetic fwld B alone is present in the system K uration. At an arbitrary point P of the reference system, vec-
(wIllIe tIte electric field E = 0), then for the system K' tor E is directed along the radius vector r drawn from the
we have point of location of the charge q to point P.
The magnitude of vector E is determined by the formula
E' = [Yo X B'l, (8.6) 1 q 1_~2
E "''' -4:rt-e-o -r-2 7":(1:-_--;;-~2;;-s-;i.:..n;;-2n{j-";;;)3'";;;/2' (8.7)
In ieed, if E = 0, then B~ = Bl.IV 1 - ~2 and Ell = 0
Ef. = [Yo, x. BlIV 1 - V Suhstituting Bl. for B and the~ where ~ = vIc, it is the angle between the radius vector r
rs.6~~r BJ. m the last cross product, we arJ'ive at formula and the vector v corresponding to the velocity of the charge.
The electric field is "flattened" in the direction of motion
Form~Ilas (8.5) and (8.6) lead to the following important of the charge (see Fig. 8.3), the degree of flattening being
conclusIOn: If only one at the fields (E or B) is present in the the larger the closer the velocity vof the diarge to the veloc-
system K, the electric and magnetic fields in the system K' ity c of light. It should be also borne in mind that the field
are mutually p,erpendicular (E' -L B'). It should be noted shoWJl in the figure "moves" together with the charge. That
t~a.t the ~p~oslte statement is true only under certain ad- is why the field E in the reference frame relative to which
dItiOnal lImitations imposed on the magnitudes of the vec- the charge moves varies with time.
tors E and B. Knowing the field E, we can find the field B in the same
Finally, since Eqs. (8.5) and (8.6) contain only the quan- reference system:
tities referring to the same system of reference, these equa- 1, 110 q[vxr] 1-~2
(8.8)
B = ~ [v< El = 4it ,.a (1- ~2 sin2 {j-)3/2 •
209
Problems
208 8. Relative Nature of Electric and Magnetic Fields
. . 'f E - 0 l' B = 0 in some reference system,
opposite IS Ba~s? true. Ither s-yste~ (This conclusion has alreadY heen
This formula is a corollary of relation (8.5) iu which we have then E'..l III any 0 .
replaced the primed quantities by unprimed ones and v drA~ifinS:llY 8~~;~ more remark: it should be borne in.mind
by -v. nilis E and B depend both on coordlllates
When v ~ c (~ «1), expressions (8.7) and (8.8) are trans- t h at genera II Y fi
". c

formed to (1.2) and (6.3) respectively. ~ B


K K'I 1

r~o
8.4. ElectYomagnetic Field Invariants -- I
I
-vo .f(11c" v
Since vectors E and B characterizing electromagnetic field I r 1 ~.
I
depend on the system' of reference (at the same point in ---- ----
space and time), a natural qu~stion arises concerning the q
q
invariants of electromagnetic field, i.e. the quantitative Fj~ 8.5
Fig. 8.01
characteristics independent of the reference system.
It can be shown that there exist two such invariants which . .... t f (8 \J) refers
d on time COllsequently, eac Il lIlvallHl 0 . d'
are the combinations of vectors E and B, viz. an . ' . 'n s ace and time of the field, the COOl' 1-
~::~eas:::i~~~~~he ~oint in different systems of .reference
jE.B=inv. and E2-c2B2=inv.1 (8.9) being connected through the Lorentz transformations.
The invariance of these quantities (relative to the Lorentz
transformations) is a consequence of field transformation Problems
formulas (8.1) or (8.2). This question is considered in greater . I f f Id transformation. A nonrelativistic
• 8.1. Specla case 0 n I~ant velocity v. Using the transforma-
detail in Problem 8.9.
The application of these invariants allows us in some l.
point charge q mo;e at a co t
field B of this charge at a point whose
. tion formulas,. fin t e ma~ne rc. defined hy the radius vector r.
cases to find the solution quickly and simply and make the h
position relatIve to tee arge IS
corresponding conclusions and predictions. We shall list t the s stem of reference K' fixed to the
Solution'h~et us go ov r ~he coJlomh fIeld with the intensity
1
the most important· conclusions. charge. In t IS system, on Y
1 q
L The invariance of E'B immediately implies that if E ..1 B E ' = - - , r,
in some reference system, Le. E· B = 0, in all other inertial systems 41tEo r
E' ..1 B' as well.
2. It follows from the invariance of E2 - c 2B2 that if E = cB is resent where we took into account that the radius :~~tof~o~~~h~
(Le. E2 - c2B2 = 0), in any other inertial system E' = cB' as well. in ~he sy~tem K' (nonrelativistic case). Leltt~s ntowth:\ystem K' at
' t th stem K that moves re a Ive 0 h
3. If in a certain reference system the angle betwl'cn vectors E system K 0 e sy. we shall use the formula for t I'
and B is acute (ohtuse) (this means that E· B is greater (Ies:;) than zero),
in any other reference system the ang-Ie between vectors E' and H' }.~i/~lf~~~(~;), ir~~v~?~(rl fi:~~l:~:r~~~m~~JI~:~g~i:Sth;I~~I~~Ai~~
by unprimed quantIties an vice '. d ' v and hence ~
will he acute (ohtus(') as well.
4. If in a certain reference system F > cB (or E < cB). (which b -Vo (Fig. 8.4). In the case under conSI ~ratJOn, vo ~ 'K' B' = 0
means that, E2 - r 2B2 is greater (or l!!ss) than zero), in any other l = B' + [v X E'l/c Considering that the system
2. III
reference system E' > cB' (or E' <; clJ 0) as well. and that c2 = 1/Eoflo, we fInd
5. If hoth invariants are equal to zero, then E 1. R ana E = cB 110 q [v X rJ
in all rf'ference systems. It will he shown later that this is precisely B= 4Jt r3 •
ohserved in electromagnetic waves.
6. If the invariant I>B alorw is equ.. 1 to zero, we can fmd a refer- We have obtained formula (6.3) which was earlier postulated as
ence system in which either E' = 0 or B' = O. The sign vI ihe other · t'JOn of experimental data.
a result of the genera 1lza
invariant determines which of these vectors is equal to zero. The
14-0181
C'

210 8. Relative Nature 0/ Electric and Magnetic Fields


Problem• 2ft
•. 8..2.. A large plate made of homogeneous dielectric with the
permlttlvltY.F 1I.lOves at a cOllstant nonrelativistic velocity v in a uni- induction B' of the llIagnetic lield at the same distance frolll the beam
form m~gneLJc. held H as shown in Fig. 8.5. Find the polarization P in the system K' that moves relative to the system K at the velocity Vo
of the 'dlelectnc and the surface density cr' of hound charges. in the direction of the proton beam.
SolUtion,. In the refereIl;ce system fixe~ to the plate, in addition to Solution. This problem can be solved in the most simple way with
the ~agne~lc field, there .wIlI he an elec,trlc field which we shall denote the help of formulas (8.1). But first we must find the induction B in
bY Eo· In ,lccord:lllce with formulas (8.4) of field transformation we the syst.em K at the same distance from the beam where the intensity E
h ave ' is given.
Using the theorem on circulation of vector B and the Gauss theo-
E~ .= [v X B).
rem for vector E, we find
The polarization of the diP!ectrie is gin'lI by
R = !JIlI/2rrr, F A/2rtfor,
, e-1
P= xeoE =. e o - - [v X HI where r is the distance from the beam, I = A.V is the current, alI,1 'A
e ' is the charge per unit length of the heam. It follows from these formulas
where we to~k into account that according to (3.29), E' = Eolt:. The that
surface denSIty of hound charges is . BIA' = fO!-tOJ/I.. = v/c 2 ,
, e-1 where c 2 = lleo!-to. Substituting the expression for B fmlll this for-
Icr I =P=lI o ---vB. mula into the last of transformation formulas (8.1), we obtain
e
At the front surface of the plate (Fig. 8.5) cr' > 0 while on the opposit B' = 1~'lv-vol
face cr' < o. " e .. ,,' c2 V1- (v o/C)2
• 8.3. S,~ppose we have an uncharged long straight wire with If in this case Vo < u, the Jines of B' form the right-handed system
a current I. ~ Illd the char~e per uni ~ length of this wire in the reference with the vector vo. If va > v, they form the left-handed system (since
system moving translatlOnally WIth a nonrelativistic velocity v the current ]' in the system K' in this case will flow in the opposite
along the conductor in the direction of the current I. 0 direction).
Soluti?n . . In aC,cordance with the transformation formulas (B.4), • 8.5. A relativistic charged particle moves in the space occupied
the electnc held E = [va X B) will appear in the moving reference by. uniform mutually perpendicular electric and magnetic Helds E
system, or
and B. The particle moves rectilinearly in the direction perpendicular
E~ = -1'0!-toII2nr (1) to the vectors E and B. Find E' and B' in the reference system moving
translationally with the particle.
where r is the distance from the wire axis.
The expression for B in this formula was obtained with the help Solution. It follows from the description of motion of the particle
of the theorem on circulation. that its velocity must satisfy the condition
On the other hand, according to the Gauss theorem (in the moving vB = E. (1)
system) we have
In accordance with the transformation formulas (B.l), we have
E; = 'A'/2m: or, (2)
where 1..' is t~le charge per ulli t length of the wire. E'= E+[vxBI =0
A companson of (1) and (2) gives Vl-~2 '
'A' = -vOIlc 2 , since in the case under consideration the Lorentz force (and hence
2
,where c = Ih 0 !:l0' The o,rigin of this charge is associated with the
the quantHy E + [v X B) is equal to zero.
According to the same transformation formulas, the magnetic
~orentz contractIOn expeflenced by the "chains" of positive and nega- field is given hy
tive charges (they have different velocities!). 2
H' = B-Iv X EI/e •
. • 8.~. Th?re is a narrow beam of protons mOVing with a relativis- l" 1_~2
tiC velOCity v II.! the ~ystem of reference K. At a certain distance from
the beam, the IlltenSlty of the electric field is equal to E. Find the The arrangement of vectors is shown in l<'ig. 8.6, from which it follows
that l v X EJ tt B. Consequently, considering that according to (1)
14*
Problems 213
212 8, Relative Nature 0/ Electric and Magnetic Fields
This equation is written for the instant t = 0, when the particle
v = EIB, we can write moved in the system K' as is shown in Fig. 8.8. Since the Lorentz
force F is always perpendicular to thE: velocity of the particle, vo =
B' = B-E'IBe' = const, and it follows from (1) that in the system K' the particle
Y1-1i' will move in a circle of radius
or in the vector form R = mvo/qB.
, Thus, the particle moves uniformly with the velocity vo in a circle
B'=BV1-(ElcB)'. in the system K' which, in turn, moves uniformly to the right with
We the same velocity vo = EIB. This motion can be compared with the
bothadvise the reader to ver'f
field invariants. I y th a t th e 0 b tamed
. expressions satisfy motion of the point q at the rim of a wheel (Fig. 8.9) rolling with the
angular velocity w = voiR = qBlm.
~ ~.~. The lIIotion of a charge ill crossed field ].' d Figure 8.9 readily shows that the coordinates of the particle q
~. Anlon-
relatIvIstIc particle with a specific charge qlrn movessI'n' aanregIOn w Iere at the instant t are given by
y x=vot-R sinwI=a (wI-sin WI),
y=R-Rcoswt=a (1-cos wt),
E
where a = mElqB2 and w = qBlm.
x • 8.7. There is only a uniform electric field E in an inertial sys
B q tern K. Find the magnitudes and directions of vectors E' and B' in
the system K' II14Yving relative to the system K at a constant relativistic
Z velocity vo at an angle a to vector E.
Solution. According to transformation formulas (8.1) and taking
Fig. 8.6 Fig. 8.7 into account that B = 0 in the system K, we obtain
u~iform, mutually perpendicular fields E. and B have been created
E'n=Ecosa, E~=Esinan/i-lit, Ii=vol c .
(FIg..8.7). At the moment t = 0 the partlcle was at point 0 d 't Hence the magnitude of vector E' is given by
velocIty was equal to zero. Find the law of motion of the pant' II s
x (t) and y (t). , a r IC e,
R'-~VE~~ I-Ei-El/(1-Bt cos' a)/(1-1i')
Solutton. The particle mov~s under the action of the Lorentz f
~t can b.e easily seen tha.t the particle always remains in the plane ~~. and the angle a' between vectors E' and vo can be determined from
tS motIOn can b~ descn~ed most easily in a certain system K' h . the formula
onI y the magnetIc field IS present. Let us find this refe n
It follows from transformations
,were tan a' = E~I Ell = tan al Vi
-lit.
.h . (84)
' that E' -_ 0 mare
.re ce sfystem.
erence
sys t em -th a t mov~s WIt a velocIty Vo satisfying the reI a t' E- The magnitude and direction of vector B' can be found in 11 similar
= -!vo XB!. It .IS more convenient to choose the s stem ~~ h - way: -
ve~ocIty Vo. IS ~hrected towards the positive value/on the ;a~~e B"=O, B~=-[voXEJ/(c'V1-1i'), B'=B~.
(FIg. 8.7), smce m such a system the particle will move perpendicula 1s
to vector B' and its motion will be the simplest r y This means that B' .l vo, and
v 2h;iB·i~h t~e l~~~er:: K' that moves to the. right at the velocity B' = voE sin al(c' V i-Ii')·
a~ce with (8e4)e d F-=- 0 8an7d onlyhthe field B' IS observed. In accord-
. an Ig. ., we ave - , • 8.8. Uniform electric and magnetic fields E and B of the same
B' = B - [yO X E]/c 2 = B (1 - v3Ic2). direction exist in a system of 'reference K. Find the magnitudes of
vectors E' and B' and the angle between these vectors in the system K'
Since for a nonrelat~vistic par~icle vo ~ c, we can assume that B' = B moving at a constant relativistic velocity Vo in the direction perpen-
, In th~ syst~m K , the partlcle will move only in the rna net' fi I .
Its ve~ocIty ~em!!, perpendicular to this field. The eqUatio~ of I~ :- d,
dicular to vectors E and B.
for thIS partIcle m the system K' will have the form 0 IOn
Solution. In accordance with formulas (8.1), in the syste lll Fe
rnv~/R = qvoB. (1)
21~ 8. Relative Nature 0/ Electric and Magnetic Fields Problem. 215

the tWo vectors E' and D' will be also perpendicular to the vector v 0 where we used the fact that [YO X BJ.Hvo X E.L] = v~J. E.L X
(Fig. 8.10). The magnitudes of the vectors E' and B' can be found by X cos a = v3B.L ·E.L (Fig. 8.11). The remaining two scalar products
the form1l\as .
in (2) are equal to zero since the vectors are mutually perpendicular.
E' =" ;- EI (VOB)I + B' = .. ;- BI+ (voElcl)S 1'hus, the right-hand side of (1) becomes
V i-(volc)I' V 1-(volc)' Erl·B{, +EJ. .BJ. =EII·B" +EJ. .B.L =E·B
The angle between the vectors E' and B' can be found from its tan- Q.E.D.
gent through the formula • 8.tO. Field E of a uniformly moving charge. A point charge q
moves uniformly and rectilinearly with a relativistic veloaity v.
Ian a' = tan (as+a B)= (tan as -Han a B)/(1- tan ai: tan ail).
Since tan ai: = voBIE and tan aB = voElelB (Fig. 8.10), we obtain
tan '"-= Vo (BI + Ellc l )
a (1-~1) EB
This formula shows that as vo -+ e (~ -+ 1), the angle a' --+ n12. The

+y'
_I-,

/ " I i)D Fig. 8.10 Fig. 8.11

l~ Find the intensity E of the fle'ld created by this charge at a point whose

lli--_
Vo 0 X'
radius vector relative to the charge is equal to r and forms an angle ~
with the vector v.
Solution. Suppose that the charge moves in the positive direction
of the X-axis in the system of reference K. Let us go over to the sys-
Fig. X.X Fil{. 8.!! tem K' at whose origin this charge is at rest (the axes X' and X of
these systems coincide while the 'Y'- and Y-axes are parallel). In 'the
opposite is also tru!': if we know E and 8 in one reference system, and system K' the field E' of the charge has a simpler form:
the angle between these vectors is less than 90°, there exist reference
systems where the two vectors E' and B' are parallel to one anotheL E' =
1
4n8o
q
---,:;r r
,
,
. • tl:9. Invaria~t E: B.. Using transformation formulas (8.1) and in the X'Y' plane we have
show thaI the quantIty E· n IS all invariant.
Solution. In the system K', this product will be E~ = 4~80 ;'3 x', E~ 4~80
='= : ' y'. (1)
E' . B' = (E'II-tE'.lHBII+BJ.)=EII·BII+EJ..BJ.'
, , " , , (1) Let us now perform the reverse transition to the initial system K,
Let us write the last term with the help of formulas (8.1): which moves relative to the system K' with the velocity -v. At the
m<tment when the charge passes through the origin. of the sys.tem K,
EJ. .BJ. = (EJ. + [Yo X BJ. I)· (BJ. - [Yo X EJ. I/e l ) the projections x and y of vector r are connected With the proJections
1-~1 (2) ;1;' and y' of vector r' through the following relations:

Considerf'hg that the vectors E J. and BJ. are perpendicular to the x=r cos 1'l'=x'Y1-Wa y=r sin {}=y' (2)
vector Yo, we transform the numerator of (2) to the form where f\ = vic. Here we took into account that the longitudin.al diIJ?-en-
sions undergo the Lorentz contraction, while the transverse. dlm~nslons
E.l.B.l-(v o!e) 2E .l.Bl. =E.l·B.l (1_~2), (3) do not change. Besides, in accordance with the transformatiOns lDvetse·
-- "'ll

9.1. Faraday's Law. Lenz's Law 217


216 8. Relative Nature of Electric and Magnetic Field.
to (8.2), we obtain
= vElc 2
, e2 = 11 follo· It should be noted that
I 2 the ratio
E:x:= E~, Ey=E~1 Vl-~I. F IF. = Follov l = (v e) ,
. ~ .e (6 5) It canI be seen that as v ---+ C,
Substituting into these expressions (1) where x' and y' are replaced h
just as in tenon relatiVIstIc h force. .
f case F'
F ten( s to e' b
by the corresponding expressions from formulas (2), we get the magnetic componentfo. ~ eaction (~pulsive force) is given y
The resultant force 0 III er
E%=_l_ L x E __ l q_ II
. _ _ 1_!fl_ yl_1\2.
4neo r'a Vl-~2' 1/- 4neo ria J/I-1\2' l=Fe-·F n1 - 4ne
o
12 .

It should be noted that E:x:IEII= xly, Le. vector E is directed


radially along the vector r. The situation is such as if the effect of lag
were absent altogether. But this is true only when v = const. If,
however, the c~arge moves with an acceleration, the field E is no longer
radial. 9. Electromagnetic Induction
K
I t remains for us to find the magni-
... v tude of vector E: d ' Law of Electromagnetic Induction.
9.1. Fara ay S Lenz's Law .
II - V~
E-
- _1_...!L /
Ex, E y - 4neo r 'S J
zS+yl
1 _ pl.
q ••- -...~ v "t R8 established that the relation
L _ Since x 2 + y2 = r 2 and In the previous chapter,~ w . 'electromagnetic field, viz.
between the "components 0 f ~n
· and tu a f7 netic fielrls, IS
Fig. 8.12 e Iec t nc ..-.• '": bv the C
mainly determmed .
a ( 1- ~2 sin 2 {t ) 3/2
= choice of the reference sy~tem. In
r 1- ~I ,
. ds., the two components
otI leI' wor I in .
in accordance with (2), the electric field intensity will be given by
of electromRgnetic fie dare -
R- _1_..!L. 1-~2 t e rlate d. We shall show here
• -- 4np. o r 2 (1-1\2 sin2 ,'})3/2 re". '11 ]osel'
that there eXIsts a stl r, I
• 8.11. Interaction between two moving charges. The relativistic .conneetlon . hetween the. ,E- IalH '
particles having the same charge q raove parallel to each other at the B-fields which is exhlllltC( I.n
same velocity v as is shown in Fig. 8.12. The separation between the henomena , 0 f e I ectroma ,..,f7 netlc
particles is [. Using expression (8.7), find the force of interaction be-
tween the particles. induction.. I 1831 Fig. 9.1
Faraday's DIscovery. n
Solution: In this case, the angle between the vector v of one of the I
particles and the direction towarrls the other particle is {t = 900. FRraday mal e.onc. of ,,' the ('1('('lrod~'lIamlcs-,
,. '11most . tl Ic, . pheno-
h t
Hence, in accordance with formula (8.7), the electric component of fl1l((lam'~lIlal ,1Iscov( IICSt, I . duction. It eonsists m t a
the Lorentz force is given by
menon of electromaKne lC {:~ induced current) appe~rs 1I1
1 q2 an electric Cl.lrrent (cRlled t hange in the maKnetlc flux
a closed conl.lc '
d t tnR l0 op 11pon a c loop
,tJ· .
Fe=qE===- 4nE o II Vi-I\I (i)
(Le. the nux of B) enclose~ ce~:-~e\lrre~t indicates that the
;y
while the magnetic component of the Lorentz force is Tile appearance of the ml U . . tile loop as a result
'
induced electromotwe . f orce.r·'nappears It ism important i here tt l~
It ql v 2
Fm=qvB= _0_
4n [I Vi-~I
,
(2) of a change of the mRgne~~ h~~'the ('lInnge in the magl\etl~
'{. rloes not at Rl! rleprnd . . I ()lllv by the rate . o!
i ' . I l ' dctel'llll1l(,(.
where we took into account that in the case under consideration B Oil " {f-. was realIzc( alH IS n'rl '1 CII'III f7 e ill the SigH
is related to E through formula (8.5) from which it follows that B = I ' . . . .I.e. by d<J>/dt.
its -,vanatlOll, , ueSl es" . < "
218
9. Electromagnetic Induction 9,1. Faraday', lAw. Len.', Law 219

of the del'ivative d<J.>/dt I d


"direction" of f . ea s to a change in the sign or regardless of the cause of the variation of the magnetic flux
i
Faraday found out that th . d embraced by the closed conducting loop. The minus sign
ated in two different wa s e In. u~e~ current may be gener- in this equation is connected with a certain sign rule. The
which shows the coil C ~iih ThIs IS Illustrated in Fig. 9.1, sign of the magnetic flux <J.> is determined by the choice of
magnetic field) and the loa clrrent I (the coil creates the the normal to the surface S bounded by the loop under
meter G which indicates th: ind con3ected to the galvano- consideration, while the sign of ~i depends on the choice
The first way consists in th ~ce current. of the positive direction of circumvention of the loop.
(or its separate parts) in the ~e1~s~~acement of t~e loop L

eb
As before we assume that the direction of the normal n
In the second case the I . the fixed call C. to the surfa~e Sand the positive direction of the loop cir-,
field varies either due to the ::~f 0 IS ¥xed b?t the magnetic cumvention are connected through the right-hand screw rule·
of a change in the current I . I.~ 0 the coIl C or as a result (Fig. 9.2). Consequently, when we choose
In all these cases the gal In I , or due to both reasons . (arbitrarily) the direction of the normal,
ence of induced current in :hanolmetelr G indicates the pres~ we denne the sign of the flux <J.> as well as
Len' L . e oop J. the sign (and hence the "direction") of the
z saw. The direction of th .
hence the sign of the induce i f )~ Induced. current (and induced e.mJ. ll' +
law: the induced current is ~.e.m .. IS determIned by Lenz's With such a choice of positive directions
cause generating it. In oth:;e~~d;o that i.t counteracts the (in accordance with the right-hand screw Fig. 9.2
creates a magnetic flux h' t r s, the Induced current rule), the qua9tities ~l and d<J.>ldt have
magnetic flux generatin~ ~~ 1 ~re;ents the variation of the opposite signs..
If, for example the 100 e Ill( ~ced,e.rn.f. The unit of the magnetic flux is the weber (Wb). If the
coil C, the magne'tic flux & L
FIg. 9.1) is.drawn to the rate of variation of the magnetic flux is 1 Wh/s, the e. m.f.
c~rre~t induced in the loop i~o~~~ the I?o~ Illcreases. The induced in the loop is equal to 1 V [see (9.1)1.
directIOn (if we look at th 1 'f .case IS In the clockwise Total Magnetic Flux (Magnetic-flux Linkage). If a closed
creates til(' magnetic fJuxe,,~?p ~od~ the right). This current loop in which an c.m.f. is illlillced contains not one but N
vents an increa!';e in It Irec e. ~o the left, which pre- turns (like H coil), 'ii will be eqnal to the sum of the e.m.f.s
induced current'. . Ie magnetIc flux, which causes the induced in each turn. And if the magnetic flux embraced by
each tUl'll is the same and equal to <Pi' the total flux <J.> through
The same situation takes I· 'f .
in the coil C, keeping fixed ihace \ we Increase the current the surface stretched over such a complex loop can be
other hand, if we decrca~e the e COl a~d the loop L. On the represented as
rent induced in the loop L 'lfu~rent Ill. the .coil C, the cur- (n.2)
now be directed countercloc~~is~ei~6I'seIlts dl~~ctioll (it wi11
Lenz's law expresses an im or we oo~ from the right). This quantity is called the total magnetic flux, or the
tendency of a system to ~ tant phYSIcal fact, viz. the magnetic-flux linkage. In this case, the e.m.f. induced in
(electromagnetic inertia) Coun eract the change in its state the loop is defined, in accordance with (9.1), by the formula
Faraday's Law of Ele~trorn .
to this law, the c.m.f i d :g~ehC Ind~ction. According 'L'. =_ )\T d<D i (9.3)
formula . n uce III a loop IS defined by the C, l' dt .

(!).1) '" If these two directions were connected through the left-hand
screw rule, Eq. (9.1) would not contain the minus si~n.
220
9. Electromagnetic Induction

9.2. Origin 01 Elet..romagnetic Induction 221


9.2. Origin of Electromagnetic Induction
Let us now consider the h . I positive direction of loop circumvention, and hence ~l
appearance of induced e f p rICa reaso~s behind· the
is a negative quantity.
induction (9.1) from whaim~e' a~n dtry to derIve the law of The product vl in (9.4) is the increment of the area bounded
two cases. . rea y know. Let us consider
by the loop per unit time (dS/dt). Hence vBl = B dS/dt =
A Loop Moves in a Per t M . . = det>/dt, where d<1> is the increment of the magnetic flux
first take a l ' man.en agnetIc Field. Let us
oop wIth a movable Jumpe~ o~ l?ngth 1 (Fig. 9.3) through the area of the loop (in our case, d<1> > 0). Thus,
.. Su~pose that It IS III a uniform mag- ~l =
-d<1>/dt. (9.5)
E=[V"B] netIc field perpendicular to the plane It can be proved in the general form that law (9.1) is
r-----€I-__ of the loop and directed behind
valid for any loop moving arbitrarily in a permanent non-
:\ the plane of the figure. We start
( : ) uniform magnetic field (see Problem 9.2).
movmg the jumper to the right with
v velo.city v. The charge carriers in Thus, the excitation of induced e.m.f. during the motion
of a loop in a permanent magnetic field is explained by the
tfe Jumper, viz. the electrons, will
action of the magnetic force proportional to [v X BI,
Fig. 9.3 a so ~tart to move with the same
which appears during the motion of the conductor.
. v~loClty. As a result, each electron
It should be noted by the way that the idea of the circuit
force F = -e [v X BJ w~ll be acted upon by a magnetic shown in Fig. 9.3 forms the basis of all induction generators
electrons will start mov·dlrected along the jumper. The in which a rofOr with a winding rotates in an external mag-
which is equivalent to ~f! d?wnwards al~:mg the jumper, netic field.
This is just the ind d ctrlC current dIrected upwards. . A Loop Is at Rest in -a Varying Magnetic Field. In this
over the surfaces ot~~e cur~nt. The ~harges redistributed case also, the induced current is an evidence of the fact that
field which will ind con ucto~s wIll create an electric
the loop. uce a current III the remaining parts of the magnetic field varying with time creates extraneous
forces in the loop. But what are these forces? What is their
The magnetic force F I h origin? Clearly, they cannot be magnetic forces proportional
The field correspondingP t~y\t .e r~~e of an extraneous force. to [v X HI, since these forces cannot set in motion the
It should be noted that h' 1 IS . = F/(-e) = [v X Bl. charges that have been at rest (v = 0). But there are no
with the help of field t t .IS expre~sIOn can also be Obtained other forces besides qE and q [v X Bl! This leaves the only
Circulation of vector r~~sformatIOn formulas (8.4). conclusion that the induced current is due to the appearance
by definition, the magnitud:r~~~~ a. cldosed contour gives, of an electric field E in the wire. It is this field which is
case under consideratI'on h e III uced e.m.f. In the responsible for the induced e.m.f. in a fixed loop placed
,we ave
into a magnetic field varying with time..
~i = -vBl, . (9.4) Maxwell assumed that a magnetic field varying with time
where the minus sign is tak . leads to the appearance in space of an electric field regardless
sign rule: the normal n t~n tf: accordance with the adopted of the presence of a conducting loop. The latter just allows
loop was chosen so that ·t· ~. surface stretched over the us to reveal the existence of this electric field due to the
Fig. 9.3 (towards the fi:ld ISB) Irect~d behind the plane of current induced in the loop.
the right-hand screw rule ,~~ he!lce, according to Thus, according to Maxwell, a magnetic field varying
vention is the clockwise l the .pOSltIV~ dlrecf.ion of circum- with time generates an electric field. Circulation of vect9r E
In this case, the extraneo~sefit~f;'E~s~s ds~own in th~ figure. of this field around any fixed loop is defined as
IS . Irected agaIllst thf
~ E dl = - ~; . (9.6)
223
9.2. Origin 0/ Electromagnetic Inductton
222 9. Electromagnetic induction
radius (see Problem 9.5). Since the electriC field is a vortexlfieldi
Here the symbol of parti I d ' . the direction of the force acting on the electrons always coincides witli
(a/at) emphasizes the fa:t t~~~v~~vel with respect to time the direction of motion; and the electrons
t h e oop and the surface continuously increase their energy. During the
~ retc ed on it are fixed. Since the flux tIl = ~ B dS time when the magnetic field increases (-- 1
ros), the electrons manage to make about a
Integration is performed aro an a. . (the
ed over the loop we are int~~eds t ed In,
y million turns and acquire an energy up to 400
!b)ltrar sur ace
we have stretch- MeV (at such energies, the electrons' veloc-
ity is almost equal to the velocity of light
a i r aB c in vacuum).
at J BdS= J at dS , Thp inductioll arcelerat.or (bptatron) re-
sembles a transformer in which the role of
the secondary winding consisting of a single
~ith
In this equalitv, we exchan d I Fig. \J.4
respect to'time and int:; a t': Ie or(\er' of differentiation turn is played by an electron beam.
IS possible since the loop anJ t~on ov~r the slll'face, which
Eq. (9.6) can be represented in theef:~~ ace are fixed. Then Conclusion. Thus, the Jaw of electromagnetic induction
(9.1) is valid when the magnetic flux through a loop varies
Jr ~
,{"Edl= _ either due to the motion of the loop or lIw tillle v,ll'iatioll
• ';Y at dS. (9.7) of the maglletic ticl!l (01' due [0 both reasons). Uowever,
we had to fcsprt to two quite different phenOi nena for ex-
becto~ j being played by the vec~r~:~/~q· ~6.17)
This equation has the same stru t
thl:' role of the plaining the f<I\V in these two cases: fol' t!le moving loop we
t. Eq.
e written in the differential form th e same as on.sequently,
(6.26), i.e. it can usatl the action of tlw maglletic force proportional to (v Y. Bl
V X E = -aB/at.
while for the magneti~ field varying with time, aB/at,
Th' . . ~~ the concept of vortex electric field E was employed.
Since there is nO unique and profound principle which
the time variation ot ma neUe IOn hetween e~ectric and magnetic
IS equatIOn expresses a local relat'
fields:
r h
the c~rl 0/ vector E at the same ~otnt !!;~d ~t gwen point determines
zero IS an evidence of the presence ~f thee aC t ~t field
eIectnc V X itself.
E differs from
.
would combine these phenomena, we must interpret the law
of electromagnetic induction as the combined effect of two
entirely different phenomena. These phenomena are gen-
The fact that the circulation of th I . erally independent and nevertheless (which is astonishing)
by a varying magnetic field d'ff e e ectflc field ind uced
~h~s electric field is not a pote:ti:~sfifl~mL~r;o indica~es that
the e.m.f. induced in a loop is always equal to the rate of va-
riation of the magnetic flux through this loop.
It IS a vortex field. Thus, electric fiel e . 1 e. magnetic field, In other words, when the field B varies with time as well
field (in electrostatics) or a vorte~ c~~l:e eIther a potential as the ConfIguration or arrangement of the loop in the field,
electrostatic field a~d the fi ld . dm~y be the sum of the
In the general case electric field E . the induced e.m.f. should be calculated by formula (9.1)
containing on its right-hand side the total time derivative
field varying in time Since the e. In :lCed by a magnetic
field is equal to ze;o, Eqs.· {9~~)~(~18\lOn of t?e elec~rostat~c
detl/dt that automatically takes into consideration the two
for the general case as well when the' Ii l~u~n. cut to ne vahd
factors. In this connection, law (9.1) can be represented
of these two ftehls.' e IS the vector sum in the form
.. all) t' Bj dl. (9.9)
<\lu E dl =, - - --\. (\) [v
at .
1D ~e induction electron accel~ratol'uavs: Ol~~ a hrilli8.~t aPillication
. Betatron. The vortex electric field' I ' J
CODSlSts of a toroidal evacuated ~h " .z. B.atron. TblS aecelerato"
0lf an electroma~et (Fig 94) ThamDer. at:rang"d between the pol<;~ The expression on the right-hand side of this equation is
e ectromagnet wmding cr~at~s ; e .varlstlon of the current in ihn the total deri vative deDI dt. Here the first term is d Ul' to
a vortex electric field. Ttl~ latt':r y\ng ,magnetic field which iIlduce;
v8: the time variation of the magnetic field, while the second
taneously retains them on an eq UJ a'~?be ~r.. 1.e3. the eillctmna and aimuI-
.. rlUm cIrcular Drbit of a certain
224 9 " 225
- - - - - , - - , - - - '. :~Irctromagnetic Induction 9.2. Origin of Electromagnetic Induction

is due. to tile mution of the 100 T1 " a permanent magnetic fIeld B is localized (it is directed perpendicularly
term IS explained in "Teater de~"1 ~e 0pl'lglll of the seconu to the plane of the figure). This region is encircled by a fixed metallic
Possible Difficulties'''' s< . al III roblern 9.2. ring R. By moving the sliding contacts to the other side of the ring
t'< J • ometllnes we cc (see Fig. 9.6), we introduce the magnetic flux l1> into the closed con-
a IOn w wn tile law of ('Iectro . . Ime across a situ- tour containing a galvanometer G (i-initial position, 2-final posi-
(9.1) tlll'f1~ Ollt to In' if!' I~agnetlc IIldllction in the form tion). Will the galvanometer show a current pulse?
difficlllti('~ associated' wi;;P~1 Ica~le. (mainly Lecallse of the Applying formally the law (9.1), we should conclude that there
I Ie, c JOIce of the contoul' itself). will be an induced current. But it is not so. There is no current since
in this case both oB/iJt and the Lorentz force are equal to zero: the
magnetic field B is constant and the closed loop moves in the region
where there is no magnetic field. Thus, we have none of the physical
'I
'I reasons underlying the law of electromagnetic induction.
On an Apparent Paradox. We know that the force acting
on an electric charge in a magnetic field is perpendicular

~s' L!
~/"':\"'}_--J.r-
to its velocity and hence accomplishes no work. Meanwhile,
during the motion of a current-carrying conductor (moving
charges!) Ampere's forces undoubtedly accomplish some
work (electric motor!). What is the matter?
Fig. 9.5 This seeming contradiction disappears if we t,ake into
Fig. 9.6
account that the*,IDotion of the current-carrying conductor
In such cases it is necessar
the Lorentz force qE + fv
t~ resort to the Lasic laws, viz.
in the magnetic neld is inevitably accompanied by electro-
magnetic induction. And since the e.m.f. induced in the
1= -aBI8t. These 1aws express
q X BI and the law -
tl J'
v E _
v /,- conductor performs work ort the charges, the total work of
aw of electromagnetic ind 1- l~ P lyslcal meaning of the the forces of the magnetic field (the work done by Ampere's
Let us consider two . utc 1O~1 In all cases.
. Ins ructlve examples. forces and by the induced e.m.f.) is equal to zero. Indeed,
E~ample 1. A conduct' t.. . upon an elementary displacement of a current loop in the
a regIOn in which a m Illg ~ rIp IS dIsplaced at a velocit ' magnetic field, Ampere's forces perform the work (see
~~~~J~~::fj~~~~ lh:~;ebI~ n;1ddo~t;J ~fr~f~e~~::!'t::St:1~oJ~~ Sec. 6.8)
.. ' . 6A A = 1 d<D (9.10)
~h~~~:~e~~~c~:n! ct;retVi~;o~~.r ~iIli ctoh~n~~\~~~~m~~~~ ~~:U~:: while the induceo\,:m.f. accomplishes during the same time
At first sight th . the work
is difficult t h e questIOn seems not so sim I . .
he "closed" ? cthoos e ~he contour itself: it is n ~ e'l smce In th!s case it 6A; = 'tJ dt = -1 d<1> (9.11)
d' III e strIp and how tho . 0 c ear where It should
f urmg. the motion of the stri I IS part of the contour will hehave
s~:i;'~IlIhb~o:rI.eSpslimmdediatelrc1:~rh~h:~v~:;~;lecrterSoonr~ ~o
tthJe Lorentz
where we took into account that c;
= -d<1>ldt. It follows
. h ace upwards h' h . S III Ie movin . from these two formulas that the total work is
In 1\ =hlui~b~o~~~~r thi~c~t,. 'd7re~~et~lo~~~~i~e an electric curren1 6A A +
6A; = O. (9.12)

~~~~?~;~:y~~g,:~~~{;:i~r!N1I{I~f:Z~!1~i~fi;r:!:;~(:~~~j
s t e case of the conducting r tr~spects, the situation is the same
III
Thus, the work of the forces of the magnetic field includes
not only the mee-hanical work (due to Ampere's forces)
but also the work done by the e.m.f. indueed during the
motion of the loop. These works are equal in magnitude and
Exam Ie 2 ., S J'Ip.
P . lhe dotted circle in F' opposite in sign, and hence their sum is equal to zero.
Ig. 9.6. shows the region in Which
15-0181
227
9. Electromagnetic induction 9.3. Self-induction

The work of Ampere's forces is done not at the expense d' If the loop is rigid and there are no ferromagnet~cs
of the energy of the external magnetic field but at the expense ~e i;:~~ighbourhood, the inductance is a constant quantlty
of the power supply which maintains the current in the loop. independent of the current /. . h
In this case, the source performs additional work against The unit of inductance is henry (H). In acco~da~ce w:~
the induced e.m.f., equal" to 6Aad 0= -'til dt = I dW, (9 14) the inductance of 1 H corresponds to t e oop e
which turns out to be the same as the work 6A A of Ampere's m~gnetic flux through which is equal to 1 Wb at the current
forces. of 1 A which means that 1 H = 1 Wb/A.
The work 6A which is done during the displacement of E ' I Find the inductance of a solenoid, neglecting edge effec~t
the loop against the impeding Ampere's forces (which xamp e. f th olenoid be V, the number of turns p~r um
Let the voluIDde hO e s t"c permeability of it substance inside the
appear due to the current induced in accordance with Lenz's length n, an t e magne I "

law) is transformed into the work of the induced e.m.f.: solenoid fJ.. 0 •

d' t (9 t4) L = $/1. Consequently, the problem IS .re-


6A = -6A A = 6.,1 i. (9.13) Accor lllg 0 '.'. f th t tal magnetic flux $ assummg
duced to the dete~ml!1atlOn. or thee cu~rent I the magnetic held in the
From the point of view of energy, this is the essence of the that t~e ~urre~ I IS ;re~il~~agnetic flux through a turn of, th~ sole~
solen?ld IS B - fJ.~o . IS whole the total magnetic flux piercIng II
operation' of all induction generators. noid IS $1 = BS - 11I1oll , I
turns is <P = N'" -
'Vi -
l B(' = r
It·" IlIlon2Vl
r '
9.3. Self-indudion where V = Sl. JIence the inductance of the solenoid is
L = l1!1o nIV. "(9.t5)
. Electromagnetic induction appears in all cases when the
magnetic flux through a loop changes. It is not important On Certain Difficultiel!l. ~t should be noted that the ~eter­
at all what causes the variation of this flux. If in a certain mination of inductance wlth the help of formula L -: wi/
loop there is a time-varying current the magnetic field of is fraught with certain difficulties. ~o m~tter how t~~n the
this current will also change. And this leads to the variation .. b its cross-sectional area IS fU11te, and we ~lmply
of the magnetic flux through the contour, and hence to the wue ma y e.'h to draw in the body of the conductor a geo-
appearance of induced e.m.f. do not k nOw ow I I t' <D The result
metrical contour required for ca eu a .mg. ' . .
Thus, the variation of current in a circuit leads to the - b' For a sufficiently thm Wlre thlS amblgu-
appearance of an induced e.m.f; in thi~ ~;ir(',pit. This phe- ~ec~m~s amatelgrl~aolusH'owever for thick wires the situation is
' ",~ " Ity IS Imm .. ., I f I I t' fL
nOmenon is called self-induction. t' ly different: in this case, the resu. toea c.u a 1O~ 0
Inductance. If there are no ferromagneiics in the space en lre t'n a gross error due to the mdetermlllacy m the
where a loop with current I is located, field B and hence ma~ cO~f a: he geometrical contour. This should be always
total magnetic flux W through the contour are proportional ~hO~C~ 'nd We shall later shOW (see Sec. 9.5) that t~ere
-to the current I, and we can write ~~ist;~~~~he~ method of determining L, which is not subJect
W = LI (9.14) to the indicated drawbacks. . .
E.M.F. of Self-induction: According to (9.1), a var~~t~~I~
where L is the coefficient called the inductance of the circuit. of the current in the circUlt leads to the appearance
In accordance with the adopted sign rule for the quantities e.m.f. of self-induction ~a
Wand I, it turns out that <1> and I always have the same
signs. This means that the inductance L is essentially a '€ _" _ d$ = _ ldd (10/). (9.16)
ea - dt t
positive quantity. . t' of
The inductance L depends on tlti~ shape and size of-tfle If the inductance L remainsconstan~ up.on a vana lon
loop as well as on t he magnetic properties of tlw surrounding the current (the configuration of the ClrcUlt does not change
15*
229
9.3. Self-induction
'228 9. Electromagnetic Induction
. for a very short time and then dis-
and there are no ferromagnetic:'», then the key short-circUIts t~e s?urc\hout breaking the latter.
connects it from the hl[h~\tnd;ctance coil L starts to decreasei.w:Jld~
The current t h roug f self-inducti~n e.mJ. ~s = -
'f B = -L ~
dt (L=const). (9.17) leads to the appearance 0

The minus sign here indic.ates that 'f s is always direeted L


so that it prevents a change in the current (in accordance
with the Lenz's law). This e.mJ. tends to keep the current
constant: it eounteracts the current when it increases and
sustains it when it decreases. In self-induction phenomena,
the current has "inertia". Therefore, the induction effects
strive to retain the magnetic flux constant just in the same
way as mechanical inertia strives to preserve the velocity
of a body.
Examples of Self-induction Effects. Typical manifesta- (a) (b) o
Fig. 9.8
tions of self-induction are observed at the moments of c.on- Fig. 9.7
nection or disconnection of an electric circuit. Tho illcrease . , law counteracts the decrea:re in
in the current before it reaches the stead v-state value after which, in accor,danc~ wIth L;~~ s th~ current in the circuit WIll be
closing the circuit and the dOGl'oase in "the current when the current. At"each ~nstant ~ ~~eiR, or
the circuit is disconnected Dceur grJdually and not insti!nt- determined by Ohm 5 law - s
.. dl (9.18)
aneously. These effects of lag are the morc pronounced the RI=-L dt-·
larger the inductance of the eircuit.
Any large electromagnet has a largo inductance. If its . hI es, we obtain
Separating t he varIa
winding is diseonnected~from the source, tlw current rapidly dl R
diminishes to zerO and creates during this process a huge T=-T dt .
e.mJ. This often leads to the appearance of a Voltaic arc . rover 1 (between I 0 and I) and over t
between the contacts of the switch which is not only very Integration of thIS ~qua {on(111'o) = -RLlt, or
dangerous for the eftlctromagnet winding but may even prove (between 0 and t) gl ves n (9.19)
I=Ioe-t/l:
fatal. For this reason, a bulb with the l'{)sistance of the
same order of magnitude as that of the wire is normally where 'ris a constant having the dimension "Of time:
connected in parallel to the electromagnet winding. In this (9.20)
or = LIR.
case, the current in the winding decreases slowly and is not the relaxatol! time). This quantity
a hazard. It is called the time constant (O\n the current: it follows fro,!, «(J.t.9)
Let us now consider in greater detail the mode of vanishing characterizes the rate .of deh-ehs~he current decreases to (tIe) tIlI!es I~S
and establishment of the current in a circuit. that or is the time durIllg w IC . f 'r the slower the decrease 1':1 ~ e
initial value. The larger th value ° f 'the dependence I(t) descrlbIllg
Example 1. Current collapse upon the disconnection of a circuit. CUff('nt. Figure 9.8 shows ~ e c~rveh °
ve 1)
the decrease of current wIth tIme (cur . I f a circuit
Suppose that a circuit consists of a coil of constant inductance L, .. t· r current upon c osure 0 •k
a resistor R, an ammeter A, an e.mJ. source~· and a special key K Example 2. Stablhza Ion 0 . 1\ . turn the switch S countercloc -
(Fig. 9.7a). Initially, the key K is in the lower position (Fig. 9.7b) At the instant t = 0, we rapH Ysition (Fig. 9.7b). We thus co.n-
and a current 10 = ~/R flows in the circuit (we assume that the re- wise from the upper to the .lo;er po coil L. The current in. the CI~­
sistance of the source of e.m.f. ~ is negligibly small). nected the source ~ to ~he III If~t~d~~tion e.mJ., coun,teractlllg t~~
At the instant t = 0, we rapidly turn key K clockwise from the cuit starts to grow, an a s~n I~ceordance with Ohm slaw, RI -
lower to the upper position (Fig. 9.7 a). This leads to the following: increase, will &iain appear.
9.4. Mutual Induction
231
230 9. Electromagnettc Induction

= ~ + ~., or of the field of the induced current and of the. ext!,!rn~l current throu~
the ring are equal in magnitude and opposite m SIgn. Hence LI -
dl
RI=~-L­ (9.21 ) = na'B, whence
dt 1= na2 B/L.
We transp?se ~ to the left-hand side of this equation and introduce This current in accordance with (6.13), creates a field B I = 1tl1oaB/2~
a new varIable u = RI -~, du = R dl. After that we transform at the cent;e of the ring. The resultant magnetic induction at this
the equation thus obtained to '
point is given by
du/u = -dtIT, B = B- B I = B (1 - 1tl1oa/2L).
res
where 't = L/R is the time constant.
Integration over u {between - 'jg and RI - ~) and over t (be-
tween 0 and t) gives n [(RI - ~)/(-~)) = -tIT, or 9.4. Mutual Induction
1= 1 0 (1_e- t /'f), (9.22) Mutual Inductance. Let us consider two fixed loops 1
where J 0 = 'jg/R is the value. of the steady-state current (for t ->- 00). and 2 (Fig. 9.9) arranged sufficiently close to each other.
EquatIOn (9.22) shows that the rate of stabilization of the currentis If current I flows in loop 1, it creates through loop 2
?etermin.ed by the same constant T. The curve I(t) characterizing the the total m~gnetic flux $2 proportional (in the absence of
II
I.
IDcrease m the current with time is shown in Fig. 9.8 (curve 2).
ferromagnetics) to the current II:
On the Conservation of Magnetic Flux. Let a current
loop move and be deformed in an arbitrary external mag- $2 = .{i21Il' (9.23)
netIC field (permanent or varying). The current induced in Similarly, if current 12 flows in
the loop in this case is given by loop 2, it creates through the
1= 'jgt+~. = __
1 d$ contour 1 the total magnetic
R R dt' flux
If the resistance of the circuit R = 0, d$/dt must also be (9.24)
equal to zero, since the current I cannot be infinitely large. 9.9
Hence it follows that cI> = const. The proportionality factors L 12
Thus, when a superconducting loop moves in a magnetic and L 21 are called mutual indue- .
field, the magnetic flux through its contour remains constant. tances of the loops. Clearly, mutual' inductance IS nu-
This conservation of the flux is ensured by induced currents merically equal to the magnetic flux through one of the
which, according to Lenz's law, prevent any change in the loops created by a unit current in the other loop. The coef-
magnetic flux through the contour. ficients L 21 and L 12 depend on the shape, size, an~ mutual
The tendency to conserve the magnetic flux through arrangement of the loops, as well as on the magnetic perme-
11 coutour' always exists but is exhibited in the clearest form
ability of the medium surrounding. the loops: These coef-
in the circll~ts of supercoilductors. ficients are measured in the same umts as the lllducta?ce L.
Reciprocity Theorem. Calculations show (an~ expenments
. .Examp.le. A superconducting ring of radius a and inductance L' confrrm) that in the absence of fel'romagnetlcs, the coef-
IS m .a u~Iform magnetic field B. In the initial position, the plane of
the rmg IS parallel to vector B, and the current in the ring is equal ficients L 12 and L 21 are equal:
t~ zero. The ring is turned to the position perpendicular to vector B. (9.25)
Fmd the current in the ring in the final position and the magnetic
induction at its centre.
. The magne~ic flux through the ring does not change upon its rota- This remarkable property of mutual inductance is usually
tIOn and remams equal to zero. This means that the magnetic fluxes called the reciprocity theorem. Owing to this theorem\ we
....,...

233
232 9. Electromagnetic Induction 9,4. Mutual Induction
--_.-=.=~-

, . theorem becomes inapplicable.


do not have to distinguish between L 12 and L.J.l and can situation, and th~ ree~I~r()cI1ty I I of the following concrete
simply speak of the mutual inductance ·of two circuits. Let us verify thiS Wltl1 t. Ie 1e p
The meaning of equality (9.25) is that in any case the U~~. .
magnetic flux <1>1 through loop 1, created by current I . l' uder of volume V has two W1D~-
in loop 2, is equal to the magnetic flux <1>2 through loop 2, Example. A long ferromagnetic ?Y l
contains nl turns per umt
jngs (one over the other). ~ne w:n ~~gFind their mutual inductance
created by the same current lin loop 1. This circumstance length while the other contams n2 ur .
often alIows us to considerably simplify the calculation, ignori~g the edge effects,
for example, of magnetic fluxes. Here are two examples. __ cD.)I This means that we must create
Accordinjt t? (9:23), L -::-alculat~ the total magnetic flux throu~h
2a
I' Example f. Two circular loops 1 current II in Wll1~llld~ 1 a; If winding 2 contains N 2 turns, we obtam
all the turns of Will lllg ,
and 2 whose centres coincide lie in a $2 = N 2B 1S,
plane (Fig. 9.10). The radii of the loops
are 4 1 and a,. Current I flows in f th c linder Considering that
where S is the cross-sectio1l1~ jreaJ~nglhe a~d B 1 ~ f!1~lonlII' where
loop 1. Find tile magnetic flux 4>2 em- e
Fig. 9.10 braced by loop 2, if 41 <: 41' N 2 = n2 l , where ~ is the cYb~l' t ; for cll'rrent II' we writ~ <1>2 ~~
is the magnetic permea I I Y
The direct calculation of the flux 4>1 f!1 VI where)1 = lS. Hence
= /l1f!onln2 l'
is clearly a rather complicated problem L 21 = f!1f!onln2 V,
since the configuration of the field
itself is complicated. However, the SimilarlY, we"lcan fwd L 12 :
application of the reciprocity theorem L 12 = f!2f!onln2 V.
greatly simplifies the solution of the • " he last two expressions are generally
problem. Indeed, let us pass the same Since the values of f!1 and fl2 II. \ I and I in ferromagnetics), the
current 1 through loop 2. Then the different (they depend on curren ~ ld 2
magnetic flux $1 created by this cur- . f L nd l do not comCI e. 'l'ng
rent through loop 1 can be easily values 0 21 a .'12 The presence of a magnetIC ~OI;IP 1 •
found, provided that al « 4 2 : it is suf- Mutual InductIOn. "f t d 'n that any varIat10n of
. 't '. mam es e l f
ficient to multiply the magnetic induc- between clrcUlS IS . . 't leads to the appearance 0
Fig. 9.1
tion B at the centre of the loop (B = current in one of t,he ~rcll~1 s , circuit. This phenomenon
=/loI12a2) by the area 1ta¥ of the circle an ind uced e .m.L 1I1 t ~ 0 leI
and take into account that 11>2 = $1 in accordance with the reci- is called mutual inJuctzon. 1 f 's appearing in
procity theorem. According to Faraday:s law, t 1e e,m ..
Example 2. A loop with current 1 has the shape of a rectangle.
Find the magnetic flux $ through the hatched half-plane (Fig. 9.ft) contours 1 and 2 are given by
whose boundary is at a given distance from the contour. Assume that d$ d/
this half-plane and the loop are in the same plane.
ce
CI- dt
1 = -L 1,)- 2
-dt '
In this case too, the magnetic field of current I has a complex con-
figuration, and hence it is very diffIcult to directly calculate the flux lD d<l>~ ell t (\:I. 2tl)
in which we are interested. However, the solution can be considerably 't z = -at =. - L 2 \ dt'
simplified by using the reciprocity theorem.
Suppose that current I flows not around the rectangular contour th at the contOtlfS are fIxed
but along the boundary of the half-plane, enveloping it at infinity. respec t'Ive 1y. Here
. ' we
, assume
a netics in the SUf!'OlllJ(I'lUgS. .
The magnetic field created by this current in the region of the rectan- and there are no fenom 11 .
d tiOil the current appeal'1ng,
gular loop hall a simple configuration-this is the field of a straight Taking into account se -lIl He'n c'urrents of hoth eircuits
current. Hence we can easily lind the magnetic flux $' through the sav, in contour 1 upon a ch~)I~ge ,1 hw thrOllft!i the formlll:J
re.ctanguJar contour (with the help of simple integration). In accordance is'determinod according- to 1m S , eo
With the reciprocity theorem, the reqUired flux $ = <1>' and the
problem is thus solved. dl( dI 2•
Rl/l=~I-LI F - L12 dt '
, However! the presence of ferro magnetics changes the
234
9. Electromagnetic Induction
9.5. Magilettc Field Erler", 235
where ~1 is the extraneous e f '
induced e.m.f.s) and L is th m : 'd In contour 1 (besides e.m.f. ~o. As we a-lready know, the current in the circuit
A similar equation can I be ~t In ufctance of contour 1. will start increasing. This leads to the appearance of a self-·
contour 2. WrI en Or the current 1
2
in induced e.m.f. ~,. In accordance with Ohm's law, RI =
It should be noted that tr f = ~o + ~" whence
for transforming currents a dans ~rmers, devices intended
principle of induction. n vo tages, are based On the lo = RI - ~•.
On • the Sign of L 12· Un l'k .' d
1 e In uctance L h' h Let us find the elementary work done by extraneous forces
mentIOned earlier is essentiall . . w IC , as was (Le. the source of ~o) during time dt. For this purpose, we
inductance L i~ an l b ! a POsItIve quantity, mutual multiply the above equality by I dt:
12 a ge rate quantity (which may in
particular, be equal to zero). This is ~oI dt = RI2 dt - ~,I dt.

00 00 due to the fact that, for instance in


(~.23) the quantities cD 2 and II r~fer
;all dtfferent contours. It immediately
o ows from Fig. 9.9 that the si n
o~ the. magnetic flux cD 2 for a giv:n
Taking into account the meaning of each term and the rela-
tion ~, == -dcD/dt, we can write
6A ntr = flQ + I d(]).

dIrectIOn of current I will d d It can be seen that in the process of current stabilization,
(a} (b) when the flux (]) varies so that dcD > 0 (if I > 0), the work
the h' f 1 epen on
c OICe 0 the normal to the Sur- done by the s(Jurce of ~o turns out to be greater than the
Fig. 9.12 fahc~ boufnhded by contour 2 (or On the Joule heat liberated in. the circuit. A part of this work
. c OICe 0 . t e positive direction of cir~ (additional work) is performed against the self-induced
The positive directi~~~!oentIOn of this contour). e.m.f. It should be noted that after the current has been
contours can alwa 'S be I' curre!1 ts (and e.m.f.s) in both stabilized, d(]) = 0, and the entire work of the source of ~o
direction of cont~ur ci~hosen ar~Itra~Ily (and the positive will be spent for the liberation of the Joule heat.
the right-hand screw rUl~)~:le~t~n IS huniquel~ (through Thus, the additional work accomplished by extraneous
normal n to the surface bo d ~ e to t e dIrectIOn of the forces against the self-induced e.m.f. in the process of current
long run with the sign of u~~lee b y th~ contour, i.e. in the . stabilization is given by
these directions are s e . magnetIc flux). As SOOn as
assumed to he Positi':e C.lr~~' the quantity !-12 should be
~hrough the contours a:e ~~~i~~~esfoof mu~~al induction
IflA add
= I d<l>·1 (9.27)
I.e. when they coincide in sign witl : If?Sltive. currents, This relation is of a general nature. It is valid in the
In other word:,;, L > 0 'f f. ~ ~e -InductIOn fluxes. presence of ferromagnetics as well, since while deriving it
tours "magnetize" ea~h otl I 0 o~ pOs.Itlve currents the con- 110 assumptions have been made concerning the magnetic
cases, the positive di~ect:~~'s o~ e~wIse, L 12 :<- O. In special properties of the surroundings.
t?UI'S can be established b f . CircumventIOn of the con- Here (and below) we shall assume that there are no ferro·
SIgH of the quantity L ,e ~Iehall(~ so that the req uired magnetics. Then dcD = L dI, and
. 12 can e obtaJlIed (Fig. U. U).
fJAadd cc= LI dI. (9.28)
9.S. Magnetic Field Energy Having integrated this equation, we obtain A add =
Magnetic Energy of C t L ' = L[2/2. According to the law of conservation of energy,
containing induction C~i~r~n ~udet us. connect a fixed circuit any work is equal to the increment of any kind of energy .
.J resistor R to a source of It is clear that a part of the work done by extraneous forces
9.5. Magnetic Field Energy 237
236 9. Electromagnetic Induetlon
Thus, as in the case of the electric field er~ergy, we ar~ive
(t 0) is spent on increasing the internal energy of conductors at the conclusion that the magnetic energy IS also localIzed
(it is associated with the liberation of the Joule heat); while in the space occupied by a magnetic field.
another part (during current stabilization) is spent on some- It follows from formulas (a.30) and (9.31) that the m~g­
thing else. This "something" is just the magnetic field, since netie energy is distributed in space with the volume density
its appearance is associated with the appearance of the
current.
Thus, we arrive at the conclusion that in the absence of Iw=¥=-,~, \ (9.32)
ferromagnetics, the contour with the induction coil £
and current [ has the energy It should be noted that tltis expl'e~sion call be applie.d

Iw = +Lf2 = i- [<I> = ; . .1
only to the media for whieh till' B VE'. II dependence is
(9.29) lin;ar, i.e. p in the relation B,= I-tl-toH is inrlependent.of H.
In other words, expressions (tl.31) (lnr! (fl.32) are applicable.
This energy is called the magnetic, or intrinsic, energy of only to para magnetics and diamagnetics. III t.he ease 01
current. It can be completely converted into the internal fefi:olllagneties, these expression::, ,Il'l' illill'plieable*
energy of conductors if we disconnect the source of to I l sho'tdd also be Hoted that
from the circuit as shown in Fig. 9.7, Le. if we rapidly turn the llwglletic elll'rgy is an essen-
the key K from position b to position a. tially positive ".quantity. This
Magnetic Field Energy. Formula (9.29) expresses the can be easily seen from the last
magnetic energy of current in terms of inductance and cur- two form ulas.
rent (in the absence of ferromagnetics). In this case, however, Another Approach to the Sub-
the energy like the electric energy of charged bodies, can be stantiation of Formula (9.32). Let
directly expressed in terms of magnetic induction B. Let us prove the validity of this formula
us show this first for a simple case of a long solenoid, ignoring by proceeding "the ot~er way roun~", Fig. 9.13
Le. let us show that If formula (9.32)
field distortions at its ends (edge effects). Substituting the is valid, the magnetic energy of the
expression L = flflon2V into (9.29) we obtain current loop is W = LJ2/2. . . .
For this purpose, we conSider the magnetIc field of an. arbltrar,Y
W = (112)£[2 = (1/2)flflon2[2V. loop with current I (Fig. !U3). Let us imagine that the entire fiel? IS
And since nI = H = B1flflo,. we have divided into elementary tubes whose generatrices are the fIeld lIn~s
of B. We isolate in one such tube a volume element dV = dt as.
W=--V=-V.
BI B.H
(9.30) According to formula (9.32), the energy ~ BH dt dS is localized in
211110 2
This formula is valid for a uniform field filling the volume V this volume. .
Let us now find the energy dW in the volume of the entire ~lementa-
(as in the case of the solenoid under consideration). ry tube. For this purpose, we integrate the latter expressIOn .alon,g
The general theory shows that the energy W can be ex- the tube axis. The flux dcD == B dS through the tuue cross-sectlOIl IS
pressed in terms of Band H in any case (but in the absence
of ferromagnetics) through the formula * This is due to the fact that in the long run expressions (9.31)
and (9.32) are the consequences of the formula 6A add =, J d(fJ and of
l~ = J B;JI dV.! (9.31)
the fact that, in the absence of hysteresis, the work 6A add i~ spent only
on increasing the magnetic energy dW. For a ferromagnetic medIUm,
the situation is different: the work 6A add is also spent on increasing
The integrand in this equation has the meaning of the the internal energy of the medium, i.e. on i~s }:eating.
energy contained in the volume element dV.
..
"!'·:1t
t-

238 9. Electromagnetic Induction 9.6. Magnetic Energy of Two Current l,oop.~ 239

constant along the tube, and hence d<1J can be taken out of the integral: Let us find this work over the time dt:
dW = dq,{ II dl = I d<1J
2 ';Y 2 •
c5Aadd = -(tal + til) II dt - (£82 + ei2) /2 dt = dW.
Here we used the theorem on circulation of vector H (in our case, pro- Considering that t SI = -LldIlldt, til = -L 12 dI 2 ldt,
jection HI = H). etc., we transform this formula as follows:
Finally, we sum up the energy of all elementary tubes:
dW = L1 / 1 d/ 1 +L 12 [1 d/ 2 1- L 2 / 2 d/ 2 1- L 21 / 2 (1I 1 •
W= i I .\ d¢J=I¢J/2=Lf2/2, Taking into account the fact that L 12 = L 21 , we represent
'" this expression in the form
where ¢J ,= LI is the total magnetic flux enveloped by the current
loop, Q.E.D. dll' = d (LJ~/2) + d (L 2 /;t'2)

Determination of Inductance from the Expression for


wlleflce
+ d (L 12 / 1 / 2 ), ~
~
Energy. We have intlfoduced the inductance L as the pro-
portionality factor between the total magnetic flux (J) and
ellrJ'Cllt l There is, however, another possibility of ealculat- IW=¥ +¥ -I LIZ/liz· \ (!I.:H) Fig. 9.14
in/.{ 1.1 froUl the exprcssioJl for energy. Indeed, COlli paring
formulas (9.31) and (9.29), we see that iIl' the absence of The first two terms in this expression are called the intrinsic
ferromagnetics, energies of CUl1'ents II and /2' while the last term, the mutual
l' BI energy of the currents. Unlike the intrinsic energies of cur-
L=Ji" \ -dV. (9.33) rents, the mutual energ¥ is an algebraic quantity. A change
• IJlJo
in the direction of one of the currents leads to the reversal
The value of L found in this way is free of indeterminacy of the sign of the last term in (9.34), viz. the mutual energy.
associated with the calculation of the magnetic flux <1>
in formula (9.14) (see p. 227). The discrepancy appearing Example. Suppose that we have two concentric loops with currents
I and I whose directions are shown in Fig. 9.14. The mutual energy
sometimes in determining L through formula (9.33) and ot these ~urrents (W12 = 1. 2111 2) depe~ds on thr~e.alge~raic.quantit~es
from the expression (9.14) for the flux is illustrated in whose signs are determineJ by t~e chOIce of pos~tlve dIrectIOns of CIr-
PrOblem 9.9 about a coaxial cable. cumvention of the two loops. It IS useful to verIfy, however, that .the
sign of W (in this case W 12 > 0) depends only on the mutual ?flen-
tation of lhe currents themselves anft is independent of the chOice of
9.6. Magnetic Energy of Two Current Loops the positive direction~ of circumventi?n of t~e loops. We recall that
the sign of the quantity La was conSidered m Sec. 9.4.
Intrinsic and Mutual Energies. Let us consider two fixed Field Treatment of Energy (9.34). There are some more
10J}ps 1 and 2, arranged near one another at a sufficiently important problems that can be solved by calculating the
small distance (so that there is a magnetic coupling between magnetic energy of two loops in a different way, viz. from
them). We assume that each loop includes its own source the point of view of localization of energy in the field.
of constant e.m.f. Let us close each loop at the instant
Let B 1 be the magnetic field of current. /1' and ~2. the
t = O. In each loop, its Own current will start being t-lsLab- field of current /2' Then, in accordance WIth the pnnciple
lished, and hence self-induced c.m.f. t 8 and the c.m.£. Ci of superposition, the field at each point is B~-= B 1 -t- B 2 ,
of mutual induction will appear. The additional work done
and according to (9.31), the field energy of this system of
in this case by the sources of eonstant e.m.£. against ~s
and t i is spent, as was shown above, for cTcating the mag- currents is W = ~ (H2/2~lflo) dV. Substituting into this
netic energy. formula .13 2 B~ + B; + 2B 1 B 2 , we obtain
nO
~ . -=.9-,-._ Electromagnetic Induction
9.7. Energy! and Forces in Magnetic Field 241


I W= i-')~
1_ _ ~l.lJ.lo
dV+ r 2LdV+
J 2l.lf ot
I' -!Jl·B
J l.ll.lo
2 dV I
•I
(9.35)
into account that
<1>} = L}1} + L 121 2 , <1>2 = L 21 2 + L 211}. (9.38)
1(he correspondence between individual terms in formulas
9.35) and (9.34) is beyolHl doubt. According to the law of conservation of energy, the work
Formulas (9.:3'1) and (9.35) lead to the following important bA * done by the sources of e.m.f. included into circuits 1
sequences. and 2 is converted into the heat /jQ and is spent to increase
1. The. magnetic energy of a system of two (or more) the magnetic energy of the system by dW (due to the motion
CUl'wnts IS an essentially positi~e quantity (W > 0). This of the loops or the variation of currents in them as well as
to accomplish the mechanical work bAm (as a result of a
rOllo\~s from tile fact that tv ex: \ Jr2 dF, wllOre the integrand displacement or deformation of the loops):
contall1S positive quantities. • bA * = bQ + dW + bAm. (9.39)
2. The ~nergy of currents is a nonadditive quantity (due
to, tll,n, ex lsten~e of m 1I1llal energy). . We assumed that the capacitance of the loops is negligibly
.3. I he last lJItogTHI .in (~).;3:l) is proportional to the pro- small and hence the electric energy can he ignored.
l!t,Ctt' 1 1/ 2I . of Cllrren t,<; .since H1 ex: 11 ('IJIII I? ., ex: I .,. 'I'] lC pI.0- Henceforth, we shall be interested not in the entire work
POl' IOJ,Ja Ily fa~tol'. (i.e. tile rpmaining- ilJtogral) turns out 6A * of the source of e.m.f. hut only in its part wllich is done
to he I'ymmetnc WIlli rospeet to indices 1 and 2 and hence against the induced and self-induced e.m.f.s (in each loop).
c({~n he denoted by L I2 01' L 21 ill accordance with formula This work (wlW;h was called the additiOl:aJ w",l\ is equal
v.34). Thus, L I2 == L 21 indeed.
. 4. Expression (9.35) leads to another defuiition of mutual to bAadd = -(~i} + ~B1) I} dt - (~12 .t- (s ,) I dt.
I~ductance L 12 . Indeed, a comparison of (9.3.')) and (934) Considering that

~ i + ~8 = -d<t>/dt fOJ C';I, loop, we
gives .
L 12 = _1_- r ]l1 ·B2 av. can write the expression for the. additional wllrk ;t lhe form
II 12 J l.lfto (9.36) 6A add = I} d<1>} + 1 2 d(l>"o (9.40)

It is just this part of the work of the e . l ' I 'iomces (the


9.7. Energy and Forces in Magnetic Field work against the induced and self-induc,"! c.nd.f') that
. The most general method for determining the forces acting is associated with the variation of the nuxes (1\ and <1>2
I? a magnetic fwld is. the energy method, in which the expres- and spent for increasing the magnetic energy of tile system
slOn for the magnetic fwld energy is used. as well as for accomplishing the mechanicnl work:
":~. ~hall confine our.selves to the ease when the system
T
(9.41)
conSIsts
.
of two loops With CULTPnts
. •
11 ~Jld 1 2' 'r-l Ie magne t'IC
(A

enel gy of such a system can be represented in the form This formula forms the basis for calcula.ting the mechanical
U? 1 work bAm and then the forces acting in a mllgnetic fwld.
rr '" - (1 1
2
-+-- 1 2(I)2)'
(Il1 1
(9.37) Formula (9.41) can be used for obtaining simpler expres-
sions for /jAm by assuming that either all the magnetic fluxes
where tV J and (1)2 are the total magnetic fluxes piercing through the loops, Or currents flowing in them remain un-
loop~ ~ all,d ~ respectivelr,. This expression can be easily changed during a displacement. Let us consider this in
?b~allIed from for~ula (\:1.01) by representing the last term greater detail.
IIllt as the sum (1i2) L 12 IJ2 + (1/2) L 2 / 2 / 1 and then taking 1. If the fluxes are constant (<t>" = const), it immediately
16-0181
9.7. Energy and Forces in Magnetic Field
243
24~ 9. Electromagnetic Induction

follows hom (9.11) that , t I' e L (x) is known. Find the Ampere's force acting on the
(IIna e x, . . d f' ,h t
jumper, in two ways: for 1 = const an or .... = cons.
. ----1~1
1=--6A-n- = (9.42)

I' where the subscript <D indicates that the increment of t.he o x X
_---1----__X
~-~
II
magnetic energy of the system must be calculated at constant
fluxes through the loops. The obtained formula is similar -.:-
I
to the corresponding formula (4.15) for the work in an elec-
tric field. . B
2. If the currents are constant (I k = const), we have Fig. lUG
Fig. \J.15
16Am=dWlr./ (9.43)
In the case under consideration, t~lC ma~netic en.ergy o~ the system
can be represented, in accordance With (\1.29), as follows.
Indeed, for 1 k const, it follows from formula (9.37) that
W = L12/2 = (P2/2L,
dWlr = ~ (11 d<t\ +1 2 d<tl 2 ), where <D = Ll. Let us displace the jumper, for example, to the right
by dr. Since 6A m = F", dx, we have
i.e. in this case, in accordance with (9.40), the increment
of the magnetic energy of the system is equal to half of the F ,,'-
OW
-'-ax-
I = - -ax,
I
1
2

2
OJ,

additional work done by the sources of e.m.f. Another half


or
of this work is spent for doing mechanical work. In other
words, when the currents are constant, dWlr = 6A m,
Q.E.D.
F= _
x ao: 1;= ~22 ~: = I; ~: '
It should ·be emphasized that the two expressions (9.42) I.e. the calculations carried out with the help of the two formulas
give in accordance with (9.44), the same result.
and (9.43) obtained by us define the mechanical work of
the same force, Le. we can write ~ample 2. Interaction between two current-carrying coil~. Two
coils 1 and 2 with currents 11 and 1 2 are mounted on a ma~nehc core
F·dl = -dWI<1> = dWlr. (9.44) (Fig. 9.16). Suppose that the mutual inductance of the ~.ods depends
on their separation x according. to a known law £12 (x). lllld the force
In order to calculate the force with the lielp of these
of interaction between the coils.
formulas, there is no need, of course, to choose a regime in The magnetic energy of the syste~ of two .coils is given by formul~
which either magnetic fluxes or currents would necessarily (9.34). For determining the force of lllteractlOn.. we shall use expres
remain constant. We must simply find the increment dW . (9 43) Let us displace coil 2 through a distance d:1: at consta!?t
of magnetic energy of the system provided that either <Ilk = 510n t' [' and 1 The correspondin<r increment of the magnetic.
curren s 1 < 2' "
= const or 1 k = const, which is a purely mathematical energy of the system is
operation. dW 11 = 1 11 2 dL I2 (x).
The value of the obtained expressions (9.42) and (9.43) Since the e1ementary Inecllanl'cal work 6A m = F2~~ dr, according to
lies in the.ir generality: they are applicable to systems con- (9.43) we obtai n
sisting of any number of contours-one, two, or more. OL I2 (x)
Let us consider several examples illustrating the applica- F 2x =1 112 ax .
tion of these formulas.
Example 1. Force in the case of one current loop. Suppose that
Let currents 11 and 1 2 magnetize each other. Then L 12 > 0, n for
dr
t: 1
> 0 the increment dD 12 < U, i.e. F 2x < U. Consequently,. t ~ orc~
we have a current loop, where AB is a movable jumper (Fig. 9.15). exerted on coil 2 by coil' is the attractive force: vect.or 1"2 IS dlrecte(
The inductance of this loo~ depends in a certain way on the coor- to the left ill the ligure.
16*
244
- ~9~._.!.Electromagnetic Induction
Problema 245
---_=....:....::..:..:..::.:::.:=-------_._----
Example 3. Magnetic pressure t·
Let us mentally increase the r (/" ac leg on SOk:noid willding.
by dr:, retaining t.be current I th:o~uh ~b th~. so~en01d cross section This formula shows that 'jgi ex: y. The minus sign indicates that ~i
A III perl' S f~rces ~ill accomplish the ';ork
uuder consideratIOn, we have
6A\\
1~lDg constant. Then
m - dW fl' In the case
in the figure acts counterclockwise
• 9.2. A loop moves arbitrarily. A closed conducting loop is moved
t'>A m = pS dr arbitrarily (even with a deformation) in a constant nonuniform mag·-
where p is the required P d ~ netic field. Show that Faraday's law (9.1) will be fulfIlled in this case.
solenoid, and· ressure an S IS the lateral surface of the
y
dW 11 ~~ d ( BS
2140
v) ccc ~
:!11o
S'd
< r.
ITpre we took into account the f t h
as well. Equating these two ex"'::o ' t at when I. = COllst, B = canst
... ~Ions, we obtam
p = BS/2p.o. f,

Magnetic Pressure The .


in Example 3 can be 'genera~f:ierJs;lO~hfor pressure obtained x ds = [ dr" d I J
netic field is different (B and B ) or d .effcase when the mag-
face with current (cond 1 t' 2 on I erent sides of a sur- Fig. 9.17 Fig. 9.18
uc IOn current Or mag t' t'
rent). In this case the mag t' . ne Iza IOn cur-
, ne IC pressure IS given by Solution.. Let u~ consider an element dt of loop length, which at
p=/. HI·H
2
I -. Bs·H.
2'
I (9,45)
a given instant moves at a velocity v in the magnetic field B. In
aeoordance with formulas (8.4) of field transformation, in the reference
system fixed to the given elemQllt the electric field E = [v X B] will
The situation is such as if the re i o ' . be observed. It should be noted that this expression can be also ob-
energy density were the region ~f ~. ~Ith a hIgher magnetic tained with the help of the Lorentz force, as it was done at the beginn-
ing of Sec. 9.2.
Helation (9.45) is one of the ba .Ig Ielr p.ressure. Circulation of vector E around the entire contour is, by definition,
hydrody . I . h SIC re atlOns in magneto
ing liq~f(~lC(i;l~~ec~~~~;fs:~e .beha."iour of electroconduct=
the induced e.mJ.:
gllleel'lng and astrophysics). ~i ~-= ~ Iv X BI dt. (1)

Let us now fllld the corresponding increment of the magnetic flux


PMlems through the loop. For this purpose, we tum to Fig. 9.18. Suppose that
• !.l.f. Jnduced emf A w· . th h the loop was displaced from position r 1 to r s during time dt. If in
is in a uniform magneii~ fiel:ran e s ape of a parabola y = kz2 the first position the magnetic flux through the surface S1 stretched
A )umper translates without initial v~~p~tndlc~ar to the plane XY. over the loop was <1>11 the corresponding magnetic flux in the second
atlOn a from the apex f th b CI yan at a constant aceeler-
induced in the formed .c~ntou~ ~:r: f~la lFi g . f9 . 1h7). Fin~ the e.m.f.
,position can be represented as (1)1 +
dl1l, Le. as the flux through the
• . DC IOn 0 t e coordmate y. surface S + dS. Here d<D is the required increment of the magnetic
Solutlon. By definition' rS - dll>ld . nux throngh the narrow strip dS between contours r l and r 2 •
to the plane of the contour'in lh~-dir / Hfvlll g chosen the normal n Using Fig. 9.18, we can write
= B dS wh dS _ . ec IOn 0 vector B, we write dl1l =
obtain' ere - 2x dy. ConSidering now that x = Vulk , we d<1>,= ~ B.dS=.\ B.[drXdIJ=-~[drXBldl. (2)
~i= -B·2 ¥ylk dYldt. Here (1) the direction of normal n is matched with the direction of
During the motion w'th circumvention of tile contour, viz. with vector dl (right-handed sys-
= '/2'-
rag, an
d I
lence
I a constant acceleration, the velocity dyldt = tem); (2) the direction of vector cL'!, viz. the area element of the strip,
is matched with the choice of normals n, and (3) the following cyclic
transposition is \lSi'll in the scalar triple product:
a·lb X e] = b·le X a) = e·[a X b) = -Ib X a)·e.
[I

247
24fl .'J. Eleclromfl!:netic Inrlnclion Problems

Dividing expression (2) by rll, we find ·te the relativistic eqnation for motion of an
Solution. Let us wrl
electron,
df!>/dt = - ~ Iv X DI dl, (3) dp/dt = eE +
e Iv X Do),
(1)

where v = dr/dt. It remains for us to compare (3) with (1), which


gives ej = -detJ/dt.
. . Id ' t
where E is the vortex electnc he
the tangent 't and the normal n to e raJec 0 .
's:
of the projections onto
~rlJ.1s t ry 'For this purpose,

. .• 9.3 .. A flat coil ,,:,ith a large number N of tightly wound turns


IS In a umform magnetic field perpendicular to the plane of the coil
(Fig. 9.19). The external radius of the coil is equal to a. The magnetic b
o
field varies in time according to the law B = B 0 sin wt.' Find the
maximum [value of the e.m.f. induced in the coil. ---+---, I
Solution. Since each turn of the coil 10 a 2 I
practically does not differ from a circle. the L.-_ _ ~ _ _ --lI
e.m. f. induced in it is given by
a O'
I'j = - df!>/dt = -rrr 2Bow cos wt,
where r is the radius of the turn under COll-
sideration. The number of turns corresponding Fig. 9.21
to the interval dr of the values of the radius Fig. ..(l.20
is dN = (N/a) dr. The turns are connected in
series, hence the total e.m.f. induced in the . h I t ' momentlllli as P p'f and lind its time derivative
c.=
we wrl te tee ec ron -..
coil is d d [,2
dp _ft'f+p-!-=.L'f!m-n, (2)
Fig. 9.19 ~j=.\ e;(r)dN. dt"- dt dt dt '0
. t that p - nlV In being the relativistic
t k into accoun -, F' 9 20)
Having integrated this expression, we obtain the following for the h
were we 00 t __ (vlro)n (this can be easily seen fr~m Ig. ' •
maximum value of the induced e.m.f.: mass, and d'f 1d - __ ( dtl )n and the rest is ObVIOUS,
Indeed, d'f = dcp'dn, -- tV Fro aday's law 2rrr oE =~ Id<I)/dt I, where
~im = (1/3)rraWBow. Besides, accor mg 0 ar '
etJ = nrfi (B). Hence
• 9.4. A coil of N turns wi th the cross-sectional area S is placed
inside a long ;;olenoid. The coil is rotated at a constant angular velocity
E ~c ~ :t (B). (3)
w around the axis coinciding with its diameter and perpendicular . k' 'nto account (2) and (3), in terms
to the axis of the solenoid. The magnetic fIeld in the solenoid varies
according to the law B =, B o sin wt. Find the e.m.L induced in the
Let us now wnte Eq'I(1), ta IlIf
I d the normal to the trajectury:
of the projections on to t Ie tangen an
coil, if at the instant t = 0 the coil axis coincided with the axis of
the solenoid. . dp-eE.-e!'!!- ~.(R), (4)
7 - ,- 2 dt
Sdlution. A t the instant t, the total magnetic flux through the l,2
coil is m---ccevBo·
etJ = NBS cos wt = NBoS sin wt·cos wt = (1/2)NB oS sin 2wt. ro
In accordance wi th Farad ay' slaw, we have The last equation can be written, after cancelling V, in the form
0j = -detJ/dt = -(1/2)NBoS·2w cos 2wt = -NBoSw eos 2wt. [J = eroBo,
• 9.5. The betatron condition. Show that electrons in a betatron . ''I resllecL to time, considering
We differentiate this equatIOn WI. I
will move in an orbit of a constant radius ro prOVided that the magnetic
field Bo on the orbit is equal to half of the value (B> of the magnetic that ro,= const:
dp . ' dB o (5)
fIeld averaged over the area inside the orbit, i.e. Eo = <:> . (it =: ei o -rit .
248
9. mectromagnclic Induclion
Problems 249
A comparison of Eqs. (5) and (4) gives
Il can be seen that the obtai:1ed quantity is independent of the induct·
~ IJ _ 1 d ance of the circuit (the situation would be oifferent if the circuit were
dt 0 -"2 dt (8). supereonriueting).
In particular, the last condition will be satislied when • 9.7. A jumper 12 of muss m slides without friction along two long
condncting rails separated by a distance l (Fig. 9.22), The sys~em, is
1 in a uniform magnetic fIeld perpendicular to the plane of the CIrCUIt,
lJo""" 2 ((I).

Practically, it is altai/It'd b Ill' f.. . L1


form (in the form of 'I hluntYI <tn,lI aclllrJIlg pole pieces in a speciul
. , - lusel cones). 1
• !1.6. Induced CUrrent A . .

R~
L2
a long straight conductl)I' w·I't'h I~qnare wire frallle with side a and
,( Ig. . 1). The inductance of the f' . . en t/ 0 I'"! 1/1
F' 9 2 a I Ireet eurr' ' the same plane
®B ..
y

fhe frame has been rUlated l' "rame loS" and Its resistance is R.
then stopped. Find the arnoun\ I~fu; '~ ,I ~(~. aroulHI Ihe axis or)' and
~he frame. The distancl' b bet ' t~cl,rI~I,ty l/}at has 111lssell throlll{h
IS assumed to be known. ween e aXIS 00 and the straight wire
1
2
X

0"-':
Fig. \).22 Fig. \.l.2:i
Sulutiun. According to Ohm's la\ I
frullle duril1" its rutali . I .v, t Ie current / appearirl" in the
,., . un IS e etermllled by the formula ,.,
Tile Idl ends.~t the rails are shunted through resistor ll. At the instant
'R 1= _ dl1> _ L ~ t ''= 0, the jllinper received the initial velocity Vo directed to the right.
dt dt • Pintl the velocity of the jumper as a function of time, ignoring the
resistances of the jumper and of the rails as well as self-inductance of
Hence the required the circuit.
arnollnt of electricity (charge) is
_r 1
q-- I I dt= - - -

\ (dIll I I dl) 1
• Solution. Let us choose the positive direction of the normal n
to the plane of the circuit away from us. This means that the positive
• R.' -=-7f(M>[-!.M).
direction of circumvention of the circuit (for induced e.m.r. and cur-
~ince the frame has been sto I, f . . rent) is chosen clockwise, in accordance with the right-hand screw
valllshes, and hence A/- 0 a Ppel ,! tcr rotatIOn, the current in it mle. It follows from Ohm's law that
of the flux Art> through 'tlll: fra~~n(r~~lJ f~r ~s to find the increment dtlJ dS
et us choose the normal n to th I 2 - 11>1)' Rl = --F== -II d't= -Hlv, (1)
rl that in the lin.al Positior; n is dire~t~/b~h~f ~he frame, for instance;'
ah?t g . B). Then It can be easily seen that'
II' I e III the initial positio~ <1>'
the
ihelPlane of the figure
e ma position 11>2 > 0
where we took into account that when the jumper moves to the right,
,JII d«1> > O. According to Lenz'" law, the induced eurrent I causes the
and A$ turns out to be' I I < 0 (the normal is opposite to B)' Ampere force counteracting the motion, directed to the left.
boumled by the jj~al aneti~~·~'leqU<~I.to the flux through the surIac~ Having chosen the X-axis to the right, we write the equation of
I Ia 'pOSI tlOns of the frame: ,.
motion of the jumper
b+a
m dvldt = /lB, (2)
All> = $2 + 1$11= \ Ba dr,
where the right-hand side is the projection of the Ampere force onto
b:a the X-axis (this quantity is negative, but we omit the minus sign
Where B is a fllnction I I sina, as can be seen from (1), current I < 0).
help. of the the~~em ~n "'ci\;c~)lset.rurm cau be easily found with the Elinlinating' I from l':qs. (1) allli (2), we obtain
FJIlall .t . a lOll.
, y, oml trng the minus, sl'gn , we 0 bt aln
. dvlv = -a dt. a = IJ2/2/mR.

q=' ~ ~lon/o I b+ a The integration of this expression, taking into account the initial
n ~ n -,;::::-a . condition, gives
In (171170) = -at, v = vee- t
251
Problems
250 9. Electromagnetic Induc~ion
(erroneous) result, namely, insteal\ of 1J1i, 1/2 is obtained in the paren-
theses. The thinner the central wire, "i.e. the larger the ratio bla,
inductan~es
. • 9.8. The role of t ranslent
Fig. 9.23 the f . processes In th . . the smaller the relative diflerence in the results of calculation by these
~1sup~rconducting ~nd
L e.:.. I!f of the source, its iniernal e .clrcUlt shown in two methods, viz. through the energy and through the flux.
currents
S
establish:d I;z
w e cOlIs after key coils :::Iktance R
K h b nown. Fwd the
the
• 9.10. Mutual indudion. A long straight wire is arranged along
and ~L:~on.
. Let us use Kirchhoff's laws for:~ec :~n
rIC closed.
circuits ~Ll
the symmetry axis of a toroidal coil of rectangular crosS section, whose

RI ='f,-L 1 dl l
dt' RI = 'fb - L __
d/ z B
2 dt

for stabilized ofcurre~t


A comparison the expresshlOns
.
s we ave shows that L t d1 1 = L zdJ z ,
while

(1)
Besides, Lt / 10
= LzJ zO'

J 10 + 1 20 = 1 0 = ~ I R (2)
From these eq ua t'IOns, we find .
Fig. U.25
1 10 ='fi,- LZ 6 L Fi~ U.24
+
R L 1 L 2 ' J 20 = -R L LL 1 .
diownsio ,Ire given in Fig. H.25. The number of tllrns on the coil
. • 9.9. Calculati f'. I', 2 is N, alltillsthe permeability of tbe surrounding medium is unity. Find
mternal solid conduc on 0 ~n~uetanee. A coaxial·
tube. of radius b. Fi~dr l! r~d1Us a and extemal thi c~b~e .
aU C?nSlsts. of the amplitude of tlte e.Ill.£' induced in this coil if the current I =
consIdering that th
e.curr~nt
t e lllductance of a unit I n ili ~he
canductmg = 1 cos wt flows along the straight wire.
I I Yc~oss
internal conductor distribution over the eng. of cable,
everywhere . IS umform. The permeabTt sectIOn
IS equal to of the
unity
m
Solution. The required e.mJ. 8; = _det>ldt, where et> = Net>l
and <1)1 is the magnetic flux through the cross section o[ the coil:
Ii

(
t~e3~)nergyrath::~ha~et~~o~:~athce ~bi d:~e:~~~~dc~~dt~~tor
thin and Inh the Case under considerati
notSolution. tl' '
Bn d::>' \ ' flo
Ih dr = . !.lohI 1 b
iSr 2.- - - - n - ,
(1
~nr
0_ -')- 0...
should 11 0 • • n a
1 ,we can write e magnetIc nux. In acco r dancems.o
WIth \
n
where B is determined with the help of the theorem on circulation
Lu =
1
J2
r.\b -;;-
BZ
2rtr dr (1 )
of vecto'rl n. Taking' the time derivative of <I)] and multiplying the
obtained resnlt by N, we fllld the following expression for the amplitude
ii"'o '
valm' of induced e.m.£.:
where r iswethemu
integral, dist fince from the cable axis In d
-circulation, we hstavend the dependence B(·r). Using
or er to
theevaluate
theoremthis
on
'0int =~(Jllm~
2il
In!!....
a
• 9.11. Calculation of mutual inductance. Two solenoids of the
- 2~
B r<a ~ !.loIz r, Ba<r<b ,- ltoI -1 same length and of practically the same cross section arc completely z
"a -' -2-- B - () inserted one into the other. The inductances of solenoids are L] and L ·
Th f n r' r>b - . (2)
Neglecting ellge cUeds, fmd their mutual inductance (modulo).
(2), integral (1) is sp r~dences
e orm of these de
obtain It mto twoisparts'\
shown in F'19. 9.24. On account of
.. s a result of integration, we Solution. By definition, the mutual inductance is
LIZ = <lIP?; (1)

Lu = .&.. (..!:-- -+- In ~ ) where <!ll is the total magnetic nux through all the turns ofI solenoidS1
~o\enoitl
2n 4 a' provitlell tbat current I"z flows in solenoid 2. The flux <D = NI!J2 ,
It should he noted that where N l i8 the number of turns in 1. 5 is \ he eross-sedlOnul
terms of magnetic flux by thenf~;~:;I~tLto~etermine this quantity in
• II - ¢\/J leads to a different
10.1 Displacement Current
-------~~
252 fJ. EleclrtJlnll!:netic Induction
------- It remains for us to substitute (2) into (1) and recal! that /8 = Pm'
Then q = floNPm/2aR, and
area of the solenoid, and B 2 = f-tJ.1onS/2' Hence formula (1) can be
written (after cancelling /2) as follows: pm = 2aRq/lloN.
I L I2 1 =f-tf-ton2NtS=llllonln2V, (2)
where we took into account that N 1 = ntl, where l is the solenoid
length :md lS ,= V i~ its volume. Expression (2) can be represented
ill terms of L 1 and ['2 as follows: 10. Maxwell's Equations.
I L 12 1= r'" f-tflon(V YflflonW= yr L1L2 • Energy of E~edromagnetic Field
It shonld be noted that this C'xpres~ion definC's the limiting (maximum)
value of I L 12 I· In general,! L 12 1 < YL 1 L 2 • .O'i. . . . e.enf Current
• 9.12. Reciprocity them·em. A small cylindrical magnet M is Maxwell's Discovery. The theory of electromagnetic field
placed at the centre of a thin coil of rarlius a, containing N tnrns founded by Faraday was mathematically completed by
(Fig. 9.26). The coil is connect.ed to a ballistic galvanometer. The Maxwell. One of the
resistalKeof the circuit is R. After the magnet had been rapidly re- most important new S' S'
moved from the coil, a chaq;e q passed through the galvanometer.
ideas put forward by
,-,
~........

Find the magnetic moment of the magnet. - (+ \ S (\.


Solution. In the process of removal of the magnet, the total mag-
Maxwell was the idea
of symmet,fy in the ~------J I ~-L~I-.n ~n
netic flux through the coil was changing, which resulted in the emerg-
ence of induced current defined by the following equation: mutual dependence
of electric and mag-
Il!:J'\. \_./
r
R / = __ d(D _ L d/ (a) (b)
dt dt . netic fields. Name-'
We multiply both sides of this
ly, since a time-var- Fig. 10.1
equation by dt and take into ying magnetic field
acconnt that / dt = dq. This (aR/at) creates an electric field, it should be expected that
Fig. 9.26 gives a time-varying electric field (aE/at) creates a magnetic field.
R dq = -d<D - L d/. This idea necessitating the existence of a new in principle
Having integrated this expression, we obtain Rq = -t.<D - L t./. phenomenon of inductioD. can be approached, for example,
Considering now that !1/ = 0 (the current was equal to zero at the by the following line of reasoning. We know that according
beginning as well as at the end of the process), we obtain to the theorem on circulation of vector H,
q= !1<D/R = $/R, (1)
where lD is the magnetic flux through the coil at the beginning ot the ~ Hdl= ) jdS. (10.1)
process (we omitted the minus sign since it is immaterial).
Thus, the problem is reduced to the calculation of the flux $ through Let us apply this theorem to the case when a preliminarily
the coil. This quantity cannot be determined directly. This difficulty, charged parallel plate capacitor is being discharged through
however, can be overcome by using the reciprocity theorem. We men-
tally replace the magnet by a small current loop creating in the sur-
a certain external resistance (Fig. 1O.1a). For the contour r
rounding space the same magnetic field as the magnet. If the area of we take a curve embracing the wire. We can stretch various
the loop is 8 and the current is /, their product will give the magnetic surfaces on this contour, for example, S and Sf. ,These two
moment I'm of the magnet: Pm e 18. According to the reciprocity surfaces have "equal rights". However, current I flows through
theorem, f'l?/= LnT and the problem is reduced 1.0 determining the' :mrface S, while through surfaco S' there is nO current!
magnetic nux through the area 8 of the loop, which creates the same
current /. but flowing in the coil. ARsnming the field to be uniform It turns out Lhat the circulation of vector ijdepends on
within the loop, we obtain the surface stretched over a given contour (?!), which i.s
<JI = BS = It oN/ S/2a. (2)
'[I

254 10. Ma.xweU's Equations. Electromagnetic Field EnerlfY 10.1. Displacement CUr/ent 255

obviously ruled out (and never happened in the case of it is sufficient to introduce on the right~hand side of Eq.
direct currents). (10.1) the total current instead of conduction current, viz.
Is it possible to change somehow the right-hand side of the quantity
(10.1) to avoid this inconvenience? Fortu nately, th is can (10.7)
be done in the following way.
First, we note that surface 8' "cuts" only the electrit Indeed, the right-hand side of (10.7) is the sum of conduc-
field. In accordance with the Gauss theorem, the flux of tion current I and displacement current I d :1 t = I I d' -+
vector D through a closed surface is ~ D dS = qj whence Let us show that the total current It will be the same for
surfaces 8 and 8' stretched over the same contour r. For
r\' ~
'j' fJt dS = !.!L
at . (10.2)
this purpose, we apply formula (10.4) to the closed surface
composed of surfaces 8 and 8' (Fig. to.1b). Taking into ac-
On the other hand, according to the continuity equation count the fact that for a closed surface the normal n is di-
(5.4), we have rected outwards, we write
,h .. dS = _.!!!L, (10.3)
It (8') + It (8) = O.
'j' J at .
Now, if we direct the normal 0' fo:-, surface 8' to the same
Summing up the left- and right-hand sides of Eqs. (10.2) side .as for surface 8, the first term in this expression will
and (10.3) separately, we obtain change the ~gn, and we obtain

~ (j+ ~~) dS=O. (10.4) It (8') = It (8),


Q.E.D.
This equation is similar to the continuity equation for Thus, the theorem on circulation of vector H, which was
direct current. It can be seen that in addition to the density j established for direct currents, can be generalized for an ar-
of the conduction current, there is one mOre term aD/at bitrary case in the following form:
whose dimensions are the same as for current density. Max-
well termed this addend the density of displacement current:
jd = aD/at. (10.5)
I~Hdl= J(j-+{¥-)dS. (10.8)

The sum of the conduction and displacement currents is In this form, the theorem on circulation of vector H is al-
called the total current. Its density is given by ways valid, which is confirmed by the agreement of this
equation with the results of experiments in all cases without
. .+ 7ft.
Jt=]
aD (10.6) any exception.

According to· (10.4), the lines of the total current are con- Ditlerential form of Eg. (10.8):
tinuous in contrast to the lines of conduction current. If
conduction currents are not closed, displacement currents VXJI=j+ ~~ , (10.9)
close them.
We shall show now that the introduction of the concept of where the curl of vector II is determined by the density j of conduction
total current eliminates the difficulty associated with the current and an/at of displacement current at the same point.
dependence of circulation of vector H on the choice of the
surface stretched over contour r. It turns out that for this Several Remarks about Displacement Current. It should
256 In. Maxwell's Equation.~. Electromagnetic Field Energy 10.2. Maxwell's Equations 257

be kept.in mind that displacement current is equivalent to (Fig. 10.2) would differ from zero, which is in cuntradi,',tiun with
conductl~m current only from the point of view of its ability Eq. (7.2). Consequently, vector B Olust be perpendicular to the radial
of creating a magnetic field. direction at the point P. But this is also impossibh>, sincl' all directions
perpendicular to the radial dirrclion arc absolutely I'quivull'nt. It
~ispla~eme~t currents exist only when an electric field remains for us to conclude that magnetic flCld is equal to zero every-
varIes with time. In dielectrics, displacement cnrrent con- where.
sists of two essentially different The absence of magnetic field in the presence of electric current
components. Since vector D = of density j indicates that in addition to conduction current j, displace-
ment current jd is also present in the system. This current is such that
= BoE+ P, it follows that the den- the total current is equal to zero eYcrywhere, Le. jd = - j at each
sity iJD/iJt of displacement CUf- point. This means that
rent is t.he sum of the densities
of the "true" displacement current, . . I q iJD
BoiJE/iJt, and of the polariztllion
Jd = J = 4nrz c= 4nrz = 7ft '
current ap/iJt. The latter quantity where we took into account that J) = q/4nr z in accordance with the
appears due to the motion of Gauss theorem.
bound charges. There is nothing un-
Fig. 10.2 expected in the fact that polari- 10.1. Maxwell's Equations
. zation currents excite a magnetic Maxwell's Equations in the Integral Form. The introduction

ch.
field~ SlIlce these currents do not differ in nature from con- of displacemtilUt current brilliantly completed the macro-
ductIOn currents. A new in principle statement consists in scopic theory of electromagnetic field. The discovery of dis-
that the other component of the displacement current placement current (aD/at) allowed Maxwell to create a
BoiJE~at, which is not connected with any motion of unified theory of electric and magnetic. phenomena. Maxwell's
and. IS only due to the variation of the electric field also theory not only explained all individual phenomena of elec-
eXCites a magnetic ~eld. Even. in ~ vacuum, any tem'poral tricity and magnetism from a single point of view, but also
change of .an electnc field eXCites In the surrounding space predicted a number of new phenomena whose existence was
a magnetiC field. subsequently confmned. . ,
The d~s~overy of this phenomenon was the most essential Hitherto, we considered separate parts of this theory.
and decIsIve s.tep made by Maxwell while constructing the Now we can represent the entire theory in the form of a sys-
electrom.agnetIc field theory. This discovery was as valuable tem of fundamental equations in electrodynamic.s, called
as ~he dlscov.ery of elec~romagnetic induction, according to l'vfaxwell's equations for stationary media. In all, theN are
whICh a varymg magnetic field excites a vortex electric field. four Maxwell's equations (we are already acquainted with
It should also be noted that the discovery of displ.arement each of these equations separately from previous sections;
cu~rent b.y Maxwell was a purely theoretical discovery of here we simply gather them together). In the integral form,
prImary Importance. the system of Maxwell's equations is written as follows:
Let liS consider an example in which displaooment cur-
rents are manifested.
Example. A metallic positively charged sphere i8 in aniBUaite
r ~Edl= -) ~~ dS, ~ DdS= ~ pdV, (10.10)

homo~~neo~s co~ducling medium (Fig. to.2). Electric currents flowing


11'! ra~lal 11IrectIOns should excIte II magnetic field. Let us find the
directIOn of vector B at an arbitrary point P. •
I'~H
1
dl = ~ (j +- ~~_ ) dS, t" BdS=O, (10.11)

First of al~, it is clear that vector B cannot have a radial compo- where p is the volume density of extraneous charges and j
nent. OtherwISC, the flux or B through the surface S of the "'re is the density of conduction ClllTent.
17-0181


258 _-.!(): J1!a.r'Nll's Equalion.~. EleclromaKnclic Field Enerr:y
_ _~10:.:.2::.--:M~ax~w::.'d:..:.:...I'.:...s. ::E:,2q..:.:.u_a__
ti_on_s 25_9
._._._ _
These et)uations express in .. concise f01'l1l all our know-
lOlL:,,> <l bout electrom:lgnl'tic fIeld. The content of these equa-
I ;OIIS ('onsis[s in the following". imen t a 1 l'esu Its , 'fllese postulates play. inI electrodynamics
'h' the
the same role as Newt.on's laws in classlca mel, alliCs or
1. Circulation of vector E al'ound any closetl contonr is
equal, wit h tlteminussign, to the time derivative of the magnet- laws i~l ther.mlodYFnamicosr· Maxwell's Equations. Equatiol1s
ic llux through an arbitrary surface bounded by the given Dillerenba OJ'lU 1. . , If,
(to 10) and (11.11) can be represented il~ dlrreJ'en~la oll.n,
contour, lIere E is not only vortex electric Jield but also elec- Le.·in the form of a system of differential equatIOns, VIZ.
trostatie lipid (whose circulation is known to be equal to
zero).
2. The fj ux of D through any closed surface is equal to v >~ E = - ~~, V· D = p, (10.13)
the algebraic sum of extraneous charges embraced by this
surface, . aD ..,. B
V>< II =J+aT' y' = 0. (10.11)
3. Circulation of vector H around an arbitrary closed Con-
tour is equal to the total current (conduction current plus
dbplacement current) through an arbitrary surface bounded E nations (10.13) show that there may b~ two causes for
by this contonr. f t ic field First, its sources are electl'lc charges, b?th
4•. The flux of B through flU arbitrary closed surface is :~t:a~::us a~d' bound (which follows f~om. th~ ~q~a~~~
always equal to zero. V . D ,= p if we take into account that = eo - .
It follows from Max\vell's eq uations for circulations of vec- - ~ , th~' V·E ex: p + p'). Second, the field ~alwa~s
V·p-
emerges when p, a magnetic
n . he'I.d vanes . . t'me '(willch
1~1 . I ) ex-
tors E and H that l'lectric and magnetic fields cannot be
treated as ipdependent: a change ill time in One of these fields F d 's law of electroffingnetlc mductlOn .
leads to the ilmergence of the other. Hence it is only a combi- pr~se~1 <1I'~h:I\and Eqs, (10.14) indicate t~lHt the magnet-
nation of these fields, describing a unique electromagnetic . fin I :e; can be e~cited by moving electrrc charges (elec-
field, that has a sense. IC
t.ric 10cUl'rents),
( , ".
by varYlllg e Iectl'le
. j'10 Ids , or" by both
V.,factors
H _'"
If the fields are stationary (E = constand B ,= const), simultaneously (this foll~ws from the eqt~t~~J~/ A J and
Maxwell's equations split into two groups of independent = .1- DD/ilt, if we take mto acco~nt ~,1U l t .. . flo. iJE/ Dt
equations V ~ J ..- c j'; then V X B ex: J J + ~P/ iJt I- eo . . : +
} ., i the magnetization current denSIty and iJP/dt IS
~ Edl =0, ~ DdS=q, ~1~e~~I~ri:atiOn current density. The flf~~ tl~~~e f~:[e~~:r:~~
(10.12) associated \V.ith moti?n ~fel~r~gex,s~~~o:'s from the eql~a­
~ Hdl~I, ~ B dS=O. is due to a time-vary lIlg I " . fi Id in na-
tion V· B = 0, there are no sources. of magnetIc Ie Id be
In this case, the electric and magnetic fIelds are indepen- tlll'e (cn-lled, by analogy, magnetic charges) til at wou
similar .1.0 electric clfla~g1:~wel1's equations in differential
dent of one another, which allowed us to study first a con-
stant electric fwld and then, independently, eonstant mag- The Importance 0 n I. b . I s
netic fIeld. .
form consists no t on 1Y I'n that they express
I tie
fi 1 Is aSlc
E andaw,B
It should be emphasize!! that the line of reasoning which of electromagnetic freid, but abo in that tie. re)l I '
has led us to Maxwell's equations can by no means be con- themselves can be found by solving (integratlllg t lCseeqll<1-

tiO;~~ether
sidered their proof. These equa tions C:l nnot be "derived" since
they are the fundamental axioms, or postulates of electro- with the equation of mot,ion of charged particles
dynamics, ohtained willi t.he help of generalization of expeJ'- nder the action of the Lorentz fOlce,
u . )
dp/dt = qE + q [v X Bl, (10,1;)
17*
..

261
260 10. Mazwelf. Equation•. Electromagnetic "eld Ener" 10.3. Properties 0/ Mazwell'. Equations

Maxwell's equations for,m a fundamental system of equa- I the well-known constants characteriz:
I
, I where e, ~, alH a are . erties of the medium
tions. In principle, this system is sufficient for describing all ing the electric and ~~gne~~l~~~ctroconductivity), and
I,

electromagnetic phenomena in which quantum effects are (permittivity, permeabIlity, force field due to chemi-
not exhibitecl. ' E* is the strength of the extraneous
Boundary Conditions. Maxwell's equations in integral calor thermal processes
form are of a more general nature than their differentia)
counterpart, since they remain valid in the presence of a 10.3. Properties of Maxwell's Equations
surjace oj discontinuity, viz. the surface on which the proper- . Tl contain only tlIe first
ties of the medium or fields change abruptly. On the other Maxwell's Equations Are LlDea~th ::: ect to time and spa-
hand, Maxwell's equations in the differential form presume derivatives of 1ields E and B WI P f densities P and j
that all~ quantities vary continuously in space and time. tial coordinates, and the first powers 0
However, the differential form of Maxwell's equations
can be imparted the same generality by supplementing them /" --,,
t~· t~
with the boundary conditions which must be observed for an s, \
electromagnetic field at the interface between two media.
These conditions are contained in the integral form of
Maxwell's equations and are already familiar to us:
DIn = D2n , Eh = E 2'f' BIn = B 2n , Hl'f = H 2'f
I
(
\ "'..... _--"",)
~r
I ebcb
E
Fig. 10.4
H

(iO.i6) Fig. 10.3


(here the fIrst and last conditions refer to the cases when there . c1Ul~g es. and curren ts '1'11e linearity
. .of Maxwell's
are neither extraneous charges nor conduction currents on the of clectl'lC t i . .' th the princIple f
0 super-
interface). It should also he noted that these boundary con- equations is dIrectly cOrnnledc et aWtiisfy Maxwell's equations,
. . 'f any two 10 S S t '
ditions are valid both for constant and for varying fields. pOSItIOn: 1 I d ' n also satisfy these equa IOns.
Material Equations. Maxwell's fundamental equations the sum o~ these !Ie ~ W{ude the Continuity Equation. This
still do not form a complete system of equations for the elec- Maxwell s EquatIons nc ation of electric charge. In
tromagnetic field. These equations are insufficient for de- expresses the law off ~~?se~;t us take an infinitely small
termining the fields from given distributions of charges and order to make sure 0 l~t n arbitrary finite surface S
currents. contour r, stredtclhl over ltracat this contour to a point, so
. 10 3) an t en COIl
(F19.
Maxwell's equations should be supplemented with rela- .<, . 'fi 'te In the limit, k.H dlvanishes,
tions which would contain the quantities characterizing in- that surface S remams III . '.)
dividual properties of a medium. These relations are CAlled surface S becomes closed, and the first of Eqs. (10.11) be-
material equations. Generally, these equations are rather com-
plicated and are not as general and fundamental as Max- comes
well's equations. ~ (j + ~~ ) dS = O.
Material equations have the simplest form for sufficiently
Hence it follows that
weak electric fields which vary in time and space at a com-' iJ t iJq
paratively low rate. In this case, for isotropic media ~ jdS= -.7ft'Y DdS=-~ -at' .
containing no ferroelectrics and ferromagnetics, the material . (4) This equatIOn
.
equations have the following (already familiar to us) form: which is just the continUlt~ eqfatlOl~ :~lu~e V t.hrough a
D = EEoE, B = /-l/-loH, j = a (E + E*), (iO.f7 J st.aWS ~hflt. the current flOWIng rom
262 10. Maxwell's Equations. Electromagnetic Field Energy
10-3. Properties of Maxwell's Equations 263
dosed surface S is equal to the decrease in charge in this
volume per unit time. . pnnci
new ill . 'pIe physical phenomenon: 1 electro.magnetic
. h 1 field
cur
The same law (viz. the continuity equation) can also be may exist independently, ~itho~t e ectnc c a~g~s all( -
obtained from Maxwell's differential equations. For this rents 'A change in its state ill thiS case necessanl~ has a
purpose, it is sufficient to take the divergence of both sides wave' nature. Such fields are called electromagnetIc wal~es.
of the first of Eqs. (10.14) and make use of the second of Eqs. In a vacuum, these waves always propagate at a velOCity
(10.13). This gives V·j = - ap/iJt. equal to the velocity of light c. ( D/;::> )
Maxwel's Equations Are Satisfied in All Inertial Systems of It also turned out that displacement cl.lrr~mt a vt
Reference. They are relativistic invariants. This is a conse- plays in this phenomenon a primary role. It IS Its presence,
quence of the relativity principle, according to which all E U
inertial systems of reference are equivalent from the physi-
cal point of view. The invariance of Maxwell's equations
(with respect to Lorentz' transformations) is confirmed by
numerous experimental data. The form of Maxwell's equa-
El _
tions remains unchanged upon a transition from one inertial
reference system to another, but the quantities contained in
them are transformed in accordance with certain rules. The
transformation of vectors E and B was considered in Sec. 8.
kr: n
o
B
0<\'
Thus, unlike, for example, Newton's equations in mechan- Fig. 10.5 Fig. 10.6
ics, Maxwell's equations are correct relativistic equations.
On the Symmetry of Maxwell's Equations. Maxwell's along with the quantity iJB/rJt, that mak.es the ~~~earalte
equations are not symmetric relative to electric and magnet- of electromagnetic waves possible. Any tll~e vana~lO~ () a
ic fields. This is again due to the fact that there are electric magnetic field induces an electric fwld, wIllIe a. va.l'latl~r! ~f
charges in nature, but as far as it is known at present, mag- an electric field, in its turn, induces a. magnetic held., 1 IllS
netic charges do not.exist. However, for aneutralhomogen- continuous~interconversionor interactIOn of the, fwlds pr~­
eous nonconducting medium, where p = 0 and j - 0, serves them, and an electromagnetic perturbatIOn propa-
Maxwell's equations acquire a symmetric form, since E gates in space. . f I
is related to aB/at in the same way as B to aE/at: Maxwell's theory not only predicted the e.Hstence 0 e ~c-
tromagnetic waves, but also made it possil~le ~o ~stabh~l~
V X E = - aB/at, V . D = 0, all their basic properties. Any electromagnetic. \\ ave, reg~r d
V X H = iJD/at, V • B = O. (10.18) less of its specific form (it can be a harm~mc wave 01, an
The symmetry of equations relative to electric and magnet- electromagnetic p~rturbation of any form) IS charactenzed
ic fields does not involve only the sign of the derivatives by the following properties: . . .
aB/at and aD/at. The difference in signs of these derivatives (1) the velocity of its propa~atlO.n m a noneonductmg
indicates that the lines of the vortex electric field induced neuttal nonferromagnetic medIUm IS equal to .
by a variation of field B form a left-hand screw system with v=c/Ve!Jo, where c=1/Veo!Joo; (10.19)
vector iJB/at, while the lines of the magnetic field induced
by a variation of D form a right-hand screw system with (2) vectors E, B, and v (wave velocity) are ITW~l~lllly per
vector aD/at (Fig. 10.4). . pendicular and form a right-hand scre~) s#stem (F Ig. 10.5)
Oil Electromagnetic Waves. Maxwell's equations lead to This riaht-handed relation is an intrtnSlC property of ~'!1
the following important conclusion about the existence of a electrOl~agnetic wave, independcnt of the choicc of COfll'( 1-
nate system;
264 10. Maxwell's E t·
---- qlla IOns. Electromagnetic F'Ie ld E nergy
10.4. Energy and linergy Flux. Poynting's Vector 265
. (3) vectors E and B in an e l e c t r ' -
cIlIate in phase (see F' 10 6 oma~netlc wave always os- This equation expresses Poynting's theorem: a decrease
~ wave), the instantan:;~ls V~ll sh~wlDg a "photograph" of in energy per unit time in a given volume is equal to the energy
lllg connected t1uough the reI t~ es EE and B at any point be- flux through the surface bounding this volume plus the work
a IOn = vB or
per unit time (Le. power P) which is accomplished by the field
. VeE oE=Vf.1f.1oH . ' (10.20) over the charges of the substance inside the given volume.
ThIs means that E and H ( B) .
maximum values vanish or sImultaneously attain theil' In Eq. (10.21), W = ) w dV, where w is the field
M I l 's b
axwe '
rilliant , e t c.. tl d
succes
tromagnetic theory of'light s m d Ie eve.lopment of the elec- J
energy density, and P= j.E dV, where j is the current
the ,possibility of existence ~asl ~~ to hIS u~derstanding of density and E is the electric field strength. The above ex-
10wlllg from differential equat~o~csI<:~~f;)~tJC waves, fol- pression for P can be obtained as follows. During the time
dt, field E will do the work ~A = qE.u dtoverapointcharge
f~.4. ,Energy and Energy Flux. Poynting's Vector q, where u is the charge velocity. Hence the power of force
PO~lltmg s Theorem. Proceedin f ' qE is P = qu·E. Going over to the volume distribution of
l?cahzation in the field itseif an g rom the ~dea of energy charges, we replace q by P dV, where p is the volume charge
clple of energy conservation d On the basIs of the prill- density. Then dP = pu· E dV = j. E dV. It remains for Us
:v
energy decreases in a certai ' e . Shou}~ conclude that if to integrate dP over the volume under consideration.
due to its "flowing out" tllrou nhl~~~on, t 11S ~ay only OCCur It should ~ noted that power P in (10.21) can be elther
under consideration (tl e
tionary), I
l . boundal'les of the region
me !lun IS assumed to be sta-
positive or negative. The latter takes place when positive
charges of the substance move against field E, or negative
In this respect thCle is a f charges move in the opposite direction. For example, such
of conservation of charge . . ormall an~logy with "the law a situation is observed at the points of a medium where in
ing of tl11S
' 'I aw consists in-ex tl pIteSEe( d by Eq . ("J. 4) . 'I'h e mean- addition to an electric field E, the field E* of extraneous
time in a given volume"
th
'I{J a ecrease in charge pel' uni.t
. IS equa to the flux f •
forces is also acting. At such points, j = (j (E +
E*),
e surface enclosing this ' I 0 vector l through and if E* tt E, and E* > E in magnitude, the product
I "I . \0 ume. j. E in the expression for P turns out to be negative.
. n.a SimI ar manner, for the law f .
\\e should assume that in Id't' 0 conservatIOn of energy Poynting obtained the expressions for the energy density
i.n " a(
a gIven regIOn, there e x i s t s , '. I IOn to the en ergy d ' w
enslty wand vector S by using Maxwell's equations (we shall not
zlllg the energy {lux density a eel tam vector S characteri- present this derivation here). If a medium contains no ferro~
. If we speak only of the en gy . electrics and ferromagnetics (Le. in the absence of hysteresis),
~ts total energy in a given ver of ~n electromagnetic field, the energy density of an electromagnetic field is given by
lllg out of this volume as ~~llre W~1l change due to its flow-
field transfers its ener t as ue to the fact that the E·D B·B
w=-Z--\--2-' (10.22)
i.e. accomplishes wor:Yov~ at~ubst~nce (charged particles),
form, this statement can bel' . ?ttSU stance. In macroscopic
wrl en as follows:
I
It should be noted that individual terms of this expres-

r--~-=---=-,y S ,fA + p. i (10.21)


sion have been obtained by us !'larHer [see (4.10) and (9.32)1.
The uensity of the electromagnetic energy flux, which is
called Poynting vector, is defined as
wb~r~ til\. is Jl surface element.
I S= [E X H]·I (10.23)
267
10.4. Energy and Energy Flux. Poynting's Vector
266 10. Maxwell's Equations. Electromagnetic Field Energy
. 'd the wire normally to its lateral sur-
Poynting vector is directed ~lsl the electromagnetic energy flows into
Strictly speaking, Maxwell's equations cannot give un- face (Fig: 10.7) COllseque? y, 1 But does it agree with the amount

~r~~?g~'~;';~ff*~;:~;'
ambiguous ex'pressions for the two quantities wand S.
The expressions presented above are the simplest from an in-
finite number of possible equations. Therefore, we should I
treat these expressions as postulates whose validity should the wire of lpngtll l.
be confirmed by the agreement with experiments of their EH .2nal = 2naH ·El = I'U = Rl2,
corollaries. differen~e acr?sts
Several following examples will demonstrate that although where U is the. potential
r
the results obtained with the help of formulas (10.22) the ends of thIS se?
ance. Thus, we arr.lve
lO:
t
d R is Its reslS -
~he conclusion that
into the
and (10.23) may sometimes seem strange, we will not be t~e electromag~edtlc ~de~~Yco~;i~telY eon-
able to find in them anything incredible, any disagreement WIre from outsl e a t gree
with experiment. And this is just the evidence that these verted into Joule's heat. We mus a lu-
that this is a rather unexpected cone Fig. 10.7
expressions are correct.
sionit should be .not~d t~~a; ~ins~ c~~~~~
Example 1. EDergy flux in an electromagnetic wavc (in vacuum),
Let us calculate energy dW passing during time dt through a unit area source, vecto~ ~hs ~~~~~ity lrthe source ,;,.-_--r_---
I
'P 2
perpendicular to the direction of propagation of the wave. I, anpd hent?nge10 ve~tor is directed outside:
the oyn 1 . gy flows
__--a.._---
I
f{J,

If we know the values of E and B in the region of location of the . th's region ~ctromagnetlc ener d
unit area, then !n \ urrdunding space. In other wor s,
!nto t eSt th t the energy from t~e source Fig. 10.8
dW = we dt, It turns ou a 1 the WIres but
is transmitted not a ong ! ductor
where w is the energy density, w = eo£2/2 +
,""oH2/2. In accordance through the space surroundlllg a con
in the form of the flux of electromagne 1
t·c energy viz. the flux of vec-
,
with (10.20), for an electromagnetic wave we have
t oE2 = llo H2 . tor S. l' of a balanced (twin) line.
Example 3. Figure 10.8.sho ws ~.sec ~~~ known as well as the fact
This means that the electric energy density in the electromagnetic The directions of curre.nts III the ::'Ires Can
wave at any instant is equal to the magnetic energy density at the that the wires' potent~als are <PI h rit) the E
same point, so we can write for the energy density we find where (to the rlgh~ or to tee
I
w = eo£2. Power source (generator) IS? .
Th nswer to this questiOIl can e
b ob-
tainedewith the help of. the ~oyntin~::~t~h~
Then
dW = EoE2 c dt = V Eo/Ilo E2 dt. In the case under conSIderatiOn, bet h'l
. E '. directed downwar dS W 1 e
Let us now see what we shall obtain with the help of Poynting wIres, vec~or . I~t d behind the flane of the
vector. The same quantity dW can be represented in terms of the mag- vector H IS dlrec e ~ -IE X II is direc1 d n

nitude of vector S as follows: figure Hence, wctor S - . th Fig. 10.9


to the' right, i.e. the power sou~ce IS on e
dW = S dt= Ell dt= lfeo / Ilo E2 dt. left and a consumer is on the rIght.
. L t take a parallehplate
Thus, both expressions (for wand for S) lead to the same result Example 4. Charging of a capacl~or. i n~rin edge effects (field
(the last two formulas). capacitor with circular plates of ra~lU~:~rg~ fiux 1hrcugh the lateral
Example 2. Evolution of heat in a coDductor. Let a current I now dissipation), fwd the. elect~omagnll? thO region the Poynting vector
through a straight circular wire of radius a (Fig. 10.7). Since the wire "surface" of the capaCitor, Sl?Ce on.y I~O 9)s
is directed inside the capaCItor (FIg. . , d t"c
has a re!listance, a certain electric field E is acting along it. The same . . electric fIeld E au a magne 1
value of E will be at the wire surface in a vacuum. Besides, the pres- On this surface, there l~ ;~ v.ar Yl'1 rding to the theorem on cir-
ence of current generates a magnetic field. According to the theorem
on circulation of vector H, near the surIace of the wire we have
field H generated by it.s vrarlllaLJOnih<
culation or vector H, It 0 OWS li
tCZnaH = na2 DD/8t. where the
2naH = I, H = 112n4. Vectors E andll are manged so that the
c

.
'
~
\ .
10.5. Electromagnetic Field Momentum
268 10. Maxwell's Equations. Electromagnetic Field Energy
of such a system can be conserved only
right-hand side contains the displacement current through the contour The momen t um . ld ( ) I esses
vided that the eleetromagnetlc fie wave a so poss .
shown in Fig. 10.9 by the dashed line. Hence H = (1/2) a aD/at. If pro t . the substance acquires the momentum due
the distance between the plates is h, the /lux of vector S through the
lateral surface is ~o ~~~e~~~~ierred to it by the electromagnetic fIeld.
a aD aD ,
EH2nah= E 2: at 2nah=E at l, (1)
E
where V = na 2h is the volume of the capacitor. We shall assume that

"I·~
this /lux is completely spent for increasing the capacitor's energy.
Then, multiplying Eq. (1) by dt, we obtain the increment of the capa-
citor's energy during the time dt: Po
---po;-
dW=EdD.V=d( ee~E2 V)=d( E: V)
B Fig. 10.11
Having integrated this equation, we find the formula for the energy W Fig. to.l0
of a charged capacitor. Thus, everything is all right in this case too.
t f momentum density G of
10.5. Electromagnetic Field Momentum Let us introduce the concep o. . 11 equal to
Pressure of Electromagnetic Wave. Maxwell showed theo- electromagnetic :re~~ea~lC\~ei~u:n~~;~ ~~~:~~aCIteulati~ns
retically that electromagnetic waves, being reflected or ah- ~~i~o~ft~~~mittedhere show that the momentum denSIty
sorbed by bodies on which they are incident, exert pressure
is given by
on these bodies. This pressure appears as a result of the action (10.24)
2
of the magnetic field of· a . wave on the electric currents '\ G=S/c , \
induced by the electric field of the same wave.
Suppose an electromagnetic wave propagates in a homo- _ [E X HI is Poynting's vector..Just a~ vector
geneous medium capable of absorption. The presence of ab- s~:'~;eIrtum density G is generally a functIOn of time and
sorption means that Joule's heat with the volume density
aE2 will be liherated in the medium. Hence a =1= 0, Le. coordinates. 'tl (10 20) for an electromagnetic wave
In accordance WI I d
the absorbing medium is conductive. ~ I -.
.
_
,
V-H Hence the energy en-
The electric field of tIle wave excites in this medium an elec- in a vacuum we have.v ~oE s-f ~;niing's vector are re-
sity wand the magllltu e 0
tric current with a density j = aE. As a result, Ampere's spectively equal to
force F u =- !j X B) =-= a [E X B] will act on a unit volume
w=BoE2/2+p.~..
f.T2/2 E2 S = EH = l r 80/ 11'0 E2.
=Bo • Y
of the medium. This force is directed towards the propaga- r
tion of the wave (Fig. 10.10) and causes the pressure of the lr- Ad' V~
It follows that S = wi Y Bol-lo·. n slUce _ wOe °T--;king'
ile -
e,leetromagnetic wave. I ' f l' ht lU vacuum S - .
In the absence of absorption, the conductivity a = 0 ~here e is the(1vOc 2°4)It~~ o~fain that for a~ electromagnetic
and F u == 0, i.e. in this case the electromagnetic wave does lUto account . ,
not exert any pressure on the medium. wave in a vacuum
Electromagnetic Field Momentum. Since an electromag-
netic wave exerts a pressure on a substance, the latter ac-
I G=wle. \
(10.25)

and momentum is inhe-


quires a certain momentum. However, if in a closed system
consisting of a substance and an electromagnetic wave only The same. relation ?et~~e~h~~~~g~f relativity) in particles
the substance possessed a momenLum, the Jaw of conservu- re~t (as IS sho~n lUTh .s is quite natural since, according to
With zero rest mass. 1
tion of momep.t'llm would be yjolated.
271
210 10. Maxwell's Equations. Electromar(_fI:!!.~c Field Ener~.__ Problems

quantum-mechanical notions, an electromagnetic wave is Problems


equivalent to the flow of photons, viz. the particles with zero A int charge q moves uniformly
rest mass. • 10 1. Displacement currenlt· · po · velocity v Find the vector
" 1- 'th a nonre a t IVIS t 1c
along n ~I.ralght Inl' WI . at a oint P lymg. .
at a d'1~'I,ance ~
Some More Remarks on the Pressure of EJcctl'Omagnelie of the di~placement curren.t ~rnl~~: (1) c~nciding with its traJectory,
Waves. Let us calculate, llsing forll1111a (10.2;"»), tlte pres- from the charge on a. stral~ t ry and passing through the charge.
(2) perpendicular t~ ItS traJec 0
sure exerted by an electromagnetic wave on a body when the
wave is incident normal to its surface and is partially reflect- dD
ed in the opposite direction. In accordance with the law of
conservation of momentum, Po = p~ -+- p, where Po, p~ are
the momenta of the incideilt and reflected waves, wltile p
is the momentum transferred to the hody (Fig. 10.11). Hav-
ing projected this equality onto the direction of the inci-
dent wave and referring all the quantities to unit time and
unit cross-sectional area, we obtain
P, D dO
P = Po + P; = (G) c + (G') c, v o r
where (G) and (G') are the average values of the moment lim Fig. 10.13
10.12
density for the incident and r.eflected waves. It remains for
us to take into account relation (10.25) between (G) and
(w) as well as the fact that (w') = P (TV), where {> is the . t 'current density jd = DD/at. Hence,
Solution. The lhsplaceme~, reducPlI to determining vector D at
ref!ectioncoefficienl. As a resu)t, the previous expression be- the solution of the problem 1;;. g . time deriv'ltive. In both cases,
comes the indicated points nTll~ toI fmdI,n It~ of r Let ~IS find the derivative
D --- qe r /4~r2
n ••. , where e r IS t Ie lITH t vee or .
P = (1 +p) (w). (10.20) iJD/Dt
(1) At pomt '2 where it is assumerl that q > 0), we
. P I (I"'lg. to .1,
have
Here the quantity P is just the pressure of the electromagnet- aD 2q or _ 2qv
ic wave on the body. In the case of total reflection, r ._= -at= - -nr 43;;-t
u
er- 4nr 3 ,
= 1 and pressure p = 2 (w) while for total absorption,
p = 0 and p = (w). h f oint P the derivative arMt = -v.
where we took into a~cfun\t fitt~rc~arge(i~ the direction of its motioiJ
It should be added that the pressure of f'lectromagnetic If point PI were not m. ron Old b directed in the same way and wou
radiation is usually very low (the exception is the pressure but behind it, vector ~d wou e.
of high-power laser radiation beams, especially after beam have the same magmtude.. . and vice versa.
focusing, and the pressure of radiation inside hot stars). Thus,
(2) At for.
POint :,ec~ort~d)
q >P 0,(FIg. •. , ttl ~b I/D = v dt/r, and hence we can
2
For example, the pressure of solar radiation on the Earth write / _ -"
amounts to about 10-6 Pa, which is smaller than the atmo- aD/at = -qv 4nr.
spheric pressure by a factor of 10 10 . In spite of insignifIcant
values of these quantities, experimental proof of the If q > 0, id tI v, and vice .vers~. '. ht solenoid with the
• 10.2. A current n?wm~ III a long str~~{1netic field inside the
existence of electromagnetic waves, viz. the pressure of radius R of cross sec~ion I~ varIed sOrtha\~h~he l~w B = ~t2, where ~
light, was obtained by Lebedev. The results of these experi- solenoid increaFs~
is a constant. 'Illl IW~~~ lt~pla~~~;~:~~rrent
.. (Iensity as a function of
ments were in agreement with the electromagneticJheoryof the distance r from the solenoId aXIS.
light.
272 10. Maxwell', Equation,. Electromagnetic Field Ener,,,
273
Problem'
Solution. In order to find the displacement current density, we
must, in accordance with (to.5), first find the electric fiehl strength
(here it will be a vortex field). Using Maxwell's equation for circula-
Let us transform the expression in the parentheses to cosine. For thi~
~~1~h:~7:t:d~~~I:n~1:t~~~~;~i~h~xF;::~I:SbJi/:: c~ 6 te~~iiwL
2
tion of vector' E, we write
,
2nrE = nr2 aBlat, E = r~t (r < R); =sin 6. This gives
2nrE = nR2 aBlat, E = R2~tlr (r> R).
Now, using the formula id = eo iJElat, we can find the displacement
H = ~ rEm va +(eeoco
2 )2 cos (cot+6).

current density: A point charge q moves in a vacuum l;lniformly ~l,d


id = eo~r (r < R); id = eo~R2/r (r > R). rec~i~:a'~y with a nonrelalivistic velocity v. Usmg Maxwe 8

The plot of the dependence ; d (r) is shown in Fig. to.13•


• to.3. A parallel-plate capacitor is formed by two discs the space
between which is filled with a homogeneous, poorly conducting medium.
The capacitor was charged and then disconnected from a power source. o
Ignoring edge effects, show that the magnetic fIeld inside the capacitor
is absent.
Solution. Magnetic field will be absent since the total current (con-
ot ! ~D q~~_V

duction current plus displacement current) is equal to zero. Let us


prove this. We consider the current density. Suppose that at a certain Fig. 10.15
Fig. 10.14
instant the density of conduction current is j. Obviously,j ex D and
D = an, where a is the surface charge density on the positively charged • . o'f t H obtain the expression 'for H
plate and n is the normal (Fig. 10.14). equati0!l for thhe CIrcuI~tt~~~ rel::i~~rto the charge is characterized by
The presence of conduction current leads to a decrease in the surface at a pOlUt P w ose POSI I
charge density a, and hence in D as well. This means that conduction radius vector r (Fig. 10.15).
current will be accompanied by the displacement current whose den-
sity is -Solution. It iShcl:~r f~om ls~~~e;?v~~~g:d~t:~~~dthba:~~~e~~o:;
jd = aD/at = (aalat) n = -jn = -j. tour arohund whi~cle wel'~hcc~:t:~ 0 (its trace is'shown in Fig. 10.16
must c oose a clr
Hence it follows that, indeed by the dashed line). Then
+ jd = O. Jr D n dS,
it = j 2nRH=..!- (1)
at
• 10.4. The space between the plates of a parallel-plate cafacitor
in the form of circular discs is filled with a homogeneous poor y con-
ducting medium with a conductivity a and permittivity e. Ignoring
wheLre~tR iSfit~e tr:ed~~xo~fte:c~~r~l~. through a surface boun~ed by this
reus n f' 1"t e shall take a spherIcal surface
edge effects, find the maci'litude of vector H between the plates at
a distance r from their axeb, if the electric field strength between the
circle. For t~e safke 0 Stlmp IC(£.lg'
with the radIUS 0 curva ure r •
fO

16) Then the flux of D through
• .
plates varies with time in accordance with the law E = an elementary ring of this spherical surface IS
= Em cos wt. q . , :1_' q. , da.'
Solution. From Maxwell's equation for circulation of vector H. D ds=--2nr SlD a ·r ..... ='2 SlUa ,
4nrll
it follows that
while the total flux through the selected surface is
2nrH=(i n + ee o 8it) nr l
.
~ DdS=~ (i-cos a). (2)
Taking into account Ohm's law jn = aE n (t). we obtain
Now, in accordance with (1), we differentiate (2) with respect to
eeo aE n )
r (E
H ="2 n+oiJ't rEnt .
=-r(acoswt-eeows!nwt). time:
8\ q. cia. (3}
8t J D dS = 2: SlD a dt .
18-0181
274 10. MGZlIIelt, Equation•. Electromagnetic Field Energy Problem. 275

For the displacement of the charge from point 1 to point 2 (Fig. 10.17) SolutiorL l"igure 10.19 shows that S tt v. Let us find the mag-
over the distanee v dt, we have 1.1 dt ·sin a. = r da., whence nitude of S: S = EH, where E and H depend on r. According to the
GaUS8 theonm, we have
da. 1.1 sin a.
2rtrE = 'AJeo,
Cit r (4)
where A is the charge per unit length of the beam. Besides, it follows
SubstitutiDg(4) into (3) and then (3) into (1), we obtain from the theorem on circulation of vector H that
H = qvr sin a./4m,a, (5) 2rtrH = f.
where we took into account that R = r sin a.. Relation (5) in vector Having deWnnined E and H from the last two equations and taking
form can be written as follows: into aee:oant. that f = 1.1.1, we obtain
H=_LJvXr) S = EH = f 8/4n 8 eol.lr2.
4n r3 e 10.8. A current flowing through the winding of a long straight
Thus we see that expression (6.3) Which we have postulated earlier dB
is a corollary of Maxwell's equations.
el0.6.CuriofE.Acertain :re2ion ofan inertial system of reference
contains a magnetic field of m'1g11itude B = const, rotating at an
l' 'k-
H
\
I \

C;;2:'$Z;::::~=Z&-~~
S

\ /
\J
p
Fig. 10.18 l"ig. 10.19

solenoid is being increased. Show that the rate of increase in the energy
of the magnetic field in the solenoid is equal to the flux of Poynting's
vector through its lateral surface.
Solution. As the current increases, the magnetic field in the !'olenoid
also increases, and hence a vortex electric field appears, Suppose that
1 2 o the radius of the solenoid cross section is equal to a. Then the strength
of the vortex electric field near the lateral surface of the solenoid
Fig. 10.16 Fig. 10,17 can be determined with the help of Maxwell's equation that expresses
the law of electromagnetic induction:
angular velocity roo Find V X E in this region as a function of vectors
and B. aB a aB
fI}
2rtaE = rta
8
at ' E =2" at·
Solution. It follows from the equation V X E = - aB/at that
vector V X E is directed oppositely to vector dB. The magnitude of The energy flux through the lateral surface of the solenoid can be re-
this vector can be calculated with the help of Fig. 10.18: presented as follows:
I dB I = Bro dt, I dB/dt I = Bro. 2 a ( B8 )
Henee cD=EH·2nal=rta / at 2f.1o '
V X E = -fro X . BI. where I is the solenoid length and rta2 l is its volume.
Thus. we see that the energy flux through the lateral surface of the
e 10.7. Poynting's vector. Protons haVing the same velocity v solenoid (the flux of vector S) is equal to the rate of variation of the
form a beam of a circular cross section with current f. Find the direc- magnetic energy inside the solenoid:
tion and magnitude of Poynting's vector S outside the beam at a
distanee r from its axia. cD = S ·2rtal = ow/at.
18*
11.1. Equation of an Olctllatory Circuit 277

216 10. Mazwtll's Equations. Eltctromagnttic Fldtl Elltr" plates be h. Then the electric energy of the capacitor is equal to
II EI II 'la l
• 10.9. The energy from a source of constant voltage U is trans- We=+'la2h=~ UfucoslCllt. (1)
mitted to a consumer via a long coaxial cable with a negligibly small
resistance. The curr~nt in the c!lble is f. Find the energy nux through The maguetic energy can be determined through the formula
the cable cross sectIOn, assumIng that the outer conducting shell of
Et~
the cable has thin walls.
Solution. The required ener-
gy llux is defined by the formula
Wm = \

:1 110
dV. (2)

The quantity B required for ~valuating this integral can be found

dfr-:=-H= S e---o <I>:=:; j


a
S·2nr dr, (1)
from the theorem on the circulation of vector H: 2'lrH = 'lr2 aDlat.
Hence, considering that H = BII10 and aDlat = -'-eo (Umlh) CIl sin CIlt,
we obtain
1 rCllU m •
Fig. to.20 B=2'llolto-h- \ SlDCIlt I· (3)
where S=EH is the nux density,
. 2'lr dr is the elementary ring of It remains for us to substitute (3) into (2), where for dV we must
width dr within which the value of S is constant, and a and b are the take an elemeutary volume in the form of a ring for which dV =
radii of the internal wire and of the outer shell of the cable (Fig. 10.20). = 2nr dr·". All a result of integration, we obtain
In order to evaluate this integral, we must know the dependence
S (r), or E (r) and H (r). 'l J.LollifalSa6Ufu '1 (4)
W m=16 h sIn CIlt.
Using the Gauss theorem, we obtain
2'lrE = '}Jllo, (2) The ratio of th.e maximum values of magnetic energy (4) and elec-
tric energy (f) is given by
where Ais the charge per unit length of the wire. Further, by the theo-
rem on the circulation' we have WmU:-ax 1 1 1
W 8 l1011oiJ cIl •
emax
2'lrH = f. (3)
,For example, for a = 6 cm and cIl = toDD S-I, this ratio is equal to
After substituting E and H from formulas (2) and (3) into expression 5 X to-D.
(1) and integrating, we get ,
AI b
<I>=--ln - (4)
2'lll o a

The values of A, a, and b are not given in the problem. Instead, II. Electric Oscillations
we know U. Let us find the relation between these quantities:
b H.t. Equation of an Oscillatory Circuit
U= E
A b
dr=--ln-.
) 2'lllo a (5) Quasi-steady Conditions. When electric oscillations occur,
a the current in a circuit varieswith time and, generally speak-
A comparison of (4) and (5) gives ing, turns out to be different at each instant of time in diff-
erent sections of the circuit (due to the fact that electromag-
III = Uf.
netic perturbations propagate although at a very high but
This coincides with the value of power liberated in the load. still finite velocity). There are, however, many cases when
• 10.tO. A parallel-plate air capacitor whose plates are made instantaneous values of current prove to be practically the
in the form of disks of radius a are connected to a source of varying same in all sections of the circuit (such a current is called
harmonic voltage of frequency CIl. Find the ratio of the maximum values quasistationary). In this case, all time variations should occur
of magnetic and electric energy inside the capacitor. SO slowly that the propagation of electromagnetic pefturba-
Solution. Let the voltage across the capacitor vary in accordance
with the law U = Urn cos rot and the distance between the capacitor
11.1. Equation of an Oscillatory Circuit 279
278 11. Electric Oscillations
in the circuit attains its maximum value (Fig. 11.1b).
tio~s c~uld be c.onsidere~ instantaneous. If 1 is the length of Starting from this moment, the current begins to decrease,
a c~rcUlt, the time reqUlred for an electromagnetic pertur- retaining its direction. It will not, however, cease immediate-
batIOn to cover the distance 1 is of the order of 1: = lie. ly since it will he sustained by self-induced e.m.f. The cur-
F~r a periodically varying current, quasi-steady condition rent recharges the capacitor, and the appearing electric field
WIll be observed if will tend to reduce the current. Finally, the current ceases,
't = lIc~T, while the charge on the capacitor attains its maximum value.
From this moment, the capacitor starts to discharge again,
where T is the period of variations. the current flows in the opposite direction, and the process
For e:ample, for a circuit of length 1 = 3 m, the time . is repeated.
't = 10 8 s, and the current can be assumed to be quasista- If the conductors constituting the oscillatory circuit have
no resistance, strictly periodic oscillations will be observed
R in the circuit. In the course of the process, the charge on the
capacitor plates, the voltage across the capacitor and the
current in the induction coil vary periodically. The oscil-
L lations are accompanied by mutual conversion of the energy
of electric and magnetic fields.
If, however.... the resistance of conductors R =1= 0, then, in
(a) (b) 6 addition to the process described above, electromagnetic
Fig. 11.1 Fig. 11.2 energy will be transfor~ed into Joule's heat.
Equation of an Oscillatory Ci~uit. Let us derive theequa-
tionary down to frequencies of 106 Hz (which corresponds to tion describing oscillations in a circuit containing series-
T = 10-6 s). connected capacitor G, induction coil L, resistor R, and
In this chapter, we shall assume everywhere that in the varying external e.m.f. liJ (Fig. 11.2).
cases under consideration quasi-steady conditions are ob- First, we choose the positive direction of circumvention,
served and currents are quasistationary. This will allow us to e.g. clockwise. We denote by q the charge on the capacitor
use formulas obtained for static fields. In particular we shall plate the ~iirection from which to the other plate coincides
b.e using the fact that instantaneous values of 'quasista- with the chosen direction of circumvention. Then current in
tIOnary currents obey Ohm's law. the circuit is defined as
Oscillatory Circuit. In a circuit including a coil of indu-
c~ance L and a capacitor of capacitance G, electric oscilla- I = dqldt. (11.1)
tIOns may appear. For this reason, such a circuit is called an Consequently, if I > 0, then dq > 0 as well, and vice versa
oscillatory circuit. Let us find out how electric oscillations (the sign of I coincides with that of dq).
emerge and are sustained in an oscillatory circuit. In accordance with Ohm's law for section lRL2 of the
. Suppose tha~ .initially, the upper plate of the capacitor circuit, we have
IS charged p~sltlvely and t~e lower plate, negatively (Fig.
11: 1a). In tIus case, the entIre energy of the oscillatory cir- RI = CPl - CP2 ~8+ +
'IS, (11.2)
CUlt IS co~centrated in the capacitor. Let us close key K.
where 'fe s is the self-induced e.m.f. In the case under con-
T~e capacItor star!s to discharge, and a current flows through
cOlI L. The electrIc energy of the capacitor is converted in- sideration,
to the magnetic energy of the,coil. This process terminates ts = - L dIldt and CP2 - CPl.= giG
when the capacitor is discharged completely, while current
:1"
........

280
11. Electric Oscillations
281
- - - - - -11.2.
-- Free Electric OscUlatlo,..
(the sign of q must coincide with th .
difference (JJ2 - (JJ since G >0) He sIgnE,of the potential The solution of this equation is the function
written in the f~rm . ence q. (H.2) can be
q = qm cos (root + a), (11.8)
L dI q where qm is the maximum value of the charge on capacitor
7t+RI+c=~, (H.3) plates, (J)o is the natural frequency of the oscillatory cir-
or, taking into account (H.t), cuit, anda is the initial phase. The value of roo is determined

L aSq
(iii"+R 7t
dq
+ c1 q=~.
I (11.4)
only by the properties of the circuit itself, while the values
of qm and a depend on the initial conditions. For these con-
ditions we can take, for example, the value of charge q

This is the equation of an 'llat . .


q
and current I = at the moment t = O.
near second-order nonho OSCl o,:y clrcU.lt, which is a H- According to (11.6), roo = lIY LC; hence the period of
constant coefficients U~i~ggenteho.us differ~ntiafl equation with free undamped oscillations is given by
~.
t) . IS equatIOn or calculat'
q ( ,we can easily obtain the voltage across th .mg To=2~Y LC (H.9)
as U c = (JJ2 - (JJI = qlG and current I b e capaCitor (Thomson's formula).
en;~~r,::::uation of an oscillatory circuit ca:b~o;~~~aa~i~:L Having found current I [by differentiating (11.8) with
respect to time] and bearing in mind that the voltage across

l·q+2Pq+ro:q=~/L,
the eapacitor~ plates is in phase with charge q, Vie can easily
see that in 1ree undamped oscillations current I leads in
(11.5) phase the voltage across the capacitor plates by n/2.
where the following notation is introduced: While solving certam problems, energy approach can al-
so be used.
. 2p = RIL, ro: = lILG. (11.6) Exat~. In an oscillatory circuit, free' undamped oscillations
TIle quantity ro' II d h occur with energy W. The capacitor plates are slowly moved apart
and p is the dar:;, I~nca e t e natural f!equency of the circuit so that the frequency of oscillations decreases by a factor of 'I. Which
will be explaine~ b~/actor. The meanmg of these quantities work against electric forces is done in this case?
If _ . ow: The required work can be represented as an increment of the energy
tions ~The~' t~ll ~cllladatlOns are usually called free oscilla- of the circuit:
R' e un mped for R = 0 and dam ed fI
WI
=F O. Let us consider consecutively all these casts. or A = W' - W = 9; (~, - ~ ) = W ( g, -1) .
On the other hand, 000 ex: HVC, and hence 'I = CilMCilI = VC/C',
11.2. Free Electric Oscillations and consequeutly
Free Undamped Oscill a t· If . . A = W ('II - 1).
IIal e.m.f. ~ and if 't lon~. a CIrCUit contains no exter- Free Damped Oscillations. Every real oscillatory circuit
. I S resistance R -. 0 the '11 t'
111 such a circuit will be free d d- , OSCI a Ions has a resistance, and the energy stored in the circuit is grad-
describin th '11" an un amped. The equation ually spent. on heating. Free oscillations will be damped.
<£' g Oese OSCI atlOns IS a particular case of Eq (11 5)
wIlen e; = and R = 0: . . We can obtain the equation for a given oscillatory circuit

(11.7)
..
by putting ~ = 0 in (H.5). This gives
.
q+ 2pq +ro:q = O. (11.10)
282
11. Electric Oscillations
11.2. Free Electric Oscillations 283
I t can be shown (we shall not d · .
ested in another as ct of 0 It here SlDce we are inter-
the solution of this ~ the pro~lem) that for ~2 < (J)I, las
omogeneous dIfferential equation ha"s - ~/roo = cos l'l, ro!Ul o = sin 15. (1.1.15)
the form
f fJ~o.ft After this, the exp~ession for I takes the following form:
-.../' q= qme - flt cos (oot+a),
I = Ulqme-flt cos (Ult + a + 15). (1.1.16)
(H.H)
where It follows from (11.15) that angle 15 lies in the second quad-
t rant (n/2 < l) < n). This means that in the case of a non-
zero resistance, the current in the circuit leads voltage in phase
(J)=V~ (1.1.14) across the capacitor by more than n/2. It should
be noted that for R = 0 this advance is l) = n/2.
Fig. 11.3 /1 (R)2 The plots of the dependences Uc (t) and I (t) have the form
= l LC - 27' (11.12)
similar to that shown in Fig. 11.3 for q (t).
qr
?n.~. and. ~ are arbitrary constants determined from th
~lll~~ c0 nd 1tlOns. The plot of the function (11.11) is sho e Example. An oscillatory circuit contains a capacitor of c:apIICi&ance
C and a coil with resistance R and inductance L. Find the ratio be-
III I~. 11:.3 It can be seen that this is not
a . ,,:n tween the energies of the magnetic and electric fields in the circuit at
fu~tlOn SInce it determines damped oscillations periodiC the moment wh:Jl the current is at a maximum.
. "'devjedrtheless, t~e quantity T = 2n/oo is call~d the _ According to equation (11.3) for an oscillatory circuit,
no 0 amped oscLllations: pe

T- 2ft To L ~; +RI+ ~ =0.


Vt-(~/CiIo)2'
wI~~e
d
r - V&>B-W'
0 is the ~er~od of free undamped oscillations.
e act?r q"!e flt III (11.11) is called the amplitude 0
(H.13) When the current is at a maximum, dIldt = 0, and RI = -q/C.
Hence the required ratio is
Wm/We = L/CR2.
a1m3Pebd osch~llatwns. Its ?ependence on time is shown in Fig1
1 . y t e dashed hne. . Quantities Characterizing Damping.
. V o~ttagKe acr~s a
CIrCUI. nOWIllg q (t) we Capacitor and Current in an Oscillatory 1. Damping factor ~ and relaxation time "t, viz. the time
p·t d , c a n fiIII d tlIe vo Itage across a ca- during which the amplitude of oscillations decreases by a
c:~~C~~O:niS ~~~e~u~~nt in a circuit. The voltage across a factor of e. It can be easily seen from formula (11.11) that
"t = 1/~. (11.17)
Uc = ~ = q~ e-lltcos(rol+a). (1.1.14) 2. Logarithmic decrement A. of damping. It is defined as the
Naperian logarithm of two successive values of amplitudes
The current in a circuit is measured in a period T of oscillations:
1=. !:!L. ~c qme-Ilt [ - Il
cos (00t + a)-rosin(oot+a)j. a (t)
dt -- .... A. = In a (t+Tl = ~T, (11.18)
i~? transform the expression in the brackets to cosine For where a is the amplitude oi the corresponding quantity
V ~2 ~urp~s~ we multiply. and. divide this expressio~ by
1- ~ "- Wo and then mtroduce angle 0 by the formu-
(q, U, or I). In a different form, we can write
(11.19)
i I

11. Electric Oscillations


11.8. Forced Electric O,cUlations
where N e is the number of oscillations during the time 't, ' g tbl'S time Ne oscillations will
i.e. the time required for the amplitude of oscillations to , - 11~ Durm
value during the time : -d f damped oscillations, t en
'h
decrease to 1/e of its initial value. This expression can be
easily obtained from formulas (11.17) and (11.18). occur.
If T is the perlo 0
't 1.{~ =_ 1 V (W _0
)2
-1.
If damping is small (/32 ~ (O~), then (0 ~ (00 = vy LC Nc=-=
T
r~ 2n
2n/V ro6-P
~
and, in accordance with (11.18), we have 2 = 1./ LC and l> = R {2L, we obtain
Considering tllat roo ---c._-
A. ~ ~. 2n/Ulo= nRY C/L. (11.20) _1._ ~ / .!£-1.
3. Q-/actor of an oscillatory circuit. By definition, N e = 2n VCR'
. . hi Ii tbe amylitude of current
Q = nlA. =
nNe' (B.21) Example 2. Fin~ t~e ti?1e duni~~nwQ-factor wH. de~rea~ to 1~
illations in a CllCUlt WIth a g of damped OSCIllatIOns IS equ
where ). is the logarithmic decrement. The smaller the damp- osc. . 't'a1 value if the frequency
of 1ts lUl 1 ~ , • • h

;mrex:
ing, the higher the Q-factor. For a weak damping (/32 ~ to roo . . . e- IlI , the time to durmg wl1!-c
~ (0:), in accordance with (11.20), we have Since the current amphtu~e of Tj is determined by the equatIOn
the amplitude decreases by a ac 0
Q~ ~ V ~. (11.22) 1\ = ept•• Hence Ill.
to = (In 1'\) p. ,

Here is one more useful formula for the Q-factor in the case of bftDd the Q-factor is also related to ~:
a weak damping: On the other 'Q = Ttl;, = n/~T = ro/2~.
W • • A f m the laa\ two equations, we obtain
Q~2n 6W' (11.23) Elimmatmg I' ro
2Q
to = c;) In Tj.
where W is the energy stored in the circuit and II W is the de-
crease in this energy during the period T of oscillations. In-
deed, the energy W is proportional to the square of the am- 11.3. Forced electric Oscillations
plitude value of the capacitor charge, i.e. W ex:. e-2pt.
Hence the relative decrease in the energy over the period is . . Let us return to equations (11.3)
6WIW = 2~T = 21... It remains for us to take into account
Steady-state OscJlla~lOns. . 't and consider the case
1 4) for an OSCillatory Clf~Ul t al emf ~ whose
the fact that according to (11.21), A. = n/Q. an d (1 . . ' 1 d a varymg ex ern .,.
C~ncluding the section, it should be noted that when when the circUlt ,mc ~ es reased by the harmonic law:
Ill> (O~, an aperiodic discharge of the capacitor will occur dependence on tlUle IS dec. (11.25)
~ = Om cos (Ot.
instead of oscillations. The resistance of the circuit for which
the aperiodic process'sets in is called the critical resistance: . .al lace owing to the proper t'les of
This law OCCUPI~S a .Sp~Cl If lo retain a harmonic form of os-
R cr =2YL/C. (11.24) the oscillatory CIrCUlt ItS~. of external harmonic c.m.!.
dIlations under the aCt' lonfor a~ oscillatory circuit IS
Let us consider two examples. In this case, the equa Ion
Example t. An oscillatory circuit has a capacitance C inductance L, written in the form (11 26)
and resistance R. Find the number of oscillations a lter which the L ~+RI+...L=~m·cos(Ot, .
amplitude of cunent decreases to fie of its initial value. dt C
The eunent amplitude (/m ex: e -Pt) decreases to fie of its initial or (11.27)
11.3. Forced Itkctrlc O.ctUatio..
11. ItZectrlc OBctli4ttoIU
·1· f '£! Taking into account relation (11.30),
As is known from mathematics, the solution of this equa- totheexterna e.m.. ",.
tion is the sum of the general solution of the homogeneous we write (11.32)
equation (wiih the zero right-hand side) and a particular U R = RI = RI m cos (rot - cp),
solution of the nonhomogeneous equation.
q --.J!!!. cos (rot _ '\jJ) = I Tn cos (rot - cp - ~ ) , (11.33)
We shall be interested only in steady-state oscillations, U e=c- c we
i.e. in the particular solution of this equation (the general
solution of the homogeneous equation exponentially attenu- UL = L ~~ = - roLlrn sin (rot - cp)
ates and after the elapse of a certain time it virtually van-
(11.34)
ro LIm cos trot - cp + T
• 11: )
ishes). It can be easily seen that this solution has the form = .
q = qrn cos (rot -'Ii')~ (11.28) V tor Diagnun The last three formulas show that U
where qrn is the amplitude of charge on the capacitor and 'Ii'
is the phase shift -between oscillations of the charge and of
is i:;hase with cu~rent I, U e lags behind I by n/2, and U
R
l.

Im
the external e.m.f. ~ (11.25).• It will be shown that qrn
and", depend only on the properties of the circuit itself and
the driving e.m.f. ~. It turns out that 'Ii' > 0, and hence q
always lags behind ~ in phase.
In order to determine the constants qrn and 'Ii', we must sub-
stitme (11.28) into the initial equation (11.27) and trans-
form the result. However, for the sake of simplicity, we shall
pr~ in a different way: first find current I and then sub-
stitute its expression into (11.26). By the way, we shall sol-
o
ve the problem of determining the constants qrn and 'Ii'. Fig. 11.5
Differentiation of (11.28) with respect to time t gives Fig. H.4

1= - roqrn sin (rot -'\jJ) = roqrn cos (rot -'\jJ + n/2). leads I by n/2. ~his
candbe. v:~ugal;~er:::;f~~~~ w;jh vt~~
help of a vector dtagram, epIC m
Let us write this expression as follows:
tages
I = I rn cos (rot - cp), (11.29) - Rf U em = 1 m/roC, U Lm = roLl rn
Uam - m,
where I rn is the current amplitude and cp is the phase shift and their vector sum which, according to (11.31), is equal
between the current and external e.m.f. ~,
to vector ~m (Fig. 11 .4). . d' can
I rn = roqrn, cp = 'Ii' - n/2. (11.30) Considering the right triangle of. thIS Iagram,;,e
easily obtain the following expressIons for I m and cpo
We aim at finding I rn and cpo For this purpose, we proceed as
~m (11.35)
follows. Let us represent the initial equation (11.26) in the
fonn 1m = l!R'+«(a}L-1/wC)Z'
wL-1/(a}(: (11.36)
UL + Un + Ue = ~rn cos rot, (11.31) tancp= R
where the left-hand side is the sum of voltages across induc-
tion coil L, capacitor C and resistor R. Thus, we see that Th the problem is solved. . d b
It :~uld be noted that the vector diagram obtame a ove
at each instant of time, the sum of these voltages is equal
288 11.3. Forced Electric Oscillattons 289
11. Electric Oscillations
--'-'-------
proves to be vel'y con . f The maximum of this function or, which is the same, the
problems. It permits a vi:~~lent 01' solving .many specific minimum of the radicand can be found by equating to zerO
various situations. ' easy, and rapId analysis of the derivative of the radicand with respect to cu. This gives
.Resonance Curves. This is the . the resonance frequency (11.38).
dependences of the folloWing q n:-~e gIven to the plots of Let us now see how the amplitudes of voltages U R' U e,
of external e.m.f. ~: current I uahn lIes on the fre~uency (J) and U L are redistributed depending on the frequency W
, c arge q on a capaCitor, and of the external e.m.f. This pattern is depicted in Fig. 11.7.
The resonance frequencies for U R' U e and U L are deter-
mined by the following formulas:

(J}e res = WoY 1- 2 (~/(l)O)2, (11.40)


c6
(J}L res = WoY 1- 2 (~/WO)2.
The smaller the value of ~, the closer are the resonance fre-
quencies for all quantities to the value Woo
Fig. 11.6
Fig. H.7 Res6n3nce UII'Ves and Q-factor. The shape of resonance
voltages UR Ue and U d fi . curves is connected in a certain way with the Q-factor of an
(11.34). " L e ned by formulas (11.32)- oscillatory circuit. This ~onnection has the simplest form
Resonance curves for current I ( ) '. for the case of weak damping, I.e. for ~2 «w~. In this case,
U.5. Expression (11.35) shows th m cu are shown m Fig. Ueres/cem = Q (11.41)
has the maximum value for cuL _ ~t the~urrent amplitude
the resonance frequency for current lcur: -: O. C?nsequentlys (Fig. 11.7). Indeed, for ~2« cu~, the quantity w res ~ W o
tural frequency of the oscl'll t .col~cldes with the na- and, according to (11.33) and (11.35), U em res = I m/cuoC =
a ory CIrCUIt:
= tm/woCR, Or Uemres!'tm = YLCICR = (1IR) YLIC
. WI reB = Wo = lIV LC. (11.37) which is, in accordance with formula (11.22), just the Q-
The maXImum at resonance i th h' factor.
the smaller the damping facto: A. ~ ~~~er and the sharper Thus, the Q-factor of a circuit (for ~2« w~) shows how
Resonance curves for the cha p L. many times the maximum value of the voltage amplitude
.shown in Fig. 11.6 (resonance rge qm (cu) On a capacitor are across the capacitor (and induction coil) exceeds the ampli-
across the capacitor have the curves for voltage Uem tude of the external e.m.f.
charge amptitude is attaine~a~~ slhlape). The maximum of The Q-factor of a circuit is also connected with another
. t e resonance frequency important characteristic of the resonance curve, viz. its
. Wqre8=.Vw~-2~2, (11.38) width. It turns out that for ~2« w~
which comes closer and closer to .
In order to obtain (11 38) W o WIth decreasing p. Q = cuolfJw, (11.42)
ance with (11.30), i~ the ~~rmust represent qm, in accord- where CU o is the resonance frequency and fJw is the width of
defined by (11.35). Then m qm = I mlw where 1 m is the resonance curve at a "height" equal to 0.7 of the peak
height, I.e. at resonance.
qm = - _ ~=/L==-_ Resonance. Resonance in the case under consideration is
V(W3-WI)2+4~IWI (11.39)
1{2 19-0181
1111
III!
11:1

I!II

290
Iii 11. Electric Oscillations
11.4, Alternating Current 291
II the excitation of strong oscillations at the frequency of ex-
I impedance:
ternal e.m.f. Or voltage. This frequency is equal to the na-
II tura~ fre~\lency of an oscillatory circuit. Resonance is used Z = V' R2 + (roL -1/coC)2. (11.46)
for slug-hng out a required component from a composite volt-
age. The entire radio reception technique is based on reso- It can be seen that for ro = roo = 1/V' LC, the impedance
has the minimum value and is equal to the resistance R.
nan~e. In order to receive with a given radio receiver the The quantity appearing in the parentheses in formula
station we are interested in, the receiver must be tuned. In
(11.46) is denoted by X and is called the reactance:
other words, by varying C and L of the oscillatory circuit
we must attain the cOincid~nce between its natural {requeuc; X = ro[, - 1/roC. (11.47)
and. the frequency of radIo wayes emitted bv the radio Here the quantity roL is called the inductive reactance,
statlop. .
while the quantity VroC is called the capacitive reactance.
:rhe phenomenon of resonance is also associated with a cer-
tam danger: the external e.m.f. Or voltage may be small but
They are denoted XL and Xc respectively. Thus,
t:le,v?ltages.acros~in(li,:i~Ju(d elem~nts of the circuit'(the XL=roL, X c =1/roC, X=XL-X c ,
(11.48)
capaCItor or lllductlOn coIl} may attalll the values utlnaerous
for people. This should always be remembered. "
z= V R2+X~".
It should be noted that the inductive reactance grows with
the frequency",w, while t~e. cap~citive rea~tan.ce decreases
with increasing ro, When It IS saId that a CircUlt .has no ca-
11.4. Alternating Current pacitance, this must be t;nderstood so that there IS no c~pac­
~otal. Hesistance (Impedance). Steady-state forced electric itive reactance which is equal to 1/roC and hence valllsh.es
if C -lo- 00, (when a capacitor is replaced by a short-CIr-
oscillatIOns can he treated as an altel'llatinrr current now-
ing in ~.circuit having a capacitance, inducta~ce, and resist- cuited section), .,
ance. Lnder the action of external voltage (which plays Finally, although the reac~anc~ is I:Ue~suredII~ t~e same
the role of external e.m. f.) units as the resistance, they dIffer 1Il prmclple. ThIS difference
consists in that only the resistance determines irreversi~le
U =" U m cos tl)t, (11.43) processes in a circuit such as, for example, the converSIon
the current in the circuit varies according to the law of electromagnetic energy into Joule's heat,
Power Liberated in an A.C. Circuit. The instantaneous
I = 1 m cos (cut '- cp), (11.44) value of power is equal to the product of instantaneous
where values of voltage and current:
I - Um roL-1/roc P {t) = VI =-c= Vml m cos rot cos (rot - cp). (11.49)
m-R2+(roL_1/roC)2' tancp= R (11.45)
Using the formula cos (rot - (f') = cos rot cos cp +
The problem is reduced to determining the current ampli- + sin rot sin if, we transform (11.49) as follows:
tude and the phase shift of the current relative to U. P (t) = U ml m (C08 2 wt cos cp + sin rot cos rot sin cp),
The obtained ~xpression for the current amplitude 1 (ro)
m Of praeticnl importance is the value of power averaged
can be formally Interpreted as Ohm's law for the amplitude
values of current and voltage. The quantity in the denomi- over a period of oscillation. ConsJdering that (cos 2 cut) =
natOr of this expression, which has the dimension of resist- =--= 1/2 and (sin wt cos wt) = 0, we obtain
ance, is denoted by Z and is called the total resistance , Or U I
(P) = m m cos cpo (11.50)
2
19*
292
11. Electric Oscillations Problems 293
Th~s expl'ession can be written' l"ff .
take lIlto account that 'IS foIl It a {I erent form If we the concept of electric resistance becomes wider since in ad-
(see Fig. 11.'1) U 0" _ nOI
ws ram the vector diagram dition to the conversion of electric. energy into Joule's
, m C S lp - m' Hence, heat, other types of energy transformations are also possi-
1 ble. For example, a part of electric energy can be converted
{P)--2 RI:". (11.51) into mechanical work (electric motors).
TI,lis power.i~ equal to that of the direct current I' _
,T' Ie quantities - I m/V2". Problems

I = I ml V2, V = V mlV2 (11.52) • 11.1. Free undamped oscillations. Free undamped oscillations
occur in an oscillatory circuit consisting of a capacitor with capacitance
are called effective (root-mea ' C and an induction coil with inductance L. The voltage amplitude
voltaO'e All a t IHsquare) values of current alld on the capacitor is Urn- Find the e.m.f. of self-induction in the coil at
'"' . mme ers 'md VO It t
r.m.s. values of curl"ellt' . d meel'S are graduated for the moments when its magnetic energy is equal to the electric energy
'('1 an vo l tage of the capacitor•
. . Ie expression for the 'Ivera .
r.m.s. \'alues of CUlTent a<nd gle power (11.50) in tenns of Solution. According to Ohm's law,
vo t age 1las the form
RI = U + ~s
. . (P) = VI cos £P, (11.53) where U is the voltage across the capacitor (U = CPl -CP2)' In our
whele the factor cos ( is u II case, R = 0 and ,hence ~s = -U.
Thus, the power developed ~~a y called. th~ power factor. It remains for-Us to find voltage U at the moments when the electric
only on the voltage and C'lrrent an a.c. Circuit depends not energy of the capacitor is equal to the magnetic energy of the coil.
between them. . but also on the phase angle Under this condition, we Clln write
CU2 C[]2. L12 CU2
When <r c~ n/2, the value (P) - 0 ~=-2--;'-2-=2 -2-'
nes of V and I In th' - regardless of the val-
quarter of a per:iod fr~s case, the energy transmitted over a whence I U 1= Um/Vi:"
is exactly equal to the ~ea generator. to an external circuit As a result, we have I ~s I = Um / ,(2."
circuit to the generatol' dU;~O't~~nsm~tted from the external 8 11.2. An oscillatory circuit consists of an induction coil with
so that the entire energy "os""11 e n~;xt quarter of the period inductance L and an uncharged capacitor of capacitance C. The resist-
generator and the external ~i~~u~tt es uselessly between the ance of the circuit R = 0. The coil is in 11 permanent magnetic field.
The total magnetic nux piercing all the turns of the coil is <1>. At the
The dependence of the pOwer 0 . moment t = 0, the magnetic field was abruptly switched off. Find
account while desianing t ~ C?S 'P should be taken into the current in the circuit as a function of time t.
-current. If loads b;ing fedr~I~~~lssrn lines for alternating Solution. Upon an abrupt switching off of the external magnetic
may be considerably less than a ligh reactance X, cos £P field at the moment t = 0, all' induced current appears, but the capac-
to transmit the required 0 one. n snch cases, in order itor still remains uncharged. In accordance with Ohm's law, we have
voltage of the O'enel'ator) ~t ~'er to a COnsumer (for a given drD dI
I, which leads to an incr~~sel~tecessary to increase current RI=-dt- L dt
ing wires. Hence loads ." useless energy losses in feed-
be always distribut~d "oln{~UCttances and capacitances should In the given case R = 0, and hence <1> +
LI = 0. This gives <1> = LIo,
.
as possible. For this purpose °
s as it· make ffi'
cos en
'Y
asease
I to One
where lois the initial current (immediately after switching off \he
field).
X ilS small as possible i 't IS su IClent to make reactance After the external fleld has been switched off, the process is de-
ductivc and capaciti:vc' r~~l'ct~n~~:u(~ tl~ equality of in- scribed by the following equation:

When the conductors fanning an a.c. cir~lif~~~ in motion


q dl
O=---L- (1)
, C dt
295
Problems
11. ElectrIc O,Ciliatton,
, d quate the obtained ex pres-
dirIerentiate (1) with respect to t,lIne ~2 e _ 00 sin a =, 0 whence
Pifferentiation of this equation with respect to time gives sion to zero at t = O. We obt:un, -r cos a '
•• 1 tan a = -IVoo. The required ratIO IS
1+ LC 1=0.
U (O) _ cos a = 1 ,- 1. " (2)
This is ihe equation of harmonic oscillations. We seek its solution --u;;;- - 11-;- tan 2 a 1 l-r(~/W)-
in the form
.. U d (. (0) are shown in Fig, 11.8.
I = 1m cos (OOot a). + The quantIt~es hm an.,.!-.
Considerlllg t at w--
2 _
Wo
P.2 we transform
p ,
(2) as follows:

The constants 1m and a can be found from the initial conditions U(O}/U m = 1rl-(~/OOo)2= jll-H CI4.L,
2

1(0).==/0 , j (0)=0 A - RI2l and w2 = lILC.


where we took into account t hat p - J 0
(the ·second condition follows from Eq. (1), since at the initial moment 't C and induc-
t = 0 the capacitor was uncharged). From these conditions we find • 11 .5. In an oscillatory circui t wi th a capacl ance
a = 0 and 1m = 10. As a result, we obtain
I = locos OOot = (11)IL) cos OOot, u
.where 000 = 1/YLC.
• 11 .3. Q-Iactor 01 a circuit. An oscillatory circuit with a low
damping has a capacitance C and inductance L. In order to sustain
in it un(lamped harmonic oscillations with the voltage amplitude Um
aeroSll the capacitor, it is necessary to supply the average power (P).
Find the Q-factor of the circuit.
Solution. Since damping is low, we can make use of formula (11.23): o
Fig. 11.9
Q = 2nW16W, (1) Fig. lUi
where W = CUfn/2 and 6W = (P) T, T being the period of damped . h' 'h the current varies with
oscillations. In our case, T ~ To = 2n l" LC. HaVing substituted tance L damped oscil!ations lCcu~,
time in accordance With the aw ,t
(T:: ;C
f 1-
e-Ilt sin oot, Find the volt-
these expreSllions into (1), we obtain
age across the capacitor as a function 0 Ime.. h 'f
Ufn .. / 0 h I kw'se directIOn as t e POSI Ive
Q= 2{P) V y. Solution. Let us ch?ose (}. e c1 ~c9) IAccording to Ohm's law for
direction of circumven~iOn. Ig. I . . RI = cp _ qJ + 0 S' In our·
• 11.4. Damped oscillations. Ali oscillatory circuit includes a ction lRL2 of the CirCUit, we lave 1 2
se • _ Ie = U c where q is the charge on
capacitor of capacitance C, an induction coil of inductance L, a resistor ~ = -"-LI and !P2 - !PI - q . '.
01 resistance R, and a key. The capacitor was charged with the key calset' 2' Hence Ohm's law can he written as
p a e . •
oren. When the key was closed, a current began to flow. Find the ratio
o the voltage across the capacitor at time t to the voltage at the initial Uc=-RI-LI,
moment (immediately after closing the key). Having substituted i,:to this formula the expression for J (t) and
SolutIon. The voltage across the capacitor depends on time in the its derivative, we obtam
same way as the charge does. Hence we can write
.- RIme-ill (_R sin oot-oo cos uH).
U=Ume-f\t cos (oot+a). (1) U c- 2~ f'
t-" ..,.

M tbe ltdtial Diomt'nt t= 0, thevoI~ge. f/(O) =V",eoa~, -here . . tl parentheses to sine. For
Let us transform the expreSSIOn III lC
U". ia tI» ..mpUfu~eat thi8 moment. We must find U (O)/U m • i.e. 1 'd 't b 1f w2 +~ = Wo and in-
d
this purpose, we multiply an (iVI e l Y
. . :~ t.hia rapose, we sb~l use anothe~ initial conditioll: at the troduce angle II through the formulas. .
(1\
mo_' t = 0, current I = fJ = O. Since q = CU, it is sufficient to
-~/ooo = cos 0, c,)/c,)o = Sill 6.
1'1
I

296
11. Electric Oscillations
Problems 297
Then
U c = Rlmw o -131 . In our case cp < 0, which means that the: .current leads the external
,
211 I' I'Jn(Wi-O)=1
m
VLiC-l3
e
t .
sm(wt-15), voltage (Fig. 11.10).
\\here angle 15 is, in accordanc " h . • t t .8. An a.c. circuit, containing a series-connected capacitor
assumes the values :r/2 15 I' "It (1), In the second ua d ' and induction coil with a certain resistance, is connected to the source
lags behind the curren~' <h n. Thus, the voltage acrossqth rant, .1.1'.
m p ase. I' capacItor

• 11.6. Steady-state os "II· t · ' .


L and resistance R. . CJ a IOns. An mduction '1 f'
external voltage U ,::slJonne~ted ll.!. the moment t =coJ t~ mductance wLl m
a function of time m cos Wi. Fmd the current in th a ~our~e of
. e ClfCUl t as R I",
Solution. In our case, RI = .
. U-LJ , or
I-~(R/L) I =c Um cos wt Im~·I
we
11'<0.1
U
The s?IUtion of this equation' t h . . '" I 1m Um-
equatIOn plus the particular ;~lut~ genefral solution of the homog we
IOn 0 the nonhom eneous ----- u~
I (i) = Ac-(R/L)I _ Urn ogeneous equation:
Fig. 11.10 Fig. 11.11 Fig. 11.12
}/R2-i- w2L2 cos (wt-<r),
where A is an arbitrar .
(11.36): tan fJJ =
The constant A . 'f
wLii
constant and angle fJJ is defined b' d"
~ con ItlOn
of external alterpating \o"oltage whoSl'> frequency can be altered without
changing its arffplitude. At frequencies Wt and Wt the current ampli-
H IS ount! from th "'a1 tudes in the circuit proved to be the same. Find the resonance frequency
ence A = -(Um /yr R2 --'-. w2L2 I' Inlh condition 1(0) = 0 of the current.
C' ) cos fJJ· This gives .
Solution. According to (11.35), the amplitudes are equal under the
I (t) == 'm
r condition
V R2 -;- w2L2 [cos (Wi - ff) _e-(R/L)t cos fJJJ.
For a sufficiently lar e
~g~~i~~ ~m;~l,. an!
I
\\~~ ~bta~~c~h~ ~~~:I~~t;~: :ic~~ts becomes
I WtL-_1_1,=1 w2L - _1_1.
WtG w,C
The maximum of the resonance curve for current corresponds to the
(1)

u Ion I (t) ex:


frequency equal to the natural frequency Wo = 1/yLC. Further•
•. 11.7. Forced oscillation'" A . let Wt < Wo < Wt (the opposite inequality is also possible, it will
~f~~:;i~~n~~l~~~ec~i~~t~h a~d ri~~~~o~,o~sac~:::e~\~do:::isting of not affect the final result). Equality (1) can then be written in a more
general form: wG/wt - WI = Wt - W~/Wh or
~~:a~K~~~~e~~d~~~:~:t~gA~;r.~~~7:Ffn'd' t:eh:h::P;~~l~t~t:~ w2 -Wt = w5 (..!._..!-) .
Solution In th WI w 2
. case under consideration
I'
Cancelling out (1)2 - WI on both sides of this equality, we obtain
U = Urn cos wt, I = 1 m cos (~t _ 1 = W5/WIW2' whence
Where fJJ is defined b f 'P),
The unknown valu~Ofocrampul~t (11.36):. tan fJJ = -t/WCR Wo= Y W I W 2'
. f aCI ance C b f, •
SlOn or the current amplitude' I _ c~n .~ ound from the expres- • t t .9. Vector diagram. A circuit consisting of a series-connected
• m - {;ml J' R2 (1/wC)! +
, whenee capacitor of capacitance C and induction coil with resistance Rand
. C=1/w V(L' m1l m)2_R2. inductance L is connected to an external voltage with the amplitude
U m and frequency w. Assuming that current in the circuit leads in
Havmg substituted this expression' t f phase the external voltage, construct the vector diagram and use it
In 0 ormula f~r tan ff, we obtain for determining the amplitude of voltage across the coil.
tan cp= -I'I"Wm/Rl m)2_1.
Solution. The vector diagram for the case under consideration is
shown in Fig. 11.11. It is readily seen from this diagram that the

20-0181
298
11. Electric Oscillations Appendkes
amplitude of voltage across the coil is

ULRm=Im YRI+COILI,
m
where 1 = Um/Y RZ + (coL - 1/COC)I. In the presence of resistance
the voltage across the coil leads the current by less than 1(/2.
• f f .10. Power in an a.c. circuit. A circuit consisting of a series.
connected resistor R with no inductance and an induction coil with
a certain resistance is connected to the main system with the r.m.s.
voltage U. Find the thermal power developed in the coil if the r.m.s. t. Notations tor Units of Measurement
valuesUzof respectively.
and voltage across the resistor R and the coil are equal to U
I
Solution. Let us use the vector diagram shown in Fig. 11.12. In A ampere min minute
accordance with the law of cosines we obtain from this diagram C coulomb Mx maxwell
eV electronvolt N newton
UZ=Ur+U~-2UIUZ cos CPL' (f) F farad Oe oersted
The power developed in the coil is G gauss Q ohm
gram rad radian
Where I = UI/R.
Pz = IU z cos CPL, (2) L
hr
henry
hour
s
S
second
siemens
Combining Egs. (1) and (2), we obtain Hz hertz T tesla
J joule V volt
P z = (Uz_ Ui -U~)/2R. K kelvin It W watt
m meter Wb weber

2. Decimal Prefixes for Units of Measurement

T, therli-, 1011 d, deci-, 10-1 n, nano, 10-'


G, giga-, 108 e, centi-, 10-1 p, pico-, 10-11
M, mega-, 10e m, milli-, 10-3
&. kilo-, 102 f.L' micro-, 10-'
h. hecto-, 101
de. deC8-, t01 I
-
3. Units of Measurement dofGEIU8Seetr~ ~~~etiC
Qnantitles in 81 an a J

Unit or mea-
Nota- 6urement SI unit
Quantity tton RatioCGS unit
SI
I CGS

Foree F N
I .
dyne 10e
Work. energy A,W J 1Q7
Charge q C e~SE unit
C 3 X lQt
-
Appendicll8

300 Table 4. ConI.

Te6le 3. COllI. Gaussian system


81
Relation

~'I_'
Unit of _ -
Quantity Nota- nnllDellt
tton COS . .,
81
I eGS Cireulation of vector ~Edl=O
E in an eleetros-
Electric field strength E VIm CGSE unit tJ(3X{O') \8de tield p=ql
Potential, voltage cp,U V CGSE unit tPJX) Electric ,moment of a
Electric ID~ent p C·m CGSE unit 3X IOU dipole' . 8E "'-{pXEl W=-p·E
Polarization P ClIO' CGSE unit 3XI06 Electric dipole p 1D F=PBr,lU- •
Vector D D ClIO' CGSE unit 12n xtOi field E
Capacitance C F cm 9xtOU Relation between pG: p=KE
Current I A CG8E unit 3 X tQll ladsation and field
Current density j Aim' CGSE unit 3xtOl strength
Resistance R Q CG8E unit t/(9X IOU) o'=pn=')(,En
Resistivity Q·m CGSE unit t/(9X {O') Relation between cr', a'=Pn=uoE n
Conductance ~ 8 CGSE unit 9X1OU pOOE D=eoE+P
D=E+4T£P
Conductivity cr 81m CG8E unit 9xtOt Definition of vector D e=1.+4T£')(,
Magnetic induction B T G 101 Relation between e s=1.+k
Magnetic flux, magnetic- D=8E
nux linkage <D,W Wb Mx 1(18 and "
Relat.ion between D D=88 0E
Magnetic moment Pm A·m' CG8E unit UP
Magnetization J A/m CG8E unit 10"4 and E
GaUll8 t.heorem for
~ DdS=4nq
Vector H H A/m Oe 4axto-a
Inductance L H cm tOt vectorD
capacit.anC8 of a ca- C=q/U
pacitor eS
4. Basic Formulas of Electricity and Magnetism
Capacitanee of a par- c= 4nh
allel-plate capaci-
in 81 and Gaussian Systems
t.or
Energy of system of
Relation 81 charges
To\81 energy of in-
teract.ion
Field E of a point
E=---
t q
Energy 01 capacitor
charge 2
4neo r Elect.ric tield energy
Field E in a parallel- densit.y
plate capacitor and
at the surface of a
E=_G_ E=~
II
80B Continuity equation
conductor
Potential of the field 1 q Ohln's law
of a point charge cp---- lp=..!L
- 4neo r r Joule-Lens law
2
LoleDU fo~
Relation between E E= - V .cp, CPl-lp,= ) E dl
and rp
1
303
Appendices
302 Appendices Table 4. Cont.

Table 4. Cont. Gaussian system


81
Relation

E'=E+IvoY~l \l'~'=E++lVOXBl
Relation 81 Gaussian system
Laws of transforma-
tionof fields E and B
Field B of a moving
charge
B=A q[vXr] B=_1_ q[vXr) for Vo « c B'=B-~l\'oXEl
c
B'=B-+lVo>~El
4n r3 c r3
E·B=inv
Biot-Savart law dB=~ [j X rjdV dB=_1 [j Xr) dV Electromagnetic field EL_B2= inv
4n r3 c r ll E2_c2B2= inv
invariants 1 dl.1l
dl.1l 't5i=-7Cft
dB= flo 1 [dJ >< r] dB,=~ l[dlXr) Induced e.m.f. ~i=-dt
4n r3 c r3 L=cl.1ljI
Field B Inductance L=l.1ljI
L=4nltn2V
(a) of straight
current B=J:L -E.- B=~~ Inductanee of a so- L=jJ.jJ. on 2 V
1 dI
4n b c b lenoid dI
~s= -L
'6>8= -(2" L dt
(b) at the centre of
a loop
B=~ 2n1 B="~ 2Jtl E.m.f. of seH-induc-
tion
at
4n R c Ii 1 LP
L[2
4Jt Energy of the'!' mag- W=-c--r-
(c) in solenoid B=--nl W=~
B=flo nI c netic field of a cur-
rent B·B
B·H w=--gn-
Ampere's law dF=I [dJ X B] dI~= !-[dl X B) Magnetic field energy w=~
c • density i aD
aD
dF=[j X B] dV
1
dF=- [j X B) dV Displacement current jd=' a t id = 4it ---at
c density 1 ~ .
~ Edl= - 7 ~
~Edl= - ~ BdS
Force of interaction Maxwell's equations BdS
between parallel F ll = ~o 21 1 1 2 F
u
= 21 1[2
in integral forlD
currents
Magnetic moment of
'in b
1
b
~ D dS= ~ pdV ~ Das=4n ~ pdV
a current loop Pm =IS Pm=-lS
c
~
~ ~
Hdl=
Magnetic dipole Poi aB Hdl= (i+D)dS
in field B F=Pm~'
Work done in displace- 1
=4: ~(i+~lt)dS
ment of a current A= - I (11)2-<1'1)
loop c . .~ ndS=O
~ BdS=O
Circulation of ma g-\
netization '~J dl=I' J dl~>~
J l'
Maxwell's e';\uation~
\ ~ X E =-7 B
1 •

Definition of ve,~tor II H = B/[lo-J II = B--411J vxE=-B


Circulation of vector II
in differential form \ V· D =4np
~ ~ II dl = 4n I V·D=P
~ ( j + ~lt)
II in a stationary II dl=I 'j t~
oJ VX H =
field
Relations between' I I vxH=j+D
\ 'V.B=O _
J and H
fJ. and Xi ,
I J =;<:11
, 1f1=1+ 4n X \ o
Band H , B=[tH
Subject Index
Appendices

Tabl. 4. Cont.

Relation 81 Gaussian system

Relation between E
and H in electro- E Y80e=H~ electrodynamic, 144
Ampere, 117
magnetic wave magnetic, 143
Poynting's vector S";"[EXHJ
c coulomb, 12
S="41tIEXHj
Betatron, 222 current, I
Density of electro- 1 boundary conditions, 80, 92, 192 alternating, 290ff
magnetic field mo- G=_1 [E XII)' G=-4- IEXn
2c nc for Band H. 182ff conduction, 174
mentum
direct, 178
displacement, 253f, 271
5. Some Physical Constants Capacitance, 57ff, 86, 90 induced, 248
of cylindrical capacitor, 60 magnetization, 174

Velocity of light in vacuum c= 2.998 X 108 m/s


G= { 6.67 X 10-11 m3/(kg·s 2)
) of isolated conductor, 57
of parallel-p\pte capacitor, 59
of parallel wires, 65
surface, 175, 178, 192
volume, 175
molecular, 174, 177
Gravitational constantl
6.67 X 10- 8 cmS/(g.s2) of spherical capacitor,.59 polarization, 256
Acceleration of free fall
Avogadro constant
g=9.807 m/s 2
N A = 6.022 X 10 23 mole-l
I unit of, 58
capacitor, 57ft, 86, 90
density of, 259
quasistationary, 278
1.602 X 10-19 C charging, 134 straight, 188
Charge of electron or proton e = { 4.80 X 10-10 CGSE units discharging, 133 wt~l, 254
Rest mass of electron m e =0.9H X 10-30 kg energy of. 110 current element(s),
e { 1.76 X 1011 C/kg parallel-plate, 103, 108f. 1.1.4f, linear, 146
Specific charge of electron
--;n;- = 5.27 X 1017 CGSE units/g 137 volume, 146
Rest mass of proton mp= 1.672 X 10-27 kg charge invariance, 198£ curve,
Electric constant 80=0.885 X 10-11 F/m circulation, magnetization, 189
1/4n80 = 9 X 109 m/F . of vector B, 166 resonance. 288f
Magnetic constant Ito = 1..257 X 10- 8 H/m of vector E, 245
l-lo/4n= 10- 7 H/m of vector H, 179, 193
Relation between velocity c=1/~ of vector J, 176ff, 193
condition, betatron, 246, 260 Damping factor, 280, 283
of light and 80 and Ito
quasi-steady, 277 diamagnetism, 180
conductor in electrostatic field. 45 dielcctric(s).
homogeneous, 120 anisotropic, 101
nonhomogeneous, 138 isotropic, 101
constant, liquid, W8f
dielectric (electric), 12, 78£. 86, polarization of, 67
102
aq

II
Ii

307
306 Subject Inckz Subject Ind,:::e.:::..c -------
--_._-------'- potential, 2G, 28, 155
dielectric susceptibility, 71 electrosta tic shielding, 51
Farad, 58f sinks of, 25
dipole, e.m.f.,
ferroelectric(s), 72 solenoidal, 155
electric, 34, 44 induced, 244 ferromagnetic(s), 180, 188f, 232
energy of, 38 of self-induction, 227 soul'ces of, 25
ferromagnetism, 188,I!H
moment of, 34ff energy, vector,
theory of, 191 circulation of, 15, 25
dipole moment, 71 of charged capacitor, 99f, 139
intrinsic, G8 field, divergence of, 24
of charged conductor, 99f
coulomb, 122 flux of, 15
domain(s), 191 of current, electric, 11, 198, 206
intrinsic, 238 in charge-carrying conductor, force(s),
Ampere, 155,157,159,169,171,
magnetic, 236 121
Edge effects, 59, 66, 8G, 100 mutual, 238 in dielectrics, 61
of electric field, 94, 100, 103, intensity (strength) of, lfff, moment of, 160, lG9
electret(s), 68, 79
105, 107, 237, 276 28 work of, 161
electric charge(s), 11, 83, 88f, 92
of electromagnetic field, 253 macroscopic, 45ff coerei ve, 1gO
bound (polarization), 69f, 73f
localization, iOOff, 112 microscopic, 45 coulomb, 122
bulk, 69f
magnetic, 243 of point charge, 12 in dielectric, 106H
at conductor surface, 83
surface, 69£ of two current loops, 238 eleclromagn,etic, 198f electric, 157
of interaction, 96ff momentSm of, 268 extraneous, 122f, 235
in current-calTying conductor induced electromotive, 217
120 ' for system of point charges, deIlsi ty of, 269
extraneous, 78, 88f, 109, 138 97, 105, 110f electrostatic, 155 force(s) ,
fictitious, 63 total, 97, 99f, 110 in vacuum, 11 of interaction,
induced, 46, 66 intrinsic, 98, 105, 110f of freely moving relativistic electric, 144£
surface, density of, 63 of magnetic field, 234 236f charge, 207 magnetic, 144£
electric circuit,
275f " in homogeneouS magnetic, 185 Lorentz, 141H, ~03, 212f
a.c., 291, 297 of current, 234 magnetic, 142, 1115, 155, 198, Il1agn('tic, 157
branched, 126££ energy flux, 266 206 in -ma~netic, 19G
density of, 264 on the axis of circular cur- surface density of, 109£
oscillatory, 293
electri'c current, 116, 119 equation(s), rent, 147 nux,
continuity, 116, 118, 261 of current-carrying plane, 153 magnetic,
density of, 117
direct, 116, 120 in differential form, 118 energy of, 2/iO conservation of, 230
linear, density of, 153 Laplace, 54f forces in, 240 total, 212
quasis ta ti 0 na ry , 133 material, 260 in magnetic, 175 of vector D, 273
steady-state, 118 Maxwell, 253ff, 257f, 272, 274 permanent, 220 formula, Thomson, 281
in differential form, 259 of solenoid, 151, 167,181 frequency,
electric oscillations, 277££
in integral form, 257 of straight current, 151 natural, 280
electric resistance, 119
methods of calcnlation 120 properties of, 261f in substance, 172ff rl'Sonance, 289
electromagnetic inductio~ 217ff
symmetry of, 262 of toroid, 152
of oscillatory circuit, 277, 279f of uniformly moving eharge, Generator, MHD, 22!t
225 "
electromagnetic wave(s), 101, 262f Poisson, 5H 143 Henry, 227
pressure of, 268, 270 equipotential surfaces 32, 47. in vacuum, 141ff hysteresis, 72, 180
57 ' varying, 221 magnetic, 190
in vacuum, 269
vortex, 222, 275
.......

308
Subject Index
Subject Index
Image charge, 56
impedance, 290f Newton's third, 95 oscillation(s) , Self-induction, 226
inductance, 226, 238 Ohm, 119, 127, 131, 134, 136ff, forced,294 solenoid, 151
calculation of, 250f 140f, 233, 249, 293, 295 free, 280 superconductor(s), 230
In differential form, 120 steady-state, 285, 296 susceptibility, magnetic, 180
mutual, 231, 240, 251 generalized, 123
induction, undamped, 280
in integral form, 124
electrosta tic, 46
for nonuniform 8ubclreult Tesla, 144
magnetic, 143ff 123, 138 ' theorem,
direct calculation of, 163 of transformation for fields E Paramagnetic(s), 180, 194, 197 on circulation of field vectors,
mutual, 233, 251 and B, 200ft, 206, 213 permeability, 180, 189 15, 25, 52, 80f, 137f, 148f,
residual, 190 logarithmic decrement of point, Curie, 191 151ft, 155, 166, 178f, 193f,
interaction(s), 11 damping, 283 polarization of dielectric, 68, 73, 195f, 211, 248, 255
electromagnetic, 11, 86, 103 Gauss, 15ft, 19, 21ft, 25, 33,
gravitational, 11 polarization, 70fI 41, 44, 47f, 59, 65, 72,
76f, 79ft, 83, 85, 871f, 9f,
of parallel currents, 168 Magnet, permanent, 68, 195 unit of, 71 93, 102, 112, 121, 137,
strong, 11 magnetlc(s), 172, f 74, 181 potential, 25, 27, 30, 33, 43f, 148, 150, 157, 168, 173,
weak, 11 homogeneous, 174 47f, 53f, 57, 61, 64, 66, 200, 211
86f, 8&r 97f, 102, 124 application of, 19
invariants of electromagnetic nonhomogeneous, 175, 177 of point charge, 28 in dHIerential form, 23ff, 73
field, 208, 214 magnetic constant, 143
of sphere, 58 • Poynting's 264
magnetic field intensity, 179 of system of charges, 29
magnetic flux, 167 of reciprocity, 231, 252
Joule heat, 235 potential difference, 86, 90, 99 uniqueness, 54ff
linkage of, 219
power liberated in a.c. circuit, theory of relativity, 269
magnetic induction, 143f, 146, 291f, 298
163, 166, 168, 187 power factor, 292 thermal power, 132
Law(s), magnetic prot~ction, 185 transformation(s), Lorentz, 20f
Ampere, 155ff pressure, magnetic, 244
magnetism, relative nature of, 204 principle of superposition, 12f,
Biot-Savart, 145ff, 149f, 153, magnetization, 172ft, 195 18, 29, 60, 66, 146, 166f
164
mechanism of, 177 process, transient, 133, 140 Vector,
of conservation, residual, 190 D, 76ft
of electric charge, 11, 118 magnetohydrodynamics, 244 B, 178
of energy, 106, 114, 130, 139 method, Poynting's, 265, 269, '274f
235, 241 '
energy, 106, 240 Q-factor, 284, 289, 294 • vector diagram, 287, 297
Coulomb, 109'
image, 54ft, 62f, 67 volt, 28
in field form, 12
molecule(s), voltage, 59
louie, 129ff
nonpolar, 68 Reflection coefficient, 270
in differential form, 131 polar, 68
Kirchhoff, 126, 256 refraction of B-lines, 184
first, 126 moment, magnetic, 157ft relaxation time, 134, 283 Work,
second, 126f resistance of conducting medium, of displacement of current
136 loop, 161ft
Lenz, 217ff, 243
Ohm, 119 resistivity, 119 of electric field, 103f
of magnetio field, in differen- oscillation(s), resonance, 289f of e.m.f. source, 139
tial form, -194
damped, 281f, 294
Also from Mir Publishers

TO THE READEH Fundamental Laws of Mechanics


. Mir Publishers would be grateful for
your comments on the content translation I. Irodov
and design of this book. We w'ould also be
pleased to receive any other suggestions The book considers the basic laws of mechanics-laws of mo-
you may wish to make. tion and laws of conservation of energy, impul~ and moment
Our address is:
of impulse. The possibilities of their applications are indicated.
Mir Publishers A large number of problems and exercises are included.
2 Pervy Rizhsky Pereulok The material. is arranged in accordance with the syllabus of
I-110, GSP, Moscow, 129820
USSR a course in general physics fo~ university students. The book
is meant f6r university students who intend to specialize in
physics.
,." .)',."
pr
{
,"C'

1 /
I/"
,
I '

, I
'I'
f
. Handbook of Physics

B. Yavorsky and A. Detlaf

A companion volume to Vygodsky's Handbook of Higher Math-


ematic" designed for use by engineers, technicians, research
workers, students, and teachers of physics. Includes definitions
of basic physical concepts, brief formulations of physical laws,
concise descriptions of phenomena, tables of physical quanti-
ties in various systems. of units, universal physical constants,
ete.

-JI-------
u~ --

LX Libris I
Amit Dhakulkar
,i

I," ,i Klan Nosferatu


Date:t2pgjOL

U
1,i i

Ii
11111111111 Iltltll IIIUllIIIJI
CBS OUTSTANDING PUBLICATIONS
~""'''''''';;;''olCalCUm In on. Vlrilti...-
j
Bille: llw 01 El.cttDlMgMlicl
Fulldlnwltlllllw 0'Mlchanicl
Ptobltmtln G_II Pt>yajct
Fu_uol 01 Physiu .
A Prob*n Book in M~ AnlIyals •
A Q>U.coon 01 Ounu_ tond ProbItomf .. Phyojgo
~Idb<>ot 01 E~ MlIllIInMio;$
00 VDol Know et.niJ,.., I
l/loI9anic CIMomosUy Vol. I & 11
au.&iwiw CIwnouI s.mimicr........lysi1
I
00An1iuo1NW AnI/yIoI
o-qry MI~tlCa
P,_i. of TlMolllohfdl..,lic
Row l.....lify
Oogtnic a-nilUy
"""*
H..t E.n.1IY Con_on

N ~ S.,._: Foun4tnon 01 Malhi (hlll... SoIwtd)


Ai:Ilfanc..t T-'ogf ~
Moc*n AlgoatMI ~
MatncS_ _

'"'.-.
HilI- D'fIII(IlieI • ..
Coi"PIu. Anltyl..
11. .1 Anllysil
Functional Aollf'"
MlilIU .. Thao..,
..
..
,
Partlll Dill".nl..1 Eq .. ~"un ..
"Ilh,m,tlc, lor JRF/CSIK r"l
M lh.mlllciio. B!h •• Lectu"rlhlp TMl
..
..

Mllhlmlliul Stlllilici ..
MIIl»m"l~ 10. LIT ..
MllrlJt Anll\'loll ..
Hyd.Oll¥nlmk:1 "
VilcOUI Fluid ..
~.a~1 M.chlnicl (Spac:. Dynlmics) ..
Htond Book 01 V.ctCft ..
H'-ndbooltol Mlthl (HiQllw&w-) ..
oPwltlon RMNfCh ..
MIII'II'DI lAS PIPI" I & II (Combirtld) "
MI~ lor M.P. (MllIla) "
NI'"..;ca1 Anal""
IFS: Mllha (lor Indian for'-. SeMe.)
..
"

~
CBS PUBLISHERS & OIsmlBUTORS R.. 55.00
1.58W1A. 11. 0..,.. Ger4. _ !)oft. 110 _ ~
-
ISIIN "·23H301-5

You might also like